JAC Class 12 History Important Questions Chapter 12 औपनिवेशिक शहर : नगर-योजना, स्थापत्य

Jharkhand Board JAC Class 12 History Important Questions Chapter 12 औपनिवेशिक शहर : नगर-योजना, स्थापत्य Important Questions and Answers.

JAC Board Class 12 History Important Questions Chapter 12 औपनिवेशिक शहर : नगर-योजना, स्थापत्य

बहुविकल्पीय प्रश्न (Multiple Choice Questions)

1. 1857 के विद्रोह से पूर्व दिल्ली के कोतवाल थे-
(क) गंगाधर नेहरू
(ख) अरुण नेहरू
(ग) मोतीलाल नेहरू
(घ) जवाहरलाल नेहरू
उत्तर:
(क) गंगाधर नेहरू

2. अखिल भारतीय जनगणना का प्रयास भारत में प्रथम बार हुआ-
(क) 1772 में
(ख) 1872 में
(ग) 1786 में
(घ) 1881 में
उत्तर:
(ख) 1872 में

3. शाहजहाँनाबाद को बसाया था-
(क) अकबर ने
(ख) औरंगजेब ने
(ग) शाहजहाँ ने
(घ) हुमायूँ ने
उत्तर:
(ग) शाहजहाँ ने

4. ब्रिटिश काल में पहला हिल स्टेशन बना-
(क) शिमला
(ख) दार्जिलिंग
(ग) नैनीताल
(घ) मनाली
उत्तर:
(क) शिमला

5. आगरा, दिल्ली तथा लाहौर की एक सामान्य विशेषता थी-
(क) तीनों शहर मिली-जुली संस्कृति के लिए जाने जाते थे।
(ख) तीनों शहर सूफी सन्तों तथा भक्तों को खूब प्रिय थे।
(ग) तीनों शहर सोलहवीं और सत्रहवीं सदी में मुगल शासन के महत्त्वपूर्ण केन्द्र थे।
(घ) तीनों शहर मुस्लिम आबादी वाले थे।
उत्तर:
(ग) तीनों शहर सोलहवीं और सत्रहवीं सदी में मुगल शासन के महत्त्वपूर्ण केन्द्र थे।

JAC Class 12 History Important Questions Chapter 12 औपनिवेशिक शहर : नगर-योजना, स्थापत्य

6. भारत में रेलवे की शुरुआत हुई
(क) 1856 में
(ग) 1855 में
(ख) 1853 में
(घ) 1851 में
उत्तर:
(ख) 1853 में

7. कलकत्ता के स्थान पर दिल्ली को राजधानी बनाया गया-
(क) 1911 में
(ख) 1916 में
(ग) 1908 में
(घ) 1899 में
उत्तर:
(क) 1911 में

8. प्लासी का युद्ध लड़ा गया था-
(क) 1755 में
(ख) 1757 में
(ग) 1765 में
(घ) 1857 में
उत्तर:
(ख) 1757 में

9. दक्षिण भारत के दो शहरों मदुराई और कांचीपुरम की मुख्य विशेषता थी-
(क) ये दोनों नगर हमलावरों से सुरक्षित थे।
(ख) इनमें मुख्य केन्द्र मन्दिर तथा महत्त्वपूर्ण व्यापारिक केन्द्र होते थे।
(ग) इनमें विश्व के प्रमुख धर्मों के त्यौहार मनाए जाते थे।
(घ) समाज के लोग इन शहरों में साहित्यिक चर्चा किया करते थे।
उत्तर:
(ख) इनमें मुख्य केन्द्र मन्दिर तथा महत्त्वपूर्ण व्यापारिक केन्द्र होते थे।

10. औपनिवेशिक शहरों में प्राय: निम्न तीन शहर शामिल किए जाते थे-
(क) दिल्ली, बम्बई, कलकत्ता
(ख) दिल्ली, मद्रास, कलकत्ता
(ग) मद्रास, कलकत्ता, बम्बई
(घ) इनमें से कोई नहीं
उत्तर:
(ग) मद्रास, कलकत्ता, बम्बई

11. मद्रास, कलकत्ता एवं बम्बई तीनों शहरों की एक सामान्य विशेषता थी-
(क) तीनों व्यापारिक राजधानी थे
(ख) तीनों मूलतः मत्स्य ग्रहण एवं बुनाई के गाँव थे
(ग) तीनों शहरों को अंग्रेजों ने बसाया था
(घ) इनमें से कोई नहीं
उत्तर:
(ख) तीनों मूलतः मत्स्य ग्रहण एवं बुनाई के गाँव थे

12. मिर्जा गालिब थे-
(क) चित्रकार
(ग) कवि
(ख) शायर
(घ) फिल्मकार
उत्तर:
(ख) शायर

13. निम्न में से किस शहर का सम्बन्ध राइटर्स बिल्डिंग से है?
(क) बम्बई
(ग) कलकत्ता
(ख) दिल्ली
(घ) मद्रास
उत्तर:
(ग) कलकत्ता

14. कासल क्या था?
(क) एक दुर्ग
(ग) एक बस्ती
(ख) एक शहर
(घ) इनमें से कोई नहीं
उत्तर:
(क) एक दुर्ग

15.
विनोदिनी दासी कौन थीं?
(क) चित्रकार
(ख) रंगकर्मी
(ग) वास्तुकार
(घ) उपर्युक्त सभी
उत्तर:
(ख) रंगकर्मी

16. फोर्ट सेंट जार्ज कहाँ स्थित है?
(क) मद्रास
(ख) दिल्ली
(ग) कलकत्ता
(घ) बम्बई
उत्तर:
(क) मद्रास

17. किस भारतीय शहर का सम्बन्ध फोर्ट विलियम से है?
(क) दिल्ली
(ग) मद्रास
(ख) कलकत्ता
(घ) जयपुर
उत्तर:
(ख) कलकत्ता

JAC Class 12 History Important Questions Chapter 12 औपनिवेशिक शहर : नगर-योजना, स्थापत्य

18. नवशास्त्रीय स्थापत्य शैली का उदाहरण है-
(क) टाउन हॉल
(ख) लाल किला
(ग) विक्टोरिया टर्मिनल
(च) ताजमहल
उत्तर:
(क) टाउन हॉल

19. सात टापुओं का शहर है-
(क) बम्बई
(ग) मद्रास
(ख) कलकत्ता
(घ) दिल्ली
उत्तर:
(क) बम्बई

20. लॉटरी कमेटी का सम्बन्ध है-
(क) पुरातत्त्व व्यवस्था से
(ख) सैनिक व्यवस्था से
(ग) प्रशासनिक व्यवस्था से
(घ) नगर नियोजन से
उत्तर:
(घ) नगर नियोजन से

21. गारेर मठ क्या था?
(क) एक मैदान
(ग) एक बस्ती
(ख) एक शहर
(घ) एक पक्षी
उत्तर:
(क) एक मैदान

रिक्त स्थानों की पूर्ति कीजिए

1. 1853 में बम्बई से ………………….. “तक रेलवे लाइन बिछाई गई।
2. 1896 में बम्बई के ………………….. -होटल में पहली बार फिल्म दिखाई गई।
3. कलकत्ता की जगह दिल्ली को ………………….. में राजधानी बनाया गया।
4. 1857 में बम्बई में पहली स्पिनिंग और ………………….. मिल की स्थापना की गई।
5. ईस्ट इण्डिया कम्पनी द्वारा ………………… में कलकत्ता में सर्वोच्च न्यायालय की स्थापना की गई।
6. एशियाटिक सोसाइटी की स्थापना ………………….. में ………………….. की गई।
7. ………………… में बम्बई को ब्रिटेन के राजा ने कम्पनी को दे दिया।
8. ग्रामीण अंचल में एक छोटे नगर को ………………. कहा जाता है।
9. ………………….. कहा का तात्पर्य एक छोटे स्थायी बाजार है।
10. पुर्तगालियों ने 1510 में ……………… तथा डचों ने 1605 में ………………. में आधार स्थापित कर लिए थे।
11. मद्रास, बम्बई और कलकत्ता का आधुनिक नाम क्रमश: …………………. , …………………… और …………………. है।
12. प्लासी का युद्ध ……………. में हुआ था।
13. सर्वे ऑफ इण्डिया का गठन ……………. ने किया गया था।
14. कपडों जबकि …………… अपने स्टील उत्पादन के लिए प्रसिद्ध था।
15 ……………… की स्थापना गुरखा युद्ध के दौरान की गई थी।
उत्तरमाला –
1. ठाणे
2. वाटसन्स
3. 1911
4 बीविंग
5. 1773
6. सर विलियम जोन्स, 1784
7. 1661
8. कस्बा
9. गंज
10. पणजी, मछलीपट्टनम
11. चेन्नई, मुम्बई और कोलकाता
12. 1757
13. 1878
14. कानपुर, जमशेदपुर
15. शिमला।

अतिलघूत्तरात्मक प्रश्न-

प्रश्न 1.
प्लासी का युद्ध कब और किसके बीच हुआ?
उत्तर:
1757 ई. में प्लासी का युद्ध अंग्रेजों और बंगाल के नवाब सिराजुद्दौला के बीच हुआ।

प्रश्न 2.
गेटवे ऑफ इण्डिया कब और कहाँ बनाया गया?
उत्तर:
गेटवे ऑफ इण्डिया 1911 में बम्बई में बनाया

प्रश्न 3.
औपनिवेशिक भारत में स्थापित दो हिल स्टेशनों का उल्लेख कीजिए।
उत्तर:
(1) शिमला
(2) माउंट आबू

JAC Class 12 History Important Questions Chapter 12 औपनिवेशिक शहर : नगर-योजना, स्थापत्य

प्रश्न 4.
ईस्ट इण्डिया कम्पनी द्वारा अपनी बस्तियों की किलेबन्दी करने का प्रमुख उद्देश्य क्या था? उत्तर- यूरोपीय कम्पनियों के बीच प्रतिस्पर्द्धा के कारण सुरक्षा बनाये रखना।

प्रश्न 5.
अखिल भारतीय जनगणना का प्रथम प्रयास किस वर्ष में किया गया था?
उत्तर:
सन् 1872 ई. में

प्रश्न 6.
भारत के मद्रास, कलकत्ता और बम्बई तीनों शहर मूलत: किस प्रकार के गाँव थे?
उत्तर:
ये तीनों शहर मत्स्य ग्रहण और बुनाई के गाँव थे।

प्रश्न 7.
इंग्लिश ईस्ट इण्डिया कम्पनी के एजेन्ट मद्रास में किस सन् में बस गये?
उत्तर:
सन् 1639 में।

प्रश्न 8.
इंग्लिश ईस्ट इण्डिया कम्पनी ने मद्रास, कलकत्ता और बम्बई में किस प्रकार के कार्यालय स्थापित किये?
उत्तर:
व्यापारिक और प्रशासनिक कार्यालय।

प्रश्न 9.
भारत में मुगलकाल में शाही प्रशासन और सत्ता के तीन महत्त्वपूर्ण केन्द्रों के नाम लिखिए।
उत्तर:
(1) आगरा
(2) दिल्ली और
(3) लाहौर।

प्रश्न 10.
दक्षिण भारत के ऐसे दो नगरों के नाम लिखिए जिनमें मुख्य केन्द्र मन्दिर होता था।
उत्तर:
मदुरई और कांचीपुरम

प्रश्न 11.
18वीं सदी के अन्त तक स्थल आधारित साम्राज्यों का स्थान कैसे साम्राज्यों ने ले लिया?
उत्तर:
जल आधारित यूरोपीय साम्राज्यों ने।

प्रश्न 12.
भारत में रेलवे की शुरुआत कब हुई?
उत्तर:
1853 में भारत में रेलवे की शुरुआत हुई। प्रश्न 13, 19वीं सदी के मध्य तक भारत में कौनसे दो औद्योगिक शहर थे?
उत्तर:
(1) कानपुर,
(2) जमशेदपुर।

प्रश्न 14.
कलकत्ता में किन लोगों ने बाजारों के आस-पास ब्लैक टाउन में परम्परागत ढंग से दालानी मकान बनवाए?
उत्तर:
अमीर भारतीय एजेन्टों और बिचौलियों ने।

प्रश्न 15.
कलकत्ता में मजदूर वर्ग के लोग कहाँ रहते में।
उत्तर:
शहर के विभिन्न इलाकों की कच्ची झोंपड़ियों

प्रश्न 16.
1857 के विद्रोह के बाद अंग्रेजों के लिए किस नाम से नए शहरी इलाके विकसित किये गये?
उत्तर:
सिविल लाइन्स’ के नाम से

प्रश्न 17.
सिविल लाइन्स में किनको बसाया गया?
उत्तर:
केवल गोरों को।

प्रश्न 18.
पहला हिल स्टेशन कब और कौनसा बनाया गया?
उत्तर:
पहला हिल स्टेशन 1815-16 में शिमला में स्थापित किया गया।

प्रश्न 19.
ईस्ट इण्डिया कम्पनी ने कलकत्ता और बम्बई में अपने व्यापारिक केन्द्र कब स्थापित किये?
उत्तर:
(1) 1661 में बम्बई में
(2) 1690 में कलकत्ता में।

प्रश्न 20.
1800 ई. तक जनसंख्या की दृष्टि से कौनसे भारतीय शहर विशालतम शहर बन गए थे?
उत्तर:
(1) मद्रास
(2) कलकत्ता
(3) बम्बई।

प्रश्न 21.
भारत में दशकीय जनगणना कब से एक नियमित व्यवस्था बन गई थी?
उत्तर:
पहला हिल-स्टेशन 1815-16 में शिमला में स्थापित किया गया।

JAC Class 12 History Important Questions Chapter 12 औपनिवेशिक शहर : नगर-योजना, स्थापत्य

प्रश्न 22.
1900 से 1940 के मध्य 40 वर्षों की अवधि में भारतीय शहरी आबादी में कितनी वृद्धि हुई?
उत्तर:
दस प्रतिशत से बढ़कर लगभग 13 प्रतिशत हो गई।

प्रश्न 23
19वीं शताब्दी में भारत में कौनसे रेलवे नगर अस्तित्व में आए?
उत्तर;
जमालपुर, वाल्टेयर तथा बरेली।

प्रश्न 24.
अंग्रेजों ने अपनी किन बस्तियों की किलेबन्दी की थी?
उत्तर:
बम्बई, मद्रास और कलकत्ता की।

प्रश्न 25.
औपनिवेशिक काल के दो औद्योगिक शहरों के नाम लिखिए।
उत्तर:
(1) कानपुर
(2) जमशेदपुर।

प्रश्न 26.
चमड़े की चीजों तथा ऊनी और सूती वस्त्रों के निर्माण के लिए कौनसा औद्योगिक नगर प्रसिद्ध था?
उत्तर:
कानपुर। था?

प्रश्न 27.
जमशेदपुर किसके उत्पादन के लिए प्रसिद्ध
उत्तर:
स्टील उत्पादन के लिए।

प्रश्न 28.
राइटर्स बिल्डिंग कहाँ पर स्थित थी?
उत्तर:
कलकत्ता में।

प्रश्न 29.
इंग्लिश ईस्ट इण्डिया कम्पनी के एजेंट कलकत्ता में किस वर्ष बसे?
उत्तर:
1690 ई. में।

प्रश्न 30.
अंग्रेजों द्वारा स्थापित दो हिल स्टेशनों के नाम बताइये ये कब स्थापित किये गए?
उत्तर:
1818 में माउण्ट आबू तथा 1835 में दार्जिलिंग।

प्रश्न 31.
सेनेटोरियम के रूप में किनका विकास किया गया था?
उत्तर:
हिल स्टेशनों का।

प्रश्न 32.
कौनसा हिल स्टेशन भारतीय सेना के कमांडर इन चीफ (प्रधान सेनापति) का भी अधिकृत वास बन गया था?
उत्तर:
शिमला।

प्रश्न 33.
भारत के नये शहरों में यातायात के कौनसे साधन थे?
उत्तर:
घोड़ागाड़ी, ट्राम तथा बस।

प्रश्न 34.
भारतीय शहरों में किस नये वर्ग का प्रादुर्भाव हुआ?
उत्तर:
मध्य वर्ग’ का ।

प्रश्न 35.
भारतीय शहरों में किन लोगों की माँग बढ़ रही थी?
उत्तर:
वकीलों, डॉक्टरों, शिक्षकों, क्लकों, इंजीनियरों, लेखाकारों की।

प्रश्न 36.
आमार कथा (मेरी कहानी) की रचना किसने की थी?
उत्तर:
विनोदिनी दास ने

प्रश्न 37.
मद्रास में ‘व्हाइट टाउन’ का केन्द्र कौन
उत्तर:
फोर्ट सेंट जार्ज।

JAC Class 12 History Important Questions Chapter 12 औपनिवेशिक शहर : नगर-योजना, स्थापत्य

प्रश्न 38.
कलकत्ता को किन तीन गाँवों को मिलाकर बनाया गया था?
उत्तर:
(1) सुतानाती
(2) कोलकाता
(3) गोविन्दपुर।

प्रश्न 39.
कलकत्ता का किला क्या कहलाता था?
उत्तर:
फोर्ट विलियम।

प्रश्न 40.
‘स्वास्थ्यकर नगर’ और ‘अस्वास्थ्यकर नगर’ कौनसे होते थे?
उत्तर:
व्हाइट टाउन’ स्वास्थ्यकर तथा ‘ब्लैक टाउन’ अस्वास्थ्यकर कहलाते थे।

प्रश्न 41.
लार्ड वेलेजली भारत के गवर्नर जनरल कब बने?
उत्तर:
1798 ई. में।

प्रश्न 42.
लार्ड वेलेजली ने कलकत्ता में अपने लिए रहने के लिए जो महल बनवाया, वह क्या कहलाता था?
उत्तर:
गवर्नमेंट हाउस

प्रश्न 43.
ग्रामीण क्षेत्रों के लोग किस प्रकार जीवन- यापन करते थे?
उत्तर:
खेती, जंगलों में संग्रहण तथा पशुपालन द्वारा।

प्रश्न 44.
ग्रामीण क्षेत्रों से कस्बों तथा शहरों को अलग दिखाने वाली एक विशेषता बताइए।
उत्तर:
कस्बों तथा शहरों की किलेबन्दी की जाती थी, जबकि ग्रामीण क्षेत्रों की नहीं। –

प्रश्न 45.
जनगणना के आँकड़ों का एक महत्त्व बताइए। उत्तर- जनगणना शहरीकरण के इतिहास का अध्ययन करने का बहुमूल्य स्रोत है।

प्रश्न 46.
आरम्भ में अधिकांश लोग जनगणना को सन्देह की दृष्टि से क्यों देखते थे?
उत्तर:
क्योंकि लोगों का मानना था कि सरकार नए कर लागू करने के लिए जाँच करवा रही है।

प्रश्न 47.
बगीचा पर’ का सम्बन्ध किस औपनिवेशिक शहर से था?
उत्तर:
कलकत्ता।

प्रश्न 48.
प्रारम्भ में बम्बई कितने टापुओं का इलाका था?
उत्तर:
सात टापुओं का।

प्रश्न 49.
बम्बई में नव-शास्त्रीय (नव क्लासिकल शैली) में निर्मित दो इमारतों के नाम लिखिए।
उत्तर:
(1) बम्बई का टाउन हाल
(2) एल्फिंस्टन सर्कल।

प्रश्न 50.
बम्बई में नव-गॉथिक शैली में निर्मित चार इमारतों के नाम लिखिए।
उत्तर:
(1) बम्बई सचिवालय
(2) बम्बई विश्वविद्यालय
(3) उच्च न्यायालय
(4) बम्बई टर्मिनस रेलवे स्टेशन।

प्रश्न 51.
नव-गॉथिक शैली में निर्मित सर्वोत्कृष्ट इमारत कौनसी थी जो बम्बई में निर्मित थी?
उत्तर:
विक्टोरिया टर्मिनस रेलवे स्टेशन

JAC Class 12 History Important Questions Chapter 12 औपनिवेशिक शहर : नगर-योजना, स्थापत्य

प्रश्न 52.
मद्रास में इण्डो सारसेनिक स्थापत्य शैली में निर्मित भवन का नाम लिखिए।
उत्तर:
मद्रास ला कोर्टस।

प्रश्न 53.
बम्बई में निर्मित ‘गेटवे ऑफ इण्डिया’ किस शैली का प्रतीक है?
उत्तर:
गुजराती शैली का।

प्रश्न 54.
राजा जार्ज पंचम और उनकी पत्नी मेरी के स्वागत के लिए कौनसी इमारत बनाई गई थी और कब?
उत्तर:
(1) गेटवे ऑफ इण्डिया
(2) 1911 में।

प्रश्न 55.
भारत में सबसे पहली रेल किन शहरों के बीच चलाई गई और कब?
उत्तर:
(1) बम्बई से ठाणे तक
(2) 1853 में।

प्रश्न 56.
बम्बई, मद्रास और कलकत्ता विश्वविद्यालयों की स्थापना कब की गई?
उत्तर:
1857 में।

प्रश्न 57.
अंग्रेजों ने कलकत्ता की जगह दिल्ली को कब राजधानी बनाया?
उत्तर;
1911 में

प्रश्न 58.
फोर्ट विलियम पर टिप्पणी लिखिए। उत्तर- फोर्ट विलियम कलकत्ता का एक प्रसिद्ध दुर्ग था। इसका निर्माण ब्रिटिश ईस्ट इण्डिया कम्पनी ने करवाया था।

प्रश्न 59
6वीं तथा 17वीं सदी में मुगलों द्वारा बनाए गए शहर किन बातों के लिए प्रसिद्ध थे? उत्तर-ये शहर जनसंख्या के केन्द्रीकरण, अपने विशाल भवनों तथा शाही शोभा व समृद्धि के लिए प्रसिद्ध थे।

प्रश्न 60.
मुगल राजधानियों दिल्ली और आगरे के राजनीतिक प्रभुत्व की समाप्ति के बाद किन क्षेत्रीय राजधानियों का महत्त्व बढ़ गया था?
उत्तर- लखनऊ, हैदराबाद, सारंगपट्म, पूना, नागपुर, बड़ौदा, तंजौर

प्रश्न 61.
पेठ और पुरम में क्या अन्तर है?
उत्तर:
पेठ तमिल शब्द है जिसका अर्थ होता है बस्ती जबकि पुरम शब्द गाँव के लिए प्रयोग किया जाता है।

प्रश्न 62.
अंग्रेजों ने अपनी व्यापारिक गतिविधियों का केन्द्र सर्वप्रथम किसे बनाया था?
उत्तर:
सूरत।

प्रश्न 63.
कलकत्ता में अंग्रेजों की सत्ता की प्रतीक इमारत कौनसी थी?
उत्तर:
गवर्नमेंट हाऊस।

प्रश्न 64.
लॉटरी कमेटी का सम्बन्ध किस औपनिवेशिक शहर से था?
उत्तर:
कलकत्ता।

प्रश्न 65.
बम्बई सचिवालय का डिजाइन किसने बनाया था?
उत्तर:
एच. एस. टी. क्लेयर विलकिन्स ने।

प्रश्न 66.
मद्रास, कलकत्ता और बम्बई में कौनसे इलाके ब्रिटिश आबादी के रूप में जाने जाते थे?
उत्तर:
मद्रास, कलकत्ता और बम्बई में क्रमश: फोर्ट सेंट जार्ज, फोर्ट विलियम और फोर्ट इलाके ब्रिटिश आबादी के रूप में प्रसिद्ध थे।

प्रश्न 67.
आजकल बम्बई, मद्रास तथा कलकत्ता किन नामों से पुकारे जाते हैं?
उत्तर:
आजकल बम्बई, मद्रास तथा कलकत्ता क्रमशः मुम्बई, चेन्नई तथा कोलकाता नाम से पुकारे जाते हैं।

प्रश्न 68.
पुर्तगालियों तथा डचों ने कब और कहाँ अपने व्यापारिक केन्द्रों की स्थापना की?
उत्तर:
पुर्तगालियों ने 1510 में पणजी में तथा डचों ने 1605 में मछलीपट्नम में अपने व्यापारिक केन्द्र स्थापित किये।

प्रश्न 69.
अंग्रेजों तथा फ्रांसीसियों ने कब और कहाँ व्यापारिक केन्द्र स्थापित किये?
उत्तर:
अंग्रेजों ने 1639 में मद्रास में तथा फ्रांसीसियों ने 1673 में पांडिचेरी में अपने व्यापारिक केन्द्रों की स्थापना की।

JAC Class 12 History Important Questions Chapter 12 औपनिवेशिक शहर : नगर-योजना, स्थापत्य

प्रश्न 70.
भारत के एक आधुनिक औद्योगिक देश न बनने का क्या कारण था?
उत्तर:
इंग्लैण्ड की पक्षपातपूर्ण औपनिवेशिक नीति के कारण।

प्रश्न 71.
‘राइटर्स बिल्डिंग’ से आप क्या समझते हैं?
उत्तर:
राइटर्स बिल्डिंग कम्पनी का एक मुख्य प्रशासकीय कार्यालय था क्लर्क ‘राइटर्स’ कहलाते थे।

प्रश्न 72.
‘दि मार्बल पैलेस’ के बारे में आप क्या जानते हैं?
उत्तर:
किसी शहरी सम्भ्रान्त वर्ग के एक भारतीय परिवार ने कलकत्ता में दि मार्बल पैलेस नामक एक अत्यंत भव्य इमारत बनवाई थी।

प्रश्न 73.
अंग्रेजों के लिए ‘सिविल लाइन्स’ में किन इमारतों का निर्माण किया गया?
उत्तर:
चौड़ी सड़कों, बड़े बगीचों में बने बंगलों, बैरकों, परेड मैदानों, चर्च आदि का निर्माण किया गया।

प्रश्न 74.
चितपुर बाजार कहाँ स्थित था?
उत्तर:
चितपुर बाजार कलकत्ता में ब्लैक टाउन और व्हाइट टाउन की सीमा पर स्थित था।

प्रश्न 75.
ब्लैक टाउन में भारतीयों द्वारा बनवाये गए मन्दिर को अंग्रेज क्या कहते थे?
उत्तर:
ब्लैक पगौडा।

प्रश्न 76.
‘व्हाइट टाउन’ से क्या अभिप्राय है?
उत्तर:
जिस शहरी इलाके में अंग्रेज (गोरे लोग ) रहते थे, वह ‘व्हाइट टाउन’ कहलाता था।

प्रश्न 77.
‘ब्लैक टाउन’ किसे कहते थे?
उत्तर:
जिस शहरी इलाके में भारतीय (काले लोग ) रहते थे, वह ‘ब्लैक टाउन’ कहलाता था।

प्रश्न 78.
प्रसिद्ध उद्योगपति जमशेदजी टाटा ने बम्बई में किस शैली में किस होटल का निर्माण करवाया था?
उत्तर:
गुजराती शैली में ताजमहल होटल का।

प्रश्न 79.
कस्बा और गंज में क्या अन्तर था?
उत्तर:
कस्बा ग्रामीण अंचल में एक छोटे नगर को तथा गंज एक छोटे स्थानीय बाजार को कहा जाता था।

प्रश्न 80.
अंग्रेजों की नजर में ‘ब्लैक टाउन’ कैसे थे?
उत्तर:
अंग्रेजों की नजर में ‘ब्लैक टाउन’ अराजकता तथा हो-हल्ला के केन्द्र व गन्दगी और बीमारियों के स्त्रोत थे।

प्रश्न 81.
अंग्रेजों द्वारा हिल स्टेशनों की स्थापना किस उद्देश्य से की गई थी?
अथवा
अंग्रेजी शासन काल में पहाड़ी शहरों (हिल स्टेशनों) का विकास क्यों किया गया?
उत्तर:
हिल स्टेशन अंग्रेज सैनिकों को ठहराने, सीमाओं की चौकसी करने और शत्रु के विरुद्ध आक्रमण करने के लिए महत्त्वपूर्ण स्थान थे।

प्रश्न 82.
किस गवर्नर जनरल ने अपनी काउंसिल कहाँ स्थानान्तरित की थी और कब?
उत्तर:
1864 में गवर्नर जनरल जान लारेन्स ने अपनी काउंसिल शिमला में स्थानान्तरित की थी।

प्रश्न 83.
मद्रास के नये ‘ब्लैक टाउन’ से क्या अभिप्राय है?
उत्तर:
नये ‘ब्लैक टाउन’ में भारतीय लोग रहते थे। यहाँ आड़ी-टेढ़ी संकरी गलियों में अलग-अलग जातियों के मोहल्ले थे।

प्रश्न 84.
लार्ड वेलेजली ने शहर में मूलभूत सुविधाएँ प्रदान करने के लिए सरकार की जिम्मेदारियों का उल्लेख किसमें किया था?
उत्तर:
1803 में ‘कलकत्ता मिनट्स’ में

JAC Class 12 History Important Questions Chapter 12 औपनिवेशिक शहर : नगर-योजना, स्थापत्य

प्रश्न 85.
अंग्रेज ‘बस्ती’ का प्रयोग किस रूप में ‘करते थे?
उत्तर:
गरीबों की कच्ची झोंपड़ियों के रूप में।

प्रश्न 86.
बम्बई अन्तर्राष्ट्रीय व्यापार का केन्द्र क्यों था?
उत्तर:
एक प्रमुख बन्दरगाह होने के नाते।

प्रश्न 87.
19वीं शताब्दी के अन्त तक भारत का आधा निर्यात तथा आयात किस शहर से होता था?
उत्तर:
बम्बई से

प्रश्न 88.
अम्बई से व्यापार की एक महत्त्वपूर्ण वस्तु कौनसी थी?
उत्तर:
अफीम

प्रश्न 89.
व्यापार की उन्नति के कारण कौनसा भारतीय शहर ‘भारत का सरताज शहर’ कहलाता था?
उत्तर:
बम्बई।

प्रश्न 90.
एशियाटिक सोसायटी ऑफ बाम्बे’ का कार्यालय कहाँ है?
उत्तर:
बम्बई के टाउनहाल में।

प्रश्न 91.
बम्बई की उस इमारत का नाम लिखिए जो ग्रीको-रोमन स्थापत्य शैली से प्रभावित है।
उत्तर:
एल्फिंस्टन सर्कल।

प्रश्न 92.
औपनिवेशिक शहरों में इमारतें बनाने के लिए कौनसी स्थापत्य शैलियों का प्रयोग किया गया?
उत्तर:
(1) नवशास्त्रीय
(2) नव-गॉथिक शैली
(3) इण्डो-सारसेनिक शैली।

प्रश्न 93.
नव-गॉथिक स्थापत्य शैली की विशेषताएँ बताइये।
उत्तर:
ऊँची उठी हुई छलें, नोकदार मेहराबें, बारीक साज-सज्जा

प्रश्न 94.
इण्डो-सारसेनिक स्थापत्य शैली क्या थी?
उत्तर;
इण्डो-सारसेनिक स्थापत्य शैली में भारतीय और यूरोपीय दोनों शैलियों के तत्त्व थे।

प्रश्न 95.
चॉल से आप क्या समझते हैं?
उत्तर:
बम्बई में जगह की कमी एवं भीड़भाड़ के कारण एक विशेष प्रकार की इमारतें बनायी गयीं जिन्हें चॉल कहा गया।

प्रश्न 96.
स्थापत्य शैलियों से क्या पता चलता है? उत्तर-स्थापत्य शैलियों से अपने समय के सौन्दर्यात्मक आदशों एवं उनमें निहित विविधताओं का पता चलता है।

लघुत्तरात्मक प्रश्न-

प्रश्न 1.
18वीं शताब्दी के अन्त तक भारत में स्थल आधारित साम्राज्यों का स्थान जल आधारित शक्तिशाली यूरोपीय साम्राज्यों ने ले लिया।” व्याख्या कीजिए।
उत्तर:
18वीं शताब्दी के अन्त तक जल आधारित शक्तिशाली यूरोपीय साम्राज्यों ने प्रमुख स्थान प्राप्त कर लिया। अन्तर्राष्ट्रीय व्यापार, वाणिज्यवाद तथा पूँजीवाद की शक्तियाँ अब समाज के स्वरूप को निर्धारित करने लगीं। अब मद्रास, कोलकाता तथा मुम्बई जैसे औपनिवेशिक बन्दरगाह शहर नई आर्थिक राजधानियों के रूप में प्रकट हुए। ये औपनिवेशिक प्रशासन और सत्ता के केन्द्र भी बन गए। अब नये भवनों और संस्थानों का विकास हुआ।

प्रश्न 2.
तर्क सहित सिद्ध कीजिये कि औपनिवेशिक शहरों का सामाजिक जीवन वर्तमान शहरों में भी दिखाई पड़ता है?
उत्तर;
औपनिवेशिक शहरों की भाँति वर्तमान शहरों में भी घोड़गाड़ी, ट्रामों, बसों का यातायात के साधनों के रूप में प्रयोग किया जाता है। उनहाल सार्वजनिक पार्क, रंगशाला, सिनेमाहाल आदि लोगों के मिलने-जुलने के स्थान हैं। वर्तमान शहरों में ‘मध्य वर्ग’ का काफी प्रभाव है। वर्तमान शहरों में स्कूल, कॉलेज, विश्वविद्यालय, लाइब्रेरी आदि शिक्षा के केन्द्र बने हुए हैं। स्वियों में भी जागृति आई है।

प्रश्न 3.
‘चाल’ से आप क्या समझते हैं? इनकी विशेषता लिखिए।
उत्तर:
शहर में जगह की कमी और भीड़भाड़ के कारण बम्बई में एक खास तरह की इमारतें बनाई गई, जिन्हें ‘चाल’ का नाम दिया गया। ये इमारतें बहुमंजिला होती थीं, जिनमें एक-एक कमरे वाली आवासीय इकाइयाँ बनाई जाती थीं। इमारत के सारे कमरों के सामने एक खुला बरामदा या गलियारा होता था और बीच में दालान होता था। इस प्रकार की इमारतों में बहुत थोड़ी सी जगह में बहुत सारे परिवार रहते थे। सभी लोग एक-दूसरे के सुख- दुःख में भागीदार होते थे।

JAC Class 12 History Important Questions Chapter 12 औपनिवेशिक शहर : नगर-योजना, स्थापत्य

प्रश्न 4.
अंग्रेजों ने भारत में भवन निर्माण के लिए यूरोपीय शैली को किस कारण चुना? तर्क सहित लिखिए।
उत्तर:
(1) इसमें एक अनजान देश में जाना- पहचाना सा भू-दृश्य रचने की और उपनिवेश में घर जैसा अनुभव करने की अंग्रेजों की चाहत दिखाई पड़ती थी।
(2) अंग्रेजों को लगता था कि यूरोपीय शैली उनकी श्रेष्ठता, अधिकार और सत्ता का प्रतीक होगी।
(3) वे सोचते थे कि यूरोपीय ढंग की दिखने वाली इमारतों से औपनिवेशिक शासकों और भारतीय प्रजा के बीच फर्क और फासला साफ दिखने लगेगा।

प्रश्न 5.
अंग्रेजों ने मद्रास (चेन्नई) के आसपास अपना वर्चस्व कैसे स्थापित किया?
उत्तर:
1611 में अंग्रेजों ने मछलीपट्नम में अपनी फैक्ट्री स्थापित की। लेकिन जल्दी ही उन्होंने मद्रास को अपना केन्द्र बनाया, जिसका पट्टा वहाँ के जा ने 1639 में उन्हें दे दिया। राजा ने उन्हें उस स्थान की किलेबन्दी करने, सिक्के ढालने तथा प्रशासन की अनुमति दे दी। यहाँ पर अंग्रेजों ने अपनी फैक्ट्री के इर्द-गिर्द एक किला बनाया, जिसका नाम फोर्ट सेंट जार्ज रखा गया 1761 में फ्रांसीसियों की हार के बाद मद्रास और सुरक्षित हो गया।

प्रश्न 6.
दक्षिण भारत की परिस्थितियाँ अंग्रेजों के लिए अधिक अनुकूल थीं, क्यों? लिखिए।
उत्तर:
दक्षिण भारत में अंग्रेजों का मुकाबला किसी ताकतवर राज्य से नहीं हुआ 1665 में तालीकोटा के युद्ध के बाद विजयनगर साम्राज्य नष्ट-भ्रष्ट हो गया और उसके स्थान पर छोटे-छोटे राज्य स्थापित हो गये, जैसे- बीदर, बरार, गोलकुण्डा आदि। इन राज्यों को अंग्रेजों ने अपनी कूटनीति के द्वारा डरा-धमका कर या लालच देकर अपने अधीन कर लिया। केवल मैसूर के राज्य से ही उन्हें टक्कर लेनी पड़ी।

प्रश्न 7.
नक्शे हमें क्या बताते हैं और क्या छिपाते है?
उत्तर:
1878 में भारत में सर्वे ऑफ इण्डिया का गठन किया गया। उस समय के नक्शों से हमें काफी जानकारी उपलब्ध होती है, साथ ही हमें अंग्रेजों की भेदभावपूर्ण सोच का भी पता लग जाता है। उदाहरण के लिए, नक्शे में गरीबों की बस्तियों को चिह्नित नहीं किया गया। इसका अर्थ यह लगाया गया कि नक्शे में रिक्त स्थान अन्य योजनाओं के लिए उपलब्ध हैं। जब इन योजनाओं को शुरू किया गया तो गरीबों की बस्तियों को वहाँ से हटा दिया गया।

प्रश्न 8.
औपनिवेशिक काल में ग्रामीण इलाकों एवं कस्बों के चरित्र में अन्तर बताइए।
उत्तर:
ग्रामीण इलाकों एवं कस्बों के चरित्र में भिन्नता के मुख्य बिन्दु निम्नलिखित हैं-
(1) ग्रामीण इलाकों के लोग खेती, पशुपालन एवं जंगलों में संग्रहण द्वारा अपनी जीविका का निर्वाह करते हैं। इसके विपरीत कस्बों में शासक, व्यापारी, प्रशासक व शिल्पकार आदि रहते थे।
(2) कस्बों एवं शहरों की प्रायः किलेबन्दी की जाती थी। यह किलेबन्दी उन्हें ग्रामीण क्षेत्रों से अलग करती थी।
(3) कस्बों पर ग्रामीण जनता का प्रभुत्व रहता था। वे खेती से प्राप्त होने वाले करों एवं अधिशेष के आधार पर निर्भर रहते थे।

प्रश्न 9.
मध्यकालीन दक्षिण भारत के शहरों की मुख्य विशेषताओं का उल्लेख कीजिए।
उत्तर:
(1) दक्षिण भारत के नगर मदुरई तथा कांचीपुरम प्रमुख केन्द्र थे।
(2) दक्षिण भारत के अनेक नगरों में बन्दरगाह होते थे।
(3) ये व्यापार के मुख्य केन्द्रों के कारण विकसित हुए थे।
(4) दक्षिण भारत के शहरों में धार्मिक उत्सव अत्यधिक धूम-धाम के साथ मनाए जाते थे।

प्रश्न 10.
कस्बा एवं गंज के बारे में आप क्या जानते हैं?
उत्तर:
कस्बा – कस्बा ग्रामीण अंचल में एक छोटे नगर को कहा जाता है जो सामान्यतः स्थानीय विशिष्ट वस्तुओं का केन्द्र होता है।
गंज-गंज एक छोटे स्थायी बाजार को कहा जाता है। कस्बा और गंज दोनों कपड़ा, फल, सब्जी एवं दुग्ध उत्पादों से सम्बन्ध थे। ये विशिष्ट परिवारों एवं सेना के लिए सामग्री उपलब्ध करवाते थे।

प्रश्न 11.
अमेरिका के गृहयुद्ध और स्वेज नहर के खुलने का भारत की आर्थिक गतिविधियों पर क्या प्रभाव पड़ा?
अथवा
अमेरिकी गृह युद्ध ने भारत में ‘रैयत’ समुदाय के जीवन को किस प्रकार प्रभावित किया?
उत्तर:
सन् 1861 में अमेरिका में गृहयुद्ध शुरू होने के कारण वहाँ से कपास का निर्यात बन्द हो गया। इससे भारतीय कपास की माँग बढ़ी, जिसकी खेती मुख्य रूप से दक्कन में होती थी। 1869 में स्वेज नहर को व्यापार के लिए खोल दिया गया, जिससे विश्व अर्थव्यवस्था के साथ-साथ बम्बई की अर्थव्यवस्था भी मजबूत हुई। बम्बई की सरकार और भारतीय व्यापारियों ने बम्बई को ‘भारत का सरताज शहर’ घोषित कर दिया।

प्रश्न 12.
18वीं शताब्दी के प्रारम्भिक वर्षों में औपनिवेशिक सरकार ने नगरों के लिए मानचित्र तैयार करने पर ध्यान क्यों दिया ?
अथवा
औपनिवेशिक सरकार ने मानचित्र तैयार करने पर विशेष ध्यान क्यों दिया?
उत्तर:
औपनिवेशिक सरकार की मान्यता थी कि किसी शहर की बनावट और भूदृश्य को समझने के लिए मानचित्र आवश्यक होते हैं। इस जानकारी के आधार पर वे इलाके पर अधिक नियन्त्रण स्थापित कर सकते थे। शहरों के मानचित्रों से हमें उस स्थान पर पहाड़ियों, नदियों व हरियाली का पता चलता है। ये समस्त बातें रक्षा सम्बन्धी उद्देश्यों के लिए योजना तैयार करने में बड़ी उपयोगी सिद्ध होती हैं। मकानों की सघनता, सड़कों की स्थिति आदि से इलाके की व्यावसायिक सम्भावनाओं की जानकारी मिलती है।

JAC Class 12 History Important Questions Chapter 12 औपनिवेशिक शहर : नगर-योजना, स्थापत्य

प्रश्न 13.
जनगणना से प्राप्त आँकड़ों का क्या महत्त्व है?
उत्तर:
(1) ये आँकड़े शहरीकरण का अध्ययन करने के लिए एक बहुमूल्य स्रोत हैं।
(2) बीमारियों से होने वाली मृत्युओं की सारणियों, आयु, लिंग, जाति व व्यवसाय के अनुसार लोगों को गिनने की व्यवस्था से संख्याओं का एक विशाल भण्डार मिलता है।
(3) जनगणना के माध्यम से आबादी के बारे में सामाजिक जानकारियों को सुगम्य आँकड़ों में बदला जा सकता था।

प्रश्न 14.
औपनिवेशिक भारत में जनगणना सम्बन्धी भ्रमों का उल्लेख कीजिये ।
उत्तर:
(1) आबादी के विभिन्न वर्गों का वर्गीकरण करने के लिए अलग-अलग श्रेणियाँ बनाई गई थीं। कई बार यह वर्गीकरण अतार्किक होता था।
(2) लोग जनगणना आयुक्तों को गलत जवाब दे देते थे।
(3) प्राय: लोग स्वयं भी जनगणना के कार्य में सहायता देने से इनकार कर देते थे। ऊँची जाति के लोग अपने परिवार की स्त्रियों के बारे में जानकारी देने से संकोच करते थे बीमारियों से सम्बन्धित आँकड़ों को कठिन था।
(4) मृत्यु दर तथा एकत्रित करना बहुत

प्रश्न 15.
” अठारहवीं शताब्दी में औपनिवेशिक शहर अंग्रेजों की वाणिज्यिक संस्कृति को प्रतिबिम्बित करते थे।” व्याख्या कीजिये।
उत्तर:
अठारहवीं शताब्दी में राजनीतिक सत्ता और संरक्षण भारतीय शासकों के स्थान पर ईस्ट इण्डिया कम्पनी के व्यापारियों के हाथों में आ गई दुभाषिए, बिचौलिए, व्यापारी और माल आपूर्तिकर्ता के रूप में काम करने वाले भारतीयों का भी इन नये शहरों में एक महत्त्वपूर्ण स्थान था। नदी या समुद्र के किनारे आर्थिक गतिविधियों से गोदियों और घाटियों का विकास हुआ। समुद्र किनारे गोदाम, वाणिज्यिक कार्यालय, बीमा एजेंसियों, यातायात डिपो और बैंकिंग संस्थानों की स्थापना हुई।

प्रश्न 16.
1857 के विद्रोह के बाद अंग्रेज़ों ने शहरी इलाकों में ‘सिविल लाइन्स’ नामक इलाके क्यों विकसित किये?
उत्तर:
1857 के विद्रोह के बाद अंग्रेज शासकों ने अनुभव किया कि अंग्रेजों को भारतीयों (देशियों) के खतरे से दूर, अधिक सुरक्षित व पृथक् बस्तियों में रहना चाहिए। अत: उन्होंने पुराने कस्बों के चारों ओर चरागाहों और खेतों को साफ कर ‘सिविल लाइन्स’ नामक नये शहरी इलाके विकसित किये। ‘सिविल लाइन्स’ में केवल गोरे लोगों को बसाया गया। चौड़ी सड़कों, बड़े बगीचों में बने बंगलों, बैरकों, परेड मैदान, चर्च आदि से लैस छावनियाँ यूरोपीय लोगों के लिए सुरक्षित आश्रय स्थल थीं।

प्रश्न 17.
हिल स्टेशनों की अंग्रेजों के लिए क्या उपयोगिता थी?
उत्तर:
(1) हिल स्टेशन अंग्रेज सैनिकों को ठहराने, सीमाओं की चौकसी करने और शत्रु के विरुद्ध आक्रमण करने के लिए महत्त्वपूर्ण स्थान थे।
(2) हिल स्टेशनों की जलवायु अंग्रेजों के लिए स्वास्थ्यप्रद थी।
(3) यहाँ अंग्रेज सैनिक हैजा, मलेरिया आदि बीमारियों से मुक्त रह सकते थे।
(4) ये हिल स्टेशन सेनेटोरियम के रूप में भी विकसित किये गए थे। यहाँ सैनिकों को विश्राम करने एवं इलाज कराने के लिए भेजा जाता था।

प्रश्न 18.
“औपनिवेशिक शहरों में नये सामाजिक समूह बने तथा लोगों की पुरानी पहचानें महत्त्वपूर्ण नहीं रहीं।” व्याख्या कीजिये ।
अथवा
औपनिवेशिक शहरों में ‘मध्य वर्ग’ के विकास का वर्णन कीजिये।
उत्तर:
अठारहवीं शताब्दी में औपनिवेशिक शहरों में समस्त वर्गों के लोग आने लगे शहरों में क्लकों, डॉक्टरों, इन्जीनियरों वकीलों, शिक्षकों तथा लेखाकारों की माँग बढ़ती जा रही थी। परिणामस्वरूप शहरों में ‘मध्य वर्ग’ का विकास हुआ। मध्य वर्ग के लोग सुशिक्षित थे तथा इनकी स्कूल, कॉलेज, लाइब्रेरी तक अच्छी पहुँच थी। वे समाज और सरकार के बारे में समाचार-पत्रों, पत्रिकाओं और सार्वजनिक सभाओं में अपने विचार व्यक्त कर सकते थे।

प्रश्न 19.
इतिहासकारों को जनगणना जैसे स्रोतों का प्रयोग करते समय सावधानी क्यों रखनी चाहिए?
उत्तर:
इतिहासकारों को जनगणना जैसे स्रोतों का प्रयोग करते समय सावधानी इसलिए रखनी चाहिए क्योंकि जनगणना के आँकड़े भ्रामक भी हो सकते हैं। इन आँकड़ों का प्रयोग करने से पहले हमें इस बात को अच्छी तरह समझ लेना चाहिए कि आँकड़े किसने इकट्ठा किए हैं तथा उन्हें क्यों व कैसे इकट्ठा किया गया था।

प्रश्न 20.
अंग्रेजों ने बंगाल में अपने शासन के शुरू से ही नगर नियोजन का कार्यभार अपने हाथों में क्यों लिया?
उत्तर- अंग्रेज व्यापारी नवाब सिराजुद्दौला की सम्प्रभुता से असन्तुष्ट थे। उसने उनसे माल गोदाम के रूप में प्रयोग किये जाने वाला छोटा किला छीन लिया था। प्लासी के युद्ध में विजय के उपरान्त अंग्रेजों ने कलकत्ता में ऐसा किला बनाने का निश्चय किया जिस पर आसानी से आक्रमण न किया जा सके।

प्रश्न 21.
नवाब सिराजुद्दौला ने कलकत्ता नगर पर हमला क्यों किया?
उत्तर:
ईस्ट इण्डिया कम्पनी के व्यापारी नवाब सिराजुद्दौला की सम्प्रभुता पर लगातार सवाल उठा रहे थे। वे न तो कस्टम ड्यूटी चुकाना चाहते थे और न ही उनके द्वारा तय की गई कारोबार की शर्तों पर काम करना चाहते थे। इसलिए नवाब सिराजुद्दौला ने 1756 ई. में कलकत्ता पर हमला करके अंग्रेजों द्वारा बनाए गए किले पर अपना अधिकार कर लिया।

JAC Class 12 History Important Questions Chapter 12 औपनिवेशिक शहर : नगर-योजना, स्थापत्य

प्रश्न 22.
अठारहवीं शताब्दी में शहरों में स्त्रियों में आई जागरूकता का रूढ़िवादी लोग विरोध क्यों करने लगे?
उत्तर:
रूढ़िवादियों को भय था कि यदि स्त्रियाँ पढ़ लिख गई, तो वे संसार में क्रान्ति ला देंगी तथा सम्पूर्ण सामाजिक व्यवस्था का आधार खतरे में पड़ जायेगा। कुछ महिला सुधारक भी स्त्रियों को माँ और पत्नी की परम्परागत भूमिकाओं में ही देखना चाहते थे। उनका कहना था फि स्त्रियों को पर की चारदीवारी के भीतर ही रहना चाहिए। वे परम्परागत पितृसत्तात्मक रीति-रिवाजों, कानूनों को परिवर्तित करने के प्रयासों से असन्तुष्ट थे।

प्रश्न 23.
‘नव-गॉथिक शैली’ से आप क्या समझते हैं ?
उत्तर:
ऊँची उठी हुई छर्ने, नोकदार मेहराबें और बारीक साज-सज्जा ‘नव- गाँधिक शैली’ की विशेषताएँ थीं। बम्बई सचिवालय, बम्बई विश्वविद्यालय, बम्बई उच्च न्यायालय आदि भव्य इमारतें समुद्र किनारे इसी शैली में बनाई गई। यूनिवर्सिटी लाइब्रेरी के घंटाघर का निर्माण प्रेमचन्द रायचन्द के धन से किया गया था। इसका नाम उनकी माँ के नाम पर राजाबाई टावर रखा गया। परन्तु नव-गॉथिक शैली का सर्वोत्कृष्ट उदाहरण विक्टोरिया टर्मिनस रेलवे स्टेशन है।

प्रश्न 24.
नवशास्त्रीय या नियोक्लासिकल स्थापत्य कला पर एक संक्षिप्त टिप्पणी लिखिए।
उत्तर:
बड़े-बड़े स्तम्भों के पीछे रेखागणितीय संरचनाओं का निर्माण नवशास्त्रीय स्थापत्य शैली की विशेषता थी। यह शैली मूल रूप से प्राचीन रोम की भवन निर्माण शैली से निकली थी। 1833 में बम्बई का टाउन हाल इसी शैली के अनुसार बनाया गया था। 1860 के दशक में अनेक व्यावसायिक इमारतों के समूह को ‘एल्फिन्स्टन सर्कल’ कहा जाता था। बाद में इसका नाम बदलकर हार्निमान सर्कल रख दिया गया था। इसमें पहली मंजिल पर ढके हुए तोरणपथ का रचनात्मक ढंग से प्रयोग किया गया।

प्रश्न 25.
भारत में रेलवे की शुरुआत कब हुई और इसके क्या प्रभाव हुए?
अथवा
1853 में रेलवे के आरम्भ की शहरीकरण की प्रक्रिया में क्या भूमिका रही?
अथवा
1853 में रेलवे की स्थापना से किस प्रकार नगरों का भाग्य बदल गया? कोई दो परिवर्तन बताइये।
उत्तर:
(1) भारत में रेलवे की शुरुआत 1853 में हुई। अब आर्थिक गतिविधियों का केन्द्र परम्परागत शहरों से दूर जाने लगा क्योंकि ये शहर पुराने मार्गों और नदियों के निकट थे।
(2) प्रत्येक रेलवे स्टेशन कच्चे माल का संग्रह केन्द्र और आयातित माल का वितरण केन्द्र बन गया था।
(3) रेलवे नेटवर्क के विस्तार के साथ रेलवे वर्कशाप, रेलवे कालोनियों भी बनने लगीं और जमालपुर, बरेली और वाल्टेयर जैसे रेलवे नगर अस्तित्व में आए।

प्रश्न 26.
अंग्रेजों ने ब्लैक टाउनों में सफाई व्यवस्था पर ज्यादा ध्यान कब व क्यों दिया? स्पष्ट कीजिए।
उत्तर:
जब ब्लैक टाउन (काले इलाके में हैजा और प्लेग जैसी महामारियाँ फैली और हजारों लोग मौत का शिकार हुए तब अंग्रेज अफसरों को स्वच्छता व सार्वजनिक स्वास्थ्य के लिए ज्यादा कठोर कदम उठाने पड़े। उनको इस बात का डर था कि कहीं ये बीमारियाँ ब्लैक टाउन से ह्वाइट टाउन में भी न फैल जायें। 1960-70 के दशकों से साफ-सफाई के बारे में कड़े प्रशासकीय उपाय लागू किए गए और भारतीय शहरों में निर्माण गतिविधियों पर अंकुश लगाया गया।

प्रश्न 27.
आमार कथा (मेरी कहानी ) क्या है और किसके द्वारा लिखी गई ?
अथवा
विनोदिनी दास पर एक संक्षिप्त टिप्पणी लिखिए।
उत्तर:
विनोदिनी दास बंगाली रंगमंच की एक प्रसिद्ध अदाकारा थीं। ‘स्टार थियेटर’, कलकत्ता की स्थापना ( 1883 ) के पीछे उनका मुख्य हाथ था 1910 से 1913 के बीच उन्होंने ‘आमार कथा’ के नाम से किस्तों में अपनी आत्मकथा लिखी। वे एक जबरदस्त व्यक्तित्व वाली महिला थीं। उन्होंने समाज में औरतों की समस्याओं पर केन्द्रित कई भूमिकाएँ निभाई। वे अभिनेत्री, संस्था निर्मात्री और लेखिका के रूप में कई भूमिकाएँ एक साथ निभाती

प्रश्न 28.
अंग्रेजों ने जहाँ पर भी किले बनाए, उनके चारों ओर खुले मैदान क्यों छोड़े और इसके पीछे क्या दलील दी ?
उत्तर:
अंग्रेजों ने कलकत्ता में फोर्ट विलियम के इर्द- गिर्द एक विशाल जगह खाली छोड़ दी। खाली मैदान रखने का उद्देश्य यह था कि किले की ओर बढ़ने वाली शत्रु की सेना पर किले से बेरोक-टोक गोलीबारी की जा सके। जब अंग्रेजों को कलकत्ता में अपनी उपस्थिति स्थायी दिखाई देने लगी, तो वे फोर्ट से बाहर मैदान के किनारे पर भी आवासीय इमारतें बनाने लगे।

प्रश्न 29.
“कलकत्ता के लिए जो पैटर्न तैयार किया गया था, उसे बहुत सारे शहरों में दोहराया गया।” व्याख्या कीजिये।
उत्तर:
1857 के विद्रोह के बाद अंग्रेज विद्रोहियों के गढ़ों को अपने लिए सुरक्षित बनाने लगे। उन्होंने दिल्ली में लाल किले पर अपना कब्जा करके वहाँ अपनी सेना तैनात कर दी। उन्होंने किले के पास बनी इमारतों को साफ करके भारतीय मोहल्लों और किले के बीच काफी फासला बना दिया। इसके पीछे उन्होंने यह दलील दी कि अगर . कभी शहर के लोग फिरंगी-राज के खिलाफ खड़े हो जाएँ तो उन पर गोली चलाने के लिए खुली जगह जरूरी थी।

JAC Class 12 History Important Questions Chapter 12 औपनिवेशिक शहर : नगर-योजना, स्थापत्य

प्रश्न 30.
कलकत्ता नगर नियोजन में लाटरी कमेटी के कार्यों की वर्तमान संदर्भ में प्रासंगिकता स्पष्ट कीजिए।
अथवा
‘लाटरी कमेटी’ क्या थी? इसकी कार्यप्रणाली का वर्णन कीजिए।
उत्तर:
1817 में कलकत्ता में एक कमेटी बनाई गई जो सरकार की मदद से नगर नियोजन का कार्य करती थी। यह कमेटी नगर सुधार के लिए धन की व्यवस्था जनता के बीच लाटरी बेचकर करती थी, इसलिए इसका नाम ‘लाटरी कमेटी’ पड़ा। लाटरी कमेटी ने एक नक्शा बनवाया, जिससे कलकत्ता शहर की एक सम्पूर्ण तस्वीर सामने आ सके। कमेटी की प्रमुख गतिविधियों में शहर के हिन्दुस्तानी आबादी वाले हिस्से में सड़क निर्माण और नदी के किनारे से अवैध कब्जे हटाना शामिल था।

प्रश्न 31.
औपनिवेशिक काल में कस्बों का स्वरूप गाँवों से भिन्न था फिर भी इनके बीच की पृथकता अनिश्चित होती थी? स्पष्ट कीजिए।
उत्तर- औपनिवेशिक काल में लोग ग्रामीण इलाकों में खेती, जंगलों में संग्रहण या पशुपालन द्वारा जीवन निर्वाह करते थे। इसके विपरीत कस्बों में शिल्पकार, व्यापारी, प्रशासक एवं शासक रहते थे। कस्बों पर ग्रामीण जनता का प्रभुत्व रहता था तथा वे खेती से प्राप्त करों एवं अधिशेष के आधार पर फलते-फूलते थे प्रायः कस्बों व शहरों की किलेबन्दी की जाती थी जो उन्हें ग्रामीण क्षेत्रों से अलग करती थी फिर भी कस्बों एवं गाँवों के मध्य की पृथकता अनिश्चित होती थी।

किसान तीर्थयात्रा करने के लिए लम्बी दूरियाँ तय करते थे एवं कस्बों से होकर गुजरते थे। दूसरी ओर लोगों और माल का कस्बे से गाँवों की ओर गमन होता रहता था। व्यापारी और फेरीवाले कस्बों से माल गाँव ले जाकर बेचते थे इससे बाजारों का फैलाव और उपभोग की नई शैलियों का उदय होता था। इसके अतिरिक्त जब कस्बों पर आक्रमण होते थे तो लोग प्रायः ग्रामीण क्षेत्रों में शरण लेते थे।

प्रश्न 32.
अठारहवीं शताब्दी के मध्य से शहरों का रूप परिवर्तन क्यों एवं किस प्रकार हुआ?
उत्तर:
अठारहवीं शताब्दी के मध्य से शहरों के रूप परिवर्तन का एक नया चरण प्रारम्भ हुआ। व्यापारिक गतिविधियों के अन्य स्थानों पर केन्द्रित होने के कारण सत्रहवीं शताब्दी में विकसित हुए शहर- सूरत, मछलीपट्टनम व ढाका पतनोन्मुख हो गए। 1757 ई. में बंगाल के नवाब सिराजुद्दौला एवं अंग्रेजों के मध्य हुए प्लासी के बुद्ध में अंग्रेजों की जीत हुई। मद्रास, बम्बई व कलकत्ता ये शहर औपनिवेशिक प्रशासन एवं सत्ता के केन्द्र भी बन गये। नए भवनों एवं संस्थानों का उदय हुआ एवं शहरी केन्द्रों को नए तरीके से पूर्णतः व्यवस्थित किया गया। इन शहरों में नये नये रोजगारों का सृजन हुआ, जिससे लोग इन शहरों में बसने लगे। लगभग 1800 ई. तक ये शहर जनसंख्या की दृष्टि से भारत के विशाल शहर बन गए।

प्रश्न 33.
प्रारम्भिक वर्षों में औपनिवेशिक सरकार ने मानचित्र बनाने पर विशेष ध्यान क्यों दिया?
उत्तर:
प्रारम्भिक वर्षों में औपनिवेशिक सरकार ने निम्नलिखित कारणों से मानचित्र बनाने पर विशेष ध्यान दिया-
(i) सरकार का मानना था कि किसी स्थान की बनावट एवं भूदृश्य को समझने के लिए मानचित्र आवश्यक होते हैं। इस जानकारी के आधार पर वे शहरी प्रदेश पर नियन्त्रण बनाये रख सकते थे।

(ii) जब शहरों का विस्तार होने लगा तो न केवल उनके विकास की योजना तैयार करने के लिए बल्कि शहर को विकसित करने एवं अपनी सत्ता मजबूत बनाने के लिए भी मानचित्र बनाये जाने लगे।

(ii) शहरों के मानचित्रों से हमें उसकी पहाड़ियों, नदियों एवं हरियाली का पता चलता है। यह जानकारी रक्षा सम्बन्धी उद्देश्यों के लिए योजना बनाने में बहुत काम आती

प्रश्न 34.
किन सरकारी नीतियों ने भारतीयों के भीतर उपनिवेशवाद विरोधी और राष्ट्रवादी भावनाओं को बढ़ावा दिया?
उत्तर:
उन्नीसवीं सदी में शहरों में सरकारी दखलन्दाजी और सख्त हो गई। इस आधार पर और अधिक तेजी से झुग्गी-झोंपड़ियों को हटाना शुरू किया गया। दूसरे इलाकों की अपेक्षा ब्रिटिश आबादी वाले हिस्सों को तेजी से विकसित किया जाने लगा। ‘स्वास्थ्यकर’ और ‘अस्वास्थ्यकर’ के नए विभेद के कारण ‘हाइट’ और ‘ब्लैक’ टाउन वाले नस्ली विभाजन को और बल मिला। इन सरकारी नीतियों के विरुद्ध जनता के प्रतिरोध ने भारतीयों के भीतर उपनिवेशवाद विरोधी और राष्ट्रवादी भावनाओं को बढ़ावा दिया।

JAC Class 12 History Important Questions Chapter 12 औपनिवेशिक शहर : नगर-योजना, स्थापत्य

प्रश्न 35.
बम्बई स्थित होटल ताजमहल के बारे में आप क्या जानते हैं?
उत्तर:
बम्बई स्थित प्रसिद्ध होटल ताजमहल का निर्माण प्रसिद्ध उद्योगपति जमशेदजी टाटा ने करवाया था। यह परम्परागत गुजराती शैली में निर्मित है। यह इमारत न केवल भारतीय उद्यमशीलता का प्रतीक है अपितु अंग्रेजों के स्वामित्व एवं नियन्त्रण वाले नस्ली क्लबों और होटलों के लिए चुनौती भी थी।

प्रश्न 36.
फोर्ट सेण्ट जॉर्ज के विषय में आप क्या जानते हैं? क्या यह व्हाइट टाउन का केन्द्र बिन्दु था?
उत्तर:
मद्रास स्थित फोर्ट सेंट जॉर्ज औपनिवेशिक शासन का मुख्य केन्द्र था। यहाँ फोर्ट सेंट व्हाइट टाउन का केन्द्र था। यहाँ अधिकांशतः यूरोपीय रहते थे इसकी दीवारों तथा बुर्जों ने इसे एक खास प्रकार की घेराबन्दी का रूप दे दिया था। किले के भीतर रहने का निर्णय रंग तथा धर्म के आधार पर किया जाता था। भारतीयों के साथ कम्पनी के कर्मचारियों अथवा उनके परिवार के सदस्यों को विवाह की अनुमति नहीं थी। यूरोपीय ईसाई होने के कारण डच तथा पुर्तगालियों को वहाँ रहने की छूट थी। व्हाइट टाउन का केन्द्रबिन्दु इस किले का विकास गोरे विशेषकर अंग्रेजों की जरूरतों एवं सुविधाओं के अनुसार किया गया था।

प्रश्न 37.
लॉर्ड वेलेजली की नगर योजना के विषय में आप क्या जानते हैं?
उत्तर:
लॉर्ड वेलेजली 1798 ई. में बंगाल का गवर्नर जनरल बना। यह अपनी नगर योजना के लिये भी जाना जाता है। वेलेजली ने कलकत्ता में अपने लिये एक गवर्नमेण्ट
हाउस नाम का एक शानदार महल बनवाया था। यह भवन अंग्रेजी सत्ता का प्रतीक था। लॉर्ड वेलेजली कलकत्ता में आ जाने के पश्चात् यहाँ की भीड़-भाड़, अत्यधिक हरियाली, गंदे तालाब तथा साँध से परेशान हो गया। वेलेजली को इन सब तत्त्वों से चिढ़ थी तथा अंग्रेजों का यह भी विचार था कि भारत की उष्णकटिबन्धीय जलवायु बीमारियों तथा महामारियों के अधिक अनुकूल है अतः वेलेजली ने शहर को अधिक स्वास्थ्यपरक बनाने के लिए अधिक खुले स्थान रखने का निर्णय लिया। 1803 ई. में वेलेजली ने नगर नियोजन की आवश्यकता पर एक प्रशासकीय आदेश जारी किया। अतः कहा जा सकता है कि वेलेजली अपनी सहायक सन्धि के कारण जितना कुख्यात है उससे अधिक विख्यात वह स्वास्थ्यपरक नगर नियोजन के लिये है।

प्रश्न 38.
इमारतें और स्थापत्य शैलियाँ क्या बताती हैं?
उत्तर:
स्थापत्य शैलियों से अपने समय के सौन्दर्यात्मक आदर्शों और उनमें निहित विविधताओं का पता चलता है। इमारतें उन लोगों की सोच और नजर के बारे में भी बताती हैं जो उन्हें बना रहे थे इमारतों के जरिये सभी शासक अपनी ताकत को अभिव्यक्त करना चाहते थे। स्थापत्य शैलियों से केवल प्रचलित रुचियों का ही पता नहीं चलता, वे उनको बदलती भी हैं। वे नई शैलियों को लोकप्रियता प्रदान करती हैं और संस्कृति की रूपरेखा तय करती हैं।

प्रश्न 39.
‘बंगला’ भवन निर्माण शैली में किस बात का द्योतक है? इसकी रूपरेखा स्पष्ट कीजिए।
उत्तर:
अंग्रेजों ने अपनी जरूरतों के मुताबिक भवन निर्माण में भारतीय शैलियों को भी अपना लिया था। ‘बंगला’ इसका स्पष्ट उदाहरण है बंगला बम्बई और पूरे देश में सरकारी अफसरों के लिए बनाए जाने वाला भवन था। औपनिवेशिक बंगला एक बड़ी जमीन पर बना होता था। इसमें परम्परागत ढलवाँ छत होती थी और चारों तरफ बरामदा होता था। बंगले के परिसर में घरेलू नौकरों के लिए अलग से क्वार्टर होते थे।

प्रश्न 40.
पूर्व औपनिवेशिक काल के शहरी केन्द्रों की प्रमुख विशेषताएँ क्या थीं?
उत्तर:
(1) ये शहर जनसंख्या के केन्द्रीकरण, अपने विशाल भवनों तथा अपनी शाही शोभा और समृद्धि के लिए प्रसिद्ध थे।
(2) मनसबदार और जागीरदार सामान्यतः इन शहरों में अपने आवास रखते थे।
(3) इन शहरी केन्द्रों में सम्राट और कुलीन वर्ग की ‘उपस्थिति के कारण, यहाँ शिल्पकार, राजकोष, सम्राट का किलेबन्द महल होता था तथा नगर एक दीवार से घिरा होता था।
(4) नगरों के भीतर उद्यान, मस्जिदें, मन्दिर, मकबरे, महाविद्यालय, बाजार तथा सराय स्थित होती थीं।

प्रश्न 41.
ब्रिटिश भारत में निर्मित भवनों में किन- किन स्थापत्य शैलियों का सम्मिश्रण देखने को मिलता है?
उत्तर:
ब्रिटिश भारत में निर्मित भवनों में ‘नव-गॉधिक शैली’, ‘नवशास्त्रीय’ तथा ‘इण्डोसारसेनिक’ स्थापत्य शैलियों का सम्मिश्रण देखने को मिलता है। ऊँची उठी हुई छलें, नोकदार मेहरावें तथा बारीक साज-सज्जा नव-गॉथिक शैली की विशेषताएँ हैं। बड़े- बड़े स्तम्भों के पीछे रेखागणितीय संरचनाओं का निर्माण नवशास्त्रीय शैली की विशेषताएँ हैं, इण्डोसारसेनिक शैली गुम्बदों, छतरियों, मेहराबों से प्रभावित थी।

प्रश्न 42.
लॉटरी कमेटी क्या थी? इसके अन्तर्गत कलकत्ता के नगर नियोजन के लिए कौन-कौनसे कदम उठाए गए?
उत्तर:
गवर्नर जनरल लार्ड वेलेजली के पश्चात् नगर नियोजन का कार्य सरकार की सहायता से लॉटरी कमेटी ने जारी रखा। लॉटरी कमेटी का यह नाम इसलिए पड़ा कि यह कमेटी नगर सुधार के लिए पैसे की व्यवस्था जनता के बीच लॉटरी बेचकर करती थी।
लॉटरी कमेटी द्वारा नगर नियोजन के लिए उठाए गए कदम –
(i) लॉटरी कमेटी ने कलकत्ता शहर का नया मानचित्र बनाया ताकि कलकत्ता को नया रूप दिया जा सके।
(ii) लॉटरी कमेटी की प्रमुख गतिविधियों में शहर में हिन्दुस्तानी जनसंख्या वाले भाग में सड़कें बनवाना एवं नदी किनारे से अवैध कब्जे हटाना सम्मिलित था।
(iii) कलकत्ता शहर के भारतीय हिस्से को साफ- सुथरा बनाने के लिए कमेटी ने बहुत सी झोंपड़ियों को साफ कर दिया एवं गरीब मजदूरों को वहाँ से बाहर निकाल दिया। उन्हें कलकत्ता के बाहरी किनारे पर निवास हेतु जगह दी गई।

प्रश्न 43.
कौन-कौनसी सरकारी नीतियों ने कलकत्ता में भारतीयों के भीतर उपनिवेशवाद विरोधी एवं राष्ट्रवादी भावनाओं को बढ़ावा दिया?
उत्तर:
19वीं शताब्दी के प्रारम्भ के साथ ही कलकत्ता शहर में सरकारी हस्तक्षेप बहुत अधिक सख्त हो चुका था। वित्त पोषण (फण्डिंग) सहित नगर नियोजन के समस्त आयामों को अंग्रेज सरकार ने अपने हाथों में ले लिया। इस आधार पर और तीव्र गति से शुग्गी-झोंपड़ियों को हटाना प्रारम्भ किया गया। दूसरे क्षेत्रों की अपेक्षा ब्रिटिश आबादी वाले हिस्सों का तेजी से विकास किया जाने लगा। स्वास्थ्यकर एवं अस्वास्थ्यकर के नये विभेद के कारण व्हाइट और ब्लैक टाउन वाले नस्ली विभाजन को और बल मिला। कलकत्ता नगर निगम में मौजूद भारतीय प्रतिनिधियों ने शहर के यूरोपीय आबादी वाले क्षेत्रों के विकास पर आवश्यकता से अधिक ध्यान दिये जाने की आलोचना की। इन सरकारी नीतियों के विरुद्ध जनता के प्रतिरोध ने कलकत्ता में भारतीयों के भीतर उपनिवेशवाद विरोधी एवं राष्ट्रवादी भावनाओं को बढ़ावा दिया।

JAC Class 12 History Important Questions Chapter 12 औपनिवेशिक शहर : नगर-योजना, स्थापत्य

प्रश्न 44.
बम्बई का वाणिज्यिक शहर के रूप में किस प्रकार विकास हुआ? टिप्पणी लिखिए।
उत्तर:
19वीं शताब्दी के अन्त तक भारत का आधार आयात तथा निर्यात वाणिज्य शहर बम्बई से होता था। इस समय व्यापार की मुख्य वस्तु अफीम तथा नील थी। यहाँ से ईस्ट इण्डिया कम्पनी चीन को अफीम का निर्यात किया करती थी। इस व्यापार से शुद्ध भारतीय पूँजीपति वर्ग का निर्माण हुआ। पारसी, मारवाड़ी, कोंकणी, मुसलमान, गुजराती, ईरानी, आर्मेनियाई, यहूदी, बोहरे तथा बनिये इत्यादि यहाँ के मुख्य व्यापारी वर्ग से सम्बन्धित थे 1869 ई. में स्वेज नहर को व्यापार के लिये खोला गया था इससे बम्बई के व्यापारिक सम्बन्ध शेष विश्व के साथ अत्यधिक मजबूत हुए। 19वीं शताब्दी के उत्तरार्द्ध तक बम्बई में भारतीय व्यापारी कॉटन मिल जैसे नवीन उद्योगों में अत्यधिक धन का निवेश कर रहे थे।

प्रश्न 45
औपनिवेशिक शहरों में स्त्रियों के सामाजिक जीवन में आए परिवर्तनों का उल्लेख कीजिए।
उत्तर:
औपनिवेशिक शहरों में महिलाओं के लिए नए अक्सर थे। वे पत्र-पत्रिकाओं, आत्मकथाओं तथा पुस्तकों के माध्यम से स्वयं को अभिव्यक्त कर रही थीं। सार्वजनिक स्थानों पर महिलाओं की उपस्थिति बढ़ रही थी। वे नौकरानी, फैक्ट्री मजदूर, शिक्षिका, रंगकर्मी और फिल्म कलाकार के रूप में शहरों के नये व्यवसायों में प्रविष्ट होने लगीं। परन्तु घर से निकलकर सार्वजनिक स्थानों में जाने वाली महिलाओं का सम्मान नहीं था।

प्रश्न 46.
मद्रास का कौनसा क्षेत्र व्हाइट टाउन कां केन्द्रक बन गया था?
उत्तर:
मद्रास में स्थित फोर्ट सेन्ट जार्ज व्हाइट टाउन .का केन्द्रक बन गया था। यहाँ अधिकतर यूरोपीय लोग रहते ‘थे। किले के अन्दर रहने का निर्णय रंग और धर्म के आधार पर किया जाता था। कम्पनी के लोगों को भारतीयों के साथ विवाह करने की अनुमति नहीं थी। संख्या की दृष्टि से कम होते हुए भी यूरोपीय लोग शासक थे और मद्रास शहर का विकास शहर में रहने वाले गोरे लोगों की आवश्यकता को ध्यान में रखकर किया जा रहा था।

प्रश्न 47.
मद्रास में स्थित ब्लैक टाउन का वर्णन कीजिये।
उत्तर:
मद्रास में ब्लैक टाउन किले के बाहर स्थित था। यहाँ आबादी को सीधी पंक्तियों में बसाया गया था। कुछ समय बाद उत्तर की दिशा में दूर जाकर एक नवा ब्लैक टाउन बसाया गया। इसमें भारतीय बुनकरों, कारीगरों, बिचौलियों, दुभाषियों को रखा गया। इसमें भारतीय लोग रहते थे। यहाँ मंदिर और बाजार के आस-पास आवासीय मकान बनाए गए थे। शहर के बीच से गुजरने वाली आड़ी-टेड़ी संकरी गलियों में अलग-अलग जातियों के मोहल्ले थे।

प्रश्न 48.
कोलकाता में नगर नियोजन को क्यों प्रोत्साहन दिया गया?
उत्तर:
कोलकाता में 1817 में हैजा फैल गया तथा 1896 में प्लेग ने शहर को अपनी चपेट में ले लिया। इस स्थिति में अंग्रेजों ने लोलकाता में नगर नियोजन पर बल दिया। घनी आबादी के इलाकों को अस्वच्छ माना जाता था। इसलिए कामकाजी लोगों की झोंपड़ियों तथा वस्तियों को वहाँ से हटा दिया गया। मजदूरों, फेरीवालों कारागरों और बेरोजगारों को दूर वाले इलाकों में ढकेल दिया गया। आग लगने की आशंका को ध्यान में रखते हुए फँस की झोंपड़ियों को अवैध घोषित कर दिया गया।

प्रश्न 49.
मुम्बई में यूरोपीय शैली की इमारतों का निर्माण क्यों किया गया?
उत्तर:
(1) यूरोपीय शैली की इमारतों में भारत जैसे अपरिचित देश में जाना पहिचाना सा भू-दृश्य रचने की और उपनिवेश में भी पर जैसा महसूस करने की अंग्रेजों की आकांक्षा प्रकट होती थी।
(2) अंग्रेजों की मान्यता थी कि यूरोपीय शैली उनकी श्रेष्ठता, अधिकार और सत्ता का प्रतीक थी
(3) वे सोचते थे कि यूरोपीय शैली में निर्मित इमारतों से अंग्रेज शासकों और भारतीय लोगों के बीच अन्तर साफ दिखाई देगा।

निबन्धात्मक प्रश्न-

प्रश्न 1.
औपनिवेशिक भारत के प्रमुख बन्दरगाहों का वर्णन कीजिये।
उत्तर:
औपनिवेशिक भारत के प्रमुख बन्दरगाह
(1) बन्दरगाहों की किलेबन्दी-अठारहवीं शताब्दी तक भारत में मद्रास, कलकत्ता तथा बम्बई महत्त्वपूर्ण बन्दरगाह बन चुके थे। यहाँ जो बस्तियों बसों, वे चीजों के संग्रह के लिए बड़ी उपयोगी सिद्ध हुई ईस्ट इण्डिया कम्पनी ने इन बस्तियों में अपने कारखाने अर्थात् वाणिज्यिक कार्यालय स्थापित किये। कम्पनी ने सुरक्षा के उद्देश्य से इन बस्तियों की किलाबन्दी की। मद्रास में फोर्ट सेन्ट जार्ज, कलकत्ता में फोर्ट विलियम और बम्बई में फोर्ट नामक किले बनाये गए।

(2) यूरोपीयों और भारतीयों के लिए अलग-अलग बस्तियों की व्यवस्था-यूरोपीय व्यापारी किलों के अन्दर रहते थे, जबकि भारतीय व्यापारी, कारीगर, कामगार आदि इन किलों के बाहर अलग बस्तियों में रहते थे। जिस बस्ती में यूरोपीय लोग रहते थे वह ‘व्हाइट टाउन’ (गोरा शहर ) तथा जिस बस्ती में भारतीय लोग रहते थे, वह ‘ब्लैक टाउन’ (काला शहर) के नाम से पुकारे जाते थे।

(3 ) देहाती एवं दूरस्थ इलाकों का बन्दरगाहों से जुड़ना-उन्नीसवीं शताब्दी के मध्य में रेलवे के विकास ने इन शहरों को शेष भारत से जोड़ दिया। परिणामस्वरूप ऐसे देहाती तथा दूरस्थ इलाके भी इन बन्दरगाहों से जुड़ गए जहाँ से कच्चा माल तथा मजदूर आते थे।

(4) कारखानों की स्थापना- कलकत्ता, बम्बई तथा मद्रास में कारखानों की स्थापना करना भी आसान था। इसका कारण यह था कि कच्चा माल निर्यात के लिए इन शहरों में आता था तथा यहाँ सस्ता श्रम उपलब्ध था। 1850 के दशक के बाद भारतीय व्यापारियों और उद्यमियों ने बम्बई में सूती कपड़े की मिलें स्थापित कीं। कलकत्ता के बाहरी इलाके में यूरोपियन लोगों ने जूट मिलों की स्थापना की। परन्तु इन शहरों की अर्थव्यवस्था मुख्य रूप से फैक्टरी उत्पादन पर आधारित नहीं थी।

JAC Class 12 History Important Questions Chapter 12 औपनिवेशिक शहर : नगर-योजना, स्थापत्य

(5) भारत का औद्योगिक देश न बन पाना 19वीं शताब्दी में भारत में केवल कानपुर तथा जमशेदपुर ही औद्योगिक शहर थे। कानपुर में चमड़े की चीजें तथा ऊनी और सूती कपड़े बनते थे, जबकि जमशेदपुर स्टील उत्पादन के लिए प्रसिद्ध था।
भारत कभी भी एक आधुनिक औद्योगिक देश नहीं बन पाया क्योंकि ब्रिटिश सरकार की पक्षपातपूर्ण नीति ने भारत के औद्योगिक विकास को कभी प्रोत्साहन नहीं दिया। यद्यपि कलकत्ता, बम्बई तथा मद्रास बड़े शहरों के रूप में तो विख्यात हुए, परन्तु इससे औपनिवेशिक भारत की सम्पूर्ण अर्थव्यवस्था में कोई क्रान्तिकारी वृद्धि नहीं हुई।

प्रश्न 2.
सन् 1800 के पश्चात् हमारे देश में शहरीकरण की गति धीमी रही। इसके लिए उत्तरदायी कारणों की व्याख्या कीजिए।
उत्तर:
शहरीकरण- ग्रामीण एवं कृषि करने वाले लोगों का गाँवों से शहरों की ओर अच्छे रोजगार अथवा काम की तलाश में पलायन करना या सामान्य गमनागमन शहरीकरण कहलाता है। यह कस्बों एवं शहरों में कुल जनसंख्या के बढ़ते हुए आनुपातिक सन्तुलन को भी इंगित करता है। कुल जनसंख्या में शहरी जनसंख्या के अनुपात से शहरीकरण के विकास की गति का मापन होता है 1800 ई. के पश्चात् हमारे देश में शहरीकरण की गति धीमी रही। सम्पूर्ण उन्नीसवीं और बीसवीं शताब्दी के पहले दो दशकों तक देश की कुल जनसंख्या में शहरी जनसंख्या का अनुपात लगभग स्थिर रहा। इसमें केवल 3 प्रतिशत को ही बढ़ोत्तरी हुई। सन् 1900 से 1940 के मध्य शहरी जनसंख्या 10 प्रतिशत बढ़कर 13 प्रतिशत हो गई।

शहरीकरण की गति के स्थिर रहने के पीछे निम्नलिखित कारण उत्तरदायी थे-
1. छोटे कस्बों के पास आर्थिक रूप से विकसित होने के पर्याप्त अवसर नहीं थे परन्तु दूसरी तरफ कलकत्ता, बम्बई और मद्रास का विस्तार तेजी से हुआ।
2. कलकता, अम्बई व मद्रास औपनिवेशिक अर्थव्यवस्था के केन्द्र होने के कारण भारतीय सूती कपड़े जैसे निर्यात होने वाले उत्पादों के संग्रहण केन्द्र थे। लेकिन इंग्लैण्ड में हुई औद्योगिक क्रान्ति के बाद इस प्रवाह की दिशा परिवर्तित हो गई। भारत से अब तैयार माल की अपेक्षा कच्चे माल का निर्यात होने लगा।

3. 1853 ई. में औपनिवेशिक सरकार ने भारत में रेलवे की शुरुआत की, इसने भारतीय शहरों को पूर्ण रूप से परिवर्तित कर दिया। प्रत्येक रेलवे स्टेशन कच्चे माल के संग्रह और आयातित वस्तुओं के वितरण का केन्द्र बन गया। उदाहरण के लिए गंगा के किनारे स्थित मिर्जापुर दक्कन से आने वाली कंपास एवं सूती वस्त्रों के संग्रह का केन्द्र था जो बम्बई तक जाने वाली रेलवे लाइन के निर्माण के पश्चात् अपनी पुरानी पहचान को खोने लगा था। भारत में रेलवे नेटवर्क के विस्तार के पश्चात् रेलवे वर्कशॉप्स और रेलवे कॉलोनियों की स्थापना होना भी प्रारम्भ हो गया। फलस्वरूप जमालपुर, बरेली व वाल्टेयर जैसे रेलवे नगरों का जन्म हुआ।

प्रश्न 3.
अंग्रेजों ने हिल स्टेशनों की स्थापना क्यों की थी?
अथवा
हिल स्टेशनों की अंग्रेजों के लिए क्या उपयोगिता थी?
उत्तर:
हिल स्टेशनों की स्थापना का प्रमुख उद्देश्य हिल स्टेशनों की स्थापना और बसावट का सम्बन्ध सबसे पहले ब्रिटिश सेना की जरूरतों से था। सिमला (शिमला) की स्थापना गुरखा युद्ध (1815-16) के दौरान, माउंट आबू की स्थापना अंग्रेज-मराठा युद्ध (1818) के कारण की गई तथा दार्जिलिंग को 1835 में सिक्किम के राजाओं से छीना गया। ये हिल स्टेशन फौजियों को ठहराने, सरहद की चौकसी करने और दुश्मन के खिलाफ हमला बोलने की दृष्टि से महत्त्वपूर्ण स्थान थे।

हिल स्टेशनों की उपयोगिता-यूरोपियनों के लिए हिल स्टेशन अग्रलिखित कारणों से उपयोगी थे-
(1) स्वास्थ्यवर्द्धक तथा ठण्डी जलवायु-भारतीय पहाड़ों की मृदु और ठण्डी जलवायु को फायदे की चीज माना जाता था, खासतौर से अंग्रेज गर्मियों के मौसम को बीमारी पैदा करने वाला मानते थे। उन्हें गर्मियों के कारण हैजा व मलेरिया की सबसे ज्यादा आशंका रहती थी।

(2) सेना की सुरक्षा – सेना की भारी भरकम मौजूदगी के कारण ये स्थान पहाड़ियों में एक छावनी के रूप में बदल गये। हिल स्टेशनों को सेनीटोरियम के रूप में भी विकसित किया गया, जहाँ सिपाही विश्राम करने व इलाज कराने के लिए जाते थे।

(3) यूरोप की जलवायु से मिलती-जुलती जलवायु हिल स्टेशनों की जलवायु यूरोप की ठण्डी जलवायु से मिलती-जुलती थी, इसलिए नए शासकों को वहाँ की जलवायु बहुत पसन्द थी 1864 में वायसराय जॉन लारेंस ने अधिकृत रूप से अपनी काउंसिल शिमला में स्थापित कर दी और इस प्रकार गर्मी में राजधानियाँ बदलने के सिलसिले पर रोक लगा दी। शिमला भारतीय सेना के कमाण्डर इन चीफ का भी अधिकृत आवास बन गया।

(4) अंग्रेजों व यूरोपियन के लिए आदर्श स्थान -हिल स्टेशन ऐसे अंग्रेजों व यूरोपियन के लिए भी आदर्श स्थान थे। अलग-अलग मकानों के बाद एक-दूसरे से सटे विला और बागों के बीच कॉटेज बनाए जाते थे। एंग्लिकन चर्च और शैक्षणिक संस्थान आंग्ल आदर्शों का प्रतिनिधित्व करते थे। सामाजिक दावत, चाय बैठक, पिकनिक, रात्रिभोज मेले, रेस और रंगमंच जैसी घटनाओं के रूप में यूरोपियों का सामाजिक जीवन भी एक खास किस्म का था।

(5) पर्वतीय सैरगाहें रेलवे के आने से पर्वतीय सैरगाहें बहुत तरह के लोगों की पहुँच में आ गई। उच्च व मध्यम वर्गीय लोग, महाराजा, वकील और व्यापारी सैर- सपाटे के लिए वहाँ जाने लये।

(6) औपनिवेशिक अर्थव्यवस्था के लिए महत्त्वपूर्ण हिल स्टेशन औपनिवेशिक अर्थव्यवस्था के लिए भी महत्त्वपूर्ण केन्द्र थे। पास के इलाकों में चाय और काफी के बागानों की स्थापना से मैदानी इलाकों से बड़ी संख्या में मजदूर वहाँ रोजगार हेतु आने लगे ।

प्रश्न 4.
कलकत्ता (वर्तमान में कोलकाता) शहर के नगर नियोजन पर एक लघु निबन्ध लिखिए।
अथवा
कलकत्ता नगर के विकास में अंग्रेजों की भूमिका का वर्णन कीजिये।
अथवा
औपनिवेशिक काल में कलकत्ता में नगर नियोजन के इतिहास की रूपरेखा प्रस्तुत कीजिये ।
उत्तर:
कलकत्ता में नगर नियोजन का विकास क्रम कलकत्ता में नगर नियोजन के विकास क्रम को अग्रलिखित बिन्दुओं के अन्तर्गत स्पष्ट किया गया है-
(1) फोर्ट विलियम और मैदान का निर्माण तथा मैदान के किनारे आवासीय इमारतें बनाना- कलकत्ता को- सुतानाती, कोलकाता और गोविन्दपुर- इन तीन गाँवों को मिलाकर बनाया गया था। कम्पनी ने गोविन्दपुर गाँव की जमीन को साफ करने के लिए वहाँ के व्यापारियों और बुनकरों को हटवा दिया। फोर्ट विलियम के इर्द-गिर्द एक विशाल खाली जगह छोड़ दी गई, जिसे स्थानीय लोग मैदान या ‘गारेर मठ’ कहने लगे।

(2) गवर्नमेंट हाउस का निर्माण- 1798 में गवर्नर जनरल लार्ड वेलेजली ने कलकत्ता में अपने लिए ‘गवर्नमेंट हाउस’ के नाम से एक महल बनवाया। यह इमारत अंग्रेजी सत्ता का प्रतीक थी।

JAC Class 12 History Important Questions Chapter 12 औपनिवेशिक शहर : नगर-योजना, स्थापत्य

(3) जन स्वास्थ्य की दृष्टि से नगर नियोजन की आवश्यकता पर बल वेलेजली ने हिन्दुस्तानी आबादी वाले भीड़भाड़ भरे गन्दे तालाबों और निकासी की खस्ता हालत को देखते हुए यह माना कि ऐसी अस्वास्थ्यकर बस्तियों से बीमारियाँ फैलती हैं। फलतः वेलेजली ने 1803 में नगर नियोजन की आवश्यकता पर एक प्रशासकीय आदेश जारी किया। बहुत सारे बाजारों, घाटों, कब्रिस्तानों और चर्मशोधन इकाइयों को हटा दिया गया। इस प्रकार ‘जन-स्वास्थ्य’ शहरों की सफाई और नगर नियोजन परियोजनाओं का मुख्य विचार बन गया।

(4) लॉटरी कमेटी द्वारा नगर नियोजन कार्य को गति प्रदान करना – वेलेजली के जाने के बाद नगर नियोजन का काम सरकार की मदद से लॉटरी कमेटी ने जारी रखा। लॉटरी कमेटी ने शहर का एक नक्शा बनवाया, जिससे कलकत्ता की पूरी तस्वीर सामने आ सके। इसके अतिरिक्त कमेटी ने शहर के हिन्दुस्तानी हिस्से में सड़क निर्माण कराया, नदी किनारे से ‘अवैध कब्जे हटाये तथा बहुत सारी झोंपड़ियों को साफ कर दिया और गरीब लोगों को वहाँ से हटाकर कलकत्ता के बाहरी किनारे पर जगह दी गई।

(5) महामारी की आशंका से नगर नियोजन कार्य में तीव्र गति – अगले कुछ दशकों में प्लेग, हैजा आदि महामारियों की आशंका से नगर नियोजन की अवधारणा को बल मिला। कलकत्ता को और अधिक स्वास्थ्यकर बनाने के लिए कामकाजी लोगों की झोंपड़ियों या बस्तियों को तेजी से हटाया गया और यहाँ के गरीब मजदूर वाशिंदों को पुनः दूर वाले इलाकों में ढकेल दिया गया। फूँस की झोंपड़ियों को अवैध घोषित कर दिया गया।

(6) नगर नियोजन के सारे आयामों को सरकार द्वारा अपने हाथ में लेना 19वीं सदी में और ज्यादा तेजी से झुग्गियों को हटाया गया तथा दूसरे इलाकों की कीमत पर ब्रिटिश आबादी वाले हिस्सों को तेजी से विकसित किया गया।

(7) ह्वाइट और ब्लैक टाउन वाले नस्ली विभाजन को बढ़ावा-‘ स्वास्थ्यकर’ और ‘अस्वास्थ्यकर’ के नये विभेद के सहारे ‘डाइट’ और ‘ब्लैक’ टाउन वाले नस्ली विभाजन को और बल मिला तथा शहर के यूरोपीय आबादी वाले इलाकों के विकास पर आवश्यकता से अधिक ध्यान दिया गया।

प्रश्न 5.
तर्क सहित सिद्ध कीजिए कि औपनिवेशिक शहरों का सामाजिक जीवन वर्तमान शहरों में भी दिखाई पड़ता है।
उत्तर:
औपनिवेशिक शहरों का सामाजिक जीवन वर्तमान शहरों में भी दिखाई देता है। इस बात के समर्थन में निम्नलिखित तर्क प्रस्तुत हैं-
(i) वर्गभेद – औपनिवेशिक शहरों में वर्गभेद स्पष्ट दिखाई देता था। एक ओर व्हाइट टाउन थे जहाँ गोरे लोग ही रह सकते थे दूसरी ओर ब्लैक टाउन में मात्र भारतीय ही रहते थे। व्हाइट टाउन में समस्त सुविधाएँ थीं, वहीं ब्लैक टाउन आवश्यक सुविधाओं एवं स्वच्छता से विहीन थे। यही स्थिति वर्तमान शहरों में देखने को मिल रही है। धनवान लोगों की कालोनियों में पर्याप्त नागरिक सुविधाएँ व स्वच्छता देखने को मिलती है।

(ii) यातायात के साधनों का विकास औपनिवेशिक शहरों में यातायात के साधनों का पर्याप्त विकास हुआ जिस कारण लोग शहर के केन्द्र से दूर जाकर भी बस सकते थे। वर्तमान शहरों में भी यही स्थिति देखने को मिलती है। लोग यातायात के साधनों के विकास के कारण शहर के केन्द्र से दूर जाकर बस रहे हैं।

(ii) मनोरंजन तथा मिलने-जुलने के नये सार्वजनिक स्थल औपनिवेशिक शहरों की तरह वर्तमान शहरों में टाउन हॉल सार्वजनिक पार्क, रंगशाला एवं सिनेमा हॉल जैसे सार्वजनिक स्थानों का निर्माण हुआ है, जिससे शहरों में लोगों को मिलने-जुलने तथा मनोरंजन के नये अवसर मिलने लगे हैं। शहरों में नये सामाजिक समूह बने हैं।

(iv) मध्यम वर्ग का विस्तार औपनिवेशिक शहरों की तरह वर्तमान में सभी वर्गों के लोग शहरों में आने लगे हैं। क्लकों, शिक्षकों, वकीलों, डॉक्टरों, इंजीनियरों, एकाउण्टेन्ट्स की माँग बढ़ने लगी है परिणामस्वरूप मध्यम वर्ग बढ़ता जा रहा है। उनके पास स्कूल, कॉलेज तथा लाइब्रेरी जैसे नये शिक्षा केन्द्रों तक अच्छी पहुँच है।

(v) महिलाओं की स्थिति में परिवर्तन औपनिवेशिक शहरों में महिलाओं के लिए नये अवसर थे। वर्तमान शहरों में भी महिलाओं के लिए नये अवसर मौजूद हैं। आज महिलाएँ घर की चारदीवारी से निकलकर समाचार पत्र, पत्र-पत्रिकाओं और पुस्तकों के द्वारा स्वयं को अभिव्यक्त कर रही हैं।

(vi) मेहनतकश, गरीबों अथवा कामगारों में वृद्धि- औपनिवेशिक शहरों की तरह वर्तमान शहरों में भी ग्रामीण क्षेत्रों से लोग रोजगार की तलाश में लगातार शहरों की ओर आ रहे हैं।

प्रश्न 6.
मद्रास शहर के नगर नियोजन की विवेचना कीजिए।
अथवा
मद्रास में बसावट और पृथक्करण पर एक लघु निबन्ध लिखिए।
उत्तर:
स्थापना- अंग्रेज व्यापारियों ने 1639 में एक व्यापारिक चौकी मद्रासपट्म में स्थापित की फ्रेंच इंस्ट इण्डिया कम्पनी के साथ प्रतिद्वंद्विता के कारण (1746- 63) अंग्रेजों को मद्रास की किलेबन्दी करनी पड़ी।
(1 ) फोर्ट सेंट जार्ज एवं हाइट टाउन का निर्माण- अंग्रेजों ने मद्रास में अपना किला बनाया जो ‘फोर्ट सेन्ट जार्ज’ कहलाता था। इसमें ज्यादातर यूरोपीय रहते थे। मद्रास शहर का विकास शहर में रहने वाले थोड़े से गोरों की जरूरतों और सुविधाओं के हिसाब से किया जा रहा।

(2) ब्लैक टाउन का विकास ब्लैक टाउन को किले के बाहर बसाया गया। इस आबादी को भी सीधी कतारों में बसाया गया जो कि औपनिवेशिक नगरों की खास विशेषता थी नये ब्लैक टाउन में बुनकरों, कारीगरों, बिचौलियों और दुभाषियों को बसाया गया, जो कम्पनी के व्यापार में महत्त्वपूर्ण भूमिका निभाते थे। वहाँ मन्दिर और बाजार के आसपास रिहायशी मकान बनाए गए। आड़ी-टेढ़ी गलियों में अलग- अलग जातियों के मोहल्ले थे। चिन्ताद्रीपेठ इलाका केवल बुनकरों के लिए था। वाशरमेन पेठ में रंगसाज और धोबी रहते थे। दुबाश एजेन्ट और व्यापारी के रूप में कार्य करते थे, इसमें सम्पन्न लोग थे। ये लोग भारतीयों एवं यूरोपियनों के बीच मध्यस्थ की भूमिका निभाते थे।

(3) स्थानीय वाशिंदे व नौकरी-शुरू में कम्पनी में कार्य करने वालों में लगभग सारे वेल्लालार होते थे। वह एक स्थानीय ग्रामीण जाति थी। ब्राह्मण भी इसी तरह के पदों के लिए जोर लगाने लगे। तेलुगू कोमाटी समुदाय एक शक्तिशाली व्यावसायिक समूह था पेरियार और बनियार गरीब कामगार वर्ग था माइलापुर और ट्रिप्लीकेन हिन्दू धार्मिक केन्द्र थे ट्रिप्लीकेन में बड़ी संख्या में मुस्लिम आबादी रहती थी सानधोम और वहाँ का चर्च रोमन कैथोलिक समुदाय का केन्द्र था।

(4) यूरोपीय लोगों द्वारा गार्डन हाउसेज का निर्माण- जैसे-जैसे अंग्रेजी सत्ता मजबूत होती गई, यूरोपीय निवासी किले के बाहर जाकर गार्डन हाउसेज का निर्माण करने लगे।

(5) सम्पन्न भारतीयों द्वारा उपशेहरी इलाकों का निर्माण धीरे-धीरे सम्पन्न भारतीय भी अंग्रेजों की तरह रहने लगे थे। परिणामस्वरूप मद्रास के इर्द-गिर्द स्थित गाँवों की जगह बहुत सारे नये उपशहरी इलाकों ने ले ली।

(6) गरीब लोगों का काम के नजदीक वाले गाँवों में निवास- गरीब लोग अपने काम की जगह से नजदीक पड़ने वाले गाँवों में बस गये। बढ़ते शहरीकरण के कारण इन गाँवों के बीच वाले इलाके शहर के भीतर आ गए। इस तरह मद्रास एक अर्द्धग्रामीण-सा शहर दिखने लगा।

प्रश्न 7.
बम्बई एक आधुनिक नगर और भारत की वाणिज्यिक राजधानी कैसे बना? इस पर प्रकाश डालिए।
अथवा
बम्बई शहर के नगर नियोजन की विशेषताओं की विवेचना कीजिए।
अथवा
मुम्बई का उदाहरण देते हुए स्पष्ट कीजिये कि अंग्रेजों ने नगर नियोजन के माध्यम से अपने औपनिवेशिक सपने को कैसे पूरा किया?
अथवा
बम्बई नगर नियोजन तथा भवन निर्माण के मुख्य चरणों की व्याख्या कीजिये मुख्य रूप से नवशास्त्रीय ( नियोक्लासिकल) शैली में बनी इमारतों का उल्लेख कीजिये।
उत्तर:
प्रारम्भ में बम्बई सात टापुओं का इलाका था। जैसे-जैसे आबादी बढ़ी, इन टापुओं को एक-दूसरे से जोड़ दिया गया और इन टापुओं के जुड़ने पर एक विशाल शहर अस्तित्व में आया।

(1) औपनिवेशिक भारत की वाणिज्यिक राजधानी अम्बई औपनिवेशिक भारत की वाणिज्यिक राजधानी थी। पश्चिमी तट पर एक प्रमुख बन्दरगाह होने के नाते यह अन्तर्राष्ट्रीय व्यापार का केन्द्र था। 19वीं सदी के अन्त तक भारत का आधा आयात-निर्यात बम्बई से ही होता था। इस व्यापार की एक महत्त्वपूर्ण वस्तु अफीम थी। अफीम के व्यापार में ईस्ट इण्डिया कम्पनी के साथ भारतीय व्यापारी और बिचौलियों की भी प्रमुख भूमिका थी। इससे भारतीय पूँजीपति वर्ग का विकास हुआ। बाद में अमेरिकी गृहयुद्ध के कारण भारतीय कपास की बढ़ती माँग और बढ़ती कीमतों तथा स्वेज नहर के खुलने से बम्बई का वाणिज्यिक विकास हुआ और बम्बई को भारत का ‘सरताज शहर’ घोषित किया गया।

(2) विशाल इमारतें तथा उनका स्थापत्य अंग्रेजों ने अनेक भव्य और विशाल इमारतों का निर्माण कराया। इनकी स्थापत्य शैली यूरोपीय शैली पर आधारित थी। धीरे-धीरे भारतीयों ने भी यूरोपीय स्थापत्य शैली को अपना लिया। अंग्रेजों ने अनेक बंगले बनवाये।
(i) अंग्रेजों ने नव-गॉथिक शैली में इमारतों का निर्माण कराया। विक्टोरिया टर्मिनस रेलवे स्टेशन इस शैलीका बेहतरीन उदाहरण है। इसके अतिरिक्त बम्बई विश्वविद्यालय, सचिवालय और उच्च न्यायालय जैसी भव्य इमारतों का निर्माण कराया गया।
(ii) नवशास्त्रीय शैली में ‘एल्फिंस्टन सर्कल’ का निर्माण किया गया।
(iii) बीसवीं सदी के प्रारम्भ में भारतीय और यूरोपीय 4 शैली को मिलाकर एक नयी शैली का विकास हुआ जो ‘इण्डोसारसेनिक’ कहलाती थी यह शैली गुम्बदों, छतरियों, जालियों और मेहराबों से प्रभावित थी गुजराती शैली में बने गेट वे ऑफ इण्डिया तथा होटल ताज प्रमुख इमारतें हैं।

JAC Class 12 History Important Questions Chapter 12 औपनिवेशिक शहर : नगर-योजना, स्थापत्य

(3) चाल – बम्बई शहर में जगह की कमी तथा भीड़-भाड़ की वजह से विशेष प्रकार की इमारतें भी बनाई गई, जिन्हें ‘चाल’ कहा जाता था। ये बहुमंजिला इमारतें होती थीं, जिनमें एक-एक कमरेवाली आवासीय इकाइयाँ बनाई जाती थीं।

प्रश्न 8.
औपनिवेशिक काल में भारत में सार्वजनिक भवनों के निर्माण के लिए कौन-कौनसी स्थापत्य शैलियों का प्रयोग किया गया?
अथवा
ब्रिटिश काल में इण्डो-सारसेनिक स्थापत्य कला की दो विशेषताएँ बताइये।
उत्तर:
सार्वजनिक भवनों के लिए स्थापत्य शैलियाँ औपनिवेशिक काल में सार्वजनिक भवनों के लिए मोटे तौर पर तीन स्थापत्य शैलियों का प्रयोग किया गया।
(1) नवशास्त्रीय या नियोक्लासिकल शैली-बड़े- बड़े स्तम्भों के पीछे रेखागणितीय संरचनाओं का निर्माण इस शैली की विशेषता थी। भारत में ब्रिटिश साम्राज्य के लिए उसे खासतौर से अनुकूल माना जाता था। अंग्रेजों को लगता था कि जिस शैली में शाही रोम की भव्यता दिखाई देती थी, उसे शाही भारत के वैभव की अभिव्यक्ति के लिए भी प्रयोग किया जा सकता है क्योंकि यह शैली मूल रूप से प्राचीन रोम की भवन निर्माण शैली से निकली थी और इसे यूरोपीय पुनर्जागरण के दौरान पुनर्जीवित, संशोधित और लोकप्रिय किया गया था।

इस स्थापत्य शैली के भूमध्यसागरीय उद्गम के कारण उसे उष्णकटिबंधीय मौसम के अनुकूल भी माना गया। 1833 में बम्बई का ‘टाउन हॉल’ इसी शैली के अनुसार बनाया गया था। 1860 में व्यावसायिक इमारतों के समूह को ‘एल्फिंस्टन सर्कल’ कहा जाता था। यह इमारत इटली की इमारतों से प्रेरित थी। इसमें पहली मंजिल पर ढँके हुए तोरण पथ का रचनात्मक ढंग से इस्तेमाल किया गया। दुकानदारों व पैदल चलने वालों को तेज धूप और बरसात से बचाने के लिए यह सुधार काफी उपयोगी था ।

(2) नव-गॉथिक शैली- एक अन्य स्थापत्य शैली जिसका काफी प्रयोग किया गया, वह थी- नव- गाँधिक शैली ऊँची उठी हुई छरों, नोकदार मेहराबें और बारीक साज-सज्जा इस शैली की खासियत थी। इस शैली की इमारतों का जन्म गिरजाघरों से हुआ था, जो मध्यकाल में यूरोप में काफी बनाए गए थे। 19वीं सदी के मध्य इंग्लैण्ड में इसे दुबारा अपनाया गया बम्बई सचिवालय, बम्बई विश्वविद्यालय और उच्च न्यायालय की इमारतें इसी शैली में बनाई गई नव-गॉथिक शैली का सबसे बेहतरीन उदाहरण ‘विक्टोरिया टर्मिनस’ है, जो ग्रेट इंडियन पेनिन्स्युलर रेलवे कम्पनी का स्टेशन और मुख्यालय हुआ करता था।

(3) इंडो-सारासेनिक शैली 20वीं सदी के प्रारम्भ में एक नयी मिश्रित स्थापत्य शैली विकसित हुई, जिसमें भारतीय और यूरोपीय, दोनों तरह की शैलियों के तत्व थे। इस शैली को ‘इंडो-सारासेनिक शैली’ का नाम दिया गया था। 1911 में बना ‘गेट वे ऑफ इण्डिया’ परम्परागत गुजराती शैली का प्रसिद्ध उदाहरण था उद्योगपति जमशेदजी टाटा ने इसी शैली में ‘ताजमहल होटल’ बनवाया था। बम्बई के ज्यादातर ‘भारतीय’ इलाकों में सजावट एवं भवन निर्माण और साज-सज्जा में इसी शैली का बोलबाला था।

प्रश्न 9.
औपनिवेशिक भारत की स्थापत्य शैलियों से किन तथ्यों के बारे में जानकारी मिलती है?
उत्तर:
औपनिवेशिक भारत की स्थापत्य शैलियों से विभिन्न तथ्यों की जानकारी
औपनिवेशिक भारत की स्थापत्य शैलियों से निम्नलिखित तथ्यों के बारे में जानकारी मिलती है-
(1) सौन्दर्यात्मक आदर्शों एवं उनमें निहित विविधताओं का बोध होना स्थापत्य शैलियों से अपने समय के सौन्दर्यात्मक आदर्शों और उनमें निहित विविधताओं का पता चलता है।

(2) भवन-निर्माताओं के दृष्टिकोण की प्रतीक ये इमारतें उन लोगों की सोच और दृष्टिकोण के बारे में भी बताती हैं, जो उन्हें बना रहे थे इमारतों के जरिये शासक अपनी ताकत का इजहार करना चाहते थे तथा इन इमारतों से उस समय की सत्ता को किस रूप में देखा जा रहा था, इसका भी ज्ञान होता है। इस प्रकार एक विशिष्ट युग की स्थापत्य शैली को देखकर हम यह समझ सकते हैं कि उस समय सत्ता को किस तरह देखा जा रहा था।

(3) प्रचलित रुचियों की जानकारी एवं नवीन शैलियों को लोकप्रियता प्रदान करना- स्थापत्य शैलियों से प्रचलित रुचियों का तो पता लगता ही है, साथ ही संजीव पास बुक्स यह भी पता लगता है कि इन शैलियों ने उन रुचियों को बदलने में भी महत्त्वपूर्ण भूमिका निभाई है तथा संस्कृति की रूपरेखा तय की है, जैसे बहुत सारे भारतीय भी यूरोपीय स्थापत्य शैलियों को आधुनिकता व सभ्यता का प्रतीक मानते हुए उन्हें अपनाने लगे थे तथा बहुतों ने उनके आधुनिक तत्वों को स्थानीय परम्पराओं के तत्वों में समाहित कर दिया। उन्नीसवीं शताब्दी के अन्त से हमें औपनिवेशिक आदर्शों से भिन्न क्षेत्रीय एवं राष्ट्रीय अभिरुचियों को परिभाषित करने के प्रयास दिखाई देते हैं। इस तरह स्थापत्य शैलियों को देखकर हम इस बात को भी समझ सकते हैं कि शाही और राष्ट्रीय तथा राष्ट्रीय और क्षेत्रीय के बीच सांस्कृतिक टकराव और राजनीतिक खींचतान किस तरह शक्ल ले रही थी।

प्रश्न 10.
औपनिवेशिक शासन किस प्रकार बेहिसाब आँकड़ों और जानकारियों के संग्रह पर आधारित था?
अथवा
औपनिवेशिक शहरों के अध्ययन में सहायक तत्त्वों की विवेचना कीजिये।
उत्तर:
औपनिवेशिक शासन में संग्रहित आँकड़े व जानकारियाँ ( औपनिवेशिक शहरों के अध्ययन में सहायक तत्त्व )
औपनिवेशिक शासन बेहिसाब आँकड़ों और जानकारियों के संग्रह पर आधारित था। यथा-
(1) व्यापारिक गतिविधियों का विस्तृत ब्यौरा रखना-अंग्रेजों ने अपने व्यावसायिक मामलों को चलाने के लिए व्यापारिक गतिविधियों का विस्तृत ब्यौरा रखा था। (2) शहरों का नियमित सर्वेक्षण तथा उनके सांख्यिकीय आँकड़े एकत्रित करना – वे बढ़ते शहरों में जीवन की गति और दिशा पर नजर रखने के लिए नियमित रूप से सर्वेक्षण करते थे। वे सांख्यिकीय आँकड़े इकट्ठा करते थे और विभिन्न प्रकार की सरकारी रिपोर्ट प्रकाशित करते थे।

(3) मानचित्र तैयार करना- प्रारम्भिक वर्षों में ही औपनिवेशिक सरकार ने मानचित्र तैयार करने पर ध्यान दिया। उसका मानना था कि किसी जगह की बनावट और भू-दृश्य को समझने के लिए नक्शे जरूरी होते हैं। इस जानकारी के सहारे वे इलाके पर ज्यादा बेहतर नियंत्रण कायम कर सकते थे। जब शहर बढ़ने लगे तो न केवल उनकी विकास की योजना तैयार करने के लिए बल्कि व्यवसाय को विकसित करने और अपनी सत्ता को मजबूत करने के लिए भी नक्शे बनाये जाने लगे। शहरों के नक्शों से हमें उस स्थान पर पहाड़ियों, नदियों व हरियाली का पता चलता है। ये सारी चीजें रक्षा सम्बन्धी उद्देश्यों के लिए योजना तैयार करने में बहुत काम आती हैं। इसके अतिरिक्त पाटों की जगहों, मकानों की समनता तथा गुणवत्ता तथा सड़कों की स्थिति आदि से इलाके की व्यावसायिक सम्भावनाओं का पता लगाने और कराधान की रणनीति बनाने में मदद मिलती थी।

(4) नगर निगमों की गतिविधियों से उत्पन्न रिकाईस – 19वीं सदी में शहरों के रख-रखाव के लिए पैसा इकट्ठा करने के लिए आंशिक लोक प्रतिनिधित्व से लैस नगर निगम जैसी संस्था की स्थापना की गई। नगर- निगमों की गतिविधियों से नए तरह के रिकार्ड्स पैदा हुए, जिन्हें नगरपालिका रिकार्ड रूम में सम्भाल कर रखा जाने
लगा।

(5) जनगणना आँकड़े शहरों के फैलाव पर नजर रखने के लिए नियमित रूप से लोगों की गिनती की जाती थी। 19वीं सदी के मध्य तक विभिन्न क्षेत्रों में कई जगह स्थानीय स्तर पर जनगणना की जा चुकी थी। अखिल भारतीय जनगणना का पहला प्रयास 1872 में किया गया। इसके बाद, 1881 से हर दस साल में जनगणना एक नियमित व्यवस्था बन गई। भारत में शहरीकरण का अध्ययन करने के लिए जनगणना से निकले आँकड़े एक बहुमूल्य स्रोत हैं। इस प्रकार जनगणना, नगरपालिका जैसे संस्थानों के सर्वेक्षण, मानचित्रों और अन्य रिकार्डों के सहारे औपनिवेशिक शहरों का पुराने शहरों के मुकाबले ज्यादा विस्तार से अध्ययन किया जा सकता है।

JAC Class 12 History Important Questions Chapter 12 औपनिवेशिक शहर : नगर-योजना, स्थापत्य

प्रश्न 11.
औपनिवेशिक काल में भारत में जनगणना की प्रक्रिया में क्या भ्रम थे? जनगणनाओं के सावधानी से अध्ययन करने पर क्या दिलचस्प रुझान सामने आते हैं? उत्तर- औपनिवेशिक भारत में जनगणना से सम्बन्धित भ्रम यद्यपि भारत में शहरीकरण का अध्ययन करने के लिए जनगणना से निकले आँकड़े एक बहुमूल्य स्रोत हैं, तथापि इस प्रक्रिया में भी कई भ्रम थे यथा-

(1) वर्गीकरण सम्बन्धी भ्रम- जनगणना आयुक्तों ने आबादी के विभिन्न तबकों का वर्गीकरण करने के लिए अलग-अलग श्रेणियाँ बना दी थीं। कई बार यह वर्गीकरण निहायत अतार्किक होता था और लोगों की परिवर्तनशील तथा परस्पर काटती पहचानों को पूरी तरह नहीं पकड़ पाता था।

(2) स्वयं लोगों द्वारा सहयोग न करना- प्राय: लोग स्वयं भी इस प्रक्रिया में सहयोग करने से इनकार कर देते थे या जनगणना आयुक्तों को गलत जवाब दे देते थे। काफी समय तक वे जनगणना कार्यों को सन्देह की दृष्टि से देखते रहे। उन्हें लगता था कि सरकार नये टैक्स लागू करने के लिए जाँच करवा रही है।

(3) औरतों के बारे में जानकारी देने में हिचकिचाना – ऊँची जाति के लोग अपने घर की औरतों के बारे में पूरी जानकारी देने से हिचकिचाते थे। महिलाओं से अपेक्षा की जाती थी कि वे घर के भीतरी हिस्से में दुनिया से कट कर रहें। उनके बारे में सार्वजनिक जाँच को सही नहीं माना जाता था।

(4) पहचान सम्बन्धी दावों का भ्रामक होना- जनगणना अधिकारियों ने यह भी पाया कि बहुत सारे लोग ऐसी पहचानों का दावा करते थे जो ऊँची हैसियत की मानी जाती थीं। उदाहरण के लिए, शहरों में ऐसे लोग भी थे जो फेरी लगाते थे या काम न होने पर मजदूरी करने लगते थे। इस तरह के बहुत सारे लोग जनगणना कर्मचारियों के सामने स्वयं को प्रायः व्यापारी बताते थे क्योंकि उन्हें मजदूर की तुलना में व्यापारी ज्यादा सम्मानप्रद लगता था।

(5) मृत्युदर और बीमारियों से सम्बन्धित आँकड़ों को इकट्ठा करना लगभग असम्भव था मृत्यु दर और बीमारियों से सम्बन्धित आंकड़ों को इकट्ठा करना भी लगभग असम्भव था। बीमार पड़ने की जानकारी भी लोग प्रायः नहीं देते थे। बहुत बार इलाज भी गैर लाइसेंसी डॉक्टरों से करा लिया जाता था। ऐसे में बीमारी या मौत की घटनाओं का सटीक हिसाब लगाना सम्भव नहीं था। जनगणनाओं का सावधानी से अध्ययन करने पर कुछ दिलचस्प रुझान जनगणनाओं का सावधानी से अध्ययन करने पर निम्नलिखित दिलचस्प रुझान सामने आते हैं-

(1) सन् 1800 के बाद हमारे देश में शहरीकरण की रफ्तार धीमी रही। पूरी 19वीं सदी और 20वीं सदी के पहले दो दशकों तक देश की कुल आबादी में शहरी आबादी का हिस्सा बहुत मामूली तथा स्थिर रहा। यह लगभग 10 प्रतिशत रहा।
(2) 1900 से 1940 के बीच 40 सालों के दरमियान शहरी आबादी 10 प्रतिशत से बढ़कर 13 प्रतिशत हो गई थी।
(3) नए व्यावसायिक एवं प्रशासनिक केन्द्रों के रूप में बम्बई, मद्रास और कलकत्ता शहर पनपे लेकिन दूसरे तत्कालीन शहर कमजोर भी हुए।

प्रश्न 5.
” भारत छोड़ो आन्दोलन ब्रिटिश शासन के खिलाफ गाँधीजी का तीसरा बड़ा आन्दोलन था।” व्याख्या कीजिए।
उत्तर:
क्रिप्स मिशन की विफलता के पश्चात् महात्मा गाँधी ने ब्रिटिश शासन के खिलाफ अपना तीसरा बड़ा आन्दोलन छेड़ने का फैसला किया। अगस्त, 1942 ई. में शुरू किए गए इस आन्दोलन को ‘अंग्रेज भारत छोड़ो’ के नाम से जाना गया। भारत छोड़ो आन्दोलन प्रारम्भ करने के कारण-
(i) अंग्रेजों की साम्राज्यवादी नीति- सितम्बर, 1939 में द्वितीय विश्व युद्ध प्रारम्भ हो गया। महात्मा गाँधी व जवाहरलाल नेहरू दोनों ही हिटलर व नात्सियों के आलोचक थे। तदनुरूप उन्होंने फैसला किया कि यदि अंग्रेज बुद्ध समाप्त होने के पश्चात् भारत को स्वतन्त्रता देने पर सहमत हों तो कांग्रेस उनके युद्ध प्रयासों में सहायता दे सकती है। ब्रिटिश सरकार ने कांग्रेस के इस प्रस्ताव को खारिज कर दिया। इस घटनाक्रम ने अंग्रेजी साम्राज्यवादी नीति के विरुद्ध आन्दोलन प्रारम्भ करने हेतु प्रोत्साहित किया।

(ii) क्रिप्स मिशन की असफलता द्वितीय विश्व युद्ध में कांग्रेस व गाँधीजी का समर्थन प्राप्त करने के लिए तत्कालीन ब्रिटिश प्रधानमन्त्री विंस्टन चर्चिल ने अपने एक मन्त्री सर स्टेफर्ड क्रिप्स को भारत भेजा। क्रिप्स के साथ वार्ता में कांग्रेस ने इस बात पर जोर दिया कि यदि धुरी शक्तियों से भारत की रक्षा के लिए ब्रिटिश शासन कांग्रेस का समर्थन चाहता है तो वायसराय को सबसे पहले अपनी कार्यकारी परिषद् में किसी भारतीय को एक रक्षा सदस्य के रूप में नियुक्त करना चाहिए। इसी बात पर वार्ता टूट गयी। क्रिप्स मिशन की विफलता के पश्चात् गाँधीजी ने अंग्रेजों भारत छोड़ो आन्दोलन प्रारम्भ करने का फैसला किया। भारत छोड़ो आन्दोलन का प्रारम्भ-9 अगस्त, 1942 ई. को गाँधीजी के नेतृत्व में भारत छोड़ो आन्दोलन प्रारम्भ हो गया।

अंग्रेजों ने इस आन्दोलन को दबाने के लिए बड़ी कठोरता से काम लिया। कांग्रेस को अवैध घोषित कर दिया गया तथा सभाओं, जुलूसों व समाचार-पत्रों पर कठोर प्रतिबन्ध लगा दिए गए। इसके बावजूद देशभर के युवा कार्यकर्ता हड़तालों एवं तोड़फोड़ की कार्यवाहियों के माध्यम से आन्दोलन चलाते रहे। कांग्रेस में जयप्रकाश नारायण जैसे समाजवादी सदस्य भूमिगत होकर अपनी गतिविधियों को चलाते रहे। आन्दोलन का अन्त-अंग्रेजों ने भारत छोड़ो आन्दोलन के प्रति कठोर रवैया अपनाया फिर भी इस विद्रोह का दमन करने में साल भर से अधिक समय लग गया।

JAC Class 12 History Important Questions Chapter 9 शासक और इतिवृत्त : मुगल दरबार

Jharkhand Board JAC Class 12 History Important Questions Chapter 9 शासक और इतिवृत्त : मुगल दरबार Important Questions and Answers.

JAC Board Class 12 History Important Questions Chapter 9 शासक और इतिवृत्त : मुगल दरबार

बहुविकल्पीय प्रश्न (Multiple Choice Questions)

1. ‘मुगल’ की उत्पत्ति किस शब्द से हुई है –
(अ) महान
(स) मुसलमान
(ब) मंगोल
(द) मोगली
उत्तर:
(ब) मंगोल

2. किस मुगल सम्राट को शेरशाह से पराजित होकर ईरान भागना पड़ा?
(अ) बाबर
(ब) अकबर
(स) हुमायूँ
(द) जहाँगीर
उत्तर:
(स) हुमायूँ

3. मुगल साम्राज्य का संस्थापक था –
(अ) बाबर
(ब) हुमायूँ
(स) अकबर
(द) जहाँगीर
उत्तर:
(अ) बाबर

JAC Class 12 History Important Questions Chapter 9 शासक और इतिवृत्त : मुगल दरबार

4. मुगल राजवंश का अन्तिम सम्राट कौन था?
(अ) मुअज्जम
(ब) फर्रुखसियर
(स) शाह आलम
(द) बहादुरशाह जफर द्वितीय
उत्तर:
(द) बहादुरशाह जफर द्वितीय

5. महाभारत का फारसी में अनुवाद किसके रूप में हुआ –
(अ) अकबरनामा
(ब) शाहीनामा
(स) एमनामा
(द) बाबरनामा
उत्तर:
(स) एमनामा

6. फारसी के हिन्दवी के साथ पारस्परिक सम्पर्क से किस नई भाषा का जन्म हुआ?
(ब) उर्दू
(द) संस्कृत
(अ) अरबी
(स) तुर्की
उत्तर:
(ब) उर्दू

7. नस्तलिक शैली के सबसे अच्छे सुलेखकों में से एक था –
(अ) रजा खाँ
(ब) इब्राहीम खाँ
(स) मन्सूर अली
(द) मुहम्मद हुसैन
उत्तर:
(द) मुहम्मद हुसैन

JAC Class 12 History Important Questions Chapter 9 शासक और इतिवृत्त : मुगल दरबार

8. एशियाटिक सोसाइटी आफ बंगाल की स्थापना किसके द्वारा की गई?
(अ) जॉन मार्शल
(ब) मैकाले
(स) विलियम जोन्स
(द) कनिंघम
उत्तर:
(स) विलियम जोन्स

9. एशियाटिक सोसाइटी ऑफ बंगाल की स्थापना की गई –
(अ) 1784 ई.
(ब) 1794 ई.
(स) 1884 ई.
(द) 1795 ई.
उत्तर:
(अ) 1784 ई.

10. अकबर ने जजिया कर कब समाप्त कर दिया?
(अ) 1573
(ब) 1564
(द) 1579
(स) 1604
उत्तर:
(ब) 1564

11. मुगल पितृ पक्ष से निम्न में से किस तुर्की शासक के वंशज थे?
(अ) तिमूर
(ब) चंगेज खाँ
(स) मंगोल
(द) उपर्युक्त सभी
उत्तर:
(अ) तिमूर

12. रज्मनामा निम्नलिखित में से किस ग्रन्थ का फारसी अनुवाद है-
(अ) रामायण
(ब) लीलावती
(स) पंचतन्त्र
(द) महाभारत
उत्तर:
(द) महाभारत

13. नस्तलिक का सम्बन्ध निम्नलिखित में से किससे है?
(अ) एक ग्रन्थ
(ब) एक कविता
(स) सुलेखन शैली
(द) इनमें से कोई नहीं
उत्तर:
(स) सुलेखन शैली

14. निम्न में से किस मुगल दरबारी इतिहासकार ने चित्रकारी को एक जादुई कला के रूप में वर्णित किया है?
(अ) मुहम्मद हुसैन
(ब) अबुल फजल
(स) अब्दुल हमीद लाहौरी
(द) अब्दुल हसन
उत्तर:
(ब) अबुल फजल

JAC Class 12 History Important Questions Chapter 9 शासक और इतिवृत्त : मुगल दरबार

15. 1560 के दशक में अकबर ने किस इमारत का निर्माण करवाया?
(अ) आगरा का किला
(ब) ताजमहल
(स) फतेहप्हपुर सीकरी
(द) सिकन्दर
उत्तर:
(अ) आगरा का किला

रिक्त स्थानों की पूर्ति कीजिए :
1. जंगल बुक …………… द्वारा लिखी गई है जिससे युवा नायक …………….. का नाम व्युत्पन्न हुआ है।
2. हुमायूँ ने सूरों को …………… में पराजित किया था।
3. अकबर का उत्तराधिकारी …………….. हुआ।
4. …………… मुगल वंश का अन्तिम मुगल शासक था।
5. अकबर, शाहजहाँ आलमगीर की कहानियों पर आधारित इतिवृत्तों के शीर्षक क्रमशः ……………. और …………. है।
6. चगताई तुर्क स्वयं को …………….. के सबसे बड़े पुत्र का वंशज मानते थे।
उत्तर:
1. रुडयार्ड किपलिंग, मोगली
2. 1555
3. जहाँगीर
4. बहादुरशाह जफर द्वितीय
5. अकबरनामा, शाहजहाँनामा, आलमगीरनामा
6. चंगेज खाँ।

अतिलघूत्तरात्मक प्रश्न

प्रश्न 1.
मुगल शासकों में सबसे महान कौन था?
उत्तर:
मुगल शासकों में अकबर सबसे महान था।

प्रश्न 2.
‘सिजदा तथा जमींबोसी ‘ का क्या अर्थ है?
उत्तर:
‘सिजदा’ का अर्थ बादशाह के सामने दंडवत लेटना तथा ‘जमींबोसी’ का अर्थ जमीन चूमना था।

प्रश्न 3.
कौन-सा किला सफावियों और मुगलों के बीच झगड़े का कारण रहा था?
उत्तर:
कन्धार का किला।

प्रश्न 4.
बादशाहनामा के दरबारी दृश्य में अंकित शान्तिपूर्वक चिपटकर बैठे हुए शेर एवं बकरी क्या संदेश देते हैं
उत्तर:
मुगल- राज्य में सबल तथा निर्बल परस्पर सद्भाव से रहते थे।

प्रश्न 5.
मुगल दरबार में प्रचलित अभिवादन के दो तरीकों का उल्लेख कीजिए।
उत्तर:
(1) सिजदा अथवा दण्डवत लेटना
(2) जमीं बरेसी (जमीन चूमना)।

प्रश्न 6.
फतेहपुर सीकरी का बुलन्द दरवाजा क्यों और किसने बनवाया?
उत्तर:
गुजरात में मुगल विजय की बाद में अकबर ने बुलन्द दरवाजा बनवाया।

JAC Class 12 History Important Questions Chapter 9 शासक और इतिवृत्त : मुगल दरबार

प्रश्न 7.
मुगलकाल में पांडुलिपि रचना का मुख्य केन्द्र क्या कहलाता था?
उत्तर:
शाही किताबखाना।

प्रश्न 8.
‘मुगल’ नाम किस शब्द से व्युत्पन्न हुआ है?
उत्तर:
मंगोल।

प्रश्न 9.
मुगल किस मूल के वंशज थे और उनकी मातृ भाषा क्या थी?
उत्तर:
(1) चगताई तुर्क
(2) तुर्की भाषा।

प्रश्न 10.
मुगल शासक अपने को तैमूरी क्यों कहते थी?
उत्तर:
क्योंकि पितृ पक्ष से वे वंशज थे। तुर्की शासक तिमूर के

प्रश्न 11.
बावर मातृपक्ष से किसका सम्बन्धी था?
उत्तर:
चंगेज खाँ का।

प्रश्न 12.
मुगल साम्राज्य का संस्थापक कौन था?
उत्तर:
जहीरुद्दीन बाबर।

प्रश्न 13.
अकबर के तीन योग्य उत्तराधिकारियों के नाम लिखिए।
उत्तर:
(1) जहाँगीर
(2) शाहजहाँ
(3) औरंगजेब।

प्रश्न 14.
अकबर, शाहजहाँ, आलमगीर (औरंगजेब ) की कहानियों पर आधारित इतिवृत्तों के शीर्षक बताइये।
उत्तर:
(1) अकबरनामा
(2) शाहजहाँनामा
(3) आलमगीरनामा।

प्रश्न 15.
संस्कृत के दो ग्रन्थों के नाम लिखिए जिनका फारसी में अनुवाद करवाया गया।
उत्तर:
(1) रामायण
(2) महाभारत

प्रश्न 16.
चुगताई तुर्क कौन थे?
उत्तर:
मुगल

JAC Class 12 History Important Questions Chapter 9 शासक और इतिवृत्त : मुगल दरबार

प्रश्न 17.
अकबर की पसंदीदा सुलेखन शैली कौनसी
उत्तर:
नस्तलिक।

प्रश्न 18.
अबुल फसल के अनुसार चित्रकारी का क्या महत्त्व था?
उत्तर:
चित्रकारी निर्जीव वस्तुओं में भी प्राण फूंक सकती थी।

प्रश्न 19.
ईरान से मुगल दरबार में आने वाले दो प्रसिद्ध चित्रकारों के नाम लिखिए।
उत्तर:
(1) मीर सैय्यद अली
(2) अब्दुस समद।

प्रश्न 20.
दो महत्त्वपूर्ण चित्रित मुगल इतिहासों के नाम लिखिए।
उत्तर:
(1) अकबरनामा
(2) बादशाहनामा।

प्रश्न 21.
एशियाटिक सोसाइटी ऑफ बंगाल की स्थापना का क्या उद्देश्य था?
उत्तर:
भारतीय पांडुलिपियों का सम्पादन, प्रकाशन और अनुवाद करना।

प्रश्न 22.
अबुल फसल के अनुसार बादशाह अपनी प्रजा के किन-किन सत्वों की रक्षा करते थे?
उत्तर:
मुगल सम्राट अपनी प्रजा के निम्न सत्त्वों की रक्षा करते थे-
(1) जीवन (जन),
(2) धन (माल),
(3) सम्मान (नामस)
(4) विश्वास (दीन)।

प्रश्न 23.
इन सवों की रक्षा के बदले में बादशाह अपनी प्रजा से क्या माँग करता था?
उत्तर:
(1) आज्ञापालन
(2) संसाधनों में हिस्से की

प्रश्न 24.
जजिया क्या था?
उत्तर:
जजिया एक कर था, जो गैर-मुसलमानों पर लगाया जाता था।

प्रश्न 25.
अकबर ने फतेहपुर सीकरी से राजधानी को कहाँ स्थानान्तरित किया और कब?
उत्तर:
1585 में अकबर ने राजधानी को लाहौर स्थानान्तरित किया।

प्रश्न 26.
मुगलकाल में दरबारियों के द्वारा बादशाह के अभिवादन के उच्चतम तरीके क्या थे?
उत्तर:
(1) सिजदा अथवा दण्डवत् लेटना
(2) चार तस्लीम
(3) जर्मी बोसी (जमीन चूमना )।

JAC Class 12 History Important Questions Chapter 9 शासक और इतिवृत्त : मुगल दरबार

प्रश्न 27.
मुगल दरबार को किन चार अवसरों पर खूब सजाया जाता था?
उत्तर:
(1) सिंहासनारोहण की वर्षगांठ
(2) ईद
(3) शबे बरात
(4) होली। प्रश्न

प्रश्न 28.
औरंगजेब ने जयसिंह तथा जसवन्त सिंह
को किसकी पदवी प्रदान की थी?
उत्तर:
मिर्जा राजा।

प्रश्न 29.
मुगल सम्राट द्वारा दिए जाने वाले उपहारों (पुरस्कारों) का उल्लेख कीजिए।
उत्तर:
(1) सम्मान का जामा (खिल्लत)
(2) सरप्या
(3) रत्नजड़ित आभूषण।

प्रश्न 30.
शाहजहाँ की कौनसी पुत्रियों को ऊंचे शाही मनसबदारों के समान वार्षिक आय होती थी?
उत्तर-
(1) जहाँआरा
(2) रोशनआरा।

प्रश्न 31.
मुगल साम्राज्य के आरम्भ से ही कौन से अभिजात अकबर की शाही सेवा में उपस्थित थे?
उत्तर:
तूरानी और ईरानी अभिजात।

प्रश्न 32.
मुगलकाल में सभी सरकारी अधिकारियों के दर्जे और पदों में दो प्रकार के संख्या-विषयक पद कौनसे होते थे?
उत्तर:
(1) जात
(2) सवार

प्रश्न 33.
केन्द्रीय सरकार के दो महत्त्वपूर्ण मंत्रियों के नामोल्लेख कीजिए।
उत्तर:
(1) दीवाने आला (वित्तमंत्री)
(2) सद्र- उस- सुदूर (मदद-ए-माशा अथवा अनुदान का मन्त्री)

प्रश्न 34.
मुगलकाल में प्रान्तीय शासन के तीन प्रमुख अधिकारियों के नामोल्लेख कीजिए।
उत्तर:
(1) दीवान
(2) बख्शी
(3) सूबेदार।

JAC Class 12 History Important Questions Chapter 9 शासक और इतिवृत्त : मुगल दरबार

प्रश्न 35.
मुगलकालीन परगनों के तीन अधिकारियों के नामोल्लेख कीजिए।
उत्तर:
(1) कानूनगो (राजस्व आलेख का रखवाला)
(2) चौधरी (राजस्व संग्रह का प्रभारी)
(3) काजी।

प्रश्न 36.
ईरानी सेना ने मुगलों के किस दुर्ग पर और कब अधिकार किया?
उत्तर:
1622 में कन्धार के दुर्ग पर

प्रश्न 37.
पहला जेसुइट शिष्टमण्डल फतेहपुर सीकरी के मुगल दरबार में कब पहुँचा?
उत्तर:
1580 ई. में

प्रश्न 38.
अकबर ने इबादतखाने का निर्माण कहाँ करवाया था?
उत्तर:
फतेहपुरी सीकरी में।

प्रश्न 39.
मुन्तखाब उत तवारीख’ की रचना किसने की थी?
उत्तर:
अब्दुल कादिर बदायूँनी ने

प्रश्न 40.
अकबर द्वारा निर्मित बुलन्द दरवाजा कहाँ स्थित है?
उत्तर:
फतेहपुर सीकरी में।

प्रश्न 41.
फतेहपुर सीकरी में शेख सलीम चिश्ती का मकबरा किस मुगल सम्राट ने बनवाया था?
उत्तर:
अकबर ने।

प्रश्न 42
आलमगीर की पदवी किसने धारण की
उत्तर:
औरंगजेब।

प्रश्न 43.
फारसी के हिन्दवी के साथ पारस्परिक सम्पर्क से किस भाषा की उत्पत्ति हुई?
उत्तर:
उर्दू।

प्रश्न 44.
‘बाबरनामा’ क्या है?
उत्तर:
‘बाबरनामा’ में बाबर के संस्मरण संकलित हैं।

JAC Class 12 History Important Questions Chapter 9 शासक और इतिवृत्त : मुगल दरबार

प्रश्न 45.
फारसी को मुगल दरबार की भाषा बनाने वाला सम्राट कौन था?
उत्तर:
अकबर।

प्रश्न 46.
अकबर के शासनकाल में ‘महाभारत’ का -फारसी में किस नाम से अनुवाद हुआ?
उत्तर:
‘रक्मनामा’।

प्रश्न 47.
अबुल फजल ने चित्रकारी को किसकी संज्ञा दी है?
था।
उत्तर:
‘जादुई कला’ की।

प्रश्न 48.
बिहजाद कौन था?
उत्तर:
बिहजाद मुगलकाल का एक प्रसिद्ध चित्रकार

प्रश्न 49.
अकबर ने धार्मिक क्षेत्र में कौनसी नीति अपनाई थी?
उत्तर:
सुलह-ए-कुल।

प्रश्न 50.
‘सुलह-ए-कुल’ का क्या अर्थ है?
उत्तर:
पूर्ण शान्ति।

प्रश्न 51.
अकबर के पश्चात् किस मुगल सम्राट ने हिन्दुओं पर जजिया कर पुनः लगा दिया?
उत्तर:
औरंगजेब ने।

प्रश्न 52.
पीड़ित लोगों को निष्पक्षतापूर्वक न्याय दिलाने के लिए अपने महल में न्याय की जंजीर लगाने वाला मुगल सम्राट कौन था?
उत्तर:
जहाँगीर।

प्रश्न 53.
झरोखा दर्शन की प्रथा किस मुगल सम्राट ने शुरू की थी?
उत्तर:
अकबर ने

प्रश्न 54.
कोर्निश का क्या अर्थ है?
उत्तर:
कोर्निश दरबार में औपचारिक अभिवादन का एक तरीका था।

प्रश्न 55.
शाहजहाँ ने कब और किसे अपनी राजधानी बनाया?
उत्तर:
1648 में शाहजहाँ ने शाहजहाँनाबाद को अपनी राजधानी बनाया।

JAC Class 12 History Important Questions Chapter 9 शासक और इतिवृत्त : मुगल दरबार

प्रश्न 56.
शाहजहाँ द्वारा निर्मित तीन इमारतों के नाम लिखिए।
उत्तर:
(1) लाल किला (दिल्ली)
(2) जामा मस्जिद (दिल्ली)
(3) ताजमहल (आगरा)।

प्रश्न 57.
हरम शब्द का क्या अर्थ है?
उत्तर:
‘पवित्र स्थान’ मुगलों का अन्तःपुर हरम कहलाता था।

प्रश्न 58.
मुगल परिवार में ‘अगाचा’ कौन होती थीं?
उत्तर:
मुगल सम्राटों की उप-पत्नियाँ।

प्रश्न 59.
‘हुमायूँनामा’ की रचना किसने की थी?
उत्तर:
गुलबदन ने।

प्रश्न 60.
अकबर की शाही सेवा में नियुक्त होने वाला प्रथम व्यक्ति कौनसा राजपूत मुखिया था?
उत्तर:
भारमल कछवाहा।

प्रश्न 61.
अकबर के साथ सर्वप्रथम किस राजपूत नरेश ने अपनी पुत्री का विवाह किया था?
उत्तर:
आम्बेर के राजा भारमल कछवाहा ने।

प्रश्न 62.
टोडरमल कौन था?
उत्तर:
टोडरमल अकबर का वित्तमन्त्री था

प्रश्न 63.
मीरबख्शी कौन था?
उत्तर:
यह सैन्य विभाग का मन्त्री था।

प्रश्न 64.
मुगलों के केन्द्रीय शासन के दो प्रमुख मन्त्रियों का उल्लेख कीजिये।
उत्तर:
(1) दीवान-ए-आला
(2) सद्र-उस- सुदूर

JAC Class 12 History Important Questions Chapter 9 शासक और इतिवृत्त : मुगल दरबार

प्रश्न 65.
मुगलकाल में प्रान्त का प्रमुख क्या कहलाता
उत्तर:
सूबेदार

प्रश्न 66.
मुगल काल में प्रत्येक सूबा किनमें विभाजित
उत्तर:
सरकारों में।

प्रश्न 67.
अकबर ने कन्धार के दुर्ग पर कब अधिकार स्थापित किया था?
उत्तर:
1595 ई. में

प्रश्न 68.
ईरानियों और मुगलों के बीच विवाद का मुख्य कारण क्या था?
उत्तर:
कन्धार पर अधिकार करना।

प्रश्न 69.
मुगलकाल में सिजदा के महत्व का उल्लेख कीजिये।
उत्तर:
‘अकबरनामा’ का लेखक अबुल फसल तथा ‘बादशाहनामा’ का लेखक अब्दुल हमीद लाहौरी था।

प्रश्न 79.
अबुल फजल के अनुसार सुलह-ए-कुल क्या था?
उत्तर:
अबुल फसल सुलह-ए-कुल के आदर्श को प्रबुद्ध शासन की आधारशिला मानता था।

प्रश्न 80.
सुलह-ए-कुल की नीति के अन्तर्गत अकबर द्वारा किए गए दो उपायों का उल्लेख कीजिए।
उत्तर:
अकबर ने 1563 में तीर्थयात्रा कर तथा 1564 में जजिया कर समाप्त कर दिया।

प्रश्न 81.
सुलह-ए-कुल का क्या आदर्श था?
उत्तर:
साम्राज्य के सभी धार्मिक तथा नृजातीय समूहों के लोगों के बीच शान्ति, सद्भावना और एकता बनाये रखना।

प्रश्न 82.
राज्यारोहण के बाद जहाँगीर ने पहला आदेश क्या दिया था?
उत्तर:
न्याय की जंजीर को लगाने का ताकि पीड़ित व्यक्ति जंजीर हिलाकर जहाँगीर का ध्यान आकर्षित कर सकें।

प्रश्न 83.
अकबर द्वारा बनाई गई राजधानियों के नाम लिखिए।
उत्तर:
(1) आगरा
(2) फतेहपुरी सीकरी
(3) लाहौर।

प्रश्न 84.
अकबर ने कब और किसे अपनी राजधानी बनाया?
उत्तर:
1570 के दशक में अकबर ने फतेहपुरी सीकरी को अपनी राजधानी बनाया।

JAC Class 12 History Important Questions Chapter 9 शासक और इतिवृत्त : मुगल दरबार

प्रश्न 85.
झरोखा दर्शन की प्रथा का क्या उद्देश्य था?
उत्तर:
जन विश्वास के रूप में शाही सत्ता की स्वीकृति को और विस्तार देना।

प्रश्न 86.
दीवान-ए-आम से क्या अभिप्राय है?
उत्तर:
दीवान-ए-आम सार्वजनिक सभा भवन था जहाँ राज्य के प्रमुख अधिकारी अपनी रिपोर्ट और प्रार्थना-पत्र प्रस्तुत करते थे।

प्रश्न 87.
‘दीवान-ए-खास’ क्या था?
उत्तर:
मुगल सम्राट दीवान-ए-खास नामक भवन में राज्य के वरिष्ठ मन्त्रियों से गोपनीय मुद्दों पर चर्चा करते थे। प्रश्न 88. मुगल सम्राट वर्ष में कौन-से तीन मुख्य त्यौहार मनाते थे?
उत्तर:
(1) सूर्य वर्ष के अनुसार सम्राट का जन्म दिन
(2) चन्द्र वर्ष के अनुसार सम्राट का जन्म दिन तथा
(3) फारसी नव वर्ष (नौ रोज)।

प्रश्न 89.
मुगल सम्राटों द्वारा अपने अमीरों को दी जाने वाली प्रमुख पदवियों का उल्लेख कीजिए।
उत्तर:
प्रश्न 90.
(1) आसफ खाँ
(2) मिर्जा राजा।मुगल परिवार में बेगम और अगहा महिलाएँ कौन होती थीं?
उत्तर:
(1) शाही परिवारों से आने वाली महिलाएँ ‘बेगम’ कहलाती थीं।
(2) कुलीन परिवार में जन्म न लेने वाली महिलाएँ ‘अगहा’ कहलाती थीं।

JAC Class 12 History Important Questions Chapter 9 शासक और इतिवृत्त : मुगल दरबार

प्रश्न 91.
गुलबदन बेगम कौन थी?
उत्तर:
गुलबदन बेगम बाबर की पुत्री, हुमायूँ की बहिन तथा अकबर की बुआ थी।

प्रश्न 92.
अभिजात कौन थे?
उत्तर:
मुगल राज्य के अधिकारियों का दल अभिजात- वर्ग कहलाता था।

प्रश्न 93.
आप ‘जात’ व ‘सवार’ से क्या समझते हैं? स्पष्ट करें।
उत्तर:
‘जात’ मनसबदार के पद, वेतन का सूचक था तथा ‘सवार’ यह सूचित करता था कि उसे कितने घुड़सवार रखने थे।

प्रश्न 94.
मीरबख्शी का क्या कार्य था?
उत्तर:
वह शाही सेवा में नियुक्ति तथा पदोन्नति के सभी उम्मीदवारों को मुगल सम्राट के सामने प्रस्तुत करता था।

प्रश्न 95.
‘तैनात ए-रकांब’ से आप क्या समझते हैं?
उत्तर:
यह अभिजातों का ऐसा सुरक्षित दल था जिसे किसी भी प्रान्त या सैन्य अभियान में प्रतिनियुक्त किया जा सकता था।

प्रश्न 96.
दरबारी लेखक या वाकिया नवीस कौन थे?
उत्तर:
दरबारी लेखक दरबार में प्रस्तुत किये जाने वाले सभी प्रार्थना-पत्रों, दस्तावेजों तथा शासकीय आदेशों का आलेख तैयार करते थे।

JAC Class 12 History Important Questions Chapter 9 शासक और इतिवृत्त : मुगल दरबार

प्रश्न 97.
नस्तलिक शैली से क्या अभिप्राय है?
उत्तर:
नस्तलिक शैली एक ऐसी शैली थी जिसे लम्बे सपाट प्रवाही ढंग से लिखा जाता था।

लघुत्तरात्मक प्रश्न-

प्रश्न 1.
जलालुद्दीन अकबर को मुगल बादशाहों में महानतम क्यों माना जाता है?
उत्तर:
जलालुद्दीन अकबर को (1556-1605) को मुगल बादशाहों में महानतम माना जाता है क्योंकि उसने न केवल मुगल साम्राज्य का विस्तार ही किया, बल्कि इसे अपने समय का विशालतम दृढ़तम तथा सबसे समृद्ध राज्य भी बना दिया। अकबर ने हिन्दूकुश पर्वत तक अपने साम्राज्य की सीमाओं का विस्तार किया। उसने ईरान के सफावियों और तूरान (मध्य एशिया) के उजबेकों की विस्तारवादी योजनाओं पर अंकुश लगाए रखा। उसने 1595 में कन्धार पर अधिकार कर लिया।

प्रश्न 2.
सुलेखन का क्या महत्त्व था?
उत्तर:
सुलेखन अर्थात् हाथ से लिखने की कला अत्यन्त महत्त्वपूर्ण कौशल मानी जाती थी। इसका प्रयोग भिन्न- भिन्न शैलियों में होता था ‘नस्तलिक’ अकबर की मनपसन्द शैली थी। यह एक ऐसी तरल शैली थी जिसे लम्बे सपाट प्रवाही ढंग से लिखा जाता था। इसे 5 से 10 मिलीमीटर की नोक वाले छटे हुए सरकंडे के टुकड़े से स्याही में डुबोकर लिखा जाता था।

प्रश्न 3.
मुगल पांडुलिपियों में चित्रों के समावेश का क्या महत्त्व था?
उत्तर:
(1) चित्र पुस्तक के सौन्दर्य में वृद्धि करते हैं।
(2) जहाँ लिखित माध्यम से राजा और राजा की शक्ति के विषय में जो बात कही न जा सके, उन्हें चित्रों के माध्यम से व्यक्त किया जा सकता था।
(3) अबुल फजल के अनुसार चित्रकला एक जादुई कला थी। उसकी राय में चित्रकला किसी निर्जीव वस्तु में भी प्राण फूँक सकती थी।

प्रश्न 4.
“अबुल फजल ने न्यायपूर्ण सम्प्रभुता को एक सामाजिक अनुबन्ध के रूप में परिभाषित किया है।” व्याख्या कीजिए।
उत्तर:
अबुल फजल के अनुसार बादशाह अपनी प्रजा के चार सत्वों की रक्षा करता है-
(1) जीवन (जन),
(2) धन (मूल्य),
(3) सम्मान (नामस),
(4) विश्वास (दीन)।
इसके बदले में वह आज्ञापालन तथा संसाधनों में हिस्से की माँग करता है। केवल न्यायपूर्ण सम्प्रभु ही शक्ति और दैवीय मार्गदर्शन के साथ इस सामाजिक अनुबन्ध का सम्मान कर पाते थे।

प्रश्न 5.
मुगल काल में न्याय के विचार को दृश्य रूप में किन प्रतीकों के माध्यम से निरूपित किया गया? इसका क्या उद्देश्य था?
उत्तर:
मुगल राजतंत्र में न्याय के विचार को सर्वोत्तम सद्गुण माना गया। कलाकारों द्वारा प्रयुक्त सर्वाधिक मनपसन्द प्रतीकों में से एक था. एक-दूसरे के साथ चिपटकर शान्तिपूर्वक बैठे हुए शेर और बकरी अथवा शेर और गाय इसका उद्देश्य राज्य को एक ऐसे क्षेत्र के रूप में दर्शाना था जहाँ दुर्बल तथा सबल सभी परस्पर सद्भाव से रह सकते थे।

JAC Class 12 History Important Questions Chapter 9 शासक और इतिवृत्त : मुगल दरबार

प्रश्न 6.
अकबर ने फतेहपुर सीकरी को अपनी राजधानी बनाने का निश्चय क्यों किया?
उत्तर:
अकबर ने फतेहपुर सीकरी को अपनी राजधानी बनाने का निश्चय किया। इसका कारण यह था कि फतेहपुर | सीकरी अजमेर को जाने वाली सीधी सड़क पर स्थित थी,
जहाँ शेख मुइनुद्दीन चिश्ती की दरगाह उस समय तक एक महत्त्वपूर्ण तीर्थ स्थल बन चुकी थी। मुगल बादशाहों के चिश्ती सिलसिले के सूफियों के साथ अच्छे सम्बन्ध थे।

प्रश्न 7.
राज्यारोहण के बाद जहाँगीर द्वारा दिए गए पहले आदेश का उल्लेख कीजिए।
उत्तर:
राज्यारोहण के बाद जहाँगीर ने जो पहला आदेश दिया, वह न्याय की जंजीर लगाने का था। इसका कारण यह था कि यदि न्याय करने वाले अधिकारी न्याय करने में देरी करें अथवा वे न्याय चाहने वाले लोगों के विषय में मिथ्याचार करें तो उत्पीड़ित व्यक्ति इस जंजीर को हिलाकर बादशाह का ध्यान आकृष्ट कर सकता था। इस जंजीर को शुद्ध सोने से बनाया गया था। यह 30 गज लम्बी थी तथा इसमें 60 पटयाँ लगी हुई थीं।

प्रश्न 8.
शाहजहाँ द्वारा किसे शाही राजधानी बनाया गया? उसका संक्षिप्त उल्लेख कीजिए।
उत्तर:
1648 में शाहजहाँ ने शाहजहांनाबाद को नई शाही राजधानी बनाया। दिल्ली के प्राचीन रिहायशी नगर में शाहजहाँनाबाद एक नई और शाही आबादी थी। यहाँ लाल किला, जामा मस्जिद, चांदनी चौक के बाजार की वृक्षवीथि और अभिजात वर्ग के बड़े-बड़े घर थे। शाहजहाँनाबाद विशाल एवं भव्य मुगल राजतंत्र की औपचारिक कल्पना को व्यक्त करता था।

प्रश्न 9.
‘कोर्निश’ से क्या अभिप्राय है?
उत्तर:
मुगल काल में ‘कोर्निश’ औपचारिक अभिवादन का एक ऐसा तरीका था, जिसमें दरबारी दाएँ हाथ की तलहधी को ललाट पर रखकर आगे की ओर सिर झुकाते थे। यह इस बात का प्रतीक था कि कोर्निश करने वाला व्यक्ति अपने इन्द्रिय और मन के स्थल को हाथ लगाते हुए शुककर विनम्रतापूर्वक शाही दरबार में अपने को प्रस्तुत कर रहा था।

प्रश्न 10.
मुगल अभिजात वर्गों के बीच हैसियत को निर्धारित करने वाले नियमों का उल्लेख कीजिए।
उत्तर:
दरबार में अभिजात वर्ग के किसी व्यक्ति की हैसियत इस बात से निर्धारित होती थी कि वह बादशाह के कितना पास और दूर बैठा है। किसी भी दरबारी को बादशाह द्वारा दिया गया स्थान बादशाही की दृष्टि में उसकी महत्ता का प्रतीक था। बादशाह को किए गए अभिवादन के तरीके से पदानुक्रम में उस व्यक्ति की हैसियत का पता चलता था।

प्रश्न 11.
भारत में मुगल राजवंश का अन्त कैसे हुआ?
उत्तर:
1707 ई. में औरंगजेब की मृत्यु के पश्चात् मुगल राजवंश की शक्ति घट गई। अतः विशाल मुगल साम्राज्य के स्थान पर कई क्षेत्रीय शक्तियाँ उभर आई, फिर भी सांकेतिक रूप से मुगल शासक की प्रतिष्ठा बनी रही। 1857 ई. में इस वंश के अन्तिम शासक बहादुरशाह जफर द्वितीय को अंग्रेजों ने उखाड़ फेंका इस प्रकार मुगल राजवंश का अन्त हो गया।

प्रश्न 12.
मुगल बादशाहों द्वारा तैयार करवाए गए इतिवृत्तों के कोई दो उद्देश्य लिखिए।
उत्तर:
मुगल बादशाहों द्वारा तैयार करवाए गए इतिवृत्तों के प्रमुख दो उद्देश्य निम्नलिखित थे.
(i) मुगल साम्राज्य के अन्तर्गत आने वाले समस्त लोगों के समक्ष मुगल राज्य को एक प्रबुद्ध राज्य के रूप में चित्रित करना।
(ii) मुगल शासन का विरोध करने वाले लोगों को यह बताना कि उनके सभी विरोधों का असफल होना निश्चित है।

प्रश्न 13.
न्याय की जंजीर किस बादशाह द्वारा लगवाई गई?
उत्तर:
न्याय की जंजीर जहाँगीर नामक बादशाह के द्वारा लगवाई गयी जो अपनी न्यायप्रियता के लिए बहुत प्रसिद्ध था जहाँगीर ने सिंहासन पर आरूढ़ होने के बाद सबसे पहला आदेश न्याय की जंजीर लगाने के सम्बन्ध में दिया। कोई भी व्यक्ति जिसे प्रशासन से न्याय न मिले, वह सीधे इस जंजीर को हिलाकर सम्बद्ध घण्टे को बजाकर न्याय की फरियाद बादशाह से कर सकता था। इस जंजीर को विशुद्ध रूप से सोने से बनाया गया था यह 30 गज लम्बी थी तथा इसमें 60 घण्टियाँ लगी हुई थीं।

JAC Class 12 History Important Questions Chapter 9 शासक और इतिवृत्त : मुगल दरबार

प्रश्न 14.
मुगल काल में अभिवादन का ‘चार तसलीम’ तरीका क्या था?
उत्तर:
मुगल काल में अभिवादन का ‘चार तसलीम’ तरीका दाएँ हाथ को जमीन पर रखने से शुरू होता था। इसमें तलहथी ऊपर की ओर होती थी। इसके बाद हाथ को धीरे-धीरे उठाते हुए व्यक्ति खड़ा होता था तथा तलहथी को सिर के ऊपर रखता था ऐसी तसलीम चार बार की जाती थी। तसलीम का शाब्दिक अर्थ आत्मनिवेदन है।

प्रश्न 15.
मुगल दरबार में राजनयिक दूत सम्बन्धी नयाचारों का उल्लेख कीजिए।
उत्तर:
मुगल बादशाह के समक्ष प्रस्तुत होने वाले राजदूत से यह अपेक्षा की जाती थी कि वह अभिवादन के मान्य रूपों में से एक या तो बहुत झुककर अथवा जमीन को चूमकर अथवा फारसी रिवाज के अनुसार छाती के सामने हाथ बाँधकर तरीके से अभिवादन करेगा। अंग्रेज राजदूत टामस रो ने यूरोपीय रिवाज के अनुसार जहाँगीर के सामने केवल झुककर अभिवादन किया और बैठने के लिए कुर्सी का आग्रह किया।

प्रश्न 16.
शब-ए-बारात’ पर संक्षिप्त टिप्पणी लिखिए।
उत्तर:
शब-ए-बारात हिजरी कैलेंडर के आठवें महीने अर्थात् चौदहवें सावन को पड़ने वाली पूर्ण चन्द्ररात्रि है। भारतीय उपमहाद्वीप में प्रार्थनाओं और आतिशबाजियों के खेल द्वारा इस दिन को मनाया जाता है। ऐसा माना जाता है कि इस रात मुसलमानों के लिए आगे आने वाले वर्ष का भाग्य निर्धारित होता है और पाप माफ कर दिए जाते हैं।

प्रश्न 17.
सम्मान का जामा’ तथा ‘सरप्पा’ नामक पुरस्कार क्या थे?
उत्तर:
योग्य व्यक्तियों को मुगल सम्राटों द्वारा पुरस्कार दिए जाते थे। इन पुरस्कारों में ‘सम्मान का जामा’ (खिल्लत) भी शामिल था जिसे पहले कभी न कभी बादशाह द्वारा पहना गया हुआ होता था इसलिए यह समझा जाता था कि वह बादशाह के आशीर्वाद का प्रतीक था ‘सरप्पा’ (सर से पाँव तक) एक अन्य उपहार था इस उपहार के तीन हिस्से हुआ करते थे- जामा, पगड़ी और पटका।

JAC Class 12 History Important Questions Chapter 9 शासक और इतिवृत्त : मुगल दरबार

प्रश्न 18.
‘शाही परिवार’ से क्या अभिप्राय है?
उत्तर:
‘हरम’ शब्द का प्रयोग प्राय: मुगलों की घरेलू दुनिया की ओर संकेत करता है। यह शब्द फारसी से निकला है जिसका तात्पर्य है पवित्र स्थान’ मुगल परिवार (शाही) में बादशाह की पत्नियों और उपपत्नियाँ, उसके निकटस्थ और दूर के सम्बन्धी (माता, सौतेली व उपमाताएँ. बहन, पुत्री, बहू, चाची-मौसी, बच्चे आदि) व महिला परिचारिकाएँ तथा गुलाम होते थे।

प्रश्न 19.
‘जात’ और ‘सवार’ मनसबदार से क्या अभिप्राय है?
उत्तर:
मुगल काल में सभी सरकारी अधिकारियों के दर्जे और पदों में दो तरह के संख्या-विषयक पद होते थे। ‘जात’ शाही पदानुक्रम में अधिकारी ( मनसबदार) के पद और वेतन का सूचक था और ‘सवार’ यह सूचित करता था कि उससे सेवा में कितने घुड़सवार रखना अपेक्षित था। सत्रहवीं शताब्दी में 1000 या उससे ऊपर जात वाले मनसबदार अभिजात ‘उमरा’ कहलाते थे।
हैं?

प्रश्न 20.
‘तैनात ए रकाब से आप क्या समझते
उत्तर:
भुगल दरबार में नियुक्त ‘तैनात ए रकाब ‘ अभिजातों का एक ऐसा सुरक्षित दल था, जिसे किसी भी प्रान्त या सैन्य अभियान में प्रतिनियुक्त किया जा सकता था। ये प्रतिदिन दो बार सुबह व शाम को सार्वजनिक सभा- भवन में बादशाह के प्रति आत्मनिवेदन करने के लिए उपस्थित होते थे। वे दिन-रात बादशाह और उसके घराने की सुरक्षा का दायित्व भी वहन करते थे।

प्रश्न 21.
अब्दुल कादिर बदायूँनी द्वारा ‘हरम में होम’ का वर्णन किस प्रकार किया है?
उत्तर:
‘अब्दुल कादिर बदायूँनी ने लिखा है कि युवावस्था के आरम्भ से ही, मुगल सम्राट अकबर अपनी पत्नियों अर्थात् भारत के राजाओं की पुत्रियों के सम्मान में ‘हरम में होम’ का आयोजन कर रहे थे। यह ऐसी धर्मक्रिया थी जो अग्नि पूजा (आतिश-परस्ती) से उत्पन्न हुई थी परन्तु अपने 25वें शासन वर्ष ( 1578) के नए वर्ष पर उसने सार्वजनिक रूप से सूर्य और अग्नि को दंडवत् प्रणाम किया।

प्रश्न 22.
मुगल सम्राट अपने राजवंश का इतिहास क्यों लिखवाना चाहते थे?
उत्तर:
मुगल सम्राटों की यह मान्यता थी कि उन्हें एक विशाल एवं विजातीय जनता पर शासन के लिए स्वयं ईश्वर ने नियुक्त किया है। ऐसा दृष्टिकोण सदैव महत्त्वपूर्ण बना रहा। इस दृष्टिकोण के प्रचार-प्रसार का एक तरीका राजवंशीय इतिहास लिखना लिखवाना था। अतः मुगल सम्राटों ने दरबारी इतिहासकारों को विवरण लिखने का कार्य सौंपा। इन विवरणों में मुगल सम्राटों के समय की घटनाओं का लेखा-जोखा प्रस्तुत किया गया।

प्रश्न 23.
मुगल इतिवृत्त / इतिहास के महत्त्व पर प्रकाश डालिए।
उत्तर:
मुगल इतिवृत्त घटनाओं का काल क्रमानुसार विवरण प्रस्तुत करते हैं। मुगलों का इतिहास लिखने के इच्छुक विद्वानों के लिए ये इतिवृत्त अनिवार्य स्रोत हैं। एक ओर तो ये इतिवृत्त मुगल साम्राज्य की संस्थाओं के बारे में तथ्यात्मक सूचनाएँ प्रदान करते थे, तो दूसरी ओर ये उन उद्देश्यों का प्रसार करते थे जिन्हें मुगल सम्राट अपने क्षेत्र में लागू करना चाहते थे। ये इतिवृत्त यह प्रकट करते हैं कि शाही विचारधाराएँ कैसे रची तथा प्रसारित की जाती थीं।

JAC Class 12 History Important Questions Chapter 9 शासक और इतिवृत्त : मुगल दरबार

प्रश्न 24.
खिल्लत, सरण्या, पद्म मुरस्सा आदि उपहारों से आप क्या समझते हैं?
उत्तर:
मुगल सम्राटों द्वारा योग्य व्यक्तियों को पदवियाँ और पुरस्कार प्रदान कर सम्मानित किया जाता था खिल्लत पुरस्कार में बादशाह अपने द्वारा पहना हुआ जामा उपहारस्वरूप देता था। इसे बादशाह के आशीर्वाद का प्रतीकं समझा जाता था सरप्पा, सिर से पाँव तक का उपहार होता था। इसमें जामा, पगड़ी और पटका शामिल होते थे। बादशाह द्वारा उपहार में दिए गए कमल की मंजरियों वाले रत्नजड़ित गहनों के सेट को पद्म मुरस्सा कहा जाता था।

प्रश्न 25.
मुगल वास्तुकला में जहाँआरा के योगदान पर संक्षिप्त टिप्पणी लिखिए।
उत्तर:
जहाँआरा मुगल बादशाह शाहजहाँ की पुत्री थी। उसने शाहजहाँ की नई राजधानी शाहजहाँनाबाद (दिल्ली) की कई वास्तुकलात्मक परियोजनाओं में भाग लिया। इनमें से आँगन एवं बाग के साथ एक दोमंजिली भव्य कारवाँ सराय थी। शाहजहाँनाबाद के मुख्य केन्द्र चाँदनी चौक की रूपरेखा भी जहाँआरा द्वारा बनाई गई थी।

प्रश्न 26.
सरकारी अधिकारियों के दर्जे जात, मनसबदार, उमरा आदि से आपका क्या तात्पर्य सवार, है?
उत्तर:
सरकारी अधिकारियों के दर्जे और पदों में दो तरह के ओहदे होते थे। यह ओहदे संख्या विषयक होते थे। शाही पदानुक्रम में जात अधिकारी जिसे मनसबदार कहा जाता था, के पद और वेतन को सूचित करता था और ‘सवार’ से यह सूचित होता था कि वह कितने घुड़सवार रख सकता था। 1000 या उससे ऊपर जात वाले मनसबदार ‘उमरा’ जो कि अमीर का बहुवचन है, कहे जाते थे।

प्रश्न 27.
मीर बख्शी कौन था? मीर बख्शी तथा उसका कार्यालय क्या कार्य करता था?
उत्तर:
मुगलकाल में मीर बख्शी सर्वोच्च वेतनदाता था। मीर बख्शी खुले दरबार में बादशाह के दायीं ओर खड़ा रहकर नियुक्ति तथा पोदन्नति पाने वाले उम्मीदवारों को प्रस्तुत करता था उसका कार्यालय उसकी मुहर एवं हस्ताक्षर के साथ-साथ बादशाह की मुहर तथा हस्ताक्षर वाले आदेश भी तैयार करता था।

प्रश्न 28.
मुगल कौन थे?
उत्तर:
‘मुगल’ शब्द की उत्पत्ति ‘मंगोल’ से हुई है। मुगल शासकों ने अपने को ‘तैमूरी’ कहा क्योंकि पितृपक्ष से वे तुर्की शासक तिमूर के वंशज थे प्रथम मुगल शासक बाबर मातृपक्ष से चंगेजखों का सम्बन्धी था। वह तुर्की भाषा बोलता था। मुगल चगताई मूल के थे तथा तुर्की उनकी मातृभाषा थी। सोलहवीं शताब्दी में यूरोपियों ने मुगल राजवंश के लिए मुगल शब्द का प्रयोग किया था उस समय से ही इस शब्द का निरन्तर प्रयोग होता रहा है।

प्रश्न 29.
” मुगल बादशाहों द्वारा तैयार करवाए गए इतिवृत्त साम्राज्य और उसके दरबार के अध्ययन के महत्त्वपूर्ण स्त्रोत हैं।” व्याख्या कीजिए।
अथवा
इतिवृत्त किस प्रकार मुगल सम्राटों के विषय में महत्त्वपूर्ण जानकारी प्रदान करते हैं?
उत्तर:
मुंगल सम्राटों द्वारा तैयार करवाए गए इतिवृत्त साम्राज्य और उसके दरबार के अध्ययन के महत्त्वपूर्ण खोत हैं। ये इतिवृत्त मुगल साम्राज्य के अन्तर्गत आने वाले सभी लोगों के सामने मुगल- राज्य को एक प्रबुद्ध राज्य के रूप में दर्शाने के उद्देश्य से लिखे गए थे। इसी प्रकार उनका उद्देश्य मुगल शासन का विरोध करने वाले लोगों को यह भी बताना था कि उनके सभी विरोधों का असफल होना निश्चित है। मुगल सम्राट यह भी चाहते थे कि भावी पीढ़ियों के लिए उनके शासन के विवरण उपलब्ध रहे।

JAC Class 12 History Important Questions Chapter 9 शासक और इतिवृत्त : मुगल दरबार

प्रश्न 30.
“मुगल इतिवृत्तों के लेखक निरपवाद रूप से दरबारी ही रहे।” व्याख्या कीजिये।
उत्तर:
मुगल इतिवृत्तों में शासक पर केन्द्रित घटनाओं, शासक के परिवार दरबार व अभिजात वर्ग, बुद्ध और प्रशासनिक व्यवस्थाओं का समावेश था। अकबर, शाहजहाँ तथा आलमगीर (औरंगजेब) के शासन काल की घटनाओं पर आधारित इतिवृत्तों के शीर्षक ‘अकबरनामा’, ‘शाहजहाँनामा’ तथा ‘आलमगीरनामा’ यह संकेत देते हैं कि इनके लेखकों की दृष्टि में साम्राज्य एवं दरबार का इतिहास और बादशाह का इतिहास एक ही था।

प्रश्न 31.
अकबर ने फारसी को दरबार की मुख्य भाषा क्यों बनाया?
उत्तर:
अकबर ने सोच समझकर फारसी को दरबार की मुख्य भाषा बनाया। सम्भवतया ईरान के साथ सांस्कृतिक और बौद्धिक सम्पर्कों के साथ-साथ मुगल दरबार में पद पाने को इच्छुक ईरानी और मध्य एशियाई प्रवासियों ने अकबर को फारसी भाषा को अपनाए जाने के लिए प्रेरित किया होगा। फारसी को दरबार की भाषा का ऊंचा स्थान दिया गया तथा उन लोगों को शक्ति तथा प्रतिष्ठा प्रदान की गई, जिनका इस भाषा पर अच्छा नियंत्रण था।

प्रश्न 32.
मुगल पाण्डुलिपियों में चित्रों को किस प्रकार संलग्न किया जाता था?
उत्तर:
मुगल पांडुलिपियों की रचना में चित्रकारों का भी महत्त्वपूर्ण योगदान था। किसी मुगल सम्राट के शासन की घटनाओं का विवरण देने वाले इतिहासों में लिखित पाठ के साथ ही उन घटनाओं को चित्रों के माध्यम से दृश्य रूप में भी वर्णित किया जाता था जब किसी पुस्तक में घटनाओं अथवा विषयों को दृश्य रूप में वर्णित किया जाना होता था, तो सुलेखक उसके आस-पास के पृष्ठों को खाली छोड़ देते थे। चित्रकार शब्दों में वर्णित विषय को अलग से चित्र के रूप में उतारकर वहाँ संलग्न कर देते थे।

प्रश्न 33.
मुगल पाण्डुलिपियों में चित्रों को संलग्न करने का क्या महत्त्व था?
उत्तर:
पाण्डुलिपियों में चित्रों का महत्त्व-
(1) चित्र पुस्तक के सौन्दर्य में वृद्धि करते थे।
(2) राजा और उसकी शक्ति के बारे में जो बात लिखित रूप से नहीं कही जा सकती थी, उसे चित्र व्यक्त कर देते थे।
(3) अबुल फजल ने चित्रकारी को एक ‘जादुई कला’ की संज्ञा दी है उसके अनुसार चित्रकला किसी निर्जीव वस्तु को भी सजीव रूप प्रदान कर सकती है।

प्रश्न 34.
मुगल सम्राट तथा उसके दरबार का चित्रण करने वाले चित्रों की रचना को लेकर शासकों और उलमा के बीच तनाव की स्थिति क्यों बनी रहती थी?
उत्तर:
उलमा रूढ़िवादी मुसलमान वर्ग के प्रतिनिधि थे। उन्होंने कुरान के साथ-साथ हदीस में प्रतिष्ठापित मानव रूपों के चित्रण पर इस्लामी प्रतिबन्ध का आह्वान किया। हदीस में एक प्रसंग में पैगम्बर साहब को प्राकृतिक तरीके से जीवित रूपों के चित्रण की मनाही करते हुए उल्लिखित किया गया है। इसका कारण यह है कि ऐसा करने से यह लगता था कि कलाकार रचना की शक्ति को अपने हाथ में लेने का प्रयास कर रहा है। यह केवल ईश्वर का ही कार्य था।

JAC Class 12 History Important Questions Chapter 9 शासक और इतिवृत्त : मुगल दरबार

प्रश्न 35.
मुगल शासनकाल की पांडुलिपियों में चित्रों को किस प्रकार संलग्न किया जाता था?
उत्तर:
मुगल शासनकाल की पांडुलिपियों की रचनाओं में चित्रकारों का भी बहुत महत्त्वपूर्ण योगदान था। किसी मुगल बादशाह के शासन की घटनाओं का विवरण देने वाले इतिहासों में लिखित पाठ के साथ-साथ उन घटनाओं को चित्रों के माध्यम से दृश्य रूप में दर्शाया जाता था। जब किसी पुस्तक में घटनाओं अथवा विषयों को दृश्य रूप में व्यक्त किया जाना होता था तो सुलेखक उसके आसपास के पृष्ठों को खाली छोड़ देते थे। चित्रकार शब्दों में वर्णित विषय को अलग से चित्रों में उतारकर वहाँ संलग्न कर देते थे। ये लघु चित्र होते थे जिन्हें पांडुलिपि के पृष्ठों पर आसानी से लगाया और देखा जा सकता था।

प्रश्न 36.
दैवीय प्रकाश से आप क्या समझते हैं? दरबारी इतिहासकारों ने इसकी व्याख्या कैसे की?
उत्तर:
मुगल दरबार के इतिहासकारों ने मुगल सम्राटों का महिमामण्डन करने के लिए उन्हें इस प्रकार वर्णित किया कि वे ईश्वर से सीधे दैवीय शक्ति या दैवीय प्रकाश प्राप्त करते थे। इतिहासकारों ने इसके लिए एक दंतकथा का वर्णन किया है। इस दंतकथा में वर्णन है कि मंगोल रानी अलानकुआ अपने शिविर में विश्राम करते हुए सूर्य की किरण द्वारा गर्भवती हुई। उससे उत्पन्न सन्तान दैवीय प्रकाश से युक्त थी। प्रसिद्ध ईरानी सूफी शिहाबुद्दीन सुहरावर्दी के अनुसार देवीय प्रकाश से युक्त राजा अपनी प्रजा के लिए आध्यात्मिक मार्गदर्शन का स्रोत बन जाता था। अबुल फजल ने ईश्वर से उद्भूत प्रकाश को ग्रहण करने वाली वस्तुओं में मुगल राजत्व को सबसे शीर्ष स्थान पर रखा।

प्रश्न 37.
मुगल किताबखानों में रचित पाण्डुलिपियों की रचना में किन-किन लोगों का योगदान होता था?
अथवा
पांडुलिपियों की रचना में विविध प्रकार के कार्य करने वाले लोगों का वर्णन कीजिये।
उत्तर:
पांडुलिपियों की रचना में विविध प्रकार के कार्य करने वाले अनेक लोग शामिल थे। कागज बनाने बालों की पांडुलिपि के पन्ने तैयार करने, सुलेखकों की पाठ की नकल तैयार करने, कोफ्तगरों की पृष्ठों को चमकाने के लिए, चित्रकारों की पाठ से दृश्यों को चित्रित करने के लिए और जिल्दसाजों की प्रत्येक पन्ने को इकट्ठा कर उसे अलंकृत आवरण में बैठाने के लिए आवश्यकता होती थी। तैयार पांडुलिपि को एक बहुमूल्य वस्तु के रूप में देखा जाता था।

प्रश्न 38.
अबुल फजल पर एक संक्षिप्त टिप्पणी लिखिए।
उत्तर:
अबुल फजल अकबर का एक प्रसिद्ध दरबारी इतिहासकार था। अबुलफजल का पालन-पोषण मुगल राजधानी आगरा में हुआ था। वह अरबी, फारसी, यूनानी दर्शन और सूफीवाद का प्रकाण्ड विद्वान था। वह एक प्रभावशाली विवादी तथा स्वतंत्र चिन्तक था। उसने सदैव रूढ़िवादी उलमा के विचारों का विरोध किया। अबुल फजल के इन गुणों से अकबर बढ़ प्रभावित हुआ। उसने अबुलफजल को अपने सलाहकार और अपनी नीतियों के प्रवक्ता के रूप में बहुत उपयुक्त पाया।

प्रश्न 39.
‘अकबरनामा’ पर एक संक्षिप्त टिप्पणी लिखिए।
उत्तर:
अबुलफजल द्वारा रचित अकबरनामा तीन जिल्दों में विभाजित है जिनमें से प्रथम दो इतिहास हैं। तीसरी जिल्द ‘आइन-ए-अकबरी’ है पहली जिल्द में आदम से | लेकर अकबर के जीवन के एक खगोलीय कालचक्र तक (30 वर्ष) का मानव जाति का इतिहास है। दूसरी जिल्द अकबर के 46वें शासन वर्ष (1601 ई.) पर समाप्त होती है ‘अकबरनामा’ में राजनीतिक घटनाओं, साम्राज्य के भौगोलिक, सामाजिक, प्रशासनिक तथा सांस्कृतिक आदि पक्षों का विस्तृत विवरण मिलता है।

JAC Class 12 History Important Questions Chapter 9 शासक और इतिवृत्त : मुगल दरबार

प्रश्न 40.
‘बादशाहनामा’ का संक्षिप्त परिचय दीजिए।
उत्तर:
‘बादशाहनामा’ का लेखक अब्दुल हमीद लाहौरी था। ‘बादशाहनामा’ सरकारी इतिहास है। यह तीन जिल्दों में विभाजित है। प्रत्येक जिल्द दस चन्द्र वर्षों का विवरण देती है। लाहौरी ने शाहजहाँ के शासन (1627-47) के पहले दो दशकों पर पहला व दूसरा दफ्तर लिखा। इन जिल्दों में बाद में शाहजहाँ के वजीर सादुल्लाखाँ ने सुधार किया। वृद्धावस्था की कमजोरियों के कारण लाहौरी तीसरे दशक के बारे में नहीं लिख सका जिसे बाद में इतिहासकार वारिस ने लिखा।

प्रश्न 41.
औपनिवेशिक काल में ऐतिहासिक पांडुलिपियों को किस प्रकार संरक्षित किया गया?
उत्तर:
1784 ई. में सर विलियम जोन्स द्वारा स्थापित ‘एशियाटिक सोसाइटी आफ बंगाल ने अनेक भारतीय पांडुलिपियों के सम्पादन, प्रकाशन और अनुवाद का दायित्व अपने ऊपर ले लिया। ‘अकबरनामा’ तथा ‘बादशाहनामा’ के सम्पादित पाठान्तर सर्वप्रथम एशियाटिक सोसाइटी द्वारा उन्नीसवीं शताब्दी में प्रकाशित किए गए। बीसवीं शताब्दी के प्रारम्भ में हेनरी बेवरिज ने ‘अकबरनामा’ का अंग्रेजी में अनुवाद किया। ‘बादशाहनामा’ के केवल कुछ ही अंशों का अभी तक अंग्रेजी में अनुवाद हुआ है।

प्रश्न 42.
” मुगल सम्राटों में दैवीय प्रकाश सम्प्रेषित होता था। व्याख्या कीजिए।
उत्तर:
दरबारी इतिहासकारों ने कई साक्ष्यों के आधार पर यह दर्शाया कि मुगल सम्राटों को सीधे ईश्वर से शक्ति मिली थी। अबुल फजल ने ईश्वरीय प्रकाश को ग्रहण करने वाली वस्तुओं में मुगल राजत्व को सबसे ऊँचे स्थान पर रखा। प्रसिद्ध ईरानी सूफी शिहाबुद्दीन सुहरावर्दी के विचार के अनुसार यह दैवीय प्रकाश राजा में सम्प्रेषित होता था जिसके बाद राजा अपनी प्रजा के लिए आध्यात्मिक मार्गदर्शन का स्रोत बन जाता था।

प्रश्न 43.
“सुलह-ए-कुल एकीकरण का एक स्रोत था।” विवेचना कीजिए।
अथवा
सुलहकुल का आदर्श क्या था? इसे किस प्रकार लागू किया गया?
उत्तर:
अबुल फसल ‘सुलह-ए-कुल’ (पूर्ण शान्ति ) के आदर्श को प्रबुद्ध शासन का आधार बताता है। यद्यपि ‘सुलह-ए-कुल’ में सभी धर्मों तथा मतों को अभिव्यक्ति की स्वतन्त्रता थी, किन्तु उसकी एक शर्त थी कि वे राज्य- सत्ता को क्षति नहीं पहुँचाएंगे अथवा आपस में नहीं लड़ेंगे। ईरानी तूरानी, अफगानी राजपूत, दक्खनी आदि अभिजातों को पद और पुरस्कार दिए गए। 1563 में अकबर ने तीर्थयात्रा कर तथा 1564 में जजिया कर को समाप्त कर दिया।

प्रश्न 44.
अबुल फजल ने प्रभुसत्ता को एक सामाजिक अनुबन्ध के रूप में परिभाषित क्यों किया? उत्तर- अबुल फजल ने लिखा है कि बादशाह अपनी प्रजा के चार सत्त्वों की रक्षा करता है –

  1. जीवन (जन),
  2. धन (माल),
  3. सम्मान (नामस) तथा
  4. विश्वास ( दोन )।

इसके बदले में वह आज्ञापालन तथा संसाधनों में हिस्से की माँग करता है। केवल न्यायपूर्ण सम्प्रभु ही शक्ति और दैवीय मार्गदर्शन के साथ इस अनुबन्ध का सम्मान कर पाते थे। न्याय के विचार को मुगल राजतन्त्र में सर्वोत्तम सद्गुण माना गया। न्याय के विचार को चित्रों में अनेक प्रतीकों के रूप में दर्शाया गया।

प्रश्न 45.
मुगल दरबार में अभिजात वर्ग के बीच उनकी हैसियत दर्शाने वाले नियमों का वर्णन कीजिए।
उत्तर:
दरबार में किसी अभिजात की हैसियत इस बात से निर्धारित होती थी कि वह बादशाह के कितने पास और दूर बैठा है। किसी भी दरबारी को बादशाह द्वारा दिया गया स्थान बादशाह की दृष्टि में उसकी महत्ता का द्योतक था दरबारी समाज में सामाजिक नियन्त्रण का व्यवहार दरबार में मान्य सम्बोधन, शिष्टाचार तथा बोलने के नियमों द्वारा होता था। शिष्टाचार का उल्लंघन करने वालों को तुरन्त ही दंडित किया जाता था।

JAC Class 12 History Important Questions Chapter 9 शासक और इतिवृत्त : मुगल दरबार

प्रश्न 46.
राजवंशीय इतिहास लेखन, इतिवृत्त से आप क्या समझते हैं?
उत्तर:
मुगलवंशीय साम्राज्य के शासकों ने अपने उद्देश्यों के प्रचार प्रसार एवं राजवंशीय इतिहास के संकलन हेतु दरबारी इतिहासकारों को इतिवृत्तों के लेखन का कार्य सौंपा। आधुनिक इतिहासकारों ने इस शैली को इतिवृत्ति इतिहास के नाम से सम्बोधित किया। इतिवृत्तों द्वारा हमें मुगल राज्य की घटनाओं का कालक्रम के अनुसार क्रमिक विवरण प्राप्त होता है इतिवृत्त मुगल राज्य की संस्थाओं के प्रामाणिक स्त्रोत थे: जिन्हें इनके लेखकों द्वारा परिश्रम से एकत्र और वर्गीकृत किया था। इतिवृत्तों के द्वारा इस बात की जानकारी प्राप्त होती है कि शाही विचारधाराएँ कैसे प्रसारित की जाती हैं।

प्रश्न 47.
मुगल दरबार में राजनयिक दूतों से सम्बन्धित न्याचारों का उल्लेख कीजिए।
उत्तर:
मुगल दरबार में बादशाह के सामने प्रस्तुत होने वाले राजदूत से यह अपेक्षा की जाती थी कि वह अभिवादन के मान्य रूपों में से एक या तो बहुत झुक कर अथवा जमीन को चूम कर अथवा फारसी रिवाज के अनुसार छाती के सामने हाथ बाँध कर- तरीके से अभिवादन करेगा। जेम्स प्रथम के अंग्रेज दूत टामसरो ने यूरोपीय रिवाज के अनुसार जहाँगीर के सामने केवल झुककर अभिवादन किया और फिर बैठने के लिए कुर्सी का आग्रह कर दरबार को आश्चर्यचकित कर दिया।

प्रश्न 48.
झरोखा दर्शन की प्रथा का उल्लेख कीजिए।
उत्तर:
मुगल काल में बादशाह अपने दिन की शुरूआत सूर्योदय के समय कुछ व्यक्तिगत धार्मिक प्रार्थनाओं से करता था। इसके बाद वह पूर्व की ओर मुँह किए एक छोटे छज्जे अर्थात् शरोखे में आता था। इसके नीचे लोगों की भारी भीड़ बादशाह की एक झलक पाने के लिए प्रतीक्षा करती थी। झरोखा दर्शन की प्रथा अकबर द्वारा शुरू की गई थी। इस प्रथा का उद्देश्य जन विश्वास के रूप में शाही सत्ता की स्वीकृति का और विस्तार करना था।

प्रश्न 49.
अबुल फजल के अनुसार बादशाह अपनी प्रजा का एक परिपूर्ण रक्षक है। स्पष्ट कीजिए।
उत्तर:
अबुल फजल के अनुसार बादशाह अपनी प्रजा का परिपूर्ण रक्षक है। वह प्रजा के जीवन, धन, सम्मान तथा विश्वास की रक्षा करता है जीवन, धन, सम्मान तथा विश्वास की रक्षा के बदले वह प्रजा से आज्ञापालन और राज्य के संसाधनों में कर के रूप में उचित हिस्सेदारी की आशा करता है। न्यायपूर्ण बादशाह ही शक्ति और दैवीय मार्गदर्शन के साथ इस सामाजिक अनुबन्ध का दृढ़तापूर्वक पालन कर सकने में समर्थ थे। मुगल सम्राटों के इन न्यायपूर्ण विचारों का तत्कालीन चित्रकारों द्वारा बादशाहनामा आदि ग्रन्थों में प्रतीकात्मक रूप से सचित्र निरूपण किया गया।

प्रश्न 50.
अबुल फजल ने अकबर के दरबार का किस प्रकार वर्णन किया है?
उत्तर:
अबुल फजल के अनुसार जब भी मुगल- सम्राट अकबर दरबार लगाते थे तो एक विशाल ढोल पीटा जाता था और इसके साथ-साथ अल्लाह का गुणगान होता था। बादशाह के पुत्र, पौत्र, दरबारी आदि दरबार में उपस्थित होते थे और कोर्निश (औपचारिक अभिवादन का एक तरीका ) कर अपने स्थान पर खड़े रहते थे प्रसिद्ध विद्वान तथा विशिष्ट कलाओं में निपुण लोग आदर व्यक्त करते थे तथा न्यायिक अधिकारी अपनी रिपोर्ट प्रस्तुत करते थे बादशाह सभी मामलों को सन्तोषजनक तरीके से निपटाते थे।

JAC Class 12 History Important Questions Chapter 9 शासक और इतिवृत्त : मुगल दरबार

प्रश्न 51.
शाहजहाँ के ‘रत्नजड़ित सिंहासन’ (तख्त- ए-मुरस्सा) पर एक संक्षिप्त टिप्पणी लिखिए।
उत्तर:
रत्नजड़ित सिंहासन की भव्य संरचना में एक छतरी थी जो द्वादशकोणीच स्तम्भों पर टिकी हुई थी। इसकी ऊँचाई सीढ़ियों से गुम्बद तक लगभग 10 फीट थी। बादशाह ने यह आदेश दिया कि 86 लाख रुपये के रत्न तथा बहुमूल्य पत्थर तथा चौदह लाख रुपये के मूल्य के एक लाख तोला सोने से इसे सुसज्जित किया जाना चाहिए। सिंहासन की साज-सज्जा में सात वर्ष लग गए। इसकी सजावट में प्रयुक्त हुए बहुमूल्य पत्थरों में रुबी था जिसका मूल्य एक लाख रुपये था।

प्रश्न 52.
मुगल दरबार में मनाए जाने वाले विशिष्ट उत्सवों का वर्णन कीजिए।
उत्तर:
सिंहासनारोहण की वर्षगाँठ, ईद, शब-ए-बारात तथा होली जैसे विशिष्ट अवसरों पर दरबार का वातावरण सजीव हो उठता था। सजे हुए डिब्बों में रखी सुगंधित मोमबत्तियाँ और महल की दीवारों पर लटक रहे रंग-बिरंगे बन्दनवार आने वाले लोगों को अत्यधिक प्रभावित करते थे। मुगल सम्राट वर्ष में तीन मुख्य त्यौहार मनाया करते थे-सूर्यवर्ष और चन्द्रवर्ष के अनुसार सम्राट का जन्म दिन और वसन्तागमन पर फारसी नववर्ष नौरोज

प्रश्न 53.
मुगल काल में शाही परिवारों में विवाहों के आयोजन का वर्णन कीजिए।
उत्तर:
मुगल काल में शाही परिवारों में विवाहों का आयोजन काफी खर्चीला होता था 1633 ई. में शाहजहाँ के ज्येष्ठ पुत्र दाराशिकोह का विवाह राजकुमार परवेज की पुत्री नादिरा से हुआ था। विवाह के उपहारों के प्रदर्शन की व्यवस्था दीवान-ए-आम में की गई थी। बादशाह तथा हरम की स्वियों दोपहर में इसे देखने के लिए आई। हिनाबन्दी (मेंहदी लगाना) की रस्म दीवान-ए-खास में अदा की गई। दरबार में उपस्थित लोगों के बीच पान, इलायची तथा मेवे बांटे गए।

प्रश्न 54.
शाहजहाँ के तख्त-ए-मुरस्सा के बारे में बादशाहनामा में क्या वर्णन किया गया है?
उत्तर:
आहरा महल के सार्वजनिक सभा भवन में रखे हुए शाहजहाँ के रत्नजड़ित सिंहासन के बारे में बादशाहनामा में निम्नलिखित प्रकार से वर्णन किया गया है-
सिंहासन की भव्य संरचना में एक छतरी है जो द्वादशकोणीय स्तम्भों पर टिकी हुई है। इसकी ऊँचाई सीढ़ियों से गुंबद तक पाँच क्यूबिट (लगभग 10 फुट) है। अपने राज्यारोहण के समय महामहिम ने यह आदेश दिया कि 86 लाख रुपए के रत्न तथा बहुमूल्य पत्थर और एक लाख तोला सोना जिसकी कीमत चौदह लाख रुपए है, से इसे सजाया जाना चाहिए। सिंहासन की साज-सज्जा में सात वर्ष लग गए। इसकी सजावट में प्रयुक्त हुए बहुमूल्य पत्थरों में रूबी था जिसकी कीमत एक लाख रुपए थी। इस रूबी पर महान बादशाह तिमूर साहिब-ए-किरान, मिर्जा शाहरुख, मिर्जा उलुग बेग और शाह अब्बास के साथ-साथ अकबर, जहाँगीर और स्वयं महामहिम के नाम अंकित थे।

JAC Class 12 History Important Questions Chapter 9 शासक और इतिवृत्त : मुगल दरबार

प्रश्न 55.
शाहजहाँ के ज्येष्ठ पुत्र दाराशिकोह के विवाह के आयोजन का वर्णन कीजिए।
उत्तर:
बादशाहनामा में दाराशिकोह के विवाह के आयोजन का वर्णन किया गया है। शाही परिवारों में विवाहों का आयोजन अत्यन्त धूम-धाम से किया जाता था। इसमें काफी धनराशि व्यय की जाती थी। 1633 ई. में दाराशिकोह और नादिरा, जो कि राजकुमार परवेज की पुत्री थी, के विवाह की व्यवस्था राजकुमारी जहाँआरा और दिवंगत महारानी मुमताज महल की प्रमुख नौकरानी सती उननिसाखानुम द्वारा की गई दीवान-ए-आम में उपहारों के प्रदर्शन की भव्य व्यवस्था की गई थी। हिनाबन्दी दीवाने खास में की गई। दरबार में उपस्थित व्यक्तियों के बीच पान, इलायची और मेवे बांटे गए। विवाह के इस आयोजन में तत्कालीन समय में 32 लाख रु. खर्च हुए। इनमें 6 लाख रु. शाही खजाने से, 16 लाख रु. जहाँआरा तथा शेष धनराशि दुल्हन की माँ के द्वारा दिए गए थे।

प्रश्न 56
मुगल परिवार में बेगमों, अगहा एवं अगाचा स्त्रियों की स्थिति का विवेचन कीजिए।
उत्तर:
मुगल परिवार में शाही परिवारों से आने वाली स्त्रियां बेगमें कहलाती थीं अगहा स्वियों ये थीं जिनका जन्म कुलीन परिवार में नहीं हुआ था। दहेज (मेहर) के रूप में विपुल धन नकद और बहुमूल्य वस्तुएँ लेने के बाद विवाह करके आई बेगमों को अपने पतियों से स्वाभाविक रूप से ‘अगहाओं’ की तुलना में अधिक ऊँचा दर्जा और सम्मान प्राप्त था। उपपत्नियों (अगाचा) की स्थिति सबसे निम्न थी। इन सभी को नकद मासिक भत्ता तथा अपने- अपने दर्जे के हिसाब से उपहार मिलते थे।

प्रश्न 57.
“राजपूत कुलों एवं मुगलों, दोनों के लिए विवाह राजनीतिक सम्बन्ध बनाने व मैत्री सम्बन्ध स्थापित करने का एक तरीका था।” व्याख्या कीजिए।
उत्तर:
विवाह में पुत्री को पेंटस्वरूप दिए जाने के साथ प्रायः एक क्षेत्र भी उपहार में दिया जाता थे। इससे विभिन्न शासक वर्गों के बीच पदानुक्रमिक सम्बन्धों की निरन्तरता निश्चित हो जाती थी इस प्रकार के विवाह और उनके फलस्वरूप विकसित सम्बन्धों के कारण ही मुगल शासक बन्धुता के एक व्यापक तन्त्र का निर्माण कर सके। इसके फलस्वरूप उनके महत्त्वपूर्ण वर्गों से घनिष्ठ सम्बन्ध स्थापित हुए और उन्हें एक विशाल साम्राज्य को सुरक्षित रखने में सहायता मिली।

JAC Class 12 History Important Questions Chapter 9 शासक और इतिवृत्त : मुगल दरबार

प्रश्न 58.
गुलबदन बेगम पर एक संक्षिप्त टिप्पणी लिखिए।
उत्तर:
गुलबदन बेगम बाबर की पुत्री, हुमायूँ की बहिन तथा अकबर की बुआ थी। उसने ‘हुमायूँनामा’ नामक पुस्तक लिखी जिससे हमें मुगलों की घरेलू दुनिया की एक झलक मिलती है। गुलबदन तुर्की तथा फारसी में धाराप्रवाह लिख सकती थी। गुलबदन ने जो लिखा वह मुगल बादशाहों की प्रशस्ति नहीं थी, बल्कि उसने राजाओं और राजकुमारों के बीच चलने वाले संघर्षों और तनावों का उल्लेख किया है। उसने परिवार की स्वियों द्वारा कुछ झगड़ों को सुलझाने का भी उल्लेख किया है।

प्रश्न 59.
“योग्य व्यक्तियों को पदवियाँ देना मुगल राजतन्त्र का एक महत्त्वपूर्ण पक्ष था।” विवेचना कीजिए।
अथवा
मुगल सम्राटों द्वारा अभिजात वर्ग के लोगों को प्रदान की जाने वाली पदवियों एवं पुरस्कारों का वर्णन कीजिए। उत्तर- ‘आसफ खाँ’ की पदवी उच्चतम मन्त्रियों को दी जाती थी। औरंगजेब ने अपने दो उच्चपदस्थ अभिजात जयसिंह और जसवन्त सिंह को ‘मिर्जा राजा’ की पदवी प्रदान की। योग्यता के आधार पर पदवियाँ अर्जित की जा सकती थीं कुछ मनसबदार धन देकर भी पदवियाँ प्राप्त करने का प्रयास करते थे। अन्य पुरस्कारों में सम्मान का ‘जामा’ (खिल्लत) भी शामिल था ‘सरप्पा’ एक अन्य उपहार था। इस उपहार के तीन भाग हुआ करते थे
(1) जामा
(2) पगड़ी तथा
(3) पटका।

प्रश्न 60.
दरबारियों एवं राजदूतों द्वारा मुगल बादशाहों को दिए जाने वाले उपहारों का वर्णन कीजिए।
उत्तर:
कोई भी दरबारी बादशाह के पास खाली हाथ नहीं जाता था। वह या तो नज के रूप में थोड़ा धन या पेशकश के रूप में मोटी रकम बादशाह की सेवा में प्रस्तुत करता था। राजनयिक सम्बन्धों में उपहारों को सम्मान और आदर का प्रतीक माना जाता था। राजदूत प्रतिद्वन्द्वी राजनीतिक शक्तियों के बीच सन्धि और सम्बन्धों के द्वारा समझौता करवाने का महत्त्वपूर्ण कार्य करते थे। ऐसी परिस्थितियों में उपहारों की महत्त्वपूर्ण प्रतीकात्मक भूमिका होती थी।

प्रश्न 61.
‘जात’ और ‘सवार’ मनसब से क्या अभिप्राय है?
उत्तर:
मुगलकाल में सभी सरकारी अधिकारियों के दर्जे और पर्दों में दो प्रकार के संख्या-विषयक पद होते थे-
(1) जात तथा
(2) सवार ‘जात’ शाही पदानुक्रम में अधिकारी (मनसबदार) के पद और वेतन का सूचक था। ‘सवार’ यह सूचित करता था कि उससे सेवा में कितने घुड़सवार रखना अपेक्षित था। सत्रहवीं शताब्दी में 1000 या उससे ऊपर जात वाले मनसबदार अभिजात ‘उमरा’ कहलाते थे।

JAC Class 12 History Important Questions Chapter 9 शासक और इतिवृत्त : मुगल दरबार

प्रश्न 62.
अभिजात वर्ग की सैनिक और असैनिक
कार्यों में क्या भूमिका थी?
उत्तर:
(1) अभिजात सैन्य अभियानों में अपनी सेनाओं के साथ भाग लेते थे।
(2) वे प्रान्तों में साम्राज्य के प्रशासनिक अधिकारियों के रूप में भी कार्य करते थे।
(3) प्रत्येक सैन्य कमांडर घुड़सवारों को भर्ती करता था। उन्हें हथियारों आदि से लैस करता था और उन्हें प्रशिक्षण देता था।
(4) घुड़सवार सेना मुगल सेना का अपरिहार्य अंग थी। घुड़सवार सैनिक उत्कृष्ट श्रेणी के घोड़े रखते थे।

प्रश्न 63,
“अभिजात वर्ग के सदस्यों के लिए शाही सेवा शक्ति, धन तथा उच्चतम प्रतिष्ठा प्राप्त करने का एक माध्यम थी।” स्पष्ट कीजिए।
उत्तर:
शाही सेवा में आने का इच्छुक व्यक्ति एक अभिजात के माध्यम से प्रार्थना-पत्र देता था जो बादशाह के सामने तजवीज प्रस्तुत करता था यदि प्रार्थी को सुयोग्य माना जाता था, तो उसे मनसब प्रदान किया जाता था। मीरबख्शी (उच्चतम वेतनदाता) खुले दरबार में बादशाह के दाएँ ओर खड़ा होता था तथा नियुक्ति और पदोन्नति के सभी उम्मीदवारों को प्रस्तुत करता था। उसका कार्यालय उसकी मुहर व हस्ताक्षर के साथ-साथ बादशाह की मुहर व हस्ताक्षर वाले आदेश तैयार करता था।

प्रश्न 64.
मुगलों के केन्द्रीय शासन पर एक संक्षिप्त टिप्पणी लिखिए।
उत्तर:
सम्राट अपने साम्राज्य का सर्वोच्च अधिकारी होता था साम्राज्य की समस्त शक्तियाँ उसी के हाथों में केन्द्रित थीं। शासन- कार्यों में सहायता व सलाह देने के लिए एक मन्त्रिपरिषद् होती थी। मीरबख्शी, दीवान-ए-आला तथा सद्र- उस सुदूर प्रमुख मन्त्री थे मीरबख्शी नियुक्ति और पदोन्नति के सभी उम्मीदवारों को प्रस्तुत करता था दीवान-ए-आला आय-व्यय का हिसाब रखता था तथा सद्र-उस-सुदूर अनुदान का मन्त्री और स्थानीय न्यायाधीशों की नियुक्ति का प्रभारी था।

प्रश्न 65.
“सटीक और विस्तृत आलेख तैयार करना मुगल प्रशासन के लिए मुख्य रूप से महत्त्वपूर्ण था। ” विवेचना कीजिए।
अथवा
मुगल दरबार की गतिविधियों से सम्बन्धित सूचनाएँ किस प्रकार लेखबद्ध की जाती थीं?
उत्तर:
मौरबख्शी दरबारी लेखकों (वाकिया नवीसों) के समूह का निरीक्षण करता था। ये लेखक ही दरबार में प्रस्तुत किए जाने वाले सभी अर्जियों एवं दस्तावेजों तथा सभी शासकीय आदेशों (फरमानों) का आलेख तैयार करते थे। इसके अतिरिक्त, अभिजातों और क्षेत्रीय शासकों के प्रतिनिधि (वकील), दरबार की बैठकों (पहर) की तिथि और समय के साथ ‘उच्च दरबार से समाचार’ (अखबारात- ए-दरबार-ए-मुअल्ला) शीर्षक के अन्तर्गत दरबार की सभी कार्यवाहियों का विवरण तैयार करते थे।

JAC Class 12 History Important Questions Chapter 9 शासक और इतिवृत्त : मुगल दरबार

प्रश्न 66.
अखबारात में मुगल दरबार से सम्बन्धित कौनसी सूचनाएँ होती थीं?
उत्तर:
अखबारात में हर प्रकार की सूचनाएँ होती थीं जैसे दरबार में उपस्थिति, पदों और पदवियों का दान, राजनयिक शिष्टमण्डलों, ग्रहण किए गए उपहारों अथवा किसी अधिकारी के स्वास्थ्य के बारे में बादशाह द्वारा की गई पूछताछ राजाओं और अभिजातों के सार्वजनिक और व्यक्तिगत जीवन का इतिहास लिखने के लिए ये सूचनाएँ बहुत उपयोगी होती थीं।

प्रश्न 67.
मुगल काल में साम्राज्य में समाचार वृत्तान्त और महत्त्वपूर्ण शासकीय दस्तावेज भेजे जाने की प्रक्रिया का वर्णन कीजिए।
उत्तर:
मुगल काल में समाचार वृत्तांत और महत्त्वपूर्ण शासकीय दस्तावेज शाही ठाक के द्वारा मुगल साम्राज्य में एक कोने से दूसरे कोने तक भेजे जाते थे। बांस के डिब्बों में लपेटकर रखे कागजों को लेकर हरकारों (कसीद अथवा पथमार) के दल दिन-रात दौड़ते रहते थे। काफी दूर स्थित प्रान्तीय राजधानियों से भी समाचार वृत्तान्त बादशाह को कुछ ही दिनों में मिल जाया करते थे। राजधानी से बाहर नियुक्त उच्च अधिकारी सूचनाएँ सन्देश वाहकों के द्वारा भेजते थे।

प्रश्न 68.
मुगलकालीन प्रान्तीय शासन का वर्णन कीजिए।
उत्तर:
मुगल साम्राज्य अनेक प्रान्तों में विभाजित था। प्रान्तीय शासन के संचालन के लिए सूबेदार, दीवान, बख्शी, सद्र आदि अधिकारी होते थे। प्रान्तीय शासन का प्रमुख गवर्नर (सूबेदार) होता था जो सीधा बादशाह को प्रतिवेदन प्रस्तुत करता था। प्रत्येक सूबा कई सरकारों में बंटा हुआ था। इन क्षेत्रों में फौजदारों को विशाल घुड़सवार सेना और तोपचियों के साथ नियुक्त किया जाता था। परगनों (उप- जिला) के प्रबन्ध के लिए कानूनगो, चौधरी तथा काजी नियुक्त किये जाते थे।

प्रश्न 69.
कन्धार सफावियों और मुगलों के बीच झगड़े की जड़ क्यों था? व्याख्या कीजिए।
उत्तर:
कन्धार ईरान के सफावियों एवं मुगलों के बीच झगड़े की जड़ था क्योंकि ईरान का शासक शाह अब्बास कन्धार पर अधिकार स्थापित करना चाहता था। 1613 में जहाँगीर ने शाह अब्बास के दरबार में कन्धार को मुगलों के आधिपत्य में रहने देने की वकालत करने के लिए एक राजनयिक दूत भेजा, परन्तु यह शिष्टमण्डल सफल नहीं हुआ। 1622 में एक ईरानी सेना ने कन्धार पर आक्रमण कर मुगल सेना को पराजित कर दिया और कन्धार के दुर्ग पर टि अधिकार कर लिया।

प्रश्न 70.
आटोमन साम्राज्य के साथ मुगलों के सम्बन्धों की विवेचना कीजिए।
उत्तर:
आरोमन साम्राज्य के साथ मुगलों ने अपने सम्बन्ध इस बात को ध्यान में रखकर बनाए कि वे आटोमन- नियन्त्रण वाले क्षेत्रों में व्यापारियों व तीर्थयात्रियों के स्वतन्त्र आवागमन को बनाये रखवा सकें मुगल बादशाह प्राय: धर्म एवं वाणिज्य के मुद्दों को मिलाने का प्रयास करता था। वह लाल सागर के बन्दरगाह अदन और मोरवा को बहुमूल्य वस्तुओं के निर्यात को प्रोत्साहन देता था और इनकी विक्री से प्राप्त आय को उस क्षेत्र के धर्म-स्थलों व फकीरों में दान में बाँट देता था।

JAC Class 12 History Important Questions Chapter 9 शासक और इतिवृत्त : मुगल दरबार

प्रश्न 71.
जेसुइट धर्म प्रचारकों के साथ मुगलों के सम्बन्धों की विवेचना कीजिए।
उत्तर:
अकबर ईसाई धर्म के विषय में जानने को बहुत उत्सुक था। उसके निमन्त्रण पर अनेक जेसुइट शिष्टमण्डल मुगल दरबार में आए। अकबर ने इन जेसुइट लोगों से ईसाई धर्म के विषय में वार्तालाप किया और उनसे बड़ा प्रभावित हुआ। जेसुइट लोगों को सार्वजनिक सभाओं में अकबर के सिंहासन के पास जगह दी जाती थी। वे उसके साथ अभियानों में जाते, उसके बच्चों को शिक्षा देते तथा उसके फुरसत के समय में वे प्रायः उसके साथ होते थे।

प्रश्न 72.
अकबर के धार्मिक विचारों के विकास का वर्णन कीजिए।
अथवा
अकबर की धार्मिक नीति का वर्णन कीजिये। उत्तर- 1575 में अकबर ने फतेहपुर सीकरी में इबादतखाने का निर्माण करवाया। उसने यहाँ इस्लाम, हिन्दू धर्म, ईसाई धर्म, जैन धर्म, पारसी धर्म के विद्वानों को आमन्त्रित किया। अकबर के धार्मिक विचार विभिन्न धर्मों व सम्प्रदायों के विद्वानों से प्रश्न पूछने और उनके धर्म-सिद्धान्तों के बारे में जानकारी एकत्रित करने से विकसित हुए। धीरे- धीरे वह धर्मों को समझने के रूढ़िवादी तरीकों से दूर प्रकाश और सूर्य पर केन्द्रित दैवीय उपासना की ओर बढ़ा।

निबन्धात्मक प्रश्न-

प्रश्न 1.
मुगल साम्राज्य के उत्कर्ष पर एक निबन्ध लिखिए।
अथवा
मुगल साम्राज्य के उत्थान व पतन का वर्णन कीजिए। उत्तर- मुगलों की उत्पत्ति ‘मुगल’ शब्द की उत्पत्ति मंगोल से हुई है। मुगलों ने स्वयं को ‘तैमूरी’ कहा क्योंकि पितृपक्ष से वे तुर्की शासक तिमूर के वंशज थे प्रथम मुगल शासक बाबर मातृपक्ष से चंगेज खाँ का सम्बन्धी था। उस समय से ही इस शब्द का निरन्तर प्रयोग होता रहा है।
मुगल साम्राज्य का उत्कर्ष
(1) जहीरुद्दीन बाबर जहीरुद्दीन बाबर मुगल- साम्राज्य का संस्थापक था उसे उसके प्रतिद्वन्द्वी उजबेकों ने उसके अपने देश फरगना से भगा दिया था। उसने सबसे पहले काबुल पर अधिकार किया। इसके बाद 1526 में अपने दल के सदस्यों की आवश्यकताएं पूरी करने के लिए क्षेत्रों और संसाधनों की खोज में वह भारतीय उपमहाद्वीप में और आगे की ओर बढ़ा। 1530 में बाबर की मृत्यु हो गई।

(2) हुमायूँ बाबर की मृत्यु के बाद 1530 में हुमायूँ गद्दी पर बैठा। उसने अपने शासनकाल (1530-40.1555- 56) में साम्राज्य की सीमाओं का विस्तार किया। परन्तु वह अफगान नेता शेरशाह सूरी से पराजित हो गया। उसे विवश होकर भारत से भागना पड़ा। उसे ईरान के सफावी शासक के दरबार में शरण लेनी पड़ी 1555 में हुमायूँ ने अफगानों को पराजित कर दिया। परन्तु एक वर्ष बाद ही उसकी मृत्यु हो गई।

(3) जलालुद्दीन अकबर हुमायूँ की मृत्यु के बाद उसका पुत्र जलालुद्दीन अकबर (1556-1605) गद्दी पर बैठा। वह मुगल राजवंश का सबसे महान शासक माना जाता है। अकबर ने हिन्दुकुश पर्वत तक अपने साम्राज्य की सीमाओं का विस्तार किया। उसने ईरान के सफ़ावियों और तूरान (मध्य एशिया) के उजबेकों की विस्तारवादी योजनाओं पर अंकुश लगाए रखा। 1605 में अकबर की मृत्यु हो गई।

JAC Class 12 History Important Questions Chapter 9 शासक और इतिवृत्त : मुगल दरबार

(4) अकबर के उत्तराधिकारी अकबर के बाद जहाँगीर (1605-27), शाहजहाँ (162858) तथा औरंगजेब (1658-1707) गद्दी पर बैठे ये तीनों ही बहुत योग्य उत्तराधिकारी सिद्ध हुए। इनके अधीन क्षेत्रीय विस्तार जारी रहा, यद्यपि इस विस्तार की गति काफी धीमी रही। उन्होंने शासन के विविध यन्त्रों को बनाए रखा और उन्हें सुदृढ़ किया।

(5) शाही संस्थाओं के ढाँचे का निर्माण इनके अन्तर्गत प्रशासन और कराधान के प्रभावशाली तरीके सम्मिलित थे। मुगलों द्वारा शुरू की गई राजनीतिक व्यवस्था सैन्य शक्ति और उपमहाद्वीप की भिन्न-भिन्न परम्पराओं को समायोजित करने की बुद्धिमत्तापूर्ण नीति पर आधारित थी।

(6) मुगल साम्राज्य का पतन 1707 में औरंगजेब की मृत्यु हो गई। उसकी मृत्यु के पश्चात् मुगलं साम्राज्य 1 का विघटन शुरू हो गया। फिर भी सांकेतिक रूप से मुगल साम्राज्य की प्रतिष्ठा बनी रही। 1857 में मुगल वंश के | अन्तिम शासक बहादुर शाह जफर द्वितीय को अंग्रेजों ने उखाड़ फेंका।

प्रश्न 2.
अबुल फजल के मुगल साम्राज्य में योगदान के विषय में आप क्या जानते हैं? विस्तार से समझाइए।
उत्तर:
अबुल फजल अकबर का एक अति महत्त्वपूर्ण दरबारी था। उसने अकबरनामा जैसे ग्रन्थों की रचना की थी। अबुल फजल का पालन-पोषण मुगल राजधानी आगरा हुआ था। वह अरबी तथा फारसी का अच्छा ज्ञाता था: साथ ही वह यूनानी दर्शन और सूफीवाद में भी रुचि रखता था अबुल फजल एक प्रभावशाली विचारक तथा तर्कशास्त्री था। उसने दकियानूसी उलेमाओं के विचारों का पूर्ण विरोध किया, जिसके कारण अकबर उससे अत्यधिक प्रभावित हुआ अकबर ने उसे अपने सलाहकार तथा अपनी नीतियों के प्रवक्ता के रूप में उपयुक्त पाया। अबुल फजल ने दरबारी इतिहासकार के रूप में अकबर के शासन से जुड़े विचारों को न केवल आकार दिया अपितु उनको स्पष्ट रूप से व्यक्त भी किया।

मुगल साम्राज्य में अबुल फजल का योगदान- अबुल फजल का महत्त्वपूर्ण योगदान अकबर के समय की ऐतिहासिक घटनाओं के वास्तविक विवरणों, शासकीय दस्तावेजों तथा जानकार व्यक्तियों के मौखिक प्रमाणों जैसे विभिन्न प्रकार के साक्ष्यों पर आधारित, अकबरनामा ग्रन्थ की रचना था। इस ग्रन्थ की रचना उसने 1589 ई. में आरम्भ की तथा इसे पूर्ण करने में उसे लगभग 13 वर्ष लगे। अबुल फजल का अकबरनामा तीन जिल्दों में है जिसमें प्रथम दो भाग इतिहास से सम्बन्धित हैं तथा तृतीय भाग आइन-ए-अकबरी है। प्रथम जिल्द में आदम से लेकर अकबर के जीवन के एक खगोलीय कालचक्र तक का मानव जाति का इतिहास है। द्वितीय जिल्द अकबर के 46वें शासन वर्ष पर खत्म होती है।

अकबरनामा का लेखन राजनीतिक रूप से महत्त्वपूर्ण पटनाओं का विवरण देने के उद्देश्य से किया गया। अकबर के साम्राज्य की तिथियों तथा समय के साथ होने वाले परिवर्तनों के उल्लेख के बिना ही भौगोलिक, सामाजिक, प्रशासनिक तथा सांस्कृतिक सभी पक्षों का विवरण प्रस्तुत करने के अभिन्न तरीके से भी इसका लेखन हुआ। अबुल फजल की आइन-ए-अकबरी में मुगल साम्राज्य को हिन्दुओं, जैनों, बौद्धों तथा मुसलमानों की भिन्न-भिन्न जनसंख्या वाले तथा एक मिश्रित संस्कृति वाले साम्राज्य के रूप में प्रस्तुत किया गया है। इससे भारतीय- फारसी शैली को दरबार में संरक्षण प्राप्त हुआ।

प्रश्न 3.
मुगलकाल में फारसी भाषा के विकास का वर्णन कीजिए।
अथवा
मुगल सम्राटों ने फारसी भाषा को महत्त्व क्यों प्रदान किया?
उत्तर:
मुगलकाल में फारसी भाषा का विकास मुगलों के दरबारी इतिहास फारसी भाषा में लिखे गए थे। दिल्ली के सुल्तानों के काल में उत्तर भारतीय भाषाओं के साथ-साथ फारसी दरवार तथा साहित्यिक रचनाओं की भाषा के रूप में खूब विकसित हुई। चूंकि मुगल चग़ताई मूल के थे, अतः तुर्की उनकी मातृभाषा थी प्रथम मुगल सम्राट बाबर ने कविताएँ और अपने संस्मरण तुर्की भाषा में ही लिखे थे।
(1) फारसी को दरबार की मुख्य भाषा बनाना- मुगल सम्राट अकबर ने फारसी को दरबार की मुख्य भाषा बनाया।

सम्भवतः ईरान के साथ सांस्कृतिक और बौद्धिक सम्पर्कों के साथ-साथ मुगल दरबार में पद पाने के इच्छुक ईरानी और मध्य एशियाई प्रवासियों ने बादशाह को फारसी भाषा को अपनाए जाने के लिए प्रेरित किया होगा। फारसी को दरबार की भाषा का ऊंचा स्थान दिया गया तथा उन लोगों को “शक्ति एवं प्रतिष्ठा प्रदान की गई जिनका इस भाषा पर अच्छा नियन्त्रण था। सम्राट शाही परिवार के लोग और दरबार के विशिष्ट सदस्य फारसी भाषा बोलते थे। कुछ और आगे, यह सभी स्तरों के प्रशासन की भाषा बन गई, जिससे लेखाकारों, लिपिकों तथा अन्य अधिकारियों ने भी इसे सीख लिया।

JAC Class 12 History Important Questions Chapter 9 शासक और इतिवृत्त : मुगल दरबार

(2) अन्य भाषाओं को प्रभावित करना- जहाँ-जहाँ फारसी प्रत्यक्ष प्रयोग में नहीं आती थी, वहाँ भी राजस्थानी, मराठी और यहाँ तक कि तमिल में शासकीय लेखकों की भाषा को फारसी की शब्दावली और मुहावरों ने व्यापक रूप से प्रभावित किया। चूंकि सोलहवीं तथा सत्रहवीं शताब्दियों के दौरान फारसी का प्रयोग करने वाले लोग उपमहाद्वीप के भिन्न-भिन्न क्षेत्रों से आए थे और वे अन्य भारतीय भाषाएँ भी बोलते थे, इसलिए स्थानीय मुहावरों को शामिल करने से फारसी का भी भारतीयकरण हो गया था। फारसी के हिन्दवी के साथ पारस्परिक सम्पर्क से उर्दू के रूप में एक नई भाषा का उद्भव हुआ।

(3) फारसी में अनुवाद ‘ अकबरनामा’ और ‘बादशाहनामा’ जैसे मुगल इतिहास फारसी में लिखे गए थे, जबकि बाबर के संस्मरणों का ‘बाबरनामा’ के नाम से तुर्की से फारसी में अनुवाद किया गया था। मुगल बादशाहों ने ‘महाभारत’ और ‘रामायण’ जैसे संस्कृत ग्रन्थों का फारसी में अनुवाद करवाया। ‘महाभारत’ का अनुवाद ‘रज्मनामा’ (युद्धों की पुस्तक) के रूप में हुआ।

प्रश्न 4.
“मुगलकाल में मानव रूपों का चित्रण निरन्तर तनाव का कारण था।” उपर्युक्त तर्क देकर इस कथन का औचित्य निर्धारित कीजिए।
उत्तर:
मुगलकाल में मानव रूपों का चित्रण निरन्तर तनाव का कारण था। इस कथन के पक्ष में निम्नलिखित तर्क प्रस्तुत हैं-
(i) मुगलकाल में बादशाह, उसके दरबार एवं उसमें भाग लेने वाले लोगों का चित्रण करने वाले चित्रों की रचना को लेकर शासकों और मुसलमानों, रूढ़िवादी वर्ग के प्रतिनिधियों अर्थात् उलेमाओं के मध्य लगातार तनाव बना रहा। उलेमाओं ने कुरान के साथ-सात हदीस जिसमें पैगम्बर मुहम्मद के जीवन से एक ऐसा ही प्रसंग वर्णित है, में प्रतिष्ठित मानव रूपों के चित्रण पर इस्लामी प्रतिबन्ध का आह्वान किया।

(ii) इस प्रसंग में पैगम्बर मुहम्मद को प्राकृतिक तरीके से जीवित रूपों के चित्रण की मनाही करते हुए उल्लिखित किया गया है। क्योंकि ऐसा करने में लगता है कि कलाकार रचना की शक्ति को अपने हाथ में लेने का प्रयास कर रहा है। यह एक ऐसा कार्य था, जो केवल ईश्वर का ही था।

(iii) समय के साथ-साथ शरिया की व्याख्याओं में भी परिवर्तन आया। विभिन्न सामाजिक समूहों ने इस्लामी परम्परा के ढाँचे की अलग-अलग तरीकों से व्याख्या की। प्रायः प्रत्येक समूह ने इस परम्परा की ऐसी व्याख्या प्रतिपादित की, जो उनकी राजनीतिक आवश्यकताओं में सबसे अधिक समानता रखती थी। जिन शताब्दियों के दौरान सामान्य निर्माण हो रहा था। उस समय कई चित्रकार स्रोतों के शासकों ने नियमित रूप से कलाकारों को उनके चित्र एवं उनके राज्य के जीवन के दृश्य चित्रित करने के लिए नियुक्त किया। मुगलकालीन भारत में ईरान से कई चित्रकार आए: जैसे—मीर सैयद अली एवं अब्दुल समद।

(iv) बादशाह एवं रूढ़िवादी मुसलमान विचारधारा के प्रवक्ताओं के मध्य मानवों के दृश्य निरूपण पर मुगल दरबार में तनाव बना हुआ था। अकबर का दरबारी इतिहासकार अबुल फजल बादशाह को यह कहते हुए उद्धृत करता है” कई लोग ऐसे हैं जो चित्रकला को नापसन्द करते हैं। पर मैं ऐसे व्यक्तियों को नापसन्द करता हूँ। मुझे ऐसा लगता है कि कलाकार के पास ख़ुदा को पहचानने का बेजोड़ तरीका है। क्योंकि कहीं न कहीं उसे ये महसूस होता है कि खुदा की रचना को वह जीवन नहीं दे सकता।”

(v) 17वीं शताब्दी में मुगल चित्रकारों ने बादशाहों को प्रभामण्डल के साथ चित्रित करना प्रारम्भ कर दिया। ईश्वर के प्रकाश के प्रतीक के रूप में इन प्रभामण्डलों को उन्होंने ईसा और वर्जिन मैरी के यूरोपीय चित्रों में देखा था।

प्रश्न 5.
मुगल पांडुलिपियों की रचना में चित्रकारों की भूमिका का वर्णन कीजिए। चित्रों के महत्त्व को स्पष्ट कीजिए। चित्रों की रचना को लेकर तनाव क्यों बना रहा? उत्तर- मुगल पांडुलिपियों की रचना में चित्रकारों की भूमिका मुगल पांडुलिपियों की रचना में चित्रकारों की महत्वपूर्ण भूमिका थी। किसी मुगल सम्राट के शासन की घटनाओं का विवरण देने वाले इतिहासों में लिखित पाठ के साथ ही उन घटनाओं को चित्रों के माध्यम से दृश्य रूप में भी वर्णित किया जाता था।
किसी
(1) चित्रों का महत्त्व –

  • सौन्दर्य में वृद्धि चित्र पुस्तक के सौन्दर्य में वृद्धि करते थे।
  • चित्रों के माध्यम से राजा की शक्ति को व्यक्त करना- लिखित माध्यम से राजा और राजा की शक्ति विषय में जो बात नहीं कही जा सकी हों, उन्हें चित्रों के माध्यम से व्यक्त कर दिया जाता था। के
  • निर्जीव वस्तुओं को सजीव बनाना इतिहासकार अबुल फजल ने चित्रकारी को एक ‘जादुई कला’ की संज्ञा दी है। उसके अनुसार चित्रकला निर्जीव वस्तुओं में भी प्राण फूंक सकती है।

(2) चित्रों की रचना को लेकर तनाव-मुगल सम्राट तथा उसके दरबार का चित्रण करने वाले चित्रों की रचना को लेकर शासकों और उलमा के बीच तनाव की स्थिति बनी रही। उलमा रूढ़िवादी मुसलमान वर्ग के प्रतिनिधि थे। उन्होंने कुरान के साथ-साथ हदीस में प्रतिष्ठापित मानव रूपों के चित्रण पर इस्लामी प्रतिबन्ध का आह्वान किया।

(3) नियमित रूप से कलाकारों को नियुक्त करना- कई एशियाई शासकों ने अपने तथा अपने राज्य के जीवन के दृश्य चित्रित करने के लिए नियमित रूप से कलाकारों को नियुक्त किया। उदाहरण के लिए, ईरान के सफावी राजाओं ने अपने दरबार की कार्यशालाओं में प्रशिक्षित एवं उत्कृष्ट कलाकारों को संरक्षण प्रदान किया। विहजाद जैसे चित्रकारों ने सफ़ावी दरबार की सांस्कृतिक प्रसिद्धि को चारों ओर फैलाने में महत्त्वपूर्ण योगदान दिया।

(4) ईरानी कलाकारों का मुगलकालीन भारत में आगमन – मुगलकाल में ईरानी कलाकार भी भारत में आए। ईरान के प्रसिद्ध चित्रकार मीर सैयद अली तथा अब्दुस समद बादशाह हुमायूँ के साथ दिल्ली आए। कुछ अन्य कलाकार संरक्षण और प्रतिष्ठा के अवसरों की तलाश में भारत पहुँचे। परन्तु बादशाह और रूढ़िवादी मुस्लिम विचारधारा के प्रवक्ताओं के बीच जीवधारियों के चित्र बनाने पर मुगल दरबार में तनाव की स्थिति बनी रही।

JAC Class 12 History Important Questions Chapter 9 शासक और इतिवृत्त : मुगल दरबार

प्रश्न 6.
अबुल फजल चित्रकला को महत्त्वपूर्ण क्यों मानता था? उसने अकबरकालीन चित्रकला की किन विशेषताओं का वर्णन किया है?
उत्तर:
चित्रकला का महत्त्व अबुल फसल निम्नलिखित आधारों पर चित्रकला को महत्त्वपूर्ण मानता था-
(1) चित्रकला एक जादुई कला इतिहासकार अबुल फजल ने चित्रकला को एक ‘जादुई कला’ की संज्ञा दी है उसके अनुसार चित्रकला किसी भी निर्जीव वस्तु में प्राण फूँक सकती है।

(2) मुगल सम्राटों की चित्रकला में रुचि अबुल फसल के अनुसार मुगल सम्राट अकबर ने अपनी युवावस्था के प्रारम्भिक दिनों से ही चित्रकला में अपनी अभिरुचि व्यक्त की थी। वह चित्रकला को अध्ययन और मनोरंजन दोनों का ही साधन मानते थे तथा इस कला को हर सम्भव प्रोत्साहन देते थे। मुगल पांडुलिपियों में चित्रों के अंकन के लिए चित्रकारों की एक बड़ी संख्या नियुक्त की गई थी। हर सप्ताह शाही कार्यालय के अनेक निरीक्षक और लिपिक बादशाह के सामने प्रत्येक कलाकार का कार्य प्रस्तुत करते थे। इन कृतियों के अवलोकन के बाद बादशाह प्रदर्शित उत्कृष्टता के आधार पर चित्रकारों को पुरस्कृत करते थे तथा उनके मासिक वेतन में वृद्धि करते थे।

(3) अकबरकालीन चित्रकला की विशेषताएँ- अकबरकालीन चित्रकला की विशेषताएँ निम्नलिखित थीं-

  • उत्कृष्ट चित्रकारों का उपलब्ध होना चित्रकारों को प्रोत्साहन दिए जाने के परिणामस्वरूप अकबर के समय में उत्कृष्ट चित्रकार मिलने लगे थे। बिहजाद जैसे चित्रकारों की सर्वोत्तम कलाकृतियों को तो उन यूरोपीय चित्रकारों की उत्कृष्ट कलाकृतियों के समक्ष रखा जा सकता था जिन्होंने विश्व में अत्यधिक ख्याति प्राप्त कर ली थी।
  • विवरण की सूक्ष्मता, परिपूर्णता तथा प्रस्तुतीकरण की निर्भीकता विवरण की सूक्ष्मता, परिपूर्णता तथा प्रस्तुतीकरण की निर्भीकता जो अकबरकालीन चित्रों में दिखाई पड़ती थी, वह अतुलनीय थी यहाँ तक कि निर्जीव वस्तुएँ भी सजीव प्रतीत होती थीं।
  • हिन्दू चित्रकारों की दक्षता अकबर के प्रोत्साहन के परिणामस्वरूप उसके दरबार में सौ से भी अधिक चित्रकार थे, जो इस कला की साधना में लगे हुए थे। हिन्दू कलाकारों के लिए यह बात विशेष तौर पर सही थी। उनके चित्र वस्तुओं की हमारी परिकल्पना से परे थे। वस्तुतः सम्पूर्ण विश्व में कुछ लोग ही उनकी तुलना कर सकते थे।

प्रश्न 7.
सोलहवीं और सत्रहवीं शताब्दी के दौरान मुगलों की राजधानियाँ तीव्रता से स्थानान्तरित होने लगीं। | व्याख्या कीजिए।
उत्तर:
राजधानी नगर मुगल साम्राज्य का केन्द्रीय स्थल होता था दरबार का आयोजन इसी केन्द्रीय स्थल राजधानी के नगर में किया जाता था मुगलों की राजधानियाँ सोलहवीं और सत्रहवीं शताब्दी में तीव्रता से एक नगर से दूसरे नगर में स्थानान्तरित होने लग। बाबर, जिसने कि मुगल साम्राज्य की स्थापना की थी, ने इब्राहीम लोदी के द्वारा स्थापित राजधानी आगरा पर अधिकार कर लिया था। अपने शासन काल के चार वर्षों में अपने दरबार भिन्न-भिन्न स्थानों पर लगाए। 1560 ई. में अकबर ने आगरा में किले का निर्माण करवाया, जिसे आस-पास की खदानों से लाए गए लाल बलुआ पत्थर से निर्मित किया गया था।

(1) फतेहपुर सीकरी – अकबर ने 1570 ई. के दशक में अपनी नई राजधानी फतेहपुर सीकरी में बनाने का निर्णय लिया। इसका कारण सम्भवतः यह माना जाता है कि अकबर प्रसिद्ध सूफी संत शेख मुईनुद्दीन चिश्ती का मुरीद था जिनकी दरगाह अजमेर में है। फतेहपुर सीकरी अजमेर जाने वाली सीधी सड़क पर स्थित था। अकबर ने फतेहपुर सीकरी में जुम्मा मस्जिद के निकट ही शेख सलीम चिश्ती के लिए संगमरमर के मकबरे का निर्माण भी करवाया। अकबर ने फतेहपुर सीकरी में ही एक विशाल मेहराबी प्रवेश द्वार: जिसे बुलन्द दरवाजा कहा जाता है, का निर्माण भी करवाया।

(2) लाहौर अकबर ने 1585 ई. में अपनी राजधानी को लाहौर स्थानान्तरित कर दिया। लाहौर में राजधानी के स्थानान्तरण का कारण उत्तरी-पश्चिमी सीमा पर प्रभावशाली नियन्त्रण स्थापित करना था।

(3) शाहजहांनाबाद में राजधानी की स्थापना- शाहजहाँ के पास भवन निर्माण के लिए व्यापक धन राशि का प्रबन्ध था। शाहजहाँ ने दिल्ली के प्राचीन रिहायशी नगर में शाहजहाँनाबाद के नाम से नई राजधानी का निर्माण किया। शाहजहांनाबाद एक नई शाही आबादी के रूप में विकसित हुआ। मुख्य बात तो यह है कि शाहजहाँ की पुत्री जहाँआरा ने इस नई राजधानी की कई वास्तुकलात्मक संरचनाओं की रूपरेखा निर्मित की: जिनमें चाँदनी चौक और कारवाँ सराय नामक दोमंजिली इमारत प्रमुख थी।

प्रश्न 8.
अकबरनामा के ऐतिहासिक महत्त्व का विवेचन कीजिए।
अथवा
‘अकबरनामा’ पर एक लघु निबन्ध लिखिए।
अथवा
‘अबुल फजल’ और ‘अकबरनामा’ का परिचय दीजिए।
उत्तर:
अबुल फजल का परिचयं अबुल फसल मुगल सम्राट अकबर का दरबारी इतिहासकार था। अबुल फजल का पालन पोषण मुगल राजधानी आगरा में हुआ था वह अरबी, फारसी, यूनानी दर्शन और सूफीवाद का प्रकाण्ड विद्वान् था। वह एक प्रभावशाली तर्कशील व्यक्ति तथा स्वतन्त्र चिन्तक था। उसने आजीवन रूढ़िवादी उलमा के विचारों का विरोध किया। अबुल फसल के इन गुणों से अकबर बहुत प्रभावित हुआ। दरबारी इतिहासकार के रूप में अबुल फजल ने अकबर के शासन से जुड़े विचारों को न केवल आकार प्रदान किया, बल्कि उनको स्पष्ट रूप से व्यक्त भी किया।

JAC Class 12 History Important Questions Chapter 9 शासक और इतिवृत्त : मुगल दरबार

‘अकबरनामा’
‘अकबरनामा’ महत्त्वपूर्ण चित्रित मुगल इतिहासों में सर्वाधिक प्रसिद्ध है। इसमें लगभग 150 पूरे अथवा दोहरे पृष्ठों पर लड़ाई, घेराबन्दी, शिकार, भवन-निर्माण, दरबारी दृश्य आदि के चित्र हैं। अबुल फजल ने 1589 में शुरू कर ‘ अकबरनामा’ पर तेरह वर्षों तक कार्य किया और इस दौरान उसने कई बार इसके प्रारूप में सुधार किया। यह इतिहास घटनाओं के वास्तविक विवरणों, शासकीय दस्तावेजों तथा जानकार व्यक्तियों के मौखिक प्रमाणों पर आधारित है।
(1) तीन जिल्दों में विभक्त’ अकबरनामा’ को तीन जिल्दों में विभाजित किया गया है, जिनमें से प्रथम दो जिल्दें इतिहास हैं। तीसरी जिल्द ‘आइन-ए-अकबरी’ है।

  • पहली जिल्द पहली जिल्द में आदम से लेकर अकबर के जीवन के एक खगोलीय कालचक्र तक (30 वर्ष) का मानव जाति का इतिहास है।
  • दूसरी जिल्द दूसरी जिल्द अकबर के 46 वें शासन वर्ष (1601 ई.) पर समाप्त होती है। 1602 में राजकुमार सलीम ने अबुल फजल की हत्या का एक षड्यन्त्र रचा जिसके अनुसार सलीम के सहापराधी बीरसिंह बुन्देला ने ‘अबुल फसल की हत्या कर दी।
  • तीसरी जिल्द-तीसरी जिल्द ‘आइन-ए-अकबरी’ है। इसमें मुगल साम्राज्य को हिन्दुओं, जैनियों, बौद्धों और मुसलमानों की भिन्न-भिन्न आबादी वाले तथा एक मिश्रित संस्कृति वाले साम्राज्य के रूप में प्रस्तुत किया गया है।

(2) ‘अकबरनामा’ का ऐतिहासिक महत्त्व इसमें तिथियों और समय के साथ होने वाले परिवर्तनों के उल्लेख के बिना ही अकबर साम्राज्य के भौगोलिक, सामाजिक, संजीव पास बुक्स प्रशासनिक और सांस्कृतिक सभी पक्षों का विवरण प्रस्तुत किया गया है।

(3) अबुल फजल की भाषा अबुल फजल की भाषा बहुत अलंकृत थी इस भारतीय फारसी शैली को मुगल दरबार में संरक्षण प्राप्त हुआ।

प्रश्न 9.
मुगल साम्राज्य के सूचना तन्त्र का वर्णन कीजिए।
उत्तर:
मुगल साम्राज्य में व्यापक सूचना तन्त्र विकसित था साम्राज्य से सम्बन्धित सभी क्रिया-कलापों का व्यापक आलेखन किया जाता था। सभी महत्त्वपूर्ण सूचनाओं को दर्ज किया जाता था। लेखन के लिए दरबारी लेखकों, जिन्हें वाकया नवीस कहा जाता था की नियुक्ति की गयी थी। मीर बख्शी इन लेखकों के कार्यों का निरीक्षण करता था। वाकया नवीस ही दरबार में प्रस्तुत की जाने वाली अर्जियों, दस्तावेजों और शासकीय आदेशों को लिपिबद्ध करते थे।

इसके अतिरिक्त अखबारात ए- दरबारी )- मुअल्ला उच्च दरबार से समाचार शीर्षक के अन्तर्गत ओभजातों और क्षेत्रीय शासकों के प्रतिनिधि दरबार की सभी कार्यवाहियों का विवरण तैयार करते थे। इन अखवारों में सभी प्रकार की सूचनाओं: जैसे- दरबार में उपस्थित लोगों का विवरण, पदों और पदवियों का दान, राजनयिक शिष्टमण्डलों से जुड़ी हुई सूचनाएँ, ग्रहण तथा दान किए गए उपहारों आदि का दिनांक और समय के साथ लेखन किया जाता था। यह सूचनाएँ राजाओं के सार्वजनिक और व्यक्तिगत जीवन का इतिहास लिखने के लिए महत्त्वपूर्ण स्रोत होती हैं।

सूचनाओं को एक स्थान से दूसरे स्थान पर भेजने के लिए पूरा साम्राज्य एक तन्त्र से जुड़ा हुआ था। समाचार वृत्तान्त और महत्त्वपूर्ण शासकीय दस्तावेज डाक के जरिए बाँस के डिब्बे में लपेटकर हरकारों के दलों द्वारा भेजे जाते थे। हरकारों के यह दल क्रमबद्ध रूप से दिन-रात दौड़ते रहते थे। इस प्रकार बादशाह को राजधानी से काफी दूर स्थित प्रान्तों के समाचार कुछ ही दिन में मिल जाते थे। प्रान्तों में राजधानी के अभिजातों के प्रतिनिधि दरवार द्वारा माँगी गई जानकारी को एकत्र कर सन्देशवाहकों के जरिए अपने अधिकारी के पास भेज दिया करते थे। विस्तृत मुगल साम्राज्य के सुचारु संचालन हेतु यह सूचना तन्त्र अति आवश्यक था।

JAC Class 12 History Important Questions Chapter 9 शासक और इतिवृत्त : मुगल दरबार

प्रश्न 10.
‘सुलह-ए-कुल’ के सिद्धान्त की विवेचना कीजिए। इसे किस प्रकार लागू किया गया?
उत्तर:
‘सुलह-ए-कुल’ के सिद्धान्त मुगल इतिवृत्त मुगल साम्राज्य को हिन्दुओं, जैनों, जरतुश्तियों और मुसलमानों के अनेक नृजातीय और धार्मिक समुदायों के समूह के रूप में प्रस्तुत करते हैं। शान्ति और स्थायित्व के स्त्रोत के रूप में मुगल सम्राट सभी धार्मिक और नृजातीय समूहों से ऊपर होता था वह इनके बीच मध्यस्थता करता था तथा यह सुनिश्चित करता था कि न्याय और शान्ति बनी रहे।
(1) प्रबुद्ध शासन का आधार अबुल फजल सुलाह- ए-कुल (पूर्ण शान्ति) के आदर्श को प्रबुद्ध शासन का आधार बताता है।
(2) अभिव्यक्ति की स्वतन्त्रता-‘सुलह-ए-कुल’ में सभी धर्मों और मतों को अभिव्यक्ति की स्वतन्त्रता थी।
(3) राज्य सत्ता को क्षति नहीं पहुँचाना-किन्तु उसकी एक शर्त थी कि वे राज्य सत्ता को क्षति नहीं पहुँचायेंगे अथवा आपस में नहीं लड़ेंगे।

सुलह-ए-कुल के आदर्श को लागू करना-
(1) अभिजात वर्ग को पद और पुरस्कार देना- सुलह-ए-कुल का आदर्श राज्य नीतियों के द्वारा लागू किया गया। मुगलों के अधीन अभिजात वर्ग में ईरानी, तूरानी, अफगानी राजपूत, दक्खनी सभी सम्मिलित थे। इन सब को पद और पुरस्कार दिए गए जो पूरी तरह से राजा के प्रति उनकी सेवा और निष्ठा पर आधारित थे।

(2) तीर्थयात्रा कर तथा जजिया समाप्त करना- 1563 में अकबर ने तीर्थयात्रा कर तथा 1564 में जजिया को समाप्त कर दिया क्योंकि ये दोनों कर धार्मिक पक्षपात पर आधारित थे।

(3) प्रशासन में सुलह-ए-कुल के नियमों का अनुपालन करवाना साम्राज्य के अधिकारियों को आदेश दिए गए कि ये प्रशासन में सुलह-ए-कुल’ के नियमों का अनुपालन करवाएं।

(4) सहिष्णुतापूर्ण धार्मिक नीति- अधिकांश मुगल बादशाहों ने सहिष्णुतापूर्ण धार्मिक नीति अपनाई। उन्होंने उपासना स्थलों के निर्माण व रख-रखाव के लिए अनुदान दिए। यहाँ तक कि युद्ध के दौरान जब मन्दिर नष्ट कर दिए जाते थे, तो बाद में सम्राटों द्वारा उनकी मरम्मत के लिए अनुदान जारी किए जाते थे। ऐसा हमें शाहजहाँ और औरंगजेब के शासन में पता चलता है, यद्यपि औरंगजेब के शासन काल में गैर-मुसलमान प्रजा पर जजिया फिर से लगा दिया गया।

प्रश्न 11.
मुगलकालीन राजधानी नगरों का वर्णन कीजिए।
उत्तर:
मुगलकालीन राजधानी नगर मुगल राजधानी नगर मुगल साम्राज्य का हृदय स्थल था। राजधानी नगर में ही दरबार लगता था। सोलहवीं और सत्रहवीं शताब्दियों के दौरान मुगलों की राजधानी तेजी से स्थानान्तरित होने लगीं। 1560 के दशक में अकबर ने आगरा के किले का निर्माण करवाया। इसे लाल बलुआ पत्थर से बनाया गया था। यह बलुआ पत्थर आस-पास के क्षेत्रों की खदानों से लाया गया था।

(1) फतेहपुर सीकरी – 1570 के दशक में अकबर ने फतेहपुर सीकरी में एक नई राजधानी बनाने का निर्णय लिया। मुगल सम्राटों के चिश्ती सिलसिले के सूफियों के साथ घनिष्ठ सम्बन्ध स्थापित हो गए थे। अकबर ने फतेहपुर सीकरी में जुम्मा मस्जिद के बगल में ही शेख सलीम चिश्ती के लिए सफेद संगमरमर का एक मकबरा बनवाया। उसने एक विशाल मेहराबी प्रवेशद्वार अथवा बुलन्द दरवाजा भी बनवाया जिसका उद्देश्य वहाँ आने वाले लोगों को गुजरात में मुगल विजय की याद दिलाना था।

(2) लाहौर 1585 ई. में उत्तर-पश्चिमी सीमा पर प्रभावशाली नियन्त्रण स्थापित करने के लिए राजधानी को लाहौर स्थानान्तरित कर दिया गया। इस प्रकार तेरह वर्षों तक अकबर ने इस सीमा पर गहरी चौकसी बनाए रखी।

JAC Class 12 History Important Questions Chapter 9 शासक और इतिवृत्त : मुगल दरबार

(3) शाहजहाँ की भवन निर्माण में गहरी रुचि- शाहजहाँ की भवन निर्माण में गहरी रुचि थी। उसने विवेकपूर्ण राजकोषीय नीतियों को आगे बढ़ाया तथा भवन निर्माण की अपनी अभिरुचि की पूर्ति के लिए पर्याप्त धन इकट्ठा कर लिया। राजकीय संस्कृतियों में भवन निर्माण कार्य राजवंशीय सत्ता, धन तथा प्रतिष्ठा का सर्वाधिक ठोस प्रतीक था मुसलमान शासकों के संदर्भ में इसे धर्मनिष्ठा के एक कार्य के रूप में भी देखा जाता था।

(4) नई राजधानी शाहजहाँनाबाद की स्थापना- 1648 में शाहजहाँ के शासनकाल में दरबार, सेना व राजसी खानदान आगरा से नई निर्मित शाही राजधानी शाहजहाँनाबाद (दिल्ली) चले गए। दिल्ली के प्राचीन रिहायशी नगर में स्थित शहजहांनाबाद एक नई और शाही आबादी थी। यहाँ लालकिला, जामा मस्जिद, चाँदनी चौक के बाजार की वृक्ष-वीथि और अभिजात वर्ग के बड़े-बड़े पर थे। शाहजहाँ द्वारा स्थापित यह नया शहर विशाल एवं भव्य मुगल राजतंत्र का प्रतीक था।

प्रश्न 12.
“मुगल दरबार का केन्द्र बिन्दु राजसिंहासन अथवा तख्त था जिसने सम्प्रभु के कार्यों को भौतिक स्वरूप प्रदान किया था।” व्याख्या कीजिए।
अथवा
मुगल दरबार में अभिजात वर्गों के बीच हैसियत को निर्धारित करने वाले नियमों, अभिवादन के तरीकों एवं दरबार में मनाए जाने वाले त्यौहारों का वर्णन कीजिए।
उत्तर:
मुगल दरबार बादशाह पर केन्द्रित दरबार की भौतिक व्यवस्था ने बादशाह के अस्तित्व को समाज के हृदय के रूप में दर्शाया। दीर्घकाल से भारत में राजतन्त्र का प्रतीक रही छतरी, बादशाह की चमक को सूर्य की चमक से अलग करने वाली मानी जाती थी।
(1) अभिजात वर्गों के बीच हैसियत को निर्धारित करने वाले नियम- मुगल इतिवृत्तों में मुगल अभिजात वर्गों के बीच हैसियत को निर्धारित करने वाले नियमों का वर्णन किया गया है।

दरबार में किसी दरबारी की हैसियत इस बात से निर्धारित होती थी कि वह बादशाह के कितना पास और दूर बैठा है। एक बार जब बादशाह सिंहासन पर बैठ जाता था तो किसी भी दरबारी को अपने स्थान से कहीं और जाने की अनुमति नहीं थी और न ही कोई अनुमति के बिना दरबार से बाहर जा सकता था।

(2) बादशाह को किए गए अभिवादन के तरीके से अभिजात वर्ग की हैसियत का बोध होना-मुगल दरबार में बादशाह को किए गए अभिवादन के तरीके से अभिजात- वर्ग के व्यक्ति की हैसियत का बोध होता था। जिस व्यक्ति के सामने अधिक झुक कर अभिवादन किया जाता था, उस व्यक्ति की हैसियत अधिक ऊँची मानी जाती थी। अभिवादन का उच्चतम रूप ‘सिजदा’ या दंडवत् लेटना था।

(3) राजनयिक दूतों से सम्बन्धित नयाचार- मुगल- सम्राट के सामने प्रस्तुत होने वाले राजदूत से यह अपेक्षा की जाती थी कि वह अभिवादन के मान्य रूपों में से एक या तो बहुत झुककर अथवा जमीन को चूमकर अथवा फारसी रिवाज के अनुसार छाती के सामने हाथ बाँधकर तरीके से अभिवादन करेगा।

(4) झरोखा दर्शन – मुगल बादशाह अपने दिन की शुरुआत सूर्योदय के समय कुछ व्यक्तिगत धार्मिक प्रार्थनाओं से करता था। इसके बाद वह पूर्व की ओर मुँह किए एक छोटे छज्जे अर्थात् झरोखे में आता था। इसके नीचे लोगों को एक भारी भीड़ बादशाह की एक झलक पाने के लिए प्रतीक्षा करती थी। इस प्रथा का उद्देश्य जन विश्वास के रूप में शाही सत्ता की स्वीकृति का और विस्तार करना था।

(5) दीवान-ए-आम एवं दीवान-ए-खास-झरोखे में एक घंटा बिताने के बाद बादशाह सार्वजनिक सभा- भवन (दीवान-ए-आम) में आता था। वहाँ राज्य के अधिकारी रिपोर्ट तथा प्रार्थना-पत्र प्रस्तुत करते थे दो घंटे बाद बादशाह दीवान-ए-खास में निजी सभाएँ करता था तथा गोपनीय मामलों पर चर्चा करता था।

(6) दरबार का जीवन्त वातावरण सिंहासनारोहण की वर्षगाँठ, ईद, शब-ए-बारात तथा होली जैसे कुछ विशिष्ट अवसरों पर दरबार का वातावरण जीवन्त हो उठता था सुसज्जित डिब्बों में रखी सुगंधित मोमबत्तियाँ और महल की दीवारों पर लटक रहे रंग-बिरंगे बंदनवार आने वाले लोगों को आश्चर्यजनक रूप से प्रभावित करते थे।

JAC Class 12 History Important Questions Chapter 9 शासक और इतिवृत्त : मुगल दरबार

(7) विवाह मुगल काल में शाही परिवारों में विवाहों का आयोजन काफी खर्चीला होता था। 1633 ई. में शाहजहाँ के ज्येष्ठ पुत्र दारा शिकोह का विवाह राजकुमार परवेज की पुत्री नादिरा से हुआ। विवाह के उपहारों के प्रदर्शन की व्यवस्था ‘दीवान-ए-आम’ में की गई थी। हिनाबन्दी (मेहंदी लगाना) की रस्म दीवान-ए-खास में अदा की गई। दरबार में उपस्थित लोगों के बीच पान, इलायची तथा मेवे वाटे गए। इस विवाह पर कुल 32 लाख रुपये खर्च हुए थे।

प्रश्न 13.
मुगल सम्राटों द्वारा अभिजात वर्ग को प्रदान की जाने वाली पदवियों और उपहारों का वर्णन कीजिए। दरबारियों और राजदूतों द्वारा बादशाहों को उपहार देने का क्या महत्त्व था?
उत्तर:
मुगल सम्राटों द्वारा अभिजात वर्ग को प्रदान की जाने वाली पदवियाँ तथा उपहार
राज्याभिषेक के समय अथवा किसी शत्रु पर विजय के. बाद मुगल बादशाह विशाल पदवियाँ ग्रहण करते थे। जब उद्घोषक (नकीब) इन पदवियों की घोषणा करते थे, तो दरबार में आश्चर्य का वातावरण बन जाता था मुगल सिक्कों पर राजसी नयाचार के साथ सत्तारूढ़ बादशाह की पूरी पदवी होती थी।

(1) अभिजात वर्ग को पदवियाँ, पुरस्कार एवं उपहार देना योग्य व्यक्तियों को पदवियाँ देना मुगल- राजतन्त्र का एक महत्त्वपूर्ण पक्ष था दरबारी पदानुक्रम में किसी व्यक्ति की उन्नति को उसके द्वारा धारण की जाने वाली पदवियों से जाना जा सकता था। ‘आसफखों की पदवी उच्चतम मन्त्रियों में से एक को दी जाती थी। औरंगजेब ने अपने दो उच्च पदस्थ अभिजातों-जयसिंह और जसवन्त सिंह को ‘मिज राजा’ की पदवी प्रदान की। योग्यता के आधार पर पदवियाँ या तो अर्जित की जा सकती थीं अथवा इन्हें प्राप्त करने के लिए धन दिया जा सकता था। मीरखान नामक एक मनसबदार ने अपने नाम में अलिफ अर्थात् ‘अ’ अक्षर लगाकर उसे अमीरखान करने के लिए औरंगजेब को एक लाख रुपये देने का प्रस्ताव किया।

( 2) ‘ सम्मान का जामा’ (खिल्लत) – अन्य पुरस्कारों में सम्मान का जामा ( खिल्लत) भी शामिल था जिसे पहले कभी-न-कभी बादशाह द्वारा पहना गया होता था। इसलिए यह समझा जाता था कि वह बादशाह के आशीर्वाद का प्रतीक है।

(3) सरप्पा (सर से पाँव तक )- बादशाह द्वारा अभिजातों को प्रदान किये जाने वाले एक अन्य उपहार

‘सरप्पा’ (सर से पाँव तक) था इस उपहार के तीन भाग हुआ करते थे-
(1) जामा
(2) पगड़ी तथा
(3) पटका।
(4) रत्नजड़ित आभूषण बादशाह द्वारा अभिजात- वर्ग के लोगों को प्रायः रत्नजड़ित आभूषण भी उपहार के रूप में दिए जाते थे। विशिष्ट परिस्थितियों में बादशाह कमल की मंजरियों वाला रत्नजड़ित गहनों का सेट (पद्ममुरस्सा) भी उपहार में प्रदान करता था।

(5) दरबारियों एवं राजदूतों द्वारा बादशाह की सेवा में उपहार प्राप्त करना कोई भी दरबारी बादशाह के पास कभी खाली हाथ नहीं जाता था। वह या तो नज़ के रूप में थोड़ा धन या पेशकश के रूप में मोटी रकम बादशाह की सेवा में प्रस्तुत करता था। राजनयिक सम्बन्धों में उपहारों को सम्मान और आदर का प्रतीक माना जाता था। राजदूत प्रतिद्वन्द्वी राजनीतिक शक्तियों के बीच सन्धि और सम्बन्धों के द्वारा समझौता करवाने के महत्त्वपूर्ण कार्य करते थे। ऐसी परिस्थितियों में उपहारों की महत्त्वपूर्ण प्रतीकात्मक भूमिका होती थी। अंग्रेज राजदूत सर टामस रो ने आसफ खाँ को एक अंगूठी भेंट की थी, परन्तु वह उसे केवल इसलिए वापस कर दी गई कि वह मात्र चार सौ रुपये मूल्य की थी।

JAC Class 12 History Important Questions Chapter 9 शासक और इतिवृत्त : मुगल दरबार

प्रश्न 14.
“मुगल साम्राज्य में शाही परिवार की स्त्रियों द्वारा निभाई गई भूमिका अत्यन्त महत्त्वपूर्ण थी। ” विवेचना कीजिए।
उत्तर:
(1) मुगल परिवार मुगल परिवार में बादशाह की पत्नियाँ और उपपत्नियाँ, उसके नजदीकी और दूर के सम्बन्धी (माता, सौतेली व उपमाताएँ, बहन, पुत्री, बहू, चाची-मौसी, बच्चे आदि) महिला परिचारिकाएँ तथा गुलाम होते थे। शासक वर्ग में बहुविवाह प्रथा व्यापक रूप से प्रचलित थी।

(2) अगहा तथा अगाचा मुगल परिवार में शाही परिवारों से आने वाली स्त्रियों (बेगमों) और अन्य स्वियों (अगहा) जिनका जन्म कुलीन परिवार में नहीं हुआ था, में अन्तर रखा जाता था। दहेज (मेहर) के रूप में विपुल नकद और बहुमूल्य वस्तुएँ लाने वाली बेगमों को अपने पतियों से स्वाभाविक रूप से ‘अगहाओं’ की तुलना में अधिक ऊंचा दर्जा और सम्मान मिलता था।

(3) वंश आधारित पारिवारिक ढाँचे का स्थायी नहीं होना – वंश आधारित पारिवारिक ढाँचा पूरी तरह स्थायी नहीं था।

(4) गुलाम- पत्नियों के अतिरिक्त मुगल परिवार में अनेक महिला तथा पुरुष गुलाम होते थे वे साधारण से साधारण कार्य से लेकर कौशल, निपुणता तथा बुद्धिमत्ता के अलग-अलग कार्य करते थे गुलाम हिजड़े (ख्वाजासर) परिवार के अन्दर और बाहर के जीवन में रक्षक, नौकर तथा व्यापार में रुचि रखने वाली महिलाओं के एजेन्ट होते थे।

(5) शाही परिवार की स्त्रियों का वित्तीय संसाधनों पर नियंत्रण – नूरजहाँ के बाद मुगल रानियों और राजकुमारियों ने महत्त्वपूर्ण वित्तीय स्रोतों पर नियंत्रण रखना शुरू कर दिया। शाहजहाँ की पुत्रियों जहाँआरा तथा रोशनआरा को ऊंचे शाही मनसबदारों के समान वार्षिक आय होती थी। इसके अतिरिक्त जहाँआरा को सूरत के बन्दरगाह नगर से राजस्व प्राप्त होता था।

(6) निर्माण कार्यों में भाग लेना मुगल परिवार की महत्त्वपूर्ण स्वियों ने इमारतों व बागों का निर्माण भी करवाया। जहाँआरा ने शाहजहाँ की नई राजधानी शाहजहांनाबाद (दिल्ली) की अनेक वास्तुकलात्मक परियोजनाओं में भाग लिया। इनमें से एक दो मंजिली भव्य इमारत कारवां सराय थी, जिसमें एक आँगन व बाग भी था। शाहजहाँनाबाद के मुख्य केन्द्र चाँदनी चौक की रूपरेखा भी जहाँआरा द्वारा बनाई गई थी।

(7) लेखन कार्य में योगदान गुलबदन बेगम एक उच्च कोटि की लेखिका थी वह बाबर की पुत्री हुमायूँ की बहिन तथा अकबर की बुआ थी वह स्वयं तुर्की तथा फारसी में धारा प्रवाह लिख सकती थी। उसने ‘हुमायूँनामा’ नामक पुस्तक लिखी जिससे हमें मुगलों की घरेलू दुनिया की एक झलक मिलती है।

प्रश्न 15.
कन्धार, आटोमन साम्राज्य तथा जैसुइट धर्म प्रचारकों के साथ मुगल सम्राटों के सम्बन्धों का विवेचन कीजिए।
उत्तर:
(1) कन्धार के साथ मुगल सम्राटों के सम्बन्ध-कन्धार ईरान के सफावियों और मुगलों के बीच झगड़े की जड़ था। कन्धार का किला नगर आरम्भ में हुमायूँ के आधिकार में था। 1595 में अकबर ने कन्धार पर पुनः अधिकार कर लिया था। यद्यपि ईरान के शासक शाह अब्बास ने मुगलों के साथ अपने राजनयिक सम्बन्ध बनाए रखे तथापि कन्धार पर वह अपना दावा करता रहा।

1613 में जहाँगीर ने शाह अब्बास के दरबार में कन्धार को मुगलों के आधिपत्य में रहने देने की वकालत करने के लिए एक राजनयिक दूत भेजा, परन्तु यह शिष्टमण्डल अपने उद्देश्यों में सफल नहीं हुआ। 1622 में एक ईरानी सेना ने कन्धार पर घेरा डाल दिया। यद्यपि मुगल सेना ने ईरानियों का मुकाबला किया, परन्तु उसे पराजय का मुंह देखना पड़ा। इस प्रकार कन्धार के किले तथा नगर पर ईरानियों का अधिकार हो गया।

(2) आटोमन साम्राज्य के साथ मुगलों के सम्बन्ध- आटोमन साम्राज्य के साथ मुगलों ने अपने सम्बन्ध इस बात को ध्यान में रखकर बनाए कि वे आटोमन नियन्त्रण वाले क्षेत्रों में व्यापारियों व तीर्थयात्रियों के स्वतंत्र आवागमन को बनाये रखवा सकें। इस क्षेत्र के साथ अपने सम्बन्धों में मुगल बादशाह, प्राय: धर्म एवं वाणिज्य के मुद्दों को मिलाने का प्रयास करता था। वह लाल सागर के बन्दरगाह अदन और मोरवा को बहुमूल्य वस्तुओं के निर्यात को प्रोत्साहन देता था और इनकी बिक्री से प्राप्त आय को उस क्षेत्र के धर्मस्थलों व फकीरों में दान में बाँट देता था।

JAC Class 12 History Important Questions Chapter 9 शासक और इतिवृत्त : मुगल दरबार

(3) जैसुइट धर्म प्रचारकों के साथ मुगलों के सम्बन्ध-पन्द्रहवीं शताब्दी के अन्त में पुर्तगाली व्यापारियों ने तटीय नगरों में व्यापारिक केन्द्रों का जाल स्थापित किया। पुर्तगाल का सम्राट भी ‘सोसाइटी ऑफ जीसस’ (जेसुइट ) के धर्म प्रचारकों की सहायता से ईसाई धर्म का प्रचार-प्रसार करना चाहता था। सोलहवीं शताब्दी के दौरान भारत आने वाले जेसुइट शिष्टमण्डल व्यापार और साम्राज्य निर्माण की इस प्रकिया के हिस्सा थे।

अकबर ईसाई धर्म के विषय में जानने को बहुत उत्सुक था। उसने जेसुइट पादरियों को आमन्त्रित करने के लिए एक दूत मंडल गोवा भेजा। पहला जेसुइट शिष्टमंडल फतेहपुर सीकरी के मुगल दरबार में 1580 में पहुँचा और वह वहाँ लगभग दो वर्ष रहा। इन जेसुइट लोगों ने ईसाई धर्म के विषय में अकबर से वार्तालाप किया और इसके सद्गुणों के विषय में उलमा से उनका वाद-विवाद हुआ।

लाहौर के मुगल दरबार में दो और शिष्टमण्डल 1591 और 1595 में भेजे गए। जेसुइट विवरण व्यक्तिगत अनुभवों पर आधारित हैं और वे मुगल सम्राट अकबर के चरित्र और सोच पर गहरा प्रकाश डालते हैं। सार्वजनिक सभाओं में जेसुइट लोगों को अकबर के सिंहासन के काफी निकट स्थान दिया जाता था। वे उसके साथ अभियानों में जाते, उसके बच्चों को शिक्षा देते तथा उसके फुरसत के समय में वे प्रायः उसके साथ होते थे। जेसुइट विवरण मुगलकाल के राज्य अधिकारियों और सामान्य जन-जीवन के विषय में फारसी इतिहासों में दी गई सूचना की पुष्टि करते हैं।

प्रश्न 16.
मुगलों के प्रान्तीय प्रशासन का वर्णन कीजिए।
उत्तर:
मुगलों का प्रान्तीय प्रशासन शासन को सुचारू रूप से चलाने के लिए मुगल- सम्राटों ने साम्राज्य को अनेक प्रान्तों में विभक्त कर दिया था प्रान्तीय शासन के प्रमुख अधिकारी निम्नलिखित थे-

  1. सूबेदार – प्रान्त का सर्वोच्च अधिकारी सूबेदार कहलाता था। सूबेदार की नियुक्ति सम्राट के द्वारा की जाती थी। वह सीधा मुगल सम्राट को प्रतिवेदन प्रस्तुत करता था।
  2. दीवान दीवान प्रान्तीय वित्त विभाग का अध्यक्ष होता था। वह प्रान्त की आय-व्यय की देख-रेख करता था।
  3. बख्शी सैनिकों की भर्ती, उनकी आवश्यकताओं एवं साज-सज्जा की पूर्ति, रसद की व्यवस्था उसके कार्य थे।
  4. सद्र सद्र का प्रमुख कार्य लोगों के नैतिक चरित्र की देखभाल करना, दान-पुण्य तथा इस्लाम के कानूनों के पालन की व्यवस्था करवाना था।
  5. काजी- काजी प्रान्त का प्रमुख न्यायिक अधिकारी होता था।

सरकार अथवा जिला प्रबन्ध-मुगलकाल में प्रान्तों को सरकारों अथवा जिलों में बाँटा गया था। सरकार का प्रबन्ध करने के लिए निम्नलिखित अधिकारी नियुक्त किए गए थे-
(1) फौजदार यह सरकार अथवा जिले का प्रमुख अधिकारी होता था। विद्रोहियों का दमन करना, अपने क्षेत्र में शान्ति एवं व्यवस्था बनाए रखना आदि उसके प्रमुख कार्य थे। उसके अधीन एक विशाल घुड़सवार सेना तथा तोपची होती थे।

(2) अमलगुजार यह सरकार का राजस्व अधिकारी परगने का प्रबन्ध- प्रत्येक सरकार अनेक परगनों में विभक्त था। परगनों का प्रबन्ध करने के लिए निम्नलिखित अधिकारी होते थे

  • कानूनगो ( राजस्व अभिलेख का रखवाला)
  • चौधरी (राजस्व संग्रह का प्रभारी) तथा
  • काजी (न्याय का अधिकारी)।

JAC Class 12 History Important Questions Chapter 8 किसान, ज़मींदार और राज्य : कृषि समाज और मुगल साम्राज्य

Jharkhand Board JAC Class 12 History Important Questions Chapter 8 किसान, ज़मींदार और राज्य : कृषि समाज और मुगल साम्राज्य Important Questions and Answers.

JAC Board Class 12 History Important Questions Chapter 8 किसान, ज़मींदार और राज्य : कृषि समाज और मुगल साम्राज्य

बहुविकल्पीय प्रश्न (Multiple Choice Questions)

1. सोलहवीं तथा सत्रहवीं शताब्दियों के कृषि इतिहास की जानकारी देने वाला प्रमुख ऐतिहासिक ग्रन्थ था –
(अ) अकबरनामा
(स) बादशाहनामा
(ब) आइन-ए-अकबरी
(द) बाबरनामा
उत्तर:
(ब) आइन-ए-अकबरी

2. बाबरनामा’ का रचयिता था –
(अ) हुमायूँ
(स) बाबर
(ब) अकबर
(द) जहाँगीर
उत्तर:
(स) बाबर

3. पंजाब में शाह नहर की मरम्मत किसके शासनकाल में करवाई गई –
(अ) बाबर
(ब) हुमायूँ
(स) अकबर
(द) शाहजहाँ
उत्तर:
(द) शाहजहाँ

4. तम्बाकू का प्रसार सर्वप्रथम भारत के किस भाग में हुआ?
(अ) उत्तर भारत
(ब) दक्षिण भारत
(स) पूर्वी भारत
(द) पूर्वोत्तर भारत
उत्तर:
(ब) दक्षिण भारत

5. किस मुगल सम्राट ने तम्बाकू के धूम्रपान पर प्रतिबन्ध लगा दिया था?
(अ) अकबर
(स) जहाँगीर
(ब) बाबर
(द) शाहजहाँ
उत्तर:
(स) जहाँगीर

6. कौनसी फसल नकदी फसल ( जिन्स-ए-कामिल) कहलाती थी –
(अ) कपास
(ब) गेहूँ
(स) जौ
(द) चना
उत्तर:
(अ) कपास

JAC Class 12 History Important Questions Chapter 8 किसान, ज़मींदार और राज्य : कृषि समाज और मुगल साम्राज्य

7. अफ्रीका और स्पेन से किसकी फसल भारत पहुँची ?
(अ) गेहूँ
(ब) बाजरा
(स) मक्का
(द) चना
उत्तर:
(स) मक्का

8. सत्रहवीं शताब्दी में लिखी गई एक पुस्तक मारवाड़ में में किसकी चर्चा किसानों के रूप में की गई है?
(अ) वैश्यों
(ब) राजपूतों
(स) ब्राह्मणों
(द) योद्धाओं
उत्तर:
(ब) राजपूतों

9. गांव की पंचायत का मुखिया कहलाता था –
(अ) फौजदार
(ब) ग्राम-प्रधान
(स) चौकीदार
(द) मुकद्दम (मंडल)
उत्तर:
(द) मुकद्दम (मंडल)

10. खुदकाश्त तथा पाहिकारत कौन थे?
(अ) किसान
(ब) सैनिक
(स) जमींदार
(द) अधिकारी
उत्तर:
(अ) किसान

JAC Class 12 History Important Questions Chapter 8 किसान, ज़मींदार और राज्य : कृषि समाज और मुगल साम्राज्य

11. मिरासदार कौन थे?
(अ) महाराष्ट्र प्रांत के धनी अथवा सम्पन्न कृषक
(ब) राजस्थान के धनी व्यापारी तथा कृषक
(स) राजस्थान में अधिकारियों का एक समूह
(द) उपर्युक्त में से कोई नहीं
उत्तर:
(अ) महाराष्ट्र प्रांत के धनी अथवा सम्पन्न कृषक

12. वह भूमि जिसे कभी खाली नहीं छोड़ा जाता था, कहलाती थी –
(अ) परौती
(ब) चचर
(स) उत्तम
(द) पोलज
उत्तर:
(द) पोलज

13. मनसबदार कौन थे?
(अ) मुगल अधिकारी
(ब) मुगल जमींदार
(स) मुगल दरबारी
(द) इनमें से कोई नहीं
उत्तर:
(अ) मुगल अधिकारी

14. इटली का यात्री जोवनी कारेरी कब भारत आया ?
(अ) 1560 ई.
(ब) 1690 ई.
(स) 1590 ई.
(द) 1670 ई.
उत्तर:
(ब) 1690 ई.

15. आइन-ए-अकबरी में कुल कितने भाग हैं?
(अ) दो
(ब) तीन
(स) चार
(द) पाँच
उत्तर:
(द) पाँच

रिक्त स्थानों की पूर्ति कीजिए

1. 1526 ई. में ……………. को पानीपत के युद्ध में हराकर बाबर पहला मुगल बादशाह बना।
2. मुगल काल के भारतीय फारसी स्रोत किसान के लिए रैयत या …………… शब्द का उपयोग करते थे।
3. सामूहिक ग्रामीण समुदाय के तीन घटक …………… और …………….. थे।
4. पंचायत का सरदार एक मुखिया होता था जिसे ………………. या मण्डल कहते थे।
5. मुद्रा की फेरबदल करने वालों को …………….. कहा जाता था।
6. जोवान्नी कारेरी ………………. का मुसाफिर था जो लगभग ……………. ई. में भारत से होकर गुजरा था।
उत्तर:
1. इब्राहिम लोदी
2. मुजरियान
3. खेतिहर किसान, पंचायत, गाँव का मुखिया
4. मुकद्दम
5. सर्राफ
6. इटली, 1690

JAC Class 12 History Important Questions Chapter 8 किसान, ज़मींदार और राज्य : कृषि समाज और मुगल साम्राज्य

अतिलघूत्तरात्मक प्रश्न

प्रश्न 1.
जिन्स-ए-कामिल’ फसल के बारे में बताइये।
उत्तर:
‘जिन्स-ए-कामिल’ सर्वोत्तम फसलें थीं जैसे कपास और गन्ने की फसलें।

प्रश्न 2.
आइन-ए-अकबरी किसके द्वारा लिखी गई ?
उत्तर:
अबुल फ़सल।

प्रश्न 3.
सोलहवीं और सत्रहवीं सदी में राज्य द्वारा जंगलों में घुसपैठ के क्या कारण थे ?
उत्तर:
(1) सेना के लिए सभी प्राप्त करना
(2) शिकार अभियान द्वारा न्याय करना।

प्रश्न 4.
सत्रहवीं शताब्दी के स्रोत भारत में कितने प्रकार के किसानों का उल्लेख करते हैं?
उत्तर:
दो प्रकार के किसानों की –
(1) खुद काश्त
(2) पाहि काश्त

प्रश्न 5.
अबुल फ़सल द्वारा रचित पुस्तक का नाम लिखिए।
उत्तर:
‘आइन-ए-अकबरी’।

प्रश्न 6.
मुगलकाल में गाँव की पंचायत के मुखिया को किस नाम से जाना जाता था?
उत्तर:
मुकद्दम या मंडल।

प्रश्न 7.
मुगलकाल के भारतीय फारसी स्रोत किसान के लिए किन शब्दों का प्रयोग करते थे?
उत्तर:
(1) रैयत
(2) मुजरियान
(3) किसान
(4) आसामी

प्रश्न 8.
खुद काश्त किसान कौन थे?
उत्तर:
खुद काश्त किसान उन्हीं गाँवों में रहते थे, जिनमें उनकी जमीनें थीं।

प्रश्न 9.
लगभग सोलहवीं सत्रहवीं शताब्दियों में कृषि के निरन्तर विस्तार होने के तीन कारक लिखिए।
उत्तर:

  • जमीन की प्रचुरता
  • मजदूरों की उपलब्धता
  • किसानों की गतिशीलता.

प्रश्न 10.
इस काल में सबसे अधिक उगाई जाने वाली तीन प्रमुख फसलों का उल्लेख कीजिए।
उत्तर:
(1) चावल
(2) गेहूँ
(3) ज्यार

JAC Class 12 History Important Questions Chapter 8 किसान, ज़मींदार और राज्य : कृषि समाज और मुगल साम्राज्य

प्रश्न 11.
चावल, गेहूँ तथा ज्वार की फसलें सबसे अधिक उगाई जाने का क्या कारण था?
उत्तर:
इस काल में खेती का प्राथमिक उद्देश्य लोगों का पेट भरना था।

प्रश्न 12.
मौसम के किन दो मुख्य चक्रों के दौरान खेती की जाती थी?
उत्तर;
(1) खरीफ (पतझड़ में) तथा
(2) रबी (बसन्त में)।

प्रश्न 13.
16वीं – 17वीं शताब्दी के दौरान भारत में कितने प्रतिशत लोग गाँवों में रहते थे?
उत्तर:
85 प्रतिशत।

प्रश्न 14.
भारत में कपास का उत्पादन कहाँ होता था?
उत्तर:
मध्य भारत तथा दक्कनी पठार में।

प्रश्न 15.
दो नकदी फसलों के नाम बताइये।
उत्तर:
(1) तिलहन (जैसे-सरसों) तथा
(2) दलहन

प्रश्न 16.
सत्रहवीं शताब्दी में दुनिया के विभिन्न भागों से भारत में पहुंचने वाली चार फसलों के नाम लिखिए।
उत्तर:

  • मक्का
  • टमाटर
  • आलू
  • मिर्च

प्रश्न 17.
सामूहिक ग्रामीण समुदाय के तीन घटकों का उल्लेख कीजिए।
उत्तर:
(1) खेतिहर किसान
(2) पंचायत
(3) गाँव का मुखिया (मुकद्दम या मंडल)।

प्रश्न 18.
1600 से 1700 के बीच भारत की जनसंख्या में कितनी वृद्धि हुई?
उत्तर:
लगभग 5 करोड़ की।

प्रश्न 19.
ग्राम की पंचायत में कौन लोग होते थे?
उत्तर:
ग्राम के बुजुर्ग।

प्रश्न 20.
ग्राम की पंचायत का मुखिया कौन होता
उत्तर:
मुकद्दम या मंडल।

JAC Class 12 History Important Questions Chapter 8 किसान, ज़मींदार और राज्य : कृषि समाज और मुगल साम्राज्य

प्रश्न 21.
पंचायत के मुखिया का प्रमुख कार्य क्या
उत्तर:
गांव की आमदनी एवं खर्चे का हिसाब-किताब अपनी निगरानी में बनवाना

प्रश्न 22.
इस कार्य में मुकद्दम की कौन राजकीय कर्मचारी सहायता करता था ?
उत्तर:
पटवारी

प्रश्न 23.
पंचायतों के दो अधिकारों का उल्लेख कीजिए।
उत्तर:
(1) जुर्माना लगाना
(2) दोषी व्यक्ति को समुदाय से निष्कासित करना।

प्रश्न 24.
पंचायत के निर्णय के विरुद्ध किसान विरोध का कौनसा अधिक उम्र रास्ता अपनाते थे? उत्तर- गाँव छोड़कर भाग जाना।

प्रश्न 25
उन दो प्रान्तों के नाम लिखिए जहाँ महिलाओं को जमींदारी उत्तराधिकार में मिलती थी जिसे बेचने व गिरवी रखने के लिए वे स्वतंत्र थीं।
उत्तर:
(1) पंजाब
(2) बंगाल

प्रश्न 26.
जंगल के तीन उत्पादों का उल्लेख कीजिए, जिनकी बहुत माँग थी।
उत्तर:
(1) शहद
(2) मधुमोम
(3) लाख

प्रश्न 27.
‘मिल्कियत’ का अर्थ स्पष्ट कीजिए।
उत्तर:
जमींदारों की विस्तृत व्यक्तिगत जमीन ‘मिल्कियत’ कहलाती थी।

प्रश्न 28.
जमींदारों की समृद्धि का क्या कारण था?
उत्तर:
जमींदारों की समृद्धि का कारण था उनकी विस्तृत व्यक्तिगत जमीन।

प्रश्न 29.
जमींदारों की शक्ति के दो स्रोत बताइये।
उत्तर:
(1) जमींदारों द्वारा राज्य की ओर से कर वसूल करना
(2) सैनिक संसाधन।

JAC Class 12 History Important Questions Chapter 8 किसान, ज़मींदार और राज्य : कृषि समाज और मुगल साम्राज्य

प्रश्न 30.
भू-राजस्व के प्रबन्ध के दो चरण कौन- कौन से थे?
उत्तर:
(1) कर निर्धारण
(2) वास्तविक वसूली।

प्रश्न 31.
अमील गुजार कौन था?
उत्तर:
अमील गुजार राजस्व वसूली करने वाला अधिकारी था।

प्रश्न 32.
अकबर ने भूमि का किन चार भागों में वर्गीकरण किया?
उत्तर:

  • पोलज
  • परौती
  • चचर
  • अंजर

प्रश्न 33.
मध्यकालीन भारत में कौन-कौनसी फसलें सबसे अधिक उगाई जाती थीं?
उत्तर:
गेहूं, चावल, ज्वार इत्यादि।

प्रश्न 34.
भारत के किस भाग में सबसे पहले तम्बाकू की खेती की जाती थी?
उत्तर:
दक्षिण भारत में।

प्रश्न 35.
मुगलकाल में खेतिहर समाज की बुनियादी इकाई क्या थी?
उत्तर:
गाँव

प्रश्न 36.
मुगल राज्य किसने को ‘जिन्स-ए-कामिल’ फसलों की खेती करने के लिए प्रोत्साहन क्यों देते थे?
उत्तर:
क्योंकि इन फसलों से राज्य को अधिक कर मिलता था।

प्रश्न 37.
चीनी के उत्पादन के लिए कौनसा प्रान्त प्रसिद्ध था?
उत्तर:
बंगाल।

प्रश्न 38.
19वीं शताब्दी के कुछ अंग्रेज अधिकारियों ने गाँवों को किसकी संज्ञा दी थी?
उत्तर:
‘छोटे गणराज्य’।

प्रश्न 39.
आइन-ए-दहसाला किसने जारी किया?
उत्तर:
अकबरअकबर ने

JAC Class 12 History Important Questions Chapter 8 किसान, ज़मींदार और राज्य : कृषि समाज और मुगल साम्राज्य

प्रश्न 40.
मुगल साम्राज्य की वित्तीय व्यवस्था की देख-रेख करने वाला कौनसा दफ्तर था?
उत्तर;
दीवान का दफ्तर।

प्रश्न 41.
अकबर के समय में वह जमीन क्या कहलाती थी, जिसे कभी खाली नहीं छोड़ा जाता था?
उत्तर:
पोलज

प्रश्न 42.
अकबर के शासन काल में वह जमीन क्या कहलाती थी, जिस पर कुछ दिनों के लिए खेती रोक दी जाती थी?
उत्तर:
परौती।

प्रश्न 43.
आइन-ए-अकबरी’ के अनुसार भू-राजस्व वसूल करने के लिए कौनसी प्रणालियाँ अपनाई जाती थीं?
उत्तर:
(1) कणाकृत
(2) बटाई
(3) खेत बटाई।

प्रश्न 44.
आइन-ए-अकबरी’ को कब पूरा किया गया?
उत्तर:
1598 ई. में

प्रश्न 45.
अकबरनामा’ की कितनी जिल्दों में रचना
की गई?
उत्तर:
तीन जिल्दों में।

प्रश्न 46.
‘अकबरनामा’ का रचयिता कौन था?
उत्तर:
अबुल फजल।

प्रश्न 47
रैयत कौन थे?
उत्तर:
मुगल काल के भारतीय फारसी खोत किसान के लिए आमतौर पर रैयत या मुजरियान शब्द का इस्तेमाल करते थे। था।

प्रश्न 48.
‘परगना’ से क्या अभिप्राय है?
उत्तर:
परगना मुगल प्रान्तों में एक प्रशासनिक प्रमंडल

JAC Class 12 History Important Questions Chapter 8 किसान, ज़मींदार और राज्य : कृषि समाज और मुगल साम्राज्य

प्रश्न 49.
ग्राम पंचायत (मुगलकाल) के मुखिया के दो कार्यों का वर्णन कीजिये।
उत्तर:
(1) गाँव के आय-व्यय का हिसाब-किताब तैयार करवाना
(2) जाति की अवहेलना को रोकने के लिए लोगों के आवरण पर नजर रखना।

प्रश्न 50.
पाहि काश्त किसान कौन थे?
उत्तर:
पाहि कारण किसान के खेतिहर थे, जो दूसरे गाँव से ठेके पर खेती करने आते थे।

प्रश्न 51.
बाबरनामा’ के अनुसार खेतों की सिंचाई साधन कौनसे थे?
गई ?
उत्तर:
(1) रहट के द्वारा
(2) बाल्टियों से।

प्रश्न 52.
भारत में नई दुनिया से कौनसी फसलें लाई
उत्तर:
टमाटर, आलू, मिर्च, अनानास, पपीता

प्रश्न 53.
पंचायत का प्रमुख कार्य क्या था?
उत्तर:
गाँव में रहने वाले अलग-अलग समुदायों के लोगों को अपनी जाति की सीमाओं के अन्दर रखना।

प्रश्न 54.
राजस्थान की जाति पंचायतों के दो अधिकारों का उल्लेख कीजिये।
उत्तर:

  • दीवानी के झगड़ों का निपटारा करना
  • जमीन से जुड़े दावेदारियों के झगड़े सुलझाना।

प्रश्न 55.
पंचायत के मुखिया का चुनाव किस प्रकार किया जाता था?
उत्तर:
(1) गाँव के बुजुर्गों की आम सहमति से और
(2) उसे इसकी स्वीकृति जमींदार से लेनी होती थी।

प्रश्न 56.
खेतिहर किन ग्रामीण दस्तकारियों में संलग्न रहते थे?
उत्तर:
रंगरेजी, कपड़े पर छपाई, मिट्टी के बर्तनों को पकाना, खेती के औजार बनाना।

प्रश्न 57.
सोलहवीं तथा सत्रहवीं सदी के प्रारम्भ में कृषि इतिहास को जानने के दो स्त्रोतों का उल्लेख करो।
उत्तर:
(1) आइन-ए-अकबरी
(2) ईस्ट इण्डिया कम्पनी के बहुत सारे दस्तावेज

JAC Class 12 History Important Questions Chapter 8 किसान, ज़मींदार और राज्य : कृषि समाज और मुगल साम्राज्य

प्रश्न 58.
गाँव के प्रमुख दस्तकारों का उल्लेख कीजिये।
उत्तर:
कुम्हार, लुहार, बढ़ई, नाई, सुनार

प्रश्न 59.
कौनसी दस्तकारियों के काम उत्पादन के लिए महिलाओं के श्रम पर निर्भर थे?
उत्तर:
सूत कातना, बर्तन बनाने के लिए मिट्टी साफ करना और गूंधना, कपड़ों पर कढ़ाई करना।

प्रश्न 60.
‘पेशकश का क्या अर्थ है?
उत्तर:
पेशकरा’ मुगल राज्य के द्वारा ली जाने वाली एक प्रकार की भेंट थी। था?

प्रश्न 61.
मुगल राज्य के लिए जंगल कैसा भू-भाग
उत्तर;
मुगल राज्य के अनुसार जंगल बदमाशों, विद्रोहियों आदि को शरण देने वाला अड्डा था।

प्रश्न 62.
जमींदार कौन थे?
उत्तर;
जमींदार अपनी जमीन के स्वामी होते थे उन्हें ग्रामीण समाज में उच्च प्रतिष्ठा प्राप्त थी।

प्रश्न 63.
जमींदारों की उच्च स्थिति के दो कारण लिखिए।
उत्तर:
(1) उनकी जाति
(2) जमींदारों के द्वारा राज्य को दी जाने वाली कुछ विशिष्ट सेवाएँ।

प्रश्न 64.
‘जमा’ और ‘हासिल’ में भेद कीजिये।
उत्तर:
‘जमा निर्धारित रकम थी, जबकि ‘हासिल’ वास्तव में वसूल की गई रकम थी।

प्रश्न 65.
जमींदारी पुख्ता करने के दो तरीकों का उल्लेख कीजिए।
उत्तर:
(1) नयी जमीनों को बसाकर
(2) अधिकारों के हस्तान्तरण के द्वारा।

प्रश्न 66.
अमीन कौन था?
उत्तर:
अमीन एक मुगल अधिकारी था, जिसका काम यह सुनिश्चित करना था कि राजकीय कानूनों का पालन हो रहा है।

प्रश्न 67.
मनसबदारी व्यवस्था क्या थी?
उत्तर:
मनसबदारी मुगल प्रशासनिक व्यवस्था के शीर्ष पर एक सैनिक नौकरशाही तत्व था, जिसे मनसबदारी व्यवस्था कहते हैं।

JAC Class 12 History Important Questions Chapter 8 किसान, ज़मींदार और राज्य : कृषि समाज और मुगल साम्राज्य

प्रश्न 68.
अकबर के समय का प्रसिद्ध इतिहासकार कौन था? उसके द्वारा रचित दो ग्रन्थों के नाम लिखिए।
उत्तर:
(1) अकबर के समय का प्रसिद्ध इतिहासकार अबुल फजल था।
(2) अबुल फजल ने ‘अकबरनामा’ तथा आइन-ए-अकबरी’ की रचना की।

प्रश्न 69.
चंडीमंगल नामक बंगाली कविता के रचयिता कौन हैं ?
उत्तर:
सोलहवीं सदी में मुकुंदराम चक्रवर्ती ने चंडीमंगल नामक कविता लिखी थी।

प्रश्न 70.
पेशकश क्या होती थी?
उत्तर:
पेशकश मुगल राज्य के द्वारा की जाने वाली एक तरह की भेंट थी।

प्रश्न 71.
गाँवों में कौनसी पंचायतें होती थीं?
उत्तर:
(1) ग्राम पंचायत
(2) जाति पंचायत ।

प्रश्न 72.
किस प्रांत में चावल की 50 किस्में पैदा की
जाती थीं?
उत्तर:
बंगाल में।

प्रश्न 73
गाँवों में पाई जाने वाली दो सामाजिक असमानताओं का उल्लेख कीजिये।
उत्तर:
(1) सम्पति की व्यक्तिगत मिल्कियत होती थी।
(2) जाति और सामाजिक लिंग के नाम पर समाज में गहरी विषमताएँ थीं।

प्रश्न 74.
शाह नहर कहाँ है?
उत्तर:
पंजाब में।

प्रश्न 75.
अकबर द्वारा अमील गुजार को क्या आदेश दिए गए थे?
उत्तर:
इस बात की व्यवस्था करना कि खेतिहर नकद भुगतान करे और वहीं फसलों में भुगतान का विकल्प भी खुला रहे।

JAC Class 12 History Important Questions Chapter 8 किसान, ज़मींदार और राज्य : कृषि समाज और मुगल साम्राज्य

प्रश्न 76
जमींदारी को पुख्ता करने के क्या तरीके थे?
उत्तर:
(1) नई जमीनों को बतकर
(2) अधिकारों के हस्तान्तरण के द्वारा
(3) राज्य के आदेश से
(4) जमीनों को खरीद कर

प्रश्न 77.
वाणिज्यिक खेती का प्रभाव जंगलवासियाँ के जीवन पर कैसे पढ़ता था?
उत्तर:
(1) शहद, मधु, मोम, लाक की अत्यधिक माँग होना
(2) हाथियों को पकड़ना और बेचना
(3) व्यापार के अन्तर्गत वस्तुओं की अदला-बदली।

प्रश्न 78.
जंगली (जंगलवासी) कौन थे?
उत्तर:
जिन लोगों का गुजारा जंगल के उत्पादों, शिकार और स्थानान्तरित खेती से होता था, वे जंगली (जंगलवासी) कहलाते थे।

लघुत्तरात्मक प्रश्न

प्रश्न 1.
खेतिहर समाज के बारे में आप क्या जानते हैं?
उत्तर:
खेतिहर समाज की मूल इकाई गाँव थी जिसमें किसान रहते थे। किसान वर्ष भर भिन्न-भिन्न मौसमें में फसल की पैदावार से जुड़े समस्त कार्य करते थे। इन कार्यों में जमीन की जुताई, बीज बोना और फसल पकने पर उसे काटना आदि सम्मिलित थे। इसके अतिरिक्त वे उन वस्तुओं के उत्पादन में भी शामिल थे जो कृषि आधारित थीं, जैसे कि शक्कर, तेल इत्यादि ।

प्रश्न 2.
सत्रहवीं शताब्दी के खोत किन दो प्रकार के किसानों का उल्लेख करते हैं?
उत्तर- सत्रहवीं शताब्दी के स्रोत निम्नलिखित दो प्रकार के किसानों का उल्लेख करते हैं- (1) खुद काश्त तथा (2) पाहि कार खुद काश्त किसान उन्हीं गाँवों में रहते थे जिनमें उनकी जमीन थी। पाहि काश्त वे किसान थे जो दूसरे गाँवों से ठेके पर खेती करने आते थे। कुछ लोग अपनी इच्छा से भी पाहि-कारत बनते थे, जैसे दूसरे गाँव में करों की शर्तें उत्तम मिलने पर कुछ लोग अकाल या भुखमरी के बाद आर्थिक परेशानी से बाध्य होकर भी पाहि कार किसान बनते थे।

प्रश्न 3.
मुगल काल में कृषि की समृद्धि से भारत में आबादी में किस प्रकार बढ़ोतरी हुई?
उत्तर:
कृषि उत्पादन में अपनाए गए विविध और लचीले तरीकों के परिणामस्वरूप भारत में आवादी धीरे-धीरे बढ़ने लगी। आर्थिक इतिहासकारों की गणना के अनुसार समय- समय पर होने वाली भुखमरी और महामारी के बावजूद 1600 से 1700 के बीच भारत की आबादी लगभग 5 करोड़ बढ़ गई। 200 वर्षों में यह लगभग 33 प्रतिशत बढ़ोतरी थी।

JAC Class 12 History Important Questions Chapter 8 किसान, ज़मींदार और राज्य : कृषि समाज और मुगल साम्राज्य

प्रश्न 4.
सोलहवीं सत्रहवीं शताब्दी का ग्रामीण समाज किन-किन रिश्तों के आधार पर निर्मित था ?
उत्तर:
सोलहवीं सत्रहवीं शताब्दी का ग्रामीण समाज छोटे किसानों एवं धनिक जमींदारों दोनों से निर्मित था। ये दोनों ही कृषि उत्पादन से जुड़े थे और फसल में हिस्सों के दावेदार थे। इससे उनके मध्य सहयोग, प्रतियोगिता एवं संघर्ष के रिश्ते निर्मित हुए कृषि से जुड़े इन समस्त रिश्तों से ग्रामीण समाज बनता था।

प्रश्न 5.
मुगल साम्राज्य के अधिकारी ग्रामीण समाज को नियन्त्रण में रखने का प्रयास क्यों करते थे?
उत्तर:
मुगल साम्राज्य अपनी आय का एक बड़ा भाग कृषि उत्पादन से प्राप्त करता था इसलिए राजस्व निर्धारित करने वाले, राजस्व की वसूली करने वाले एवं राजस्व का विवरण रखने वाले अधिकारी ग्रामीण समाज को नियन्त्रण में रखने का पूरा प्रयास करते थे। वे चाहते थे कि खेती की नियमित जुताई हो एवं राज्य को उपज से अपने हिस्से का कर समय पर मिल जाए।

प्रश्न 6.
ग्रामीण भारत के खेतिहर समाज पर संक्षेप में तीन पंक्तियाँ लिखिए।
उत्तर;
खेतिहर समाज की मूल इकाई गाँव थी; जिमसें किसान रहते थे। किसान वर्ष भर भिन्न-भिन्न मौसमों में फसल की पैदावार से जुड़े समस्त कार्य करते थे। इन कार्यों में जमीन की जुताई, बीज बोना एवं फसल पकने पर उसकी कटाई करना आदि कार्य सम्मिलित थे। इसके अतिरिक्त वे उन वस्तुओं के उत्पादन में भी सम्मिलित थे जो कृषि आधारित थीं जैसे कि शक्कर, तेल आदि ।

प्रश्न 7.
” सत्रहवीं सदी में दुनिया के अलग-अलग भागों से कई नई फसलें भारतीय उपमहाद्वीप पहुंचीं।” स्पष्ट कीजिए।
उत्तर:
सत्रहवीं सदी में दुनिया के विभिन्न भागों से कई नई फसलें भारतीय उपमहाद्वीप पहुंचीं। उदाहरणार्थ, मक्का भारत में अफ्रीका और स्पेन से पहुँचा और सत्रहवीं सदी तक इसकी गिनती पश्चिम भारत की मुख्य फसलों में होने लगी। टमाटर, आलू और मिर्च जैसी सब्जियाँ नई दुनिया से लाई गई। इसी प्रकार अनानास तथा पपीता जैसे फल भी नई दुनिया से आए।

प्रश्न 8.
मुगल काल में गाँव की पंचायत का गठन किस प्रकार होता था?
उत्तर:
गाँव की पंचायत बुजुर्गों की सभा होती थी। प्रायः वे गाँव के महत्त्वपूर्ण लोग हुआ करते थे जिनके पास अपनी सम्पत्ति के पैतृक अधिकार होते थे। जिन गाँवों में कई जातियों के लोग रहते थे, वहाँ प्रायः पंचायत में भी विविधता पाई जाती थी यह एक ऐसा अल्पतंत्र था, जिसमें गाँव के अलग-अलग सम्प्रदायों एवं जातियों का प्रतिनिधित्व होता था। पंचायत का निर्णय गाँव में सबको मानना पड़ता था।

प्रश्न 9.
गाँव की पंचायत का जाति सम्बन्धी प्रमुख काम क्या था?
उत्तर:
गांव की पंचायत का जाति सम्बन्धी प्रमुख काम यह सुनिश्चित करना था कि गाँव में रहने वाले अलग-अलग सम्प्रदायों के लोग अपनी जाति की सीमाओं के अन्दर रहें। पूर्वी भारत में सभी विवाह मंडल की उपस्थिति में होते थे। दूसरे शब्दों में ” जाति की अवहेलना के लिए” लोगों के आचरण पर नजर रखनां गाँव की पंचायत के मुखिया की एक महत्त्वपूर्ण जिम्मेदारी थी।

JAC Class 12 History Important Questions Chapter 8 किसान, ज़मींदार और राज्य : कृषि समाज और मुगल साम्राज्य

प्रश्न 10.
गाँव की पंचायतों को कौनसे न्याय सम्बन्धी अधिकार प्राप्त थे?
उत्तर:
गाँव की पंचायतों को दोषियों पर जुर्माना लगाने और समुदाय से निष्कासित करने जैसे दंड देने के अधिकार प्राप्त थे। समुदाय से निष्कासित करना एक कड़ा कदम था, जो एक सीमित समय के लिए लागू किया जाता था। इसके अन्तर्गत दण्डित व्यक्ति को (निर्धारित समय के लिए) गाँव छोड़ना पड़ता था। इस अवधि में वह अपनी जाति तथा व्यवसाय से हाथ धो बैठता था। इन नीतियों का उद्देश्य जातिगत परम्पराओं की अवहेलना को रोकना था।

प्रश्न 11.
सत्रहवीं सदी में गाँवों में मुद्रा के प्रचलन के बारे में फ्रांसीसी यात्री ज्याँ बैप्टिस्ट तैवर्नियर ने क्या लिखा है?
उत्तर:
सत्रहवीं सदी में भारत की यात्रा करने वाले फ्रांसीसी यात्री ज्याँ बैप्टिस्ट तैवर्नियर ने गाँवों में मुद्रा के प्रचलन के बारे में लिखा है कि, “भारत में वे गाँव बहुत ही छोटे कहे जायेंगे, जिनमें मुद्रा की फेरबदल करने वाले हाँ ये लोग सराफ कहलाते थे। एक बैंकर की भाँति सराफ हवाला भुगतान करते थे और अपनी इच्छा के अनुसार पैसे के मुकाबले रुपयों की कीमत तथा कौड़ियों के मुकाबले पैसों की कीमत बढ़ा देते थे।”

प्रश्न 12.
सोलहवीं तथा सत्रहवीं शताब्दी में भारत द में जंगलों के प्रसार का वर्णन कीजिए।
उत्तर:
सोलहवीं तथा सत्रहवीं शताब्दी में भारत में जमीन के विशाल भाग जंगल या शादियों से घिरे हुए थे। के ऐसे प्रदेश झारखंड सहित सम्पूर्ण पूर्वी भारत, मध्य भारत, उत्तरी क्षेत्र (जिसमें भारत-नेपाल की सीमावर्ती क्षेत्र की ि तराई शामिल है), दक्षिण भारत का पश्चिमी घाट और प दक्कन के पठारों में फैले हुए थे। समसामयिक स्रोतों से प्राप्त जानकारी के आधार पर यह अनुमान लगाया जा सकता है कि यह भारत में जंगलों के फैलाव का औसत लगभग 40 प्रतिशत था।

प्रश्न 13.
जंगलवासियों पर कौनसे नए सांस्कृतिक प्रभाव पड़े?
उत्तर:
जंगलवासियों पर नए सांस्कृतिक प्रभाव पड़े। कुछ इतिहासकारों की यह मान्यता है कि नए बसे इलाकों के खेतिहर समुदायों ने जिस प्रकार से धीरे-धीरे इस्लाम को अपनाया, उसमें सूफी सन्तों (पीर) ने एक महत्त्वपूर्ण भूमिका निभाई थी। इस प्रकार जंगली प्रदेशों में नए सांस्कृतिक प्रभावों के विस्तार से शुरूआत हुई। प्रश्न 14. जमींदार कौन थे?
उत्तर- जमींदारों की आय तो खेती से आती थी, परन्तु ये कृषि उत्पादन में सीधे भागीदारी नहीं करते थे। वे अपनी जमीन के मालिक होते थे। उन्हें ग्रामीण समाज में ऊँची स्थिति के कारण कुछ विशेष सामाजिक और आर्थिक सुविधाएँ प्राप्त थीं। जमींदारों की ऊँची स्थिति के दो कारण थे-
(1) उनकी जाति तथा
(2) उनके द्वारा राज्य को कुछ विशेष प्रकार की सेवाएं देना।

प्रश्न 15.
जमींदारों की समृद्धि का क्या कारण था?
उत्तर:
जमींदारों की विस्तृत व्यक्तिगत जमीन उनकी समृद्धि का कारण था उनकी व्यक्तिगत जमीन मिल्कियत कहलाती थी अर्थात् सम्पत्ति मिल्कियत जमीन पर जमींदार के निजी प्रयोग के लिए खेती होती थी। प्रायः इन जमीनों पर दिहाड़ी मजदूर या पराधीन मजदूर कार्य करते थे जमींदार अपनी इच्छानुसार इन जमीनों को बेच सकते थे, किसी और के नाम कर सकते थे या उन्हें गिरवी रख सकते थे।

प्रश्न 16.
जमींदारी को पुख्ता करने की प्रक्रिया का वर्णन कीजिए।
उत्तर:
जमींदारी को पुख्ता करने की प्रक्रिया धीमी थी। इसके निम्नलिखित तरीके थे—

  • नयी जमीनों को बसा कर
  • अधिकारों के हस्तान्तरण के द्वारा
  • राज्य के आदेश से
  • या फिर खरीदकर इन प्रक्रियाओं के द्वारा अपेक्षाकृत ‘निचली जातियों के लोग भी जमींदारों के दर्जे में शामिल हो सकते थे, क्योंकि इस काल में जमींदारी धड़ल्ले से खरीदी और बेची जाती थी।

प्रश्न 17.
1665 में औरंगजेब ने जमा निर्धारित करने के लिए अपने राजस्व अधिकारियों को क्या आदेश दिया?
उत्तर:
1665 में औरंगजेब ने जमा निर्धारित करने के लिए अपने राजस्व अधिकारियों को वह आदेश दिया कि वे परगनाओं के अमीनों को निर्देश दें कि वे प्रत्येक गाँव, प्रत्येक किसान (आसामीवार) के बारे में खेती की वर्तमान स्थितियों का पता करें बारीकी से उनकी जाँच करने के बाद सरकार के वित्तीय हितों व किसानों के कल्याण को ध्यान में रखते हुए जमा निर्धारित करें।

JAC Class 12 History Important Questions Chapter 8 किसान, ज़मींदार और राज्य : कृषि समाज और मुगल साम्राज्य

प्रश्न 18.
मुगल काल में कृषि की समृद्धि और भारत की आबादी की बढ़ोत्तरी में क्या सम्बन्ध था?
उत्तर:
मुगलकाल में शासकों की दूरदर्शिता द्वारा किसानों के हितों एवं कृषि उत्पादन के लिए विविध तकनीकों के प्रयोग के कारण कृषि आधारित समृद्धि में व्यापक वृद्धि हुई। इसके परिणामस्वरूप आबादी धीरे-धीरे बढ़ने लगी। आर्थिक इतिहासकारों की गणना के अनुसार 1600 से 1700 के बीच समय-समय पर होने वाली भुखमरी और महावारी के उपरान्त भी भारत की आबादी लगभग 5 करोड़ बढ़ गई। 200 वर्षों में आबादी की यह बढ़ोत्तरी करीब 33 प्रतिशत थी।

प्रश्न 19
सत्रहवीं शताब्दी में दुनिया के अलग- अलग हिस्सों से कई नई फसलें भारतीय उपमहाद्वीप पहुँची।” कथन को सोदाहरण स्पष्ट कीजिए।
अथवा
नई फसलों जैसे मक्का, ज्वार, आलू आदि का भारत में प्रवेश कैसे हुआ?
उत्तर:
सत्रहवीं शताब्दी में दुनिया के अलग-अलग हिस्सों से व्यापारिक आवागमन में वृद्धि के कारण कई नई फसलें भारतीय उपमहाद्वीप पहुँचीं। उदाहरण के रूप में मक्का भारत में अफ्रीका व स्पेन से पहुँची और सत्रहवीं शताब्दी तक इसकी गिनती पश्चिम भारत की मुख्य फसलों में होने लगी। टमाटर, आलू व मिर्च जैसी सब्जियाँ नई दुनिया से लाई गई। इसी प्रकार अनन्नास एवं पपीता भी नई दुनिया से आए।

प्रश्न 20.
जजमानी व्यवस्था क्या थी?
उत्तर:
18वीं शताब्दी में बंगाल में जमींदार लोहारों, बढ़ई व सुनारों जैसे ग्रामीण दस्तकारों को उनकी सेवाओं के बदले दैनिक भत्ता तथा खाने के लिए नकदी देते थे। इस व्यवस्था को जजमानी व्यवस्था कहा जाता था। यद्यपि यह प्रथा सोलहवीं व सत्रहवीं शताब्दी में अधिक प्रचलित नहीं थी।

प्रश्न 21.
मुगलकाल में गाँवों और शहरों के मध्य व्यापार से गाँवों में नकदी लेन-देन होना प्रारम्भ हुआ, सोदाहरण कथन को स्पष्ट कीजिए।
उत्तर:
मुगल काल में गाँवों और शहरों के मध्य व्यापार के कारण गाँवों में भी नकदी लेन-देन होने लगा। मुगलों के केन्द्रीय प्रदेशों से भी कर की गणना और वसूली नकद में दी जाती थी जो दस्तकार निर्यात के लिए उत्पादन करते थे, उन्हें उनकी मजदूरी अथवा पूर्ण भुगतान नकद में ही किया जाता था। इसी प्रकार कपास, रेशम या नील जैसी व्यापारिक फसलें उत्पन्न करने वाले किसानों का भुगतान भी नकदी में ही होता है।

JAC Class 12 History Important Questions Chapter 8 किसान, ज़मींदार और राज्य : कृषि समाज और मुगल साम्राज्य

प्रश्न 22.
आइन-ए-अकबरी’ में किन मसलों पर विस्तार से चर्चा की गई है?
अथवा
‘आइन-ए-अकबरी’ पर टिप्पणी लिखिए।
उत्तर:
आइन-ए-अकबरी’ में अनेक मसलों पर विस्तार से चर्चा की गई है; जैसे दरबार, प्रशासन और सेना का गठन, राजस्व के स्रोत और अकबरी साम्राज्य के प्रान्तों का भूगोल, लोगों के साहित्यिक, सांस्कृतिक एवं धार्मिक रिवाज अकबर की सरकार के समस्त विभागों और प्रान्तों (सूखों) के बारे में जानकारी दी गई है। ‘आइन’ में इन सूबों के बारे में जटिल और आँकड़ेबद्ध सूचनाएँ बड़ी बारीकी से दी गई हैं।

प्रश्न 23.
‘आइन’ कितने भागों (दफ्तरों) में विभक्त है? उनका सक्षिप्त उल्लेख कीजिए।
उत्तर- ‘आइन’ पाँच भागों (दफ्तरों में विभक्त है-
(1) मंजिल आबादी, शाही घर-परिवार और उसके रख- रखाव से सम्बन्ध रखती है
(2) सिपह- आबादी- सैनिक व नागरिक प्रशासन और नौकरों की व्यवस्था के बारे में है।
(3) मुल्क आबादी में साम्राज्य व प्रान्तों के वित्तीय पहलुओं तथा राजस्व की दरों के आँकड़े वर्णित हैं तथा बारह प्रान्तों का वर्णन है। चौथे और पाँचवें भाग भारत के लोगों के धार्मिक, साहित्यिक और सांस्कृतिक रीति-रिवाजों से सम्बन्ध रखते हैं।

प्रश्न 24.
अकबर के समय भूमि का वर्गीकरण किस प्रकार किया गया था?
अथवा
चाचर और बंजर भूमि की परिभाषा लिखिए।
उत्तर:

  • पोलज पोलज भूमि में एक के बाद एक हर फसल की वार्षिक खेती होती थी जिसे कभी खाली नहीं छोड़ा जाता था।
  • परीती परौती जमीन पर कुछ दिनों के लिए खेती रोक दी जाती थी ताकि वह अपनी खोई हुई उर्वरा शक्ति वापस पा सके।
  • चचर पचर जमीन तीन या चार वर्षों तक खाली रहती थी।
  • बंजर – बंजर जमीन वह थी जिस पर पाँच या उससे अधिक वर्षों से खेती नहीं की जाती थी।

प्रश्न 25.
कणकुत प्रणाली क्या थी?
उत्तर:
अकबर के समय कनकृत प्रणाली राजस्व निर्धारण की एक प्रणाली थी। हिन्दी में ‘कण’ का अर्थ है ‘अनाज’ और कुंत का अर्थ है अनुमान’। इसके अनुसार फसल को अलग-अलग पुलिन्दों में काटा जाता था-
(1) अच्छा
(2) मध्यम
(3) खराब इस प्रकार सन्देह दूर किया जाना चाहिए। प्रायः अनुमान से किया गया जमीन का आकलन भी पर्याप्त रूप से सही परिणाम देता था।

JAC Class 12 History Important Questions Chapter 8 किसान, ज़मींदार और राज्य : कृषि समाज और मुगल साम्राज्य

प्रश्न 26.
भारत में सोलहवीं तथा सत्रहवीं सदी के ग्रामीण समाज की क्या स्थिति थी?
उत्तर:
सोलहवीं तथा सत्रहवीं सदी में भारत में लगभग 85 प्रतिशत लोग गाँवों में रहते थे छोटे खेतिहर तथा भूमिहर जमींदार दोनों ही कृषि उत्पादन से जुड़े हुए थे और दोनों ही फसल के हिस्सों के दावेदार थे। इससे उनके बीच सहयोग, प्रतियोगिता तथा संघर्ष के सम्बन्ध बने खेती से जुड़े इन समस्त सम्बन्धों के ताने-बाने से गाँव का समाज बनता था। मुगल-राज्य अपनी आय का बहुत बड़ा भाग कृषि उत्पादन से वसूल करता था राजस्व अधिकारी ग्रामीण समाज पर नियन्त्रण रखते थे

प्रश्न 27.
खेतिहर समाज का परिचय दीजिए। कृषि समाज की भौगोलिक विविधताओं का भी उल्लेख कीजिए।
उत्तर:
खेतिहर समाज की मूल इकाई गाँव थी, जिसमें किसान रहते थे किसान वर्ष भर फसल की पैदावार से जुड़े समस्त कार्य करते थे। इन कार्यों में जमीन की जुताई, बीज बोना और फसल पकने पर उसे काटना आदि सम्मिलित थे। इसके अतिरिक्त वे उन वस्तुओं के उत्पादन में भी शामिल थे, जो कृषि आधारित थीं जैसे शक्कर, तेल इत्यादि । परन्तु सूखी भूमि के विशाल हिस्सों से लेकर पहाड़ियों वाले क्षेत्र में उस प्रकार की खेती नहीं हो सकती थी जैसी कि अधिक उपजाऊ जमीनों पर।

प्रश्न 28.
सोलहवीं तथा सत्रहवीं शताब्दी का कृषि- इतिहास लिखने के लिए ‘आइन-ए-अकबरी’ का मुख्य स्रोत के रूप में विवेचन कीजिए।
उत्तर:
सोलहवीं तथा सत्रहवीं शताब्दी का कृषि इतिहास लिखने के लिए ‘आइन-ए-अकबरी’ एक महत्त्वपूर्ण स्रोत है। ‘आइन-ए-अकबरी’ की रचना मुगल सम्राट अकबर के दरवारी इतिहासकार अबुल फतल ने की थी खेतों की नियमित जुलाई सुनिश्चित करने के लिए राज्य के प्रतिनिधियों द्वारा करों की वसूली करने के लिए तथा राज्य व जमींदारों के बीच के सम्बन्धों के नियमन के लिए राज्य द्वारा किये गए प्रबन्धों का लेखा-जोखा ‘आइन’ में प्रस्तुत किया गया है।

प्रश्न 29.
आइन-ए-अकबरी’ के लेखन का मुख्य उद्देश्य स्पष्ट कीजिये।
उत्तर:
आइन-ए-अकबरी’ का मुख्य उद्देश्य अकबर के साम्राज्य की एक ऐसी रूप-रेखा प्रस्तुत करना था, जहाँ एक सुदृढ़ सत्ताधारी वर्ग सामाजिक मेल-जोल बनाकर रखता था। अबुल फजल के अनुसार मुगल साम्राज्य के विरुद्ध कोई भी विद्रोह या किसी भी प्रकार की स्वायत्त सत्ता की दावेदारी का असफल होना निश्चित था अतः ‘आइन’ से किसानों के बारे में जो कुछ पता चलता है, वह मुगल शासक वर्ग का ही दृष्टिकोण है।

प्रश्न 30.
सोलहवीं सत्रहवीं शताब्दी के कृषि- इतिहास की जानकारी के लिए ‘आइन’ के अतिरिक्त अन्य स्रोतों का वर्णन कीजिए।
उत्तर:
(1) ‘आइन’ के अतिरिक्त सत्रहवीं व अठारहवीं सदियों के गुजरात, महाराष्ट्र और राजस्थान से उपलब्ध होने वाले वे दस्तावेज भी हैं जो सरकार की आय की विस्तृत जानकारी देते हैं।
(2) इसके अतिरिक्त ईस्ट इण्डिया कम्पनी के भी बहुत से दस्तावेज हैं जो पूर्वी भारत में कृषि सम्बन्धों की उपयोगी रूपरेखा प्रस्तुत करते हैं। इन सभी स्रोतों में किसानों, जमींदारों तथा राज्य के बीच होने वाले झगड़ों के विवरण मिलते हैं। इनसे किसानों के राज्य के प्रति दृष्टिकोण की जानकारी मिलती है।

प्रश्न 31.
सोलहवीं तथा सत्रहवीं शताब्दी में किसान की समृद्धि का मापदंड क्या था ?
उत्तर:
सोलहवीं तथा सत्रहवीं शताब्दी में उत्तर भारत के एक औसत किसान के पास शायद ही कभी एक जोड़ी बैल तथा दो हलों से अधिक कुछ होता था। अधिकांश किसानों के पास इससे भी कम था। गुजरात के वे किसान समृद्ध माने जाते थे जिनके पास 6 एकड़ के लगभग जमीन थी दूसरी ओर, बंगाल में एक औसत किसान की जमीन की ऊपरी सीमा 5 एकड़ थी वहाँ 10 एकड़ जमीन वाले किसान को धनी माना जाता था। खेती व्यक्तिगत स्वामित्व के सिद्धान्त पर आधारित थी।

JAC Class 12 History Important Questions Chapter 8 किसान, ज़मींदार और राज्य : कृषि समाज और मुगल साम्राज्य

प्रश्न 32.
‘बाबरनामा’ में बाबर ने कृषि-समाज की विशेषताओं का किस प्रकार वर्णन किया है?
उत्तर:
‘बाबरनामा’ के अनुसार भारत में बस्तियाँ और गाँव, वस्तुत: शहर के शहर, एक क्षण में ही बीरान भी हो जाते थे और बस भी जाते थे। दूसरी ओर, यदि वे किसी पर बसना चाहते थे तो उन्हें पानी के रास्ते खोदने की आवश्यकता नहीं पड़ती थी क्योंकि उनकी समस्त फसलें वर्षा के पानी में उगती थीं भारत की अनगिनत आबादी होने के कारण लोग उमड़ते चले आते थे । यहाँ झोंपड़ियाँ बनाई जाती थीं और अकस्मात एक गाँव या शहर तैयार हो जाता था।

प्रश्न 33.
भारत में सोलहवीं तथा सत्रहवीं शताब्दी में कृषि क्षेत्र में हुए विकास का वर्णन कीजिए।
उत्तर:
जमीन की प्रचुरता, मजदूरों की उपलब्धता तथा किसानों की गतिशीलता के कारण कृषि का निरन्तर विकास हुआ। चूँकि खेती का प्राथमिक उद्देश्य लोगों का पेट भरना था, इसलिए दैनिक भोजन में काम आने वाले खाद्य पदार्थों जैसे चावल, गेहूं, ज्वार इत्यादि फसलें सबसे अधिक उगाई जाती थीं। जिन प्रदेशों में प्रतिवर्ष 40 इंच या उससे अधिक वर्षा होती थी, वहाँ न्यूनाधिक चावल की खेती होती थी कम वर्षा वाले प्रदेशों में गेहूँ तथा ज्वार बाजरे की खेती होती थी।

प्रश्न 34.
भारत में सोलहवीं तथा सत्रहवीं शताब्दी में सिंचाई के साधनों में हुए विकास का वर्णन कीजिए।
उत्तर:
आज की भाँति सोलहवीं सत्रहवीं शताब्दी में भी मानसून को भारतीय कृषि की रीढ़ माना जाता था। परन्तु कुछ फसलों के लिए अतिरिक्त पानी की आवश्यकता थी। इसके लिए सिंचाई के कृत्रिम साधनों का सहारा लेना पड़ा। राज्य की ओर से सिंचाई कार्यों में सहायता दी जाती थी उदाहरणार्थ, उत्तर भारत में राज्य ने कई नई नहरें व नाले खुदवाये तथा कई पुरानी नहरों की मरम्मत करवाई। शाहजहाँ के शासन काल में पंजाब में ‘शाह नहर’ इसका उदाहरण है।

प्रश्न 35.
मुगलकालीन कृषि समाज में महिलाओं की स्थिति बताइए।
उत्तर:
मुगलकालीन कृषि समाज में महिलाएँ व पुरुष मिलजुलकर खेती का कार्य करते थे। पुरुष खेत जोतते थे तथा हल चलाते थे जबकि महिलाएँ बुआई, निराई व कटाई के साथ-साथ पकी हुई फसल से दाना निकालने का कार्य करती थीं। सूत कातना, वर्तन बनाने के लिए मिट्टी को साफ करना व गूंथना तथा कपड़ों पर कढ़ाई जैसे दस्तकारी के कार्य भी महिलाएँ ही करती थीं महिलाओं पर परिवार और समुदाय के पुरुषों का नियन्त्रण बना रहता था।

प्रश्न 36.
सामाजिक कारणों से जंगलवासियों के जीवन में किस प्रकार परिवर्तन आया?
उत्तर:
सामाजिक कारणों से जंगलवासियों के जीवन में परिवर्तन आया। ग्रामीण समुदाय के बड़े आदमियों की तरह जंगली कबीलों के भी सरदार होते थे। कई कबीलों के सरदार धीरे-धीरे जमींदार बन गए। कुछ तो राजा भी बन गए। 16वीं- 17वीं शताब्दी में कुछ राजाओं ने पड़ोसी कवीलों के साथ एक के बाद एक युद्ध किया और उन पर अपना अधिकार स्थापित कर लिया।

JAC Class 12 History Important Questions Chapter 8 किसान, ज़मींदार और राज्य : कृषि समाज और मुगल साम्राज्य

प्रश्न 37.
जंगलवासियों पर कौनसे नए सांस्कृतिक प्रभाव पड़े?
उत्तर:
जंगलवासियों पर नए-नए सांस्कृतिक प्रभाव पड़े कुछ इतिहासकारों की यह मान्यता है कि नए बसे क्षेत्रों के खेतिहर समुदायों ने जिस प्रकार से धीरे-धीरे इस्लाम को अपनाया, उसमें सूफी सन्तों ने एक बड़ी भूमिका निभाई थी।

प्रश्न 38.
सोलहवीं तथा सत्रहवीं शताब्दियों में भारत में खेती के विकास के लिए किसानों द्वारा अपनायी गई तकनीकों का वर्णन कीजिए।
उत्तर:
(1) किसान लकड़ी के ऐसे हल्के हल का प्रयोग करते थे जिसको एक छोर पर लोहे की नुकीली धार या फाल लगा कर सरलता से बनाया जा सकता था।
(2) बैलों के जोड़े के सहारे खींचे जाने वाले बरमे का प्रयोग बीज बोने के लिए किया जाता था परन्तु बीजों को हाथ से छिड़क कर बोने की पद्धति अधिक प्रचलित थी।
(3) मिट्टी की गुड़ाई और निराई के लिए लकड़ी के मूठ वाले लोहे के पतले धार प्रयुक्त किए जाते थे।

प्रश्न 39.
भारत में तम्बाकू का प्रसार किस प्रकार हुआ? जहाँगीर ने तम्बाकू के धूम्रपान पर प्रतिबन्ध क्यों लगा दिया?
उत्तर:
तम्बाकू का पौधा सबसे पहले दक्षिण भारत पहुँचा और वहाँ से सरहवीं शताब्दी के प्रारम्भिक वर्षों में इसे उत्तर भारत लाया गया। अकबर और उसके अमीरों ने 1604 ई. में पहली बार तम्बाकू देखा सम्भवत: इसी समय से भारत में तम्बाकू का धूम्रपान (हुक्के या चिलम में) करने के व्यसन ने जोर पकड़ा। परन्तु जहाँगीर ने तम्बाकू के धूम्रपान पर प्रतिबन्ध लगा दिया। इस प्रतिबन्ध का कोई प्रभाव नहीं पड़ा क्योंकि इसका सत्रहवीं शताब्दी के अन्त तक सम्पूर्ण भारत में प्रचलन था।

प्रश्न 40.
मुगलकाल में मौसम के चक्रों में उगाई जाने वाली विविध फसलों का वर्णन कीजिए।
उत्तर:
मुगल काल में भारत में मौसम के दो मुख्य चक्रों के दौरान खेती की जाती थी एक खरीफ (पतझड़ में) तथा दूसरी रवी (बसन्त में) अधिकतर स्थानों पर वर्ष में कम से कम दो फसलें उगाई जाती थीं जहाँ वर्षा अथवा सिंचाई के अन्य साधन हर समय उपलब्ध थे, वहाँ यो वर्ष में तीन फसलें भी उगाई जाती थीं ‘आइन-ए- अकबरी’ के अनुसार दोनों मौसमों को मिलाकर मुगल- प्रान्त आगरा में 39 तथा दिल्ली प्रान्त में 43 फसलों की उगाई की जाती थी।

प्रश्न 41.
सोलहवीं तथा सत्रहवीं शताब्दियों में भारत ‘में ‘जिन्स-ए-कामिल’ नामक फसलें क्यों उगाई जाती थीं?
उत्तर:
सोलहवीं तथा सत्रहवीं शताब्दियों में भारत में ‘जिन्स-ए-कामिल’ (सर्वोत्तम फसलें) नामक फसलें भी उगाई जाती थीं मुगल राज्य भी किसानों को ‘जिन्स-ए- कामिल’ नामक फसलों की खेती करने के लिए प्रोत्साहन देता था क्योंकि इनसे राज्य को अधिक कर मिलता था। कपास और गने जैसी फसलें श्रेष्ठ ‘जिन्स-ए-कामिल’ श्रीं तिलहन (जैसे सरसों) तथा दलहन भी नकदी फसलों में सम्मिलित थीं।

JAC Class 12 History Important Questions Chapter 8 किसान, ज़मींदार और राज्य : कृषि समाज और मुगल साम्राज्य

प्रश्न 42.
जाति और अन्य जाति जैसे भेदभावों के कारण खेतिहर किसान किन समूहों में बँटे हुए थे?
उत्तर:
खेतों की जुताई करने वालों में एक बड़ी संख्या उन लोगों की थी जो निकृष्ट समझे जाने वाले कामों में लगे थे अथवा फिर खेतों में मजदूरी करते थे कुछ जातियों के लोगों को केवल निकृष्ट समझे जाने वाले काम ही दिए जाते थे। इस प्रकार वे गरीबी का जीवन व्यतीत करने के लिए बाध्य थे। गाँव की आबादी का बहुत बड़ा भाग ऐसे ही समूहों का था। इनके पास संसाधन सबसे कम थे तथा ये जाति व्यवस्था के प्रतिबन्धों से बंधे हुए थे इनकी स्थिति न्यूनाधिक आधुनिक भारत के दलितों जैसी थी।

प्रश्न 43.
“समाज के निम्न वर्गों में जाति, गरीबी तथा सामाजिक हैसियत के बीच सीधा सम्बन्ध था। परन्तु ऐसा बीच के समूहों में नहीं था।” स्पष्ट कीजिए।
उत्तर- सत्रहवीं सदी में मारवाड़ में लिखी गई एक पुस्तक में राजपूतों का उल्लेख किसानों के रूप में किया गया है। इस पुस्तक के अनुसार जाट भी किसान थे परन्तु जाति व्यवस्था में उनका स्थान राजपूतों की तुलना में नीचा था सत्रहवीं सदी में वृन्दावन (उत्तर प्रदेश) के क्षेत्र में रहने वाले गौरव समुदाय ने भी राजपूत होने का दावा किया, यद्यपि वे जमीन की जुताई के काम में लगे थे। अहीर, गुज्जर तथा माली जैसी जातियों के सामाजिक स्तर में भी वृद्धि हुई।

प्रश्न 44.
पंचायत का मुखिया कौन होता था? उसके कार्यों का उल्लेख कीजिए।
उत्तर:
पंचायत का सरदार एक मुखिया होता था, जिसे ‘मुकदम’ चा ‘मंडल’ कहते थे। मुखिया का चुनाव गाँव के बुजुर्गों की आम सहमति से होता था। इस चुनाव के बाद उन्हें इसकी स्वीकृति जमींदार से लेनी पड़ती थी। मुखिया अपने पद पर तभी तक बना रह सकता था, जब तक गाँव के बुजुगों को उस पर भरोसा था। गाँव की आप व खर्चे का हिसाब-किताब अपनी निगरानी में बनवाना मुखिया का प्रमुख कार्य था। इस कार्य में पंचायत का पटवारी उसको सहायता करता था।

प्रश्न 45.
मुगल काल में गाँव के ‘आम खजाने’ से पंचायत के कौनसे खर्चे बलते थे?
उत्तर:
(1) मुगल काल में पंचायत का खर्चा के आम खजाने से चलता था इस खजाने से उन कर अधिकारियों की खातिरदारी का खर्चा भी किया जाता था, जो समय-समय पर गाँव का दौरा किया करते थे।
(2) इस आम खजाने का प्रयोग बाढ़ जैसी प्राकृतिक विपदाओं से निपटने के लिए भी किया जाता था।
(3) इस कोष का प्रयोग ऐसे सामुदायिक कार्यों के लिए भी किया जाता था जो किसान स्वयं नहीं कर सकते थे, जैसे छोटे-मोटे बांध बनाना या नहर खोदना

प्रश्न 46.
मुगलकालीन जाति पंचायतों के कार्यों का वर्णन कीजिए।
उत्तर:

  • राजस्थान में जाति पंचायतें अलग-अलग जातियों के लोगों के बीच दीवानी के झगड़ों का निपटारा करती थीं।
  • वे जमीन से जुड़े दावेदारियों के झगड़े सुलझाती
  • वे यह निश्चित करती थीं कि विवाह जातिगत मानदंडों के अनुसार हो रहे हैं या नहीं।
  • वे यह निश्चित करती थीं कि गाँव के आयोजन में किसको किसके ऊपर प्राथमिकता दी जाएगी। कर्मकाण्डीय वर्चस्व किस क्रम में होगा।

प्रश्न 47
पंचायतों में ग्रामीण समुदाय के निम्न वर्ग के लोग उच्च जातियों तथा राज्य के अधिकारियों के विरुद्ध कौनसी शिकायतें प्रस्तुत करते थे?
उत्तर:
पश्चिमी भारत, विशेषकर राजस्थान और महाराष्ट्र जैसे प्रान्तों से प्राप्त दस्तावेजों में ऐसे कई प्रार्थना-पत्र हैं, जिनमें पंचायत से ‘सी’ जातियों अथवा राज्य के अधिकारियों के विरुद्ध जबरन कर वसूली अथवा बेगार वसूली की शिकायत की गई है। इनमें किसी जाति या सम्प्रदाय विशेष के लोग अभिजात वर्ग के समूहों की उन माँगों के विरुद्ध अपना विरोध दर्शाते थे जिन्हें वे नैतिक दृष्टि से अवैध मानते थे। उनमें एक मांग बहुत अधिक कर की माँग थी।

प्रश्न 48
पंचायतों द्वारा इन शिकायतों का निपटारा किस प्रकार किया जाता था ?
उत्तर:
निचली जाति के किसानों और राज्य के अधिकारियों या स्थानीय जमींदारों के बीच झगड़ों में पंचायत के निर्णय अलग-अलग मामलों में अलग-अलग हो सकते थे। अत्यधिक कर की मांगों में पंचायत प्रायः समझौते का सुझाव देती थीं जहाँ समझौते नहीं हो पाते थे, वहाँ किसान विरोध के अधिक उम्र साधन अपनाते थे, जैसे कि गाँव छोड़ कर भाग जाना।

JAC Class 12 History Important Questions Chapter 8 किसान, ज़मींदार और राज्य : कृषि समाज और मुगल साम्राज्य

प्रश्न 49.
खेतिहर लोग किन ग्रामीण दस्तकारियों में संलग्न रहते थे?
उत्तर:
खेतिहर और उसके परिवार के सदस्य विभिन्न प्रकार की वस्तुओं के उत्पादन में संलग्न रहते थे जैसे-रंगरेजी, कपड़े पर छपाई, मिट्टी के बर्तनों को पकाना, खेती के औजारों को बनाना अथवा उनकी मरम्मत करना। जब किसानों को खेती के काम से अवकाश मिलता था, जैसे-बुआई और सुड़ाई के बीच या सुहाई और कटाई के बीच, उस अवधि में ये खेतिहर दस्तकारी के काम में संलग्न रहते थे।

प्रश्न 50.
ग्रामीण दस्तकार किस रूप में अपनी सेवाएँ गाँव के लोगों को देते थे? इसके बदले गाँव के लोग उन सेवाओं की अदायगी किन तरीकों से करते थे?
उत्तर:
कुम्हार, लोहार, बढ़ई, नाई, यहाँ तक कि सुनार जैसे ग्रामीण दस्तकार अपनी सेवाएँ गाँव के लोगों को देते थे, जिसके बदले गाँव वाले उन्हें विभिन्न तरीकों से उन सेवाओं की अदायगी करते थे। प्रायः वे या तो उन्हें फसल का एक भाग दे देते थे या फिर गाँव की जमीन का एक टुकड़ा अदायगी का तरीका सम्भवतः पंचायत ही तय करती थी कुछ स्थानों पर दस्तकार तथा प्रत्येक खेतिहर परिवार आपसी बातचीत करके अदायगी का तरीका तय कर लेते थे।

प्रश्न 51.
कुछ अंग्रेज अफसरों द्वारा भारतीय गाँवों को ‘छोटे गणराज्य’ क्यों कहा गया?
अथवा
ग्रामीण समुदाय के महत्त्व का विवेचन कीजिए।
अथवा
‘छोटे गणराज्य’ के रूप में भारतीय गाँवों का महत्त्व स्पष्ट कीजिए।
उत्तर:
उन्नीसवीं सदी के कुछ अंग्रेज अधिकारियों ने भारतीय गाँवों को एक ऐसे ‘छोटे गणराज्य’ की संज्ञा दी, जहाँ लोग सामूहिक स्तर पर भाई चारे के साथ संसाधनों तथा श्रम का विभाजन करते थे परन्तु ऐसा नहीं लगता कि गाँव में सामाजिक समानता थी सम्पत्ति पर व्यक्तिगत स्वामित्व होता था। इसके साथ ही जाति और लिंग के नाम पर समाज में गहरी विषमताएँ थीं कुछ शक्तिशाली लोग गाँव की समस्याओं पर निर्णय लेते थे और कमजोर वर्गों के लोगों का शोषण करते थे।

प्रश्न 52.
भूमिहर भद्रजनों में महिलाओं को प्राप्त पैतृक सम्पत्ति के अधिकार का वर्णन कीजिए।
उत्तर:
भूमिहर भद्रजनों में महिलाओं को पैतृक सम्पत्ति का अधिकार प्राप्त था। पंजाब में महिलाएँ (विधवा महिलाएँ भी) पैतृक सम्पत्ति के विक्रेता के रूप में ग्रामीण जमीन के क्रय-विक्रय में सक्रिय भागीदारी रखती थीं हिन्दू और मुसलमान महिलाओं को जमींदारी उत्तराधिकार में मिलती थी, जिसे बेचने अथवा गिरवी रखने के लिए वे स्वतंत्र थीं। बंगाल में भी महिला जमींदार थीं। वहाँ राजशाही की जमींदारी की कर्ता-धर्ता एक स्वी थी।

प्रश्न 53.
‘जंगली’ कौन थे? मुगलकाल में जंगली शब्द का प्रयोग किन लोगों के लिए किया जाता था?
उत्तर:
समसामयिक रचनाएँ जंगल में रहने वालों के लिए ‘जंगली’ शब्द का प्रयोग करती हैं। परन्तु जंगली होने का अर्थ यह नहीं था कि वे असभ्य थे। मुगलकाल में जंगली शब्द का प्रयोग ऐसे लोगों के लिए होता था जिनका निर्वाह जंगल के उत्पादों, शिकार और स्थानान्तरीच खेती से होता था। ये काम मौसम के अनुसार होते थे। उदाहरण के लिए, भील बसन्त के मौसम में जंगल के उत्पाद इकट्ठा करते थे, गर्मियों में मछली पकड़ते थे तथा मानसून के महीनों में खेती करते थे।

JAC Class 12 History Important Questions Chapter 8 किसान, ज़मींदार और राज्य : कृषि समाज और मुगल साम्राज्य

प्रश्न 54.
राज्य की दृष्टि में जंगल किस प्रकार का क्षेत्र था?
उत्तर:
राज्य की दृष्टि में जंगल राज्य की दृष्टि में जंगल उलटफेर वाला इलाका था अपराधियों को शरण देने वाला अड्डा राज्य के अनुसार अपराधी और विद्रोही जंगल में शरण लेते थे अपराध करने के बाद ये लोग जंगल में छिप जाते थे। बाबर के अनुसार जंगल एक ऐसा रक्षाकवच था जिसके पीछे परगना के लोग कड़े विद्रोही हो जाते थे और कर चुकाने से मुकर जाते थे।”

प्रश्न 55.
मुगल काल में बाहरी शक्तियाँ जंगलों में किसलिए प्रवेश करती थीं?
उत्तर:
(1) मुगलकाल में बाहरी शक्तियाँ जंगलों में कई तरह से प्रवेश करती थीं। उदाहरणार्थ, राज्य को सेना के लिए हाथियों को आवश्यकता होती थी इसलिए जंगलवासियों से ली जाने वाली भेंट में प्रायः हाथी भी सम्मिलित होते थे।
(2) शिकार अभियान के नाम पर मुगल सम्राट अपने साम्राज्य के भिन्न-भिन्न भागों का दौरा करते थे और लोगों की समस्याओं और शिकायतों पर उचित ध्यान देते थे।

प्रश्न 56.
मुगल-राजनीतिक विचारधारा में शिकार अभियान का क्या महत्त्व था?
उत्तर:
मुगल राजनीतिक विचारधारा में गरीबों और अमीरों सहित सबको न्याय प्रदान करने के उद्देश्य से ‘शिकार अभियान’ प्रारम्भ किया गया। शिकार अभियान के नाम पर मुगल सम्राट अपने विशाल साम्राज्य के कोने-कोने का दौरा करता था। इस प्रकार वह अलग-अलग प्रदेशों के लोगों की समस्याओं और शिकायतों पर व्यक्तिगत रूप से ध्यान देता था और न्याय प्रदान करता था।

प्रश्न 57.
जंगलवासियों के जीवन पर बाहरी कारक के रूप में वाणिज्यिक खेती का क्या प्रभाव पड़ा?
उत्तर:
जंगलवासियों के जीवन पर वाणिज्यिक खेती का काफी प्रभाव पड़ता था। शहद, मधुमोम तथा लाक जैसे जंगल के उत्पादों की बहुत माँग थी। लाक जैसी कुछ वस्तुएँ तो सत्रहवीं सदी में भारत से समुद्र पार होने वाले निर्यात की मुख्य वस्तुएँ थीं। हाथी भी पकड़े और बेचे जाते थे व्यापार के अन्तर्गत वस्तुओं की अदला-बदली भी होती थी। कुछ कवीले भारत और अफगानिस्तान के बीच होने वाले स्थलीय व्यापार में लगे हुए थे वे पंजाब के गाँवों और शहरों के बीच होने वाले व्यापार में भी भाग लेते थे।

प्रश्न 58.
सामाजिक कारणों से जंगलवासियों के जीवन में क्या परिवर्तन हुए?
उत्तर:
सामाजिक कारणों से जंगलवासियों के जीवन में महत्त्वपूर्ण परिवर्तन हुए। जंगली कबीलों के भी सरदार होते थे। कई जंगली कबीलों के सरदार जमींदार बन गए। कुछ तो राजा भी बन गए। ऐसे में उन्हें सेना का गठन करने की आवश्यकता हुई। अतः उन्होंने अपने ही वंश के लोगों को सेना में भर्ती किया अथवा फिर उन्होंने अपने ही भाई- बन्धुओं से सैन्य-सेच की मांग की। सिन्ध प्रदेश की कमीलाई सेनाओं में 6000 घुड़सवार और 7000 पैदल सैनिक होते थे।

प्रश्न 59.
मुगलकालीन भारत में जमींदारों की शक्ति के स्रोतों का उल्लेख कीजिए।
उत्तर:
(1) जमींदारों को शक्ति इस बात से मिलती धी कि वे प्रायः राज्य की ओर से कर वसूल कर सकते थे। इसके बदले उन्हें वित्तीय मुआवजा मिलता था।
(2) सैनिक संसाधन उनकी शक्ति का एक और स्रोत था अधिकतर जमींदारों के पास अपने किले भी थे और अपनी सैनिक टुकड़ियाँ भी, जिनमें घुड़सवारों, तोपखाने तथा पैदल सैनिकों के जत्थे होते थे।

JAC Class 12 History Important Questions Chapter 8 किसान, ज़मींदार और राज्य : कृषि समाज और मुगल साम्राज्य

प्रश्न 60.
जमींदारों की उत्पत्ति का स्रोत क्या था?
उत्तर:
समसामयिक दस्तावेजों से प्रतीत होता है कि युद्ध में विजय जमींदारों की उत्पत्ति का सम्भावित लोत रहा होगा। प्रायः जमींदारी के प्रसार का एक तरीका था शक्तिशाली सैनिक सरदारों द्वारा कमजोर लोगों को बेदखल करना। परन्तु इसकी सम्भावना कम ही दिखाई देती है कि राज्य किसी जमींदार को इतने आक्रामक रुख अपनाने की अनुमति देता हो, जब तक कि एक राज्यादेश के द्वारा इसकी पुष्टि पहले ही नहीं कर दी गई हो।

प्रश्न 61.
कृषि के विकास में जमींदारों के योगदान का वर्णन कीजिए।
उत्तर:

  • जमींदारों ने कृषि योग्य जमीनों को बसने में नेतृत्व प्रदान किया और खेतिहरों को खेती के उपकरण व उधार देकर उन्हें वहाँ बसने में भी सहायता की।
  • जमींदारी के क्रय-विक्रय से गाँवों के मौद्रीकरण की प्रक्रिया में तेजी आई।
  • जमींदार अपनी मिल्कियत की जमीनों की फसल भी बेचते थे।
  • कुछ साक्ष्य दर्शाते हैं कि जमींदार प्रायः बाजार (हाट) स्थापित करते थे जहाँ किसान भी अपनी फसलें बेचने आते थे।

प्रश्न 62.
“किसानों से जमींदारों के सम्बन्धों में पारस्परिकता, पैतृकवाद तथा संरक्षण का पुट था।” व्याख्या कीजिए।
उत्तर:
यद्यपि जमींदार शोषण करने वाला वर्ग था, परन्तु किसानों से उनके सम्बन्धों में पारस्परिकता, पैतृकवाद तथा संरक्षण का पुट था। भक्ति सन्तों ने बड़ी निर्भीकतापूर्वक जातिगत तथा अन्य प्रकार के अत्याचारों की कटु निन्दा की, परन्तु उन्होंने जमींदारों को किसानों के शोषक या उन पर अत्याचार करने वाले के रूप में नहीं दर्शाया। सत्रहवीं सदी में हुए कृषि विद्रोहों में राज्य के विरुद्ध जमींदारों को प्राय: किसानों का समर्थन मिला।

प्रश्न 63.
मुगलकालीन भू-राजस्व प्रणाली की विवेचना कीजिए।
अथवा
अकबर के शासन में भूमि के वर्गीकरण किये जाने के बारे में अबुल फजल ने अपने ग्रन्थ ‘आइन’ में क्या वर्णन किया है?
अथवा
अकबर के समय में प्रचलित भू-राजस्व व्यवस्था का वर्णन कीजिए।
अथवा
मुगलों की भू-राजस्व व्यवस्था का वर्णन कीजिये ।
अथवा
अकबर की भू-राजस्व व्यवस्था की मुख्य विशेषताओं का उल्लेख कीजिये।
उत्तर:
अकबर ने जमीनों का वर्गीकरण किया और प्रत्येक वर्ग की जमीन के लिए अलग-अलग राजस्व निर्धारित किया। भूमि को चार वर्गों में बाँटा गया-
(i) पोलज
(ii) पोलज और परौती की तीन मध्यम
(iii) चचर
(iv) बंजर। और प्रत्येक वर्ग की तीन किस्में थीं-
(1) अच्छी
(2) किस्म
(3) खराब प्रत्येक की जमीन की उपज को जोड़ा जाता था और उसका तीसरा हिस्सा औसत उपज माना जाता था इसका एक- तिहाई भाग राजकीय शुल्क माना जाता था।

JAC Class 12 History Important Questions Chapter 8 किसान, ज़मींदार और राज्य : कृषि समाज और मुगल साम्राज्य

प्रश्न 64.
अकबर के शासन काल में राजस्व की वसूली के लिए अपनाई गई प्रणालियों का ‘आइन’ में किस प्रकार वर्णन किया गया है?
उत्तर:
अकबर के शासन काल में फसल के रूप में राजस्व वसूली की पहली प्रणाली कुणकुत प्रणाली थी। इसमें फसल को तीन अलग-अलग पुलिन्दों में काटा जाता था दूसरी प्रणाली बटाई या भाओली थी, जिसमें फसल काट कर जमा कर लेते थे और फिर सभी पक्षों की सहमति से बंटवारा किया जाता था। खेत बटाई तीसरी प्रणाली थी जिसमें बीज बोने के बाद खेत बाँट लेते थे। लाँग बटाई चौथी प्रणाली थी जिसमें फसल काटने के बाद उसे आपस में बाँट लेते थे।

प्रश्न 65.
मनसबदारी व्यवस्था’ पर एक संक्षिप्त टिप्पणी लिखिए।
अथवा
मनसबदारी व्यवस्था की मुख्य विशेषताओं का वर्णन कीजिए।
अथवा
अकबर की मनसबदारी व्यवस्था की विवेचना कीजिये।
उत्तर:
मनसबदारी मुगल प्रशासनिक व्यवस्था के शीर्ष पर एक सैनिक नौकरशाही तन्त्र था। इस पर राज्य के सैनिक व नागरिक मामलों की जिम्मेदारी थी कुछ मनसबदारों को नकदी भुगतान किया जाता था परन्तु अधिकांश मनसबदारों को साम्राज्य के भिन्न-भिन्न भागों में राजस्व के आवंटन के द्वारा भुगतान किया जाता था। सम्राट द्वारा समय-समय पर उनका स्थानान्तरण किया जाता था। प्रायः योग्य और अनुभवी व्यक्ति को ही मनसबदार के पद पर नियुक्त किया जाता था।

प्रश्न 66.
सोलहवीं तथा सत्रहवीं सदी में भारत में चाँदी का बहाव क्यों और किस प्रकार हुआ? उत्तर- सोलहवीं और सत्रहवीं सदी में भारत के व्यापार में अत्यधिक उन्नति हुई। इस कारण भारत के समुद्र पार व्यापार में एक ओर भौगोलिक विविधता आई, तो दूसरी ओर अनेक नई वस्तुओं का व्यापार भी शुरू हुआ। निरन्तर बढ़ते व्यापार के साथ, भारत से निर्यात होने वाली वस्तुओं का भुगतान करने के लिए एशिया में भारी मात्रा में चाँदी आई। इस चाँदी का एक बड़ा भाग भारत पहुँचा। यह भारत के लिए लाभप्रद था; क्योंकि यहाँ चाँदी के प्राकृतिक संसाधन नहीं थे।

प्रश्न 67.
सोलहवीं तथा सत्रहवीं शताब्दी में भारत में चाँदी के बहाव के क्या परिणाम हुए?
उत्तर:
(1) सोलहवीं से अठारहवीं सदी के बीच भारत में धातु मुद्रा विशेषकर चाँदी के रुपयों की प्राप्ति में अच्छी स्थिरता बनी रही।
(2) अर्थव्यवस्था में मुद्रा संचार और सिक्कों की तलाई में अभूतपूर्व विस्तार हुआ तथा मुगल राज्य को नकदी कर वसूलने में आसानी हुई।
(3) इटली के यात्री जोवान्नी कारेरी के अनुसार चाँदी समस्त विश्व से होकर भारत पहुँचती थी तथा सत्रहवी शताब्दी में भारत में अत्यधिक मात्रा में नकदी और वस्तुओं का आदान-प्रदान हो रहा था।

प्रश्न 68
आइन-ए-अकबरी’ की त्रुटियों पर प्रकाश डालिए।
उत्तर:
(1) जोड़ करने में कई गलतियाँ पाई गई हैं।
(2) ‘आइन’ के संख्यात्मक आँकड़ों में विषमताएँ हैं।
(3) जहाँ सूबों से लिए गए राजकोषीय आँकड़े बड़े विस्तार से दिए गए हैं, वहीं उन्हीं प्रदेशों से कीमतों और मजदूरी से सम्बन्धित आँकड़े इतने विस्तार के साथ नहीं दिए गए हैं।
(4) मूल्यों और मजदूरी की दरों की सूची मुगल साम्राज्य की राजधानी आगरा या उसके आस-पास के प्रदेशों से ली गई है।

प्रश्न 69.
“कुछ त्रुटियों के होते हुए भी ‘आइन- ए-अकबरी’ अपने समय के लिए एक असाधारण एवं अनूठा दस्तावेज है।” व्याख्या कीजिए।
उत्तर:
‘आइन-ए-अकबरी’ अपने समय के लिए एक असाधारण एवं अनूठा दस्तावेज हैं। मुगल साम्राज्य के गठन और उसकी संरचना की अत्यन्त आकर्षक झलकियाँ दर्शाकर और उसके निवासियों व उत्पादों के सम्बन्ध में सांख्यिकीय आँकड़े प्रस्तुत कर, अबुल फजल मध्यकालीन इतिहासकारों से कहीं आगे निकल गए। ‘आइन’ में भारत के लोगों और मुगल साम्राज्य के बारे में विस्तृत सूचनाएँ दी गई हैं। इस प्रकार यह ग्रन्थ सत्रहवीं शताब्दी के भारत के अध्ययन के लिए संदर्भ-बिन्दु बन गया है।

JAC Class 12 History Important Questions Chapter 8 किसान, ज़मींदार और राज्य : कृषि समाज और मुगल साम्राज्य

प्रश्न 70.
बटाई प्रणाली तथा लाँग बढाई पद्धतियों | का वर्णन कीजिये।
उत्तर:
(1) कटाई प्रणाली में फसल काटकर जमा कर लेते थे और फिर सभी पक्षों की उपस्थिति तथा सहमति से बँटवारा कर लेते थे।
(2) लॉंग बटाई – इस प्रणाली में फसल काटने के बाद उसके ढेर बना लिए जाते थे तथा फिर उन्हें आपस में बाँट लेते थे। इसमें हर पक्ष अपना हिस्सा घर ले जाता था।

प्रश्न 71.
मुगल सम्राट शिकार अभियानों का क्यों आयोजन करते थे?
उत्तर:
मुगल सम्राट गरीबों और अमीरों सहित सबके साथ न्याय करना अपना प्रमुख कर्त्तव्य मानते थे। इसलिए ये शिकार अभियानों का आयोजन करते थे। वे शिकार आयोजन के नाम पर अपने विशाल साम्राज्य के कोने-कोने का दौरा करते थे और विभिन्न क्षेत्रों के लोगों की समस्याओं और शिकायतों को सुनकर उनका निराकरण करते थे।

प्रश्न 72.
कृषि के अतिरिक्त महिलाएँ दस्तकारी के किन कार्यों में संलग्न रहती थीं?
उत्तर:
कृषि के अतिरिक्त महिलाएं सूत कातने वर्तन बनाने के लिए मिट्टी को साफ करने और गूंधने, कपड़ों पर कढ़ाई जैसे दस्तकारी के कार्यों में संलग्न रहती थीं किसी वस्तु का जितना वाणिज्यीकरण होता था, उसके उत्पादन के लिए महिलाओं के श्रम की उतनी ही माँग होती थी। इस प्रकार महिलाएँ न केवल खेतों में कार्य करतीं थीं, बल्कि नियोक्ताओं के घरों पर भी जाती थीं।

प्रश्न 73.
मुगलकाल में कृषि में प्रयोग होने वाली मुख्य तकनीकी कौनसी थी? संक्षेप में लिखिए।
उत्तर:
मुगलकाल में कृषि अपनी उच्च अवस्था में थी क्योंकि उस समय अनेक तकनीकों का प्रयोग किया जाता था। वस्तुत: उस समय प्रमुख कृषि तकनीकी पशु बल पर आधारित थी। इसे हम निम्नलिखित बिन्दुओं के माध्यम से समझ सकते हैं-
(1) लकड़ी का हल एक प्रमुख कृषि उपकरण था। इसे एक छोर पर लोहे की नुकीली धार अथवा फाल लगाकर बनाया जाता था ऐसे हल मिट्टी को अधिक गहरा नहीं खोदते थे। जिसके कारण तेज गर्मी के महीनों में मिट्टी में पर्याप्त नमी बनी रहती थी जो उपज अथवा फसल के लिए अत्यधिक उपयोगी होती थी
(2) मिट्टी की निराई तथा गुड़ाई के लिए लकड़ी के मूठ वाले लोहे के पतले धार वाले उपकरण प्रयोग होते थे।
(3) बैलों के जोड़े के सहारे खींचे जाने वाले बरमे का प्रयोग बीज बोने के लिए किया जाता था, किन्तु बीजों को अधिकतर हाथ से छिड़ककर ही बोया जाता था
(4) रहट, जो उस समय सिंचाई का मुख्य साधन था, में भी बैलों का प्रयोग किया जाता था।

प्रश्न 74.
फ्रांसीसी यात्री ज्यां बैप्टिस्ट तैवर्नियर ने गाँवों में मुद्रा के प्रचलन के सम्बन्ध में अपने यात्रा वृत्तान्त में क्या वर्णन किया है?
उत्तर:
सत्रहवीं सदी में भारत में आए फ्रांसीसी यात्री ज्यां बैप्टिस्ट तैवर्नियर ने भारत के गाँवों में मुद्रा के प्रचलन के सम्बन्ध में अपने यात्रा वृत्तान्त में उल्लेख किया है कि “भारत के गाँव बहुत ही छोटे कहे जाएँगे, जिसमें मुद्रा के फेर-बदल करने वालों को ‘सर्राफ’ कहा जाता है। सर्राफ का कार्य बैंकरों की भाँति होता था, एक बैंक की भाँति सर्राफ हवाला भुगतान करते हैं और अपनी मर्जी के अनुसार पैसे के मुकाबले रुपयों की कीमत और कौड़ियों के मुकाबले पैसे की कीमत बढ़ा देते हैं।”

प्रश्न 75
ग्राम पंचायतों के कार्यों का उल्लेख कीजिये।
उत्तर:
(1) जाति की अवहेलना को रोकने के लिए लोगों के आचरण पर नजर रखना। (2) दोषियों पर जुर्माना लगाना और समुदाय से निष्कासित करना। इसके अन्तर्गत दण्डित व्यक्ति को निर्धारित समय के लिए गाँव छोड़ना पड़ता था। इस दौरान वह अपनी जाति और व्यवसाय से बचित हो जाता था।

JAC Class 12 History Important Questions Chapter 8 किसान, ज़मींदार और राज्य : कृषि समाज और मुगल साम्राज्य

प्रश्न 76.
मध्यकालीन भारत में सिंचाई के कौनसे साधन प्रचलित थे?
उत्तर:
मानसून भारतीय कृषि का रीढ़ था। खेती कृषि पर निर्भर करती थी परन्तु कुछ ऐसी फसलें भी थीं, जिनके लिए अतिरिक्त जल की आवश्यकता थी। अतः कुआँ, नहरों, नदियों, जलाशयों आदि से भी खेती की सिंचाई की जाती थी। बाबर के अनुसार रहट या बाल्टियों से सिंचाई की जाती थी।

निबन्धात्मक प्रश्न

प्रश्न 1.
आप कृषि का इतिहास लिखने के लिए ‘आइन’ का स्रोत के रूप में किस प्रकार उपयोग करेंगे? समझाइये
अथवा
कृषि-इतिहास के स्रोत के रूप में ‘आइन-ए- अकबरी’ का वर्णन कीजिये।
अथवा
सोलहवीं तथा सत्रहवीं शताब्दी के कृषि इतिहास की जानकारी के मुख्य स्रोतों का विवेचन कीजिए।
उत्तर:
सोलहवीं तथा सत्रहवीं शताब्दी के कृषि – इतिहास की जानकारी के मुख्य स्रोत सोलहवीं तथा सत्रहवीं शताब्दी के कृषि इतिहास की जानकारी के मुख्य स्रोतों का विवेचन निम्नलखित बिन्दुओं के अन्तर्गत किया जा सकता है –
(1) मुगल दरबार की निगरानी में लिखे गए ऐतिहासिक ग्रन्थ व दस्तावेज-किसान अपने बारे में स्वयं नहीं लिखा करते थे इसलिए खेतों में काम करने वाले ग्रामीण समाज के क्रिया-कलापों की जानकारी हमें उन लोगों से प्राप्त नहीं होती। परिणामस्वरूप सोलहवीं तथा सत्रहवीं शताब्दियों के कृषि इतिहास की जानकारी देने वाले हमारे मुख्य स्रोत वे ऐतिहासिक ग्रन्थ एवं दस्तावेज हैं, जो मुगल दरबार की निगरानी में लिखे गए थे।

(2) ‘आइन-ए-अकबरी’- सोलहवीं या सत्रहवीं शताब्दियों का कृषि इतिहास लिखने के लिए, ‘आइन- ए-अकबरी’ एक अत्यन्त महत्त्वपूर्ण ग्रन्थ है ‘ आइन-ए- अकबरी’ की रचना मुगल सम्राट अकबर के दरबारी इतिहासकार अबुल फसल ने की थी। ‘आइन-ए-अकबरी’ का मुख्य उद्देश्य अकबर के साम्राज्य की एक ऐसी रूप- रेखा प्रस्तुत करना था, जहाँ एक शक्तिशाली सत्ताधारी वर्ग सामाजिक मेल-जोल बना कर रखता था। ‘आइन’ के तीसरे भाग ‘मुल्क आबादी में उत्तर भारत के कृषि-समाज के बारे में विस्तृत विवरण प्रस्तुत किया गया है। इसमें साम्राज्य व प्रान्तों के वित्तीय पहलुओं तथा | राजस्व की दरों के आंकड़ों की विस्तृत जानकारी दी गई है। इसमें प्रत्येक प्रान्त तथा उसकी अलग-अलग इकाइयों की कुल मापी गई जमीन और निर्धारित राजस्व का विवरण भी प्रस्तुत किया गया है।

(3) अन्य स्रोत- सोलहवीं तथा सत्रहवीं शताब्दियों के कृषि इतिहास की जानकारी के लिए हम निम्नलिखित स्रोतों का भी प्रयोग कर सकते हैं-
(i) मुगलों की राजधानी से दूर के प्रदेशों में लिखे गए ग्रन्थ व दस्तावेज’ आइन’ की जानकारी के साथ- साथ हम उन लोगों का भी प्रयोग कर सकते हैं, जो मुगलों की राजधानी से दूर के प्रदेशों में लिखे गए थे। इनमें सत्रहवीं व अठारहवीं सदियों के गुजरात, महाराष्ट्र और राजस्थान से उपलब्ध होने वाले वे दस्तावेज सम्मिलित हैं. जो सरकार की आय की विस्तृत जानकारी देते हैं।

(ii) ईस्ट इण्डिया कम्पनी के दस्तावेज इसके अतिरिक्त ईस्ट इण्डिया कम्पनी के बहुत से दस्तावेज भी हैं जो पूर्वी भारत में कृषि सम्बन्धों की उपयोगी जानकारी देते हैं। इन सभी स्रोतों में किसानों, जमींदारों तथा राज्य के बीच होने वाले झगड़ों के विवरण मिलते हैं। इन स्रोतों से हमें यह समझने में सहायता मिलती है कि किसानों का राज्य के प्रति क्या दृष्टिकोण था तथा राज्य से उन्हें कैसे न्याय की आशा थी।

प्रश्न 2.
खुद काश्त व पाहि काश्त से आप क्या समझते हैं? मुगलकाल में किसानों की दशा का वर्णन कीजिए।
उत्तर:
किसानों को मुगलकाल में रैयत या मुजरिवान कहा जाता था। किसानों के लिए सत्रहवीं सदी के भारतीय- फारसी स्रोत में इसी शब्द का प्रयोग किया गया है। किसानों के लिए आसामी शब्द का प्रयोग भी किया जाता था। यह शब्द गाँवों में अब भी प्रचलित है। सत्रहवीं शताब्दी के ग्रामीण समाज से सम्बन्धित स्रोतों में दो प्रकार के किसानों का उल्लेख किया गया है; जो इस प्रकार है – खुद काश्त किसन जो उन्हीं गाँवों में निकास करते थे जिनमें उनकी जमीन होती थी अपनी जमीन पर खेती करने वाले किसानों को खुद काश्त किसान कहा जाता था। पाहि काश्त पाहि काश्त उन किसानों को कहा जाता था जो दूसरे गाँवों से आकर ठेके पर खेती लेकर कृषि कार्य करते थे। कभी-कभी लोग दूसरे गाँवों में खेती की अच्छी सम्भावनाओं, जैसेकि ठेके आदि की कम दरें और अच्छी उपज के लालच में पाहि काश्तकारी करते थे।

आर्थिक परेशानी, अकाल या भुखमरी तथा खुद की जमीन न होने पर भी लोग पाहि काश्तकारी करते थे। किसानों के पास उपलब्ध कृषि भूमि किसानों के पास उपलब्ध कृषि भूमि उनकी समृद्धि का मापदण्ड थी। सामान्य तौर पर उत्तर भारत के एक औसत किसान के पास अधिक भूमि न होने के कारण एक जोड़ी बैल और दो हल से अधिक कुछ नहीं होता था। अधिकांश किसानों के पास इससे भी न्यून साधन होते थे किसानों के पास उपलब्ध साधनों के आधार पर हम कह सकते हैं कि भूमि की उपलब्धता कम थी। गुजरात में 6 एकड़ जमीन का स्वामित्व रखने वाले किसान समृद्ध कहे जाते थे। बंगाल में औसत किसान के पास अधिक से अधिक 5 एकड़ जमीन होती थी।

10 एकड़ जमीन रखने वाले किसान अमीर समझे जाते थे। कृषि भूमि की मिल्कियत जिस प्रकार अन्य सम्पत्तियाँ खरीदी और बेची जाती थीं, उसी प्रकार कृषि भूमि भी व्यक्तिगत मिल्कियत के सिद्धान्त पर आधारित थी। व्यक्तिगत मिल्कियत होने के नाते किसान कृषि भूमि तथा क्रय-विक्रय अन्य सम्पत्तियों की भाँति कर सकते हैं। किसानों की मिल्कियत के बारे में एक विवरण उन्नीसवीं सदी के दिल्ली-आगरा प्रदेश के किसानों के सम्बन्ध में प्राप्त होता है। उपर्युक्त विवरण सत्रहवीं सदी के किसानों पर भी इसी भाँति लागू होता है। किसानों की दशा तत्कालीन समय के इतिहासकारों में किसानों की सामाजिक व आर्थिक स्थिति के बारे में मतभेद है।

JAC Class 12 History Important Questions Chapter 8 किसान, ज़मींदार और राज्य : कृषि समाज और मुगल साम्राज्य

प्रश्न 3.
मुगलकाल में प्रचलित सिंचाई साधनों की वर्तमान कृषि में उपयोगिता को समझाइये।
अथवा
‘बाबरनामा’ में वर्णित भारत में सिंचाई के साधनों एवं उपकरणों का विवेचन कीजिए।
उत्तर:
(1) मुगलकाल में प्रचलित सिंचाई साधनों की वर्तमान कृषि में उपयोगिता आज की भाँति मानसून मुगलकाल में भी भारतीय कृषि को रोड़ थी परन्तु कुछ ऐसी फसलें भी थीं जिनके लिए अतिरिक्त पानी की आवश्यकता थी। अतः मुगल काल में वर्षा के अतिरिक्त सिंचाई तालाबों, कुओं, नहरों एवं बांधों के द्वारा होती थी। मुगल काल में प्रचलित सिंचाई साधनों की वर्तमान कृषि में काफी उपयोगिता है। आज भी देश की अधिकांश आबादी गांवों में रहती है। यद्यपि आज धनी लोगों द्वारा ट्यूबवैल आदि आधुनिक साधनों का उपयोग किया जाता है, परन्तु अधिकांशतः खेतों की सिंचाई वर्षा, कुओं, तालाबों, नहरों, बांधों के द्वारा ही की जाती है।

(2) ‘बाबरनामा में वर्णित भारत में सिंचाई के साधन-
(i) बहते पानी के प्रबन्ध का अभाव- भारत का -अधिकतर भाग मैदानी जमीन पर बसा हुआ है। यद्यपि यहाँ शहर और खेती योग्य जमीन की प्रचुरता थी, परन्तु कहीं भी बहते पानी का प्रबन्ध नहीं किया जाता था। इसलिए फसलें उगाने या बागानों के लिए पानी की बिल्कुल आवश्यकता नहीं थी। शरद ऋतु की फसलें वर्षा के पानी से ही पैदा हो जाती थीं यह बड़े आश्चर्य की बात है कि बसन्त ऋतु की फसलें तो वर्षा के अभाव में भी पैदा हो जती थीं।

(ii) रहट द्वारा सिंचाई ‘बाबरनामा’ के अनुसार छोटे पेड़ों तक रहट या बाल्टियों के द्वारा पानी पहुंचाया जाता था लाहौर, दीपालपुर (वर्तमान में पाकिस्तान में) और ऐसे दूसरे स्थानों पर लोग रहट के द्वारा सिंचाई करते थे लोग रस्सी के दो गोलाकार फंदे बनाते थे, जो कुएँ की गहराई के अनुसार लम्बे होते थे। इन फन्दों में थोड़ी-थोड़ी दूरी पर लकड़ी के गुटके लगाए जाते थे और इन गुटकों पर पड़े बांध दिए जाते थे।

लकड़ी के गुटकों और घड़ों से बंधी इन रस्सियों को कुएँ के ऊपर पहियों से लटकाया जाता था। पहिये की धुरी पर एक ओर पहिया होता था। इस अन्तिम पहिए को बैल के द्वारा घुमाया जाता था इस पहिये के दाँत पास के दूसरे पहिए के दांतों को जकड़ लेते थे और इस प्रकार घड़ों वाला पहिया घूमने लगता था। घड़ों से जहाँ पानी गिरता था, वहाँ एक संकरा नाला खोद दिया जाता था और इस विधि से प्रत्येक स्थान पर पानी पहुँचाया जाता था।

(iii) बाल्टियों से सिंचाई ‘बाबरनामा’ के अनुसार वर्तमान उत्तर प्रदेश में स्थित आगरा, चांदवर तथा बवाना में और ऐसे अन्य प्रदेशों में भी लोग बाग बाल्टियों से सिंचाई करते थे। वे कुएँ के किनारे पर लकड़ी के कन्ने गाड़ देते थे। वे इन फन्नों के मध्य बेलन टिकाते थे, एक बड़ी बाल्टी में रस्सी बांधते थे रस्सी को बेलन पर लपेटते थे और इसके दूसरे छोर को बैल से बाँध देते थे। एक व्यक्ति बैल हाँकता था तथा दूसरा बाल्टी से पानी निकालता था।

प्रश्न 4.
भारत में सोलहवीं तथा सत्रहवीं शताब्दियों में ग्रामीण दस्तकारों की स्थिति पर प्रकाश डालिए। उत्तर भारत में सोलहवीं तथा सत्रहवीं शताब्दियों में ग्रामीण दस्तकारों की स्थिति –
(1) गाँवों में बड़ी संख्या में दस्तकारों की उपलब्धता-अंग्रेजी शासन के प्रारम्भिक वर्षों में किए गए गाँवों के सर्वेक्षण तथा मराठाओं के दस्तावेजों से ज्ञात होता है कि गाँवों में दस्तकार काफी बड़ी संख्या में रहते थे। कुछ गाँवों में तो कुल घरों के 25 प्रतिशत घर दस्तकारों के थे।

(2) किसानों और दस्तकारों के बीच सम्बन्ध- कभी-कभी किसानों और दस्तकारों के बीच अन्तर करना कठिन होता था क्योंकि कई ऐसे समूह से जो दोनों प्रकार के काम करते थे। खेतिहर और उसके परिवार के सदस्य अनेक प्रकार की वस्तुओं के उत्पादन में भाग लेते थे। उदाहरणार्थ, वे रंगरेजी, कपड़ों पर छपाई, मिट्टी के बर्तनों को पकाने, खेती के औजार बनाने या उनकी मरम्मत का काम करते थे। बुआई और सुहाई के बीच या सुहाई और के बीच की अवधि में किसानों के पास खेती का काम नहीं होता था, इसलिए इस अवधि में ये खेतिहर दस्तकारी का काम करते थे।

JAC Class 12 History Important Questions Chapter 8 किसान, ज़मींदार और राज्य : कृषि समाज और मुगल साम्राज्य

(3) ग्रामीण दस्तकारों द्वारा गाँव के लोगों को सेवाएँ देना कुम्हार, लोहार, बढ़ई, नाई और सुनार जैसे ग्रामीण दस्तकार अपनी सेवाएं गाँव के लोगों को देते थे। इसके बदले गाँव के लोग उन्हें अलग-अलग तरीकों से उन सेवाओं की अदायगी करते थे। प्रायः वे या तो उन्हें फसल का एक भाग दे दिया करते थे या फिर गाँव की जमीन का एक टुकड़ा यह जमीन का टुकड़ा ऐसा था जो खेती योग्य होने के बावजूद बेकार पड़ा रहता था भुगतान का तरीका सम्भवतः पंचायत ही निश्चित करती थी। महाराष्ट्र में ऐसी जमीन दस्तकारों की ‘मीरास’ या ‘वतन’ बन गई जिस पर दस्तकारों का पैतृक अधिकार होता था।

यही व्यवस्था कभी-कभी थोड़े परिवर्तित रूप में भी पाई जाती थी। इसमें दस्तकार और प्रत्येक खेतिहर परिवार आपसी बातचीत द्वारा सेवाओं के भुगतान की कोई एक व्यवस्था तय कर लेते थे। इस स्थिति में प्रायः वस्तुओं और सेवाओं का विनिमय होता था उदाहरणार्थ, अठारहवीं सदी के स्त्रोतों से ज्ञात होता है कि बंगाल में जमींदार उनकी सेवाओं के बदले लोहारों, बढ़ई और सुनारों तक को “दैनिक भत्ता और खाने के लिए नकदी देते थे। इस व्यवस्था को ‘जजमानी’ कहते थे। इन तथ्यों से पता चलता है कि गाँव के छोटे स्तर पर विनिमय के सम्बन्ध काफी जटिल थे। परन्तु ऐसा नहीं है कि नकद अदायगी का प्रचलन बिल्कुल ही नहीं था।

प्रश्न 5.
भारत में सोलहवीं तथा सत्रहवीं शताब्दियों में कृषि समाज में महिलाओं की भूमिका का विवेचन कीजिए। समाज में उनकी स्थिति पर प्रकाश डालिए । उत्तर- भारत में सोलहवीं तथा सत्रहवीं शताब्दियों में कृषि समाज में महिलाओं की भूमिका-
(1) उत्पादन की प्रक्रिया में पुरुष और महिलाओं की भूमिका उत्पादन की प्रक्रिया में पुरुष और महिलाएँ विशेष प्रकार की भूमिकाएं निभाते हैं। भारत में सोलहवीं तथा सत्रहवीं शताब्दियों में भी महिलाएँ और पुरुष कन्धे से कंधा मिलाकर खेतों में काम करते थे।

पुरुष हल जोतते थे तथा हल चलाते थे और महिलाएँ बुआई, निराई और कटाई के साथ-साथ पकी हुई फसल से दाना निकालने का काम करती थीं। फिर भी महिलाओं की जैव वैज्ञानिक क्रियाओं को लेकर लोगों के मन में पूर्वाग्रह बने रहे। उदाहरण के लिए, पश्चिमी भारत में मासिक धर्म वाली महिलाओं को हल या कुम्हार का चाक छूने की अनुमति नहीं थी। इसी प्रकार बंगाल में अपने मासिक धर्म के समय महिलाएँ पान के बागान में प्रवेश नहीं कर सकती थीं।

(2) दस्तकारी के कामों में महिलाओं का योगदान- सूत कातने वर्तन बनाने के लिए मिट्टी को साफ करने और गूंथने, कपड़ों पर कढ़ाई जैसे दस्तकारी के कार्य महिलाओं के श्रम पर निर्भर थे।

(3) महत्त्वपूर्ण संसाधन के रूप में महिलाएं श्रम प्रधान समाज में बच्चे उत्पन्न करने की अपनी योग्यता के कारण महिलाओं को महत्त्वपूर्ण संसाधन के रूप में देखा
जाता था।

समाज में महिलाओं की स्थिति-
(1) विवाहित महिलाओं की कमी तत्कालीन युग में विवाहित महिलाओं की कमी थी क्योंकि कुपोषण, बार-बार माँ बनने और प्रसव के समय मृत्यु हो जाने के कारण महिलाओं में मृत्यु दर बहुत अधिक थी इससे किसान और दस्तकार समाज में ऐसे सामाजिक रीति- रिवाज उत्पन्न हुए, जो अभिजात वर्ग के लोगों से बहुत अलग थे। कई ग्रामीण सम्प्रदायों में विवाह के लिए ‘दुल्हन की कीमत अदा करनी पड़ती थी, न कि दहेज की। तलाकशुदा महिलाएँ और विधवाएँ दोनों ही कानूनी रूप से विवाह कर सकती थीं।

(2) महिलाओं पर नियन्त्रण रखना प्रचलित रिवाज के अनुसार घर का मुखिया पुरुष होता था। इस प्रकार महिला पर परिवार और समुदाय के पुरुषों का पूर्ण नियन्त्रण बना रहता था दूसरे पुरुषों के साथ अवैध सम्बन्ध रखने के शक पर ही महिलाओं को भयानक दंड दिए जाते थे।

(3) महिलाओं द्वारा अपने पतियों की बेवफाई का विरोध-राजस्थान, गुजरात और महाराष्ट्र आदि पश्चिमी भारत के प्रदेशों से प्राप्त दस्तावेजों में ऐसे प्रार्थना-पत्र मिले हैं, जो महिलाओं ने न्याय और मुआवजे की आशा से ग्राम पंचायतों को भेजे थे। इनमें पत्नियाँ अपने पतियों की बेवफाई का विरोध करती हुई दिखाई देती हैं या फिर गृहस्थी के पुरुष पर पत्नी और बच्चों की अवहेलना करने का आरोप लगाती हुई नजर आती हैं।

JAC Class 12 History Important Questions Chapter 8 किसान, ज़मींदार और राज्य : कृषि समाज और मुगल साम्राज्य

(4) महिलाओं को पैतृक सम्पत्ति का अधिकार- भूमिहर भद्रजनों में महिलाओं को पैतृक सम्पत्ति का अधिकार प्राप्त था।

प्रश्न 6.
मुगलकाल में सिंचाई, कृषि तकनीक व फसलों का वर्णन कीजिए।
उत्तर:
सिंचाई- भारत के किसानों की पारिश्रमिक प्रवृत्ति और कृषि से प्राप्त उनकी गतिशीलता, कृषि योग्य भूमि व श्रम की उपलब्धता के कारण कृषि का निरन्तर विस्तार हुआ खाद्यान्न फसलों, जैसे गेहूं, चावल, ज्वार आदि की खेती प्रचुरता के साथ की जाती थी। कृषि मुख्य रूप से मानसून पर आधारित थी: अधिक वर्षा वाले इलाकों में धान की खेती एवं कम वर्षा वाले इलाकों में गेहूं व बाजरे आदि की खेती की जाती थी कुछ फसलें ऐसी थीं, जिनके लिए अधिक पानी की आवश्यकता पड़ती थी।

ऐसी फसलों के लिए सिंचाई के कृत्रिम साधन विकसित करने पड़े। सिंचाई के कृत्रिम उपायों के विकास में युगल शासकों ने व्यापक रुचि ली। किसानों को व्यक्तिगत रूप से कुएँ तालाब, बाँध आदि बनाने के लिए राज्य से सहायता दी गई। कई-कई नहरों व नालों का निर्माण राज्य द्वारा करवाया गया।

शाहजहाँ के शासन काल में पुरानी नहरों की मरम्मत करवाई गई, जैसेकि पंजाब की शाह नहर कृषि तकनीकें खेती में मानवीय श्रम को कम करने के लिए किसानों ने पशुबल पर आधारित तकनीकों में नए सुधार किए। पहले जुताई के लिए लकड़ी के फाल वाले हल्के हल का प्रयोग किया जाता था।

लकड़ी के फाल वाले हल जमीन की गहरी जुताई कर सकने में सक्षम नहीं थे इसलिए किसानों ने नुकीली धार वाले लोहे के फाल का प्रयोग करना प्रारम्भ कर दिया लोहे की फाल लगा हल आसानी से गहरी जुताई कर सकता था और हल खींचने वाले पशुओं पर जोर भी कम पड़ता था। मिट्टी की गुड़ाई और निराई के लिए लोहे के पतले धार वाले यन्त्र का प्रयोग किया जाने लगा। बीजों की बुआई के लिए बरमे का प्रयोग किया जाता था, जो बैलों द्वारा खींचा जाता था। हाथ से छिड़क कर भी बीज बोए जाते थे।

भरपूर फसलें मौसम के चक्रों पर आधारित स्त्री और खरीफ की फसलें मुख्य थीं खरीफ की फसल पतझड़ में और रबी की फसल वसंत में ली जाती थी। जहाँ पर सिंचाई की पर्याप्त व्यवस्था होती थी, उन क्षेत्रों में तीन फसलें ली जाती थीं। मुख्यतया खाद्यान्न फसलों की खेती पर अधिक जोर दिया जाता था परन्तु कुछ नकदी फसलें भी उगाई जाती थी, जिन्हें ‘जिन्स-ए-कामिल’ म्यानि सर्वोत्तम फसलें कहा जाता था, जैसे- गन्ना, कपास, सरसों, नील आदि ।

प्रश्न 7.
आइन-ए-अकबरी’ मुगलकालीन
शासन व्यवस्था की जानकारी का एक महत्त्वपूर्ण स्त्रोत है।” विवेचना कीजिए।
अथवा
‘आइन-ए-अकबरी’ पर एक संक्षिप्त निबन्ध लिखिए।
अथवा
‘आइन-ए-अकबरी’ के ऐतिहासिक महत्व की विवेचना कीजिए।
उत्तर:
अबुल फजल की ‘आइन-ए-अकबरी’
(1) आइन-ए-अकबरी’ का संक्षिप्त परिचय- मुगल सम्राट अकबर के आदेश पर उसके दरबारी इतिहासकार अबुल फजल ने ‘आइन-ए-अकबरी’ की रचना की थी। 1598 ई. में पाँच संशोधनों के बाद, इस ग्रन्थ को पूरा किया गया। ‘आइन’ इतिहास लेखन की एक ऐसी विशाल परियोजना का भाग थी, जिसकी पहल अकबर ने की थी। ‘आइन” अक्रबरनामा’ की तीसरी जिल्द थी।

(2) ‘आइन’ की विषय-वस्तु’ आइन’ में निम्नलिखित विषयों पर विस्तार से चर्चा की गई है-

  • दरबार, प्रशासन और सेना का संगठन
  • राजस्व के स्रोत और अकबर के साम्राज्य के प्रान्तों का भूगोल ।
  • लोगों के साहित्यिक, सांस्कृतिक एवं धार्मिक रिवाज।
  • अकबर की सरकार के समस्त विभागों एवं प्रान्तों (सूर्वो) के बारे में विस्तार से जानकारी देने के अतिरिक्त, ‘आइन’ इन सूबों के बारे में जटिल और आँकड़ेबद्ध सूचनाएँ बड़ी बारीकी से देती है।

(3) महत्त्व इन सूचनाओं को इकट्ठा करके, उन्हें ‘क्रमबद्ध तरीके से संकलित करना एक महत्त्वपूर्ण शाही प्रक्रिया थी। इसने सम्राट को साम्राज्य के समस्त प्रदेशों में प्रचलित रिवाजें और व्यवसायों की जानकारी दी। इस प्रकार हमारे लिए’ आइन’ अकबर के शासनकाल से सम्बन्धित मुगल साम्राज्य के बारे में सूचनाओं की खान है।

(4) आइन’ के दफ्तर (भाग) आइन पांच भागों (दफ्तरों) का संकलन है। इसके पहले तीन भाग प्रशासन का विवरण देते हैं।
(i) मंजिल आबादी ‘मंजिल आबादी’ के नाम से पहला ग्रन्थ शाही घर-परिवार और उसके रख-रखाव से सम्बन्ध रखता है।

(ii) सिपह- आबादी- दूसरा भाग ‘सिपह आबादी’ सैनिक व नागरिक प्रशासन और नौकरों की व्यवस्था के बारे में है। इस भाग में शाही अधिकारियों (मनसबदारों), विद्वानों, कवियों और कलाकारों की संक्षिप्त जीवनियाँ सम्मिलित हैं।

(iii) मुल्क आबादी- तीसरे भाग ‘मुल्क- आबादी’ में साम्राज्य व प्रान्तों के वित्तीय पहलुओं तथा राजस्व की दरों के आँकड़ों की विस्तृत जानकारी दी गई है। इसके बाद ‘बारह प्रान्तों का वर्णन’ दिया गया है। इस खण्ड में सांख्यिकी सूचनाएँ विस्तार से दी गई हैं। इसमें सूबों (प्रान्तों) और उनकी समस्त प्रशासनिक तथा वित्तीय इकाइयों (सरकार, परगना और महल) के भौगोलिक, स्थलाकृतिक और आर्थिक रेखाचित्र भी सम्मिलित हैं। इसी खण्ड में प्रत्येक प्रान्त और उसकी अलग-अलग इकाइयों की कुल मापी गई जमीन और निर्धारित राजस्व (जमा) की जानकारी भी दी गई है।

JAC Class 12 History Important Questions Chapter 8 किसान, ज़मींदार और राज्य : कृषि समाज और मुगल साम्राज्य

‘सरकार’ सम्बन्धी जानकारी-
सूबा स्तर की विस्तृत जानकारी देने के बाद, ‘आइन’ हमें सूबों के नीचे की इकाई ‘सरकारों’ के बारे में विस्तृत जानकारी देती है। ये सूचनाएँ तालिकाबद्ध तरीके से दी गई हैं। हर तालिका में आठ खाने हैं जो हमें निम्नलिखित सूचनाएं देते हैं-

परगनात / महल
(ii) किला
(iii) अराजी और जमीन-ए- पाईंमूद (मापे गए इलाके)
(iv) नकदी (नकद निर्धारित राजस्व)
(v) सुयूगल (दान में दिया गया राजस्व अनुदान)
(vi) जमींदार
सातवें तथा आठवें खानों में जमींदारों की जातियों और उनके घुड़सवार, पैदल सैनिक (प्यादा) व हाथी (फ्री) सहित उनकी सेना की जानकारी दी गई है। ‘मुल्क- आबादी’ नामक खण्ड उत्तर भारत के कृषि-समाज का विस्तृत, आकर्षक और जटिल चित्र प्रस्तुत करता है।
(iv) और
(v) भाग-ये भाग भारतवासियों के धार्मिक, साहित्यिक तथा सांस्कृतिक रीति-रिवाजों से सम्बन्ध रखते हैं।

प्रश्न 8.
आइन-ए-अकबरी के महत्त्व एवं सीमाओं की विवेचना कीजिए।
अथवा
महत्व की विवेचना ‘आइन-ए-अकबरी’ के कीजिए और इसकी त्रुटियों पर प्रकाश डालिए।
उत्तर:
‘आइन-ए-अकबरी’ का महत्त्व एवं त्रुटियाँ (सीमाएँ ) यद्यपि ‘आइन-ए-अकबरी’ को मुगल सम्राट अकबर के शासन की सुविधा के लिए आधिकारिक तौर पर विस्तृत सूचनाएँ संकलित करने के लिए प्रायोजित किया गया था, मा फिर भी यह ग्रन्थ आधिकारिक दस्तावेजों की नकल नहीं है। अबुलफजल की ग्रन्थ लेखन में बड़ी रुचि थी इसी कारण उसने इसकी पांडुलिपि को पाँच बार संशोधित किया था।

त्रुटियाँ इतिहासकारों के अनुसार ‘आइन’ की प्रमुख त्रुटियाँ (सीमाएँ ) निम्नलिखित हैं-
(1) जोड़ करने में गलतियाँ जोड़ करने में कई गलतियाँ पाई गई हैं। ऐसा माना जाता है कि या तो ये अंकगणित की छोटी-मोटी चूक है अथवा फिर नकल उतारने के दौरान अबुल फजल के सहयोगियों की भूल।
(2) संख्यात्मक आँकड़ों में विषमताएँ आइन’ के संख्यात्मक आँकड़ों में विषमताएँ हैं सभी सूबों (प्रान्तों) से आँकड़े एक ही रूप में एकत्रित नहीं किए गए हैं।
(3) कीमतों और मजदूरी से सम्बन्धित आँकड़ों का विस्तार से वर्णन नहीं करना इसी प्रकार जहाँ सूर्यो से लिए गए राजकोषीय आँकड़े बड़े विस्तार से दिए गए हैं, वहीं उन्हीं प्रदेशों से कीमतों और मजदूरी जैसे इतने ही महत्वपूर्ण आँकड़े इतने विस्तार से साथ नहीं दिए गए हैं।
(4) मूल्यों और मजदूरों की दरों की अपर्याप्त सूची- मूल्यों और मजदूरी की दरों की जो विस्तृत सूची ‘आइन’ में दी भी गई है, वह मुगल साम्राज्य की राजधानी आगरा या उसके आस-पास के प्रदेशों से ली गई है।

निष्कर्ष –
(1) असामान्य एवं अनूठा ग्रन्थ- कुछ त्रुटियों के होते हुए भी, ‘आइन’ अपने समय के लिए एक असामान्य एवं अनूठा दस्तावेज है। मुगल साम्राज्य के गठन और उसकी संरचना की अत्यन्त आकर्षक झलकियाँ दर्शा कर और उसके निवासियों व उत्पादों के सम्बन्ध में सांख्यिकीय आँकड़े देकर, अबुल फसल मध्यकालीन इतिहासकारों से कहीं आगे निकल गए।

(2) भारत के लोगों और उनके व्यवसायों के बारे में विस्तृत जानकारी देना मध्यकालीन भारत में अबुल फसल से पहले के इतिहासकार अधिकतर राजनीतिक घटनाओं के बारे में ही लिखते थे युद्ध-विजय, राजनीतिक प या वंशीय उथल-पुथल से सम्बन्धित विवरण ग्रन्थों में रहते थे। भारत के लोगों और मुगल साम्राज्य के बारे में विस्तृत सूचनाएँ देकर ‘आइन’ ने स्थापित परम्पराओं को पीछे छोड़ दिया और इस प्रकार सत्रहवीं सदी के भारत के अध्ययन के लिए एक संदर्भ बिन्दु बन गया। जहाँ तक कृषि सम्बन्धों का प्रश्न है, ‘आइन’ के सांख्यिकी प्रमाणों के महत्व को चुनौती नहीं दी जा सकती।

JAC Class 12 History Important Questions Chapter 8 किसान, ज़मींदार और राज्य : कृषि समाज और मुगल साम्राज्य

प्रश्न 9.
मुगलों की भू-राजस्व प्रणाली की विवेचना कीजिए।
अथवा
अकबर की भू-राजस्व व्यवस्था का विवेचन कीजिए।
उत्तर:
मुगलों अथवा अकबर की भू-राजस्व व्यवस्था मुगलों अथवा अकबर की भू-राजस्व व्यवस्था की प्रमुख विशेषताओं का विवेचन निम्नलिखित बिन्दुओं के अन्तर्गत किया जा सकता है
(1) भू-राजस्व – मुगल साम्राज्य की आय का प्रमुख साधन-भू-राजस्व मुगल साम्राज्य की आय का प्रमुख साधन था अतः कृषि उत्पादन पर नियंत्रण रखने के लिए सम्पूर्ण साम्राज्य में राजस्व आकलन तथा वसूली के लिए एक प्रशासनिक तन्त्र की स्थापना की गई।
(2) भू-राजस्व के सम्बन्ध में सूचनाएँ एकत्रित करना भू-राजस्व प्रबन्ध के अन्तर्गत दो चरण थे-
(1) कर निर्धारण तथा
(2) वास्तविक वसूली जमा निर्धारित रकम थी तथा हासिल वास्तविक वसूली गई रकम राजस्व निर्धारित करते समय राज्य अपना हिस्सा अधिक से अधिक रखने का प्रयास करता था, परन्तु कभी-कभी वास्तव में इतनी वसूली कर पाना सम्भव नहीं हो पाता था।
(3) भूमि की पैमाइश अकबर ने जुती हुई जमीन तथा खेती योग्य भूमि दोनों की पैमाइश करवाई।
(4) भूमि का वर्गीकरण- कृषि योग्य भूमि को निम्नलिखित चार वर्गों में बांटा गया-

  • पोलज पोलज भूमि सबसे उत्तम भूमि थी जिस पर प्रत्येक वर्ष खेती होती थी।
  • परीती इस भूमि पर भी वर्ष भर खेती होती थी, परन्तु उसे पुनः उर्वरा शक्ति प्रदान करने के लिए एक अथवा दो वर्ष के लिए खाली छोड़ दिया जाता था।
  • चचर ऐसी भूमि जिस पर तीन वर्ष या चार वर्ष खेती नहीं की जाती थी, चचर भूमि कहलाती थी।
  • बंजर इस भूमि पर पाँच अथवा इससे भी अधिक वर्षों तक खेती नहीं की जाती थी।

(5) भूमि कर निर्धारण पोलज तथा परौती भूमि को तीन श्रेणियों में बाँटा गया था
(1) अच्छी
(2) मध्यम
(3) खराब प्रत्येक भूमि की औसत पैदावार का पता लगाया जाता था और इसका तीसरा भाग औसत पैदावार माना जाता था। इस औसत पैदावार का एक तिहाई हिस्सा भूमि-कर के रूप में लिया जाता था।

(6) भूमि कर व्यवस्था की अन्य प्रणालियाँ- अकबर ने आदेश दिया कि भूमि कर केवल नकदी के रूप में नहीं, बल्कि फसलों के रूप में भी वसूल किया जाए। इसके लिए निम्नलिखित प्रणालियाँ अपनाई गई

  • कणकुत प्रणाली- इसके अनुसार खड़ी फसल के अनुमान के आधार पर सरकारी लगान निश्चित किया जात था और फसल कटने पर सरकार अपना भाग ले लेती थी।
  • बटाई (भाओली) प्रणाली- इसमें फसल काट कर जमा कर लेते थे और फिर सभी पक्षों की उपस्थिति में व सहमति से बंटवारा करते थे।
  • खेत बटाई- इसमें बीज बोने के बाद खेत बाँट लिए जाते थे।
  • लाँग बटाई- इसमें फसल काटने के बाद उसके ढेर बना लिए जाते थे और फिर उसे आपस में बाँट लेते थे।

(7) अकबर के बाद मुगलों की भू-राजस्व व्यवस्था-अकबर के बाद के मुगल सम्राटों के शासन- काल में भी भूमि की नपाई के प्रयास जारी रहे। 1665 में औरंगजेब ने अपने राजस्व अधिकारियों को आदेश दिया कि प्रत्येक गाँव में खेतिहरों की संख्या का वार्षिक हिसाब रखा जाए। परन्तु इसके बावजूद सभी प्रदेशों की नपाई सफलतापूर्वक नहीं हुई।

प्रश्न 10.
मुगलकाल में भू-राजस्व निर्धारण की कौनसी प्रणालियाँ प्रचलित थीं?
उत्तर:
मुगलकाल के भू-राजस्व निर्धारण की प्रणालियाँ अकबर ने राजस्व अधिकारियों को आदेश दिया था कि वे लगान नकदी में लेने की आदत न डालें, बल्कि फसल भी लेने के लिए तैयार रहें। अतः अकबर के शासनकाल में भूमि कर निर्धारण के लिए निम्नलिखित प्रणालियाँ अपनाई गई-
(1) कणकुत प्रणाली हिन्दी भाषा में ‘कण’ का अर्थ है-‘ अनाज’ और ‘कुल’ का अर्थ है-‘ अनुमान’ इसके अन्तर्गत फसल को तीन अलग-अलग पुलिन्दों में काटा जाता था

  • अच्छा
  • मध्यम
  • खराब इस प्रणाली के अनुसार खड़ी फसल के अनुमान के आधार पर सरकारी लगान निश्चित किया जाता था और फसल कटने पर सरकार अपना भाग लेती थी। प्रायः अनुमान से किया गया जमीन का आकलन भी पर्याप्त रूप से सही परिणाम देता था।

(2) बटाई या भाओली इस प्रणाली के अन्तर्गत फसल काटकर जमा कर लेते थे और फिर सभी पक्षों की उपस्थिति में व आपसी सहमति से बँटवारा कर लेते थे। इसमें फसल का कुछ भाग सरकार द्वारा से लिया जाता था। परन्तु इसमें कई बुद्धिमान निरीक्षकों की आवश्यकता पढ़ती थी वरना दुष्ट बुद्धि मक्कार और धोखेबाज लोग बेइमानी करते थे तथा किसानों को धोखा देते थे।

(3) खेत बढाई इसमें खेत में बीज बो दिए जाते थे और बीज बोने के बाद खेत बाँट लेते थे। जिस प्रकार की फसल बोई जाती थी, उसी के अनुसार लगान निर्धारित होता था।

(4) लॉंग बटाई इसमें फसल काटने के बाद उसके देर बना लिए जाते थे और फिर उन्हें आपस में बाँट लेते थे प्रत्येक पक्ष फसल का अपना भाग लेकर घर ले जाता था और उससे मुनाफा कमाता था।

JAC Class 12 History Important Questions Chapter 8 किसान, ज़मींदार और राज्य : कृषि समाज और मुगल साम्राज्य

प्रश्न 11.
भारत में सोलहवीं तथा सत्रहवीं सदी में चाँदी के प्रवाह का वर्णन कीजिए। भारत में चाँदी के प्रवाह के बारे में इटली के यात्री जोवान्नी कारेरी ने क्या वर्णन किया है?
उत्तर:
भारत में सोलहवीं तथा सत्रहवीं सदी में चाँदी का प्रवाह सोलहवीं तथा सत्रहवीं सदी में भारत में व्यापार की बहुत अधिक उन्नति हुई खोजी यात्राओं से और ‘नई दुनिया’ की खोज से यूरोप के साथ भारत के व्यापार में अत्यधिक विस्तार हुआ। इसके परिणामस्वरूप भारत के समुद्र पार व्यापार की महत्त्वपूर्ण उन्नति हुई तथा कई नई वस्तुओं का व्यापार भी शुरू हो गया। निरन्तर बढ़ते व्यापार के साथ भारत से निर्यात होने वाली वस्तुओं का भुगतान करने के लिए एशिया में भारी मात्रा में चाँदी का आगमन हुआ। इस चाँदी का एक बड़ा हिस्सा भारत पहुँचा। यह स्थिति भारत के लिए लाभप्रद थी क्योंकि यहाँ चाँदी के प्राकृतिक संसाधन नहीं थे। परिणामस्वरूप, सोलहवीं से अठारहवीं सदी के बीच भारत में धातु मुद्रा – विशेषकर चांदी के रुपयों की प्राप्ति में अच्छी स्थिरता बनी रही। इसके दो लाभ हुए-
(1) अर्थव्यवस्था में मुद्रा- संचार तथा सिक्कों की ढलाई में अभूतपूर्व विस्तार हुआ तथा –
(2) मुगल राज्य को नकदी कर वसूलने में आसानी हुई।

इटली के यात्री जोवान्नी कारेरी का वृत्तांत-
लगभग 1690 ई. में इटली का यात्री जोवान्नी कारेरी भारत से होकर गुजरा था वह यहाँ चाँदी की प्रचुरता देखकर आश्चर्यचकित हो गया। उसने अपने यात्रा वृत्तांत में लिखा कि मुगल साम्राज्य में चाँदी भारी मात्रा में समस्त विश्व में से होकर भारत पहुँचती थी। उसने लिखा है कि समस्त है संसार में विचरण करने वाला समस्त सोना-चाँदी अन्ततः भारत पहुँच जाता है। इसका बहुत बड़ा भाग अमेरिका से आता है और यूरोप के कई राज्यों से होते हुए, इसका कुछ भाग कई प्रकार की वस्तुओं के लिए तुर्की में जाता है और थोड़ा-सा भाग रेशम के लिए फारस पहुँचता है। तुर्की लोग कॉफी से अलग नहीं रह सकते और न ही फारस अरविया तथा तुर्की के लोग भारत की वस्तुओं के बिना रह सकते हैं।

चे मुद्रा की विशाल मात्रा मोचा भेजते हैं। इसी प्रकार वे ये वस्तुएँ बसरा भेजते हैं। बाद में यह समस्त सम्पत्ति जहाजों से भारत भेज दी जाती है भारतीय जहाजों के अतिरिक्त जो डच, अंग्रेजी और पुर्तगाली जहाज प्रतिवर्ष भारत की वस्तुएँ लेकर पेगू, स्याम, श्रीलंका, मालद्वीप, मोजाम्बीक और अन्य स्थानों पर ले जाते हैं, इन्हीं जहाजों को निश्चित तौर पर विपुल सोना-चाँदी इन देशों से लेकर भारत पहुँचना पड़ता है। वह सब कुछ, जो डच लोग जापान की खानों से प्राप्त करते हैं अन्ततः भारत चला जाता है। यहाँ से यूरोप को जाने वाली समस्त वस्तुएँ चाहे वे फ्रांस जाएँ या इंग्लैण्ड या पुर्तगाल, सभी नकद में खरीदी जाती हैं, जो नकद भारत में रह जाता है।

प्रश्न 12.
उन स्त्रोतों का उल्लेख कीजिए जिनका प्रयोग अबुल फजल ने अपने ग्रन्थ ‘आइन-ए-अकबरी’ को पूरा करने के लिए किया।
उत्तर:
अबुल फजल द्वारा ‘अकबरनामा’ को पूरा करने के लिए प्रयुक्त किए गए स्त्रोत
(1) दस्तावेजों तथा प्रमाणों को एकत्रित करना- मुगल सम्राट अकबर ने अबुल फसल को आदेश दिया था कि वह साम्राज्य की शानदार घटनाओं और राज्य क्षेत्र अधीन करने वाली उनकी विजयों का ईमानदारी से वर्णन करे। अबुल फसल ने सम्राट के आदेश का पालन करते हुए अनेक दस्तावेज तथा प्रमाण इकट्ठे किये और राज्य के कर्मचारियों तथा शाही परिवार के सदस्यों से सूचनाएँ माँगीं।

JAC Class 12 History Important Questions Chapter 8 किसान, ज़मींदार और राज्य : कृषि समाज और मुगल साम्राज्य

(2) लोगों के विवरणों को नोट करना अबुल फसल ने सच बोलने वाले बुद्धिमान बुजुर्गों और तीव्र बुद्धिवाले, सत्यकर्मी जवानों दोनों की जाँच-पड़ताल की और उनके विवरणों को लिखित रूप में नोट किया।

(3) पुराने कर्मचारियों द्वारा लिखित संस्मरणों का अवलोकन करना- सूबों (प्रान्तों) को शाही आदेश जारी किया गया कि पुराने कर्मचारी अपने संस्मरण लिखें और उसे दरबार भेज दें। इसके बाद एक और शाही आदेश जारी किया गया कि जो सामग्री जमा की जाए, उसे सम्राट की उपस्थिति में पड़कर सुनाया जाए और बाद में जो कुछ भी

लिखा जाना हो, उसे उस महान ग्रन्थ में परिशिष्ट के रूप में जोड़ दिया जाए। अबुल फजल को आदेश दिया गया कि वह ऐसे विवरण जो उसी समय परिणति तक न लाए जा सकें, को बाद में नोट करे –

(4) घटनाओं का इतिहासवृत्त प्राप्त करना इस शाही आदेश के बाद अबुल फजल ने ऐसे कच्चे प्रारूप लिखने शुरू किये जिसमें शैली या विन्यास का सौन्दर्य नहीं था। उन्होंने इलाही संवत के उन्नीसवें वर्ष से जब बादशाह ने दस्तावेज कार्यालय स्थापित किया था, घटनाओं का इतिहासवृत्त प्राप्त किया और उसके पृष्ठों से कई मामलों का वृत्तान्त प्राप्त किया। उन सभी आदेशों की मूल प्रति वा नकल प्राप्त की गई, जो राज्याभिषेक से लेकर आज तक सूबों को जारी किए गए थे। अबुल फसल ने उनमें से कई रिपोर्टों को भी शामिल किया, जो साम्राज्य के विभिन्न मामलों या दूसरे देशों में घटी घटनाओं के बारे में थीं और जिन्हें उच्च अधिकारियों और मन्त्रियों ने भेजा था।

(5) कच्चे मसौदे तथा ज्ञानपत्र प्राप्त करना अबुल | फसल ने वे कच्चे मसौदे तथा ज्ञानपत्र भी प्राप्त किये जिन्हें जानकार और दूरदर्शी लोगों ने लिखा था।

प्रश्न 13.
मुगल काल में कृषि कीजिए।
की दशा का वर्णन
उत्तर:
मुगल काल में कृषि की दशा मुगल काल में कृषि की दशा का वर्णन निम्नलिखित बिन्दुओं के अन्तर्गत किया जा सकता है-
(1) फसलें मुगलकाल में भी कृषि भारतीयों के जीवन निर्वाह करने का प्रमुख साधन था किसान फसल की पैदावार से सम्बन्धित विविध प्रकार के कार्य करते थे, जैसे कि जमीन की जुताई, बीज बोना तथा फसल पकने पर उसकी कटाई वे उन वस्तुओं के उत्पादन में भी सम्मिलित होते थे जो कृषि आधारित थीं, जैसे कि शक्कर, तेल इत्यादि।

(2) फसलों की भरमार मौसम के दो मुख्य चक्रों के दौरान खेती की जाती थी एक खरीफ (पतझड़ में) तथा दूसरी रबी (बसन्त में)। अधिकांश स्थानों पर वर्ष में कम से कम दो फसलें होती थीं। जहाँ वर्षा या सिंचाई के अन्य साधन हर समय उपलब्ध थे वहाँ तो वर्ष में तीन फसलें भी उगाई जाती थीं। दैनिक आहार की खेती पर अधिक जोर दिया जाता था। कपास और गन्ना जैसी फसलें श्रेष्ठ ‘जिन्स- ए-कामिल थीं। मध्यभारत तथा दक्कनी पठार में कपास उगाई जाती थी। बंगाल गन्ने के उत्पादन के लिए प्रसिद्ध था। तिलहन (सरसों) और दलहन भी नकदी फसलों में गिनी जाती थीं। मक्का भारत में अफ्रीका और स्पेन से पहुँचा टमाटर, आलू व मिर्च जैसी सब्जियाँ नई दुनिया से लाई गई।

(3) सिंचाई के साधन आज की भाँति मानसून मुगल काल में भी भारतीय कृषि की रीढ़ था परन्तु कुछ ऐसी फसलें भी थीं, जिनके लिए अतिरिक्त पानी की आवश्यकता थी। ‘बाबरनामा’ से ज्ञात होता है कि छोटे पेड़ों तक रहट एवं बाल्टियों के द्वारा पानी पहुँचाया जाता था। इसके अतिरिक्त सिंचाई तालाबों, कुओं, नहरों एवं बाँधों के द्वारा भी होती थी। सिंचाई कार्यों को राज्य की ओर से प्रोत्साहन दिया जाता था।

(4) किसानों द्वारा तकनीकों का प्रयोग किसान प्राय: पशुबल पर आधारित तकनीकों का प्रयोग करते थे। वे लकड़ी के उस हल्के हल का प्रयोग करते थे जिसको एक छोर पर लोहे की नुकीली धार या फाल लगाकर आसानी से बनाया जा सकता था। ऐसे हल मिट्टी को बहुत गहरे नहीं खोदते थे जिसके कारण तेज गर्मी के महीनों में नमी बची रहती थी। बैलों के जोड़ों के सहारे खींचे जाने वाले बरमे का प्रयोग बीज बोने के लिए किया जाता था। मिट्टी की गुड़ाई और साथ-साथ निराई के लिए लकड़ी के मूठ वाले लोहे के पतले धार काम में लाए जाते थे।

(5) फलों के बाग मुगल सम्राटों ने फलों के अनेक बाग लगवाये।

(6) कृषि की दशा को सुधारने के लिए मुगल- सम्राटों के प्रयास मुगल सम्राटों ने किसानों को अच्छी फसलें गन्ना, कपास, तिलहन, दलहन आदि का उत्पादन करने के लिए प्रोत्साहित किया।

प्रश्न 14.
सोलहवीं तथा सत्रहवीं शताब्दी में किसानों की दशा का वर्णन कीजिए।
उत्तर:
सोलहवीं तथा सत्रहवीं शताब्दी में किसानों की दशा सोलहवीं तथा सत्रहवीं शताब्दी में किसानों की दशा का वर्णन निम्नलिखित बिन्दुओं के अन्तर्गत किया जा सकता है-
(1) किसानों के प्रकार-सोलहवीं तथा सत्रहवीं शताब्दी में भारतीय फारसी लोत ‘किसान’ के लिए चार शब्दों का प्रयोग करते थे-
(1) रैयत
(2) मुजरियान
(3) किसान और
(4) आसामी सत्रहवीं शताब्दी के स्रोत निम्नलिखित दो प्रकार के किसानों का उल्लेख करते हैं-
(i) खुद काश्त एवं
(ii) पाहि काश्त

(i) खुद काश्तखुद काश्त किसान वे थे जो उन्हीं गाँवों में रहते थे जिनमें उनकी जमीन थी।
(ii) पाहि काश्त पाहि कारण वे खेतिहर थे जो दूसरे गाँवों से ठेके पर खेती करने आते थे लोग अपनी इच्छा से भी पाहि-कारत बनते थे। उदाहरणार्थ-
(1) यदि करों की शर्तें किसी दूसरे गाँव में अधिक अच्छी मिलती थीं तथा
(2) अकाल या भुखमरी के बाद आर्थिक परेशानी के कारण।

(2) किसानों की समृद्धि का मापदंड उत्तर भारत में एक औसत किसान के पास सम्भवतः एक जोड़ी बैल तथा दो हल से अधिक कुछ नहीं होता था अधिकतर किसानों के पास इससे भी कम था। गुजरात में जिन किसानों के पास 6 एकड़ के लगभग जमीन थी, वे समृद्ध माने जाते थे। दूसरी ओर बंगाल में एक औसत किसान की जमीन की ऊपरी सीमा 5 एकड़ भी वहीं 10 एकड़ जमीन वाले आसामी को धनी समझा जाता था।

JAC Class 12 History Important Questions Chapter 8 किसान, ज़मींदार और राज्य : कृषि समाज और मुगल साम्राज्य

(3) व्यक्तिगत मिल्कियत खखेती व्यक्तिगत मिल्कियत के सिद्धान्त पर आधारित थी। किसानों की जमीन अन्य सम्पति मालिकों की भाँति खरीदी और बेची जाती थी। उन्नीसवीं सदी के दिल्ली-आगरा के प्रदेश में किसानों की मिल्कियत का यह विवरण सत्रहवीं सदी पर भी उतना ही लागू होता है- “हल जोतने वाले खेतिहर किसान हर जमीन की सीमाओं पर मिट्टी, ईंट और काँटों से पहचान के लिए निशान लगाते हैं और जमीन के ऐसे हजारों टुकड़े किसी भी गाँव में देखे जा सकते हैं।”

JAC Class 9 Hindi रचना पत्र लेखन-औपचारिक पत्र

Jharkhand Board JAC Class 9 Hindi Solutions Rachana पत्र लेखन-औपचारिक पत्र Questions and Answers, Notes Pdf.

JAC Board Class 9 Hindi Rachana पत्र लेखन-औपचारिक पत्र

आज के वैज्ञानिक युग में चाहे दूरभाष, वायरलेस, ई-मेल, स्काइप आदि के प्रयोग से दूर स्थित सगे-संबंधियों, सरकारी-गैर-सरकारी संस्थानों तथा व्यापारिक प्रतिष्ठानों से पल भर में बात की जा सकती है पर फिर भी पत्र – लेखन का अभी भी हमारे जीवन में महत्वपूर्ण स्थान है। पत्र – लेखन विचारों के आपसी आदान-प्रदान का सशक्त, सुगम और सस्ता साधन है। पत्र – लेखन केवल विचारों का आदान- प्रदान ही नहीं है बल्कि इससे पत्र – लेखक के व्यक्तित्व, दृष्टिकोण, चरित्र, संस्कार, मानसिक स्थिति आदि का ज्ञान हो जाता है। पत्र लिखते समय अनेक सावधानियाँ अवश्य ध्यान में रखनी चाहिए, जिनमें से कुछ प्रमुख हैं –

  • सरल और स्पष्ट भाषा का प्रयोग।
  • स्नेह, शिष्टता और भद्रता का निर्वाह।
  • पत्र प्राप्त करने वाले का पद, योग्यता, संबंध, सामर्थ्य, स्तर, आयु आदि।
  • अनावश्यक विस्तार से बचाव।
  • विषय-वस्तु की पूर्णता।
  • कड़वे, अशिष्ट और अनर्गल भावों तथा शब्दों का निषेध।

JAC Class 9 Hindi रचना पत्र लेखन-औपचारिक पत्र

पत्रों के भेद :

प्रमुख रूप से पत्रों को दो प्रकार का माना जाता है, जिनके आगे अनेक भेद – विभेद हो सकते हैं। वे हैं-
(क) औपचारिक पत्र (Formal Letters)
(ख) अनौपचारिक पत्र ( Informal Letters)

(क) औपचारिक पत्र (Formal Letters) :

जिन लोगों के साथ औपचारिक संबंध होते हैं, उन्हें औपचारिक पत्र लिखे जाते हैं। इन पत्रों में व्यक्तिगत और आत्मीय विचार प्रकट नहीं किए जाते। इनमें अपनेपन का भाव पूरी तरह गायब रहता है। इनमें अपने विचारों को भली-भाँति सोच-विचारकर प्रकट किया जाता है। प्रायः औपचारिक – पत्र सरकारी और गैर-सरकारी कार्यालयों के अधिकारियों, निजी संस्थानों, पत्र-पत्रिकाओं के संपादकों, अध्यापकों, प्रधानाचार्यों, व्यापारियों आदि को लिखे जाते हैं। औपचारिक पत्रों की रूपरेखा प्रायः निम्नलिखित आधारों पर निर्धारित की जाती है –

1. पत्र का क्रमांक
2. विभाग / कार्यालय / मंत्रालय का नाम
3. दिनांक
4. प्रेषक का नाम तथा पद
5. प्राप्तकर्ता का नाम तथा पद
6. विषय का संक्षिप्त उल्लेख
7. संबोधन
8. विषय-वस्तु
9. समापन शिष्टता
10. प्रेषक के हस्ताक्षर
11. प्रेषित का पद / नाम / पता

JAC Class 9 Hindi रचना पत्र लेखन-औपचारिक पत्र

औपचारिक पत्र का प्रारूप –

भारत सरकार के गृह मंत्रालय के सचिव की ओर से मुख्य सचिव, पंजाब राज्य को एक सरकारी- पत्र लिखें, जिसमें राज्य में कानून और व्यवस्था की बिगड़ती हुई स्थिति पर चिंता व्यक्त की गई हो।

JAC Class 9 Hindi रचना पत्र लेखन-औपचारिक पत्र 1

औपचारिक-पत्र के आरंभ और समापन की विधि –

JAC Class 9 Hindi रचना पत्र लेखन-औपचारिक पत्र 2

(ख) अनौपचारिक पत्र (Informal Letters) :

जिन लोगों के आपसी संबंध आत्मीय होते हैं, उनके द्वारा एक-दूसरे को लिखे जाने वाले पत्र अनौपचारिक पत्र कहलाते हैं। ऐसे पत्र प्रायः रिश्तेदारों, मित्रों, स्नेही संबंधियों आदि के द्वारा लिखे जाते हैं। इन पत्रों में एक-दूसरे के प्रति प्रेम, लगाव, गुस्से, उलाहने, आदर आदि के भाव अपनत्व के आधार पर स्पष्ट दिखाई देते हैं। इन पत्रों में बनावटीपन की मात्रा कम होती है। वाक्य संरचना बातचीत के स्तर पर आ जाती है।

मन के भाव और विचार बिना किसी संकोच के प्रकट किए जा सकते हैं। इनमें औपचारिकता का समावेश नहीं किया जाता। प्रायः सभी प्रकार के सामाजिक पत्रों को इसी श्रेणी में सम्मिलित कर लिया जाता है। विवाह, जन्म-दिन, गृहप्रवेश, मुंडन संस्कार, शोक सूचनाओं आदि को इन्हीं के अंतर्गत ग्रहण किया जाता है। अनौपचारिक पत्रों की रूपरेखा प्रायः निम्नलिखित आधारों पर निर्धारित की जाती है –

1. पत्र के बाईं ओर भेजने वाले का पता
2. दिनांक
3. पत्र प्राप्त करने के प्रति संबोधन
4. पत्र प्राप्त करने वाले से संबंधानुसार अभिवादन
5. पत्र में अनुच्छेदानुसार पत्र लिखने का कारण, विषय का विस्तार तथा परिवार के अन्य सदस्यों के प्रति अभिवादन
6. समापन
7. बाईं ओर पत्र – लेखक का पत्र प्राप्त करनेवाले से संबंध
8. पत्र लिखनेवाले का नाम

JAC Class 9 Hindi रचना पत्र लेखन-औपचारिक पत्र

अनौपचारिक पत्र का प्रारूप –

अपने मित्र को अपने विद्यालय में हुए वार्षिकोत्सव के विषय में पत्र लिखिए।

JAC Class 9 Hindi रचना पत्र लेखन-औपचारिक पत्र 3

JAC Class 9 Hindi रचना पत्र लेखन-औपचारिक पत्र

अनौपचारिक पत्र के आरंभ और समापन की विधि

JAC Class 9 Hindi रचना पत्र लेखन-औपचारिक पत्र 4

I. औपचारिक पत्र

(क) कार्यालयी – पत्र :

1. भारत सरकार के शिक्षा मंत्रालय की ओर से महाराष्ट्र सरकार को नई शिक्षा नीति लागू करने के लिए पत्र लिखिए।

रजत शर्मा
सचिव, भारत सरकार
शिक्षा मंत्रालय
दिनांक : 12 मार्च, 20XX
सेवा में
सचिव
शिक्षा विभाग
महाराष्ट्र राज्य मुंबई
विषय – नई शिक्षा नीति लागू करने हेतु पत्र।
महोदय

मुझे आपको यह सूचित करने का निर्देश दिया गया है कि भारत सरकार के निश्चयानुसार शिक्षा सत्र 20…. से संपूर्ण देश में नई शिक्षा नीति को लागू कर दिया जाए। महाराष्ट्र में भी इस नीति को क्रियान्वित किया जाए तथा इस योजना को लागू करने के लिए किए गए प्रयासों से मंत्रालय को भी परिचित कराया जाए।
आपका विश्वासपात्र
ह० रजत शर्मा
(रजत शर्मा)
सचिव, शिक्षा मंत्रालय

JAC Class 9 Hindi रचना पत्र लेखन-औपचारिक पत्र

2. उपायुक्त, सिरसा की ओर से मुख्य सचिव हरियाणा सरकार को एक पत्र लिखकर बाढ़ पीड़ितों की सहायता के लिए अनुरोध कीजिए।

बी० एस० यादव
उपायुक्त, सिरसा
दिनांक : 16 अगस्त, 20XX
सेवा में
मुख्य सचिव
हरियाणा राज्य
चंडीगढ़
विषय – बाढ़ पीड़ितों की सहायता हेतु पत्र।
महोदय
1. मैं आपको सूचित करता हूँ कि इन दिनों हुई भयंकर वर्षा के परिणामस्वरूप सिरसा के आसपास के गाँवों में भीषण बाढ़ की स्थिति उत्पन्न हो गई है। खड़ी फसलें नष्ट हो गई हैं तथा जन-धन और संपत्ति की भी बहुत हानि हुई है। मैंने अन्य जिला अधिकारियों के साथ स्थिति का निरीक्षण भी किया है।
2. ज़िला – स्तर पर बाढ़ पीड़ितों की यथासंभव सहायता की जा रही है, जो अपर्याप्त है। आपसे प्रार्थना है कि बाढ़ पीड़ितों को राज्य सरकार की ओर से विशेष आर्थिक सहायता प्रदान की जाए, जिससे वे अपने टूटे-फूटे मकान बना सकें तथा नष्ट हुई फ़सल के स्थान पर अगली फसल की तैयारी कर सकें।
3. बाढ़ के कारण बीमारियों का प्रकोप भी बढ़ गया है। जिला स्वास्थ्य अधिकारी इन बीमारियों पर नियंत्रण करने में पूर्णरूप से सक्षम नहीं हैं। अतः विशेष चिकित्सकों के दल को भी भेजने का प्रबंध करें।
भवदीय
बी० एस० यादव
उपायुक्त, सिरसा

JAC Class 9 Hindi रचना पत्र लेखन-औपचारिक पत्र

3. भारत सरकार के उद्योग मंत्रालय की ओर से हिमाचल प्रदेश सरकार के उद्योग सचिव को राज्य में उद्योग-धंधों को प्रोत्साहित करने के लिए एक पत्र लिखिए।

भारत सरकार
उद्योग मंत्रालय
नई दिल्ली
दिनांक : 22 सितंबर, 20XX
श्री पी० एस० यादव
सचिव, उद्योग विभाग, हिमाचल प्रदेश
विषय – उद्योग धंधों को प्रोत्साहित करने हेतु पत्र।
प्रिय श्री यादव जी
विगत दिनों हिमाचल प्रदेश के कुछ उद्योगपतियों ने इस मंत्रालय को राज्य में उद्योग-धंधों की दयनीय स्थिति से अवगत कराया। मुझे बहुत दुख हुआ कि उद्योग-धंधों की उपेक्षा के कारण जहाँ हिमाचल प्रदेश की आर्थिक स्थिति को आघात पहुँचा है, वहीं देश की प्रगति भी अवरुद्ध हो रही है। इस संबंध में आप व्यक्तिगत रूप से रुचि लें तथा प्रदेश में उद्योग-धंधों को प्रोत्साहित करने के लिए विभिन्न योजनाएँ चलाएँ। इस संदर्भ में विस्तृत योजनाएँ आपको अलग से भेजी जा रही हैं।
इन योजनाओं को लागू करने में यदि आपको कोई कठिनाई हो तो मुझसे संपर्क कर सकते हैं।
शुभकामनाओं सहित
आपकी सद्भावी
पूनम शर्मा
शिमला

JAC Class 9 Hindi रचना पत्र लेखन-औपचारिक पत्र

4. शारदा विद्या मंदिर, लखनऊ के प्राचार्य की ओर से महर्षि दयानंद विद्यालय के प्राचार्य को वार्षिक पुरस्कार वितरण समारोह में मुख्य अतिथि पद को स्वीकार करने हेतु एक पत्र लिखिए।

शारदा विद्या मंदिर
लखनऊ
दिनांक : 18 अक्तूबर, 20XX
सेवा में
डॉ० नरेश चंद्र मिश्र
प्राचार्य
महर्षि दयानंद विद्यालय
लखनऊ
विषय – मुख्य अतिथि पद के निमंत्रण हेतु पत्र।
प्रिय डॉ० मिश्र जी
हमारे विद्यालय का वार्षिक पुरस्कार वितरण समारोह 18 जनवरी, 20…. को आयोजित करने का निश्चय किया गया है। इसमें
मुख्य अतिथि
पद को सुशोभित करने हेतु आपसे निवेदन है कि आप अपने अति व्यस्त समय में से कुछ अमूल्य क्षण प्रदान कर हमें अनुगृहीत करें। इस समारोह में विद्यार्थियों को शैक्षणिक तथा विभिन्न सांस्कृतिक गतिविधियों में श्रेष्ठता प्रदर्शित करने के लिए पुरस्कृत किया जाएगा। यदि आप इस विषय में अपनी सुविधानुसार समय दे सकें तो मैं आपका विशेष रूप से आभारी होऊँगा।
शुभकामनाओं सहित
शुभाकांक्षी
डॉ० के० के० मेनन

JAC Class 9 Hindi रचना पत्र लेखन-औपचारिक पत्र

5. अपने क्षेत्र में मलेरिया फैलने की संभावना को देखते हुए स्वास्थ्य अधिकारी को पत्र लिखिए।

14 / 131, सुभाष नगर
सेलम
दिनांक : 17 मई, 20XX
स्वास्थ्य अधिकारी
नगर-निगम
सेलम
विषय – मलेरिया की रोकथाम हेतु पत्र।
आदरणीय महोदय
इस पत्र के द्वारा मैं अपने क्षेत्र ‘सुभाष नगर’ की सफ़ाई व्यवस्था की ओर आपका ध्यान दिलाना चाहता हूँ। आप जानते हैं कि यह क्षेत्र पर्याप्त ढलान पर बसा हुआ है। बरसात में यहाँ स्थान-स्थान पर पानी रुक जाता है। यही नहीं सड़कों के दोनों ओर गंदगी के ढेर लगे रहते हैं। सफाई कर्मचारी कोई ध्यान नहीं देते। परिणामस्वरूप यहाँ मक्खियों और मच्छरों का जमघट बना रहता है। रुका हुआ पानी मच्छरों की संख्या में तीव्र गति से वृद्धि करता है। अतः इस तरफ़ ध्यान न दिया गया तो इस क्षेत्र में मलेरिया फैलने की पूरी संभावना है। अतः आपसे प्रार्थना है कि इस क्षेत्र की तुरंत सफ़ाई करवाने का आदेश दें और मलेरिया की रोकथाम के लिए समुचित व्यवस्था करें। यही नहीं यहाँ के निवासियों को आवश्यक दवाइयाँ भी मुफ़्त उपलब्ध करवाई जाएँ।
आशा है कि आप मेरी प्रार्थना की ओर ध्यान देंगे और उचित प्रबंध द्वारा इस क्षेत्र के निवासियों को मलेरिया के प्रकोप से बचा लेंगे। भवदीय
अनुराग छाबड़ा

6. अपने नगर के जलापूर्ति अधिकारी को पर्याप्त और नियमित रूप से पानी न मिलने की शिकायत करते हुए पत्र लिखिए।

512, शास्त्री नगर
रोहतक
दिनांक : 14 मार्च, 20XX
जलापूर्ति अधिकारी
नगरपालिका
रोहतक
विषय – पेयजल की कठिनाई हेतु शिकायती पत्र।
मान्यवर
बड़े खेद की बात है कि पिछले एक मास से ‘शास्त्री नगर’ क्षेत्र में पेयजल की कठिनाई का अनुभव किया जा रहा है। नगरपालिका की ओर से पेयजल की सप्लाई बहुत कम होती जा रही है। कभी-कभी तो पूरा-पूरा दिन पानी नलों में नहीं आता। मकानों की पहली दूसरी मंजिल तक तो पानी चढ़ता ही नहीं। आप पानी की आवश्यकता के विषय में जानते हैं।
पानी की कमी के कारण यहाँ के लोगों का जीवन कष्टमय बन गया है। आप से प्रार्थना है कि इस क्षेत्र में पेयजल की समस्या का समाधान करें।
आशा है कि आप इस विषय पर ध्यान देंगे और शीघ्र ही उचित कार्रवाई करेंगे।
भवदीय
रामसिंह

JAC Class 9 Hindi रचना पत्र लेखन-औपचारिक पत्र

7. अपने मोहल्ले में वर्षा के कारण उत्पन्न हुए जल भराव की समस्या की ओर ध्यान आकृष्ट करने के लिए नगरपालिका अधिकारी को पत्र लिखिए।

4/307, जगाधरी गेट
अंबाला
दिनांक : 18 सितंबर, 20XX
स्वास्थ्य अधिकारी
नगरपालिका
अंबाला
विषय – जल भराव की समस्या हेतु शिकायती पत्र
महोदय
निवेदन यह है कि मैं जगाधरी गेट का निवासी हूँ। यह क्षेत्र सफ़ाई की दृष्टि से पूरी तरह उपेक्षित है। वर्षा के दिनों में तो इसकी और बुरी हालत हो जाती है। इन दिनों प्रायः सभी गलियों में वर्षा का जल भरा हुआ है। नगरपालिका इस ओर कोई ध्यान नहीं दे रही। इस जल- भराव के कारण आवागमन में ही कठिनाई नहीं होती बल्कि बीमारी फैलने का भी खतरा है।
आपसे नम्र निवेदन है कि आप स्वयं एक बार आकर इस जल भराव को देखें। तभी आप हमारी कठिनाई को समझ पाएँगे। कृपया इस क्षेत्र से जल निकास का शीघ्र प्रबंध करें।
आशा है कि आप मेरी प्रार्थना की तरफ़ ध्यान देंगे।
भवदीय
महेश बजाज

8. चुनाव के दिनों में कार्यकर्ता घर, विद्यालयों और मार्गदर्शक आदि पर बेतहाशा पोस्टर लगा जाते हैं। इससे लोगों को होने वाली असुविधा पर अपने विचार व्यक्त करते हुए ‘दैनिक लोक वाणी’ समाचार-पत्र के ‘जनमत’ कॉलम के लिए पत्र लिखिए।

480, नई बस्ती
चंबा
दिनांक : 22 जनवरी, 20XX
संपादक
दैनिक लोकवाणी
चेन्नई
विषय – जगह-जगह राजनीतिक पोस्टर लगाने हेतु शिकायती पत्र।
आदरणीय महोदय
मैं आपके लोकप्रिय पत्र के ‘जनमत कॉलम’ के लिए एक पत्र लिख रहा हूँ। इसे प्रकाशित करने की कृपा करें। आप जानते हैं कि चुनाव के दिनों में कार्यकर्ता बिना सोचे-समझे, विद्यालयों, घरों तथा मार्गदर्शन चित्रों आदि पर अत्यधिक पोस्टर लगा जाते हैं। घर अथवा विद्यालय कोई राजनीतिक संस्थाएँ नहीं। उन पर लगे पोस्टरों से यह भ्रम हो जाता है कि घर तथा विद्यालय भी किसी राजनीतिक दल से संबद्ध है। इन पोस्टरों से मार्गदर्शन चित्र इस तरह ढक जाते हैं कि अजनबी को रास्ते के बारे में कुछ पता नहीं चलता। इन पोस्टरों को लगानेवालों की भीड़ के कारण आम लोगों को तथा छात्रों को असुविधा होती है। अतः मैं सरकार तथा राजनीतिक दलों का ध्यान इस तरफ़ दिलाना चाहता हूँ।
आशा है कि आप लोगों की सुविधा का ध्यान रखते हुए मेरे इस पत्र को प्रकाशित करने का आदेश देंगे।
भवदीय
मोहन मेहता

JAC Class 9 Hindi रचना पत्र लेखन-औपचारिक पत्र

(ख) आवेदन-पत्र

9. दिल्ली नगर निगम के अध्यक्ष को एक आवेदन-पत्र लिखिए जिसमें लिपिक के पद के लिए आवेदन किया गया हो।

4/19, अशोक नगर
नई दिल्ली
दिनांक : 15 दिसंबर, 20XX
अध्यक्ष
नगर-निगम
दिल्ली
विषय – लिपिक के पद हेतु आवेदन पत्र।
मान्यवर
निवेदन है कि मुझे विश्वस्त सूत्रों से ज्ञात हुआ है कि आपके कार्यालय में एक क्लर्क की आवश्यकता है। मैं इस रिक्ति के लिए अपनी सेवाएँ प्रस्तुत करता हूँ। मेरी योग्यताएँ इस प्रकार हैं –
(i) बी० ए० द्वितीय श्रेणी
(ii) टाइप का पूर्ण ज्ञान-गति 50 शब्द प्रति मिनट
(iii) अनुभव – एक वर्ष
(iv) आयु – 24वें वर्ष में प्रवेश
मुझे अंग्रेज़ी एवं हिंदी का अच्छा ज्ञान है। पंजाबी भाषा पर भी अधिकार है।
आशा है कि आप मेरी योग्यता को देखते हुए अपने अधीन काम करने का अवसर प्रदान करेंगे। मैं सच्चाई एवं ईमानदारी के साथ काम करने का आश्वासन दिलाता हूँ।
योग्यता एवं अनुभव संबंधी प्रमाण-पत्र इस प्रार्थना-पत्र के साथ संलग्न कर रहा हूँ।
धन्यवाद सहित
भवदीय
सुखदेव गोयल
संलग्न : उपर्युक्त

10. ‘जनसंख्या विभाग’ को घर-घर जाकर सूचनाएँ एकत्रित करने वाले ऐसे सर्वेक्षकों की आवश्यकता है, जो हिंदी और अंग्रेजी में भली-भाँति बात कर सकते हों और सूचनाओं को सही-सही दर्ज कर सकते हों। इसके साथ ही आवेदकों में विनम्रतापूर्वक बात करने की योग्यता होनी चाहिए। इस काम में अपनी रुचि प्रदर्शित करते हुए जनसंख्या विभाग के सचिव को आवेदन-पत्र लिखिए।

7, रामलीला मार्ग
हमीरपुर
दिनांक : 14 दिसंबर, 20XX
प्रशासनिक अधिकारी
जनसंख्या विभाग
कोयंबटूर
विषय – सर्वेक्षक की नौकरी हेतु आवेदन-पत्र।
आदरणीय महोदय
मुझे आज के दैनिक समाचार-पत्र में प्रकाशित विज्ञापन से ज्ञात हुआ है कि आपको घर-घर जाकर सूचनाएँ एकत्रित करने के लिए सर्वेक्षकों की आवश्यकता है। मैं आपके कार्यालय द्वारा निर्धारित योग्यताओं को पूर्ण करता हूँ।
मैंने इसी वर्ष बारहवीं कक्षा की परीक्षा उत्तीर्ण की है। कुछ समय तक मैंने स्थानीय कार्यालय में कार्य भी किया है। मैं हिंदी तथा अंग्रेज़ी बोलने तथा इन दोनों भाषाओं में वार्तालाप करने में दक्ष हूँ।
मैं अपनी सेवाएँ प्रदान करना चाहता हूँ। आशा है कि आप मुझे अवसर प्रदान करेंगे। आवश्यक प्रमाण-पत्र मैं भेंटवार्ता के अवसर पर साथ लेकर आऊँगा।
भवदीय
महेश महाजन

JAC Class 9 Hindi रचना पत्र लेखन-औपचारिक पत्र

11. ‘प्रचंड भारत’ दैनिक समाचार-पत्र को खेल विभाग के लिए संवाददाताओं की आवश्यकता है। पद के लिए खेलों का ज्ञान और रुचि के साथ-साथ हिंदी भाषा में अच्छी गति से लिखने का अभ्यास अनिवार्य है। इस पद के लिए आवेदन-पत्र लिखिए।

14, पश्चिम विहार,
कोटा
दिनांक : 15 सितंबर, 20XX
संपादक
दैनिक ‘प्रचंड भारत’
कोटा
विषय – खेल संवाददाता के पद हेतु आवेदन पत्र।
मान्यवर
आपके लोकप्रिय समाचार-पत्र में प्रकाशित विज्ञापन से ज्ञात हुआ कि आपको खेल विभाग के लिए संवाददाताओं की आवश्यकता है। इस पद के लिए मैं अपनी सेवाएँ अर्पित करना चाहता हूँ।
मेरी योग्यताओं का विवरण इस प्रकार है –
शैक्षिक योग्यता बी० ए० प्रथम श्रेणी – 2012, पत्रकारिता में डिप्लोमा – 2013
विद्यालय की क्रिकेट टीम का कप्तान – अवधि 2 वर्ष
मुझे खेलों की सामग्री का पर्याप्त अनुभव है। मुझे हिंदी भाषा का अच्छा ज्ञान है। विद्यालय तथा महाविद्यालय की पत्रिका में मेरे लेख भी प्रकाशित होते रहे हैं।
आवश्यक प्रमाण-पत्र इस आवेदन पत्र के साथ संलग्न हैं
आशा है कि आप मुझे सेवा का अवसर प्रदान करेंगे।
भवदीय
प्रतीक राय

(ग) व्यावसायिक पत्र :

12. रिक्तियों के लिए समाचार-पत्र में विज्ञापन के प्रकाशन हेतु पत्र लिखिए।
रेलवे रोड
जालंधर
दिनांक : 29 सितंबर, 20XX
विज्ञापन व्यवस्थापक
दैनिक ट्रिब्यून
चंडीगढ़
विषय – विज्ञापन प्रकाशन हेतु पत्र।
महोदय
इस पत्र के साथ एक विज्ञापन का प्रारूप भेज रहा हूँ जिसे कृपया 7 अक्तूबर तथा 14 अक्तूबर के अंकों में प्रकाशित कर दें। आपका बिल प्राप्त होते ही उसका भुगतान कर दिया जाएगा।

विज्ञापन का प्रारूप

आवश्यकता है एक ऐसे अनुभवी सेल्स मैनेजर की जो स्वतंत्र रूप से प्रतिष्ठान के बिक्री काउंटर को सँभाल सके। वेतन ₹15000-20000 तथा निःशुल्क आवास की व्यवस्था। बहुत योग्य तथा अनुभवी अभ्यार्थी को योग्यतानुसार उच्च वेतन भी दिया जा सकता है। निम्नलिखित पते पर
15 नवंबर, 20XX तक लिखें या मिलें –
मल्होत्रा बुक डिपो,
रेलवे रोड, जालंधर शहर।
धन्यवाद
भवदीय
एस० के० सिक्का
प्रबंधक
संलग्न : विज्ञापन

JAC Class 9 Hindi रचना पत्र लेखन-औपचारिक पत्र

13. आपको दिल्ली के किसी पुस्तक-विक्रेता से कुछ पुस्तकें मँगवानी हैं। वी० पी० पी० द्वारा मँगवाने के लिए पत्र लिखिए।

5/714, तिलक नगर
कटक
दिनांक : 17 नवंबर, 20XX
प्रबंधक
मल्होत्रा बुक डिपो
गुलाब भवन
6, बहादुर शाह ज़फ़र मार्ग
नई दिल्ली- 110002
विषय – पुस्तकें मँगवाने हेतु पत्र।
महोदय
निम्नलिखित पुस्तकें स्थानीय पुस्तक – विक्रेता से नहीं मिल रहीं अतः ये पुस्तकें वी० पी० पी० द्वारा शीघ्र भेजने का कष्ट करें। यदि अग्रिम भेजने की आवश्यकता हो तो सूचित करें। पुस्तकों पर उचित कमीशन काटना न भूलें। पुस्तकें नवीन पाठ्यक्रम के अनुसार होनी चाहिए।
1. हिंदी गाइड (कक्षा नौ) …… एक प्रति
2. इंगलिश गाइड (कक्षा नौ) ……… एक प्रति
3. नागरिक शास्त्र के सिद्धांत (कक्षा नौ) …… एक प्रति
4. भूगोल (कक्षा नौ) ……… एक प्रति
5. विज्ञान (कक्षा नौ) ……… एक प्रति
भवदीय
निशा सिंह

JAC Class 9 Hindi रचना पत्र लेखन-औपचारिक पत्र

(घ) प्रार्थना-पत्र :

14. अपने प्रधानाचार्य को एक प्रार्थना-पत्र लिखिए जिसमें कई महीने से अंग्रेज़ी की पढ़ाई न होने के कारण उत्पन्न कठिनाई का वर्णन किया गया हो।

प्रधानाचार्य
केंद्रीय विद्यालय
पुड्डूचेरी
विषय – अंग्रेज़ी की पढ़ाई न हो पाने हेतु शिकायती पत्र।
महोदय
आप भली-भाँति जानते हैं कि हमारे अंग्रेज़ी के अध्यापक श्री ज्ञानचंद जी पिछले तीन सप्ताह से बीमार हैं। अभी भी उनके स्वास्थ्य में विशेष सुधार नहीं हो रहा। डॉक्टर का कहना है कि अभी श्री ज्ञानचंद जी को स्वस्थ होने में कम-से-कम दो सप्ताह और लगेंगे। पिछले तीन सप्ताह से हमारी अंग्रेज़ी की पढ़ाई ठीक ढंग से नहीं हो रही। आपने जो प्रबंध किया है, वह संतोषजनक नहीं है। उधर परीक्षा सिर पर आ रही हैं। अभी पाठ्यक्रम भी समाप्त नहीं हुआ है। पुनरावृत्ति के लिए भी समय नहीं बचेगा। अतः आपसे विनम्र प्रार्थना है कि आप हमारी अंग्रेज़ी की पढ़ाई का उचित एवं संतोषजनक प्रबंध करें। यदि कुछ कालांश बढ़ा दिए जाएँ तो और भी अच्छा रहेगा।
आशा है कि हमारी कठिनाई को शीघ्र ही दूर करने का प्रयास करेंगे।
आपका आज्ञाकारी
नवनीत (नौ ‘क’)

15. अपने प्रधानाचार्य को प्रार्थना-पत्र लिखिए जिसमें पुस्तकालय के लिए नई पुस्तकों और बाल-पत्रिकाओं को मँगाने की प्रार्थना की गई हो।

प्रधानाचार्य
राजकीय उच्चतर माध्यमिक विद्यालय
जम्मू
विषय – पुस्तकालय में पुस्तकें मँगवाने हेतु पत्र।
मान्यवर महोदय
गत सप्ताह आपने प्रार्थना सभा में विद्यार्थियों को संबोधित करते हुए विद्यालय के पुस्तकालय की उपयोगिता पर प्रकाश डाला था। आपने छात्रों को पुस्तकालय से अच्छी-अच्छी पुस्तकें निकलवाकर उनका अध्ययन करने की भी प्रेरणा दी थी। लेकिन पुस्तकालय में छात्रों ने जिन पुस्तकों तथा बाल – पत्रिकाओं की माँग की, वे प्रायः विद्यालय में उपलब्ध नहीं हैं।
आपसे नम्र निवेदन है कि कुछ नई पुस्तकें तथा बालोपयोगी पत्रिकाओं को मँगवाने की व्यवस्था करें। पुस्तकों तथा पत्रिकाओं के अध्ययन से जहाँ छात्र वर्ग के ज्ञान में वृद्धि होती है, वहाँ उनका पर्याप्त मनोरंजन भी होता है।
आशा है कि आप हमारी प्रार्थना पर शीघ्र ध्यान देंगे।
आपका आज्ञाकारी शिष्य
मनोज कुमार
तिथि : 7.11.20XX

JAC Class 9 Hindi रचना पत्र लेखन-औपचारिक पत्र

16. विद्यालय के प्राचार्य को संध्याकालीन खेलों के उत्तम प्रबंध के लिए प्रार्थना-पत्र लिखिए।

प्राचार्य
नवयुग विद्यालय
वर्धमान
विषय – संध्याकालीन खेलों के उत्तम प्रबंध हेतु पत्र।
महोदय
निवेदन यह है कि आप ने प्रातःकालीन सभा में खेलों के महत्त्व पर प्रकाश डाला था। हम विद्यालय परिसर में संध्या के समय खेलना चाहते हैं, परंतु हमारे विद्यालय के परिसर का खेल का मैदान साफ़ नहीं है तथा हमें खेल की उचित सामग्री भी प्राप्त नहीं होती।
आपसे अनुरोध है कि खेल के मैदान को साफ़ करवाने की तथा खेलों की सामग्री दिलवाने की कृपा करें।
धन्यवाद
भवदीय
देबाशीष डे
तथा कक्षा नौवीं के अन्य विद्यार्थी
दिनांक : 24 अप्रैल, 20XX

17. अध्यक्ष परिवहन निगम को अपने गाँव तक बस सुविधा उपलब्ध कराने के लिए प्रार्थना-पत्र लिखिए।

अध्यक्ष
परिवहन निगम
दिल्ली
विषय – बस सुविधा उपलब्ध करवाने हेतु पत्र।
आदरणीय महोदय
मैं गाँव ‘सोनपुरा’ का एक निवासी हूँ। वह गाँव दिल्ली से लगभग 20 किलोमीटर की दूरी पर है। खेद की बात है कि परिवहन निगम की ओर से इस गाँव तक कोई बस सेवा उपलब्ध नहीं है। यहाँ के किसानों को नगर से कृषि के लिए आवश्यक सामान लाने में बड़ी कठिनाई का सामना करना पड़ता है। छात्र – छात्राओं को अपने-अपने विद्यालयों तथा महाविद्यालयों में पहुँचने के लिए भी कठिनाई का सामना करना पड़ता है। अतः आपसे प्रार्थना है कि इस गाँव तक बस सेवा उपलब्ध करवाने की कृपा करें।
आशा है कि हमारी प्रार्थना पर शीघ्र ध्यान दिया जाएगा।
धन्यवाद सहित।
भवदीय
रमेश ‘बाल रत्न’
निवासी ‘सोनपुरा गाँव’
तिथि : 18 मार्च, 20XX

JAC Class 9 Hindi रचना पत्र लेखन-औपचारिक पत्र

18. अपने विद्यालय के प्रधानाध्यापक को एक प्रार्थना-पत्र लिखिए जिसमें खेलों की आवश्यक तैयारी तथा खेल का सामान उपलब्ध करवाने की प्रार्थना की गई हो।

प्रधानाध्यापक
केंद्रीय विद्यालय
पणजी
विषय – खेलों की आवश्यक तैयारी तथा खेल का सामान उपलब्ध करवाने हेतु पत्र।
मान्यवर
आपने एक छात्र – सभा में भाषण देते हुए इस बात पर बल दिया था कि छात्रों को शिक्षा के साथ-साथ खेलों का महत्व भी समझना चाहिए। खेल शिक्षा का एक महत्वपूर्ण अंग है। खेद की बात यह है कि आप खेलों की आवश्यकता तो खूब समझते हैं पर खेल सामग्री के अभाव की ओर कभी आपका ध्यान नहीं गया। खेलों के अनेक लाभ हैं। ये स्वास्थ्य के लिए बड़ी उपयोगी हैं। ये अनुशासन, समयपालन, सहयोग तथा सद्भावना का भी पाठ पढ़ाते हैं।
अतः आपसे नम्र निवेदन है कि आप खेलों का सामान उपलब्ध करवाने की कृपा करें। इससे छात्रों में खेलों के प्रति रुचि बढ़ेगी। वे अपनी खेल प्रतिभा का विकास कर सकेंगे।
आशा है कि आप मेरी प्रार्थना पर उचित ध्यान देंगे और आवश्यक आदेश जारी करेंगे।
आपका आज्ञाकारी शिष्य
रजत शर्मा
कक्षा : नौवीं ‘ब’
दिनांक : 17 अप्रैल, 20XX

19. अपने विद्यालय के प्रधानाचार्य को भाई के विवाह के कारण चार दिन का अवकाश प्रदान करने हेतु प्रार्थना पत्र लिखिए।

प्रधानाचार्य
एस० एम० उच्च विद्यालय
अहमदाबाद
विषय – भाई के विवाह के कारण अवकाश हेतु पत्र।
महोदय
सविनय प्रार्थना है कि मेरे बड़े भाई का शुभ विवाह 12 अक्तूबर को होना निश्चित हुआ है। मेरा इसमें सम्मिलित होना अति आवश्यक है। इसलिए इन दिनों मैं विद्यालय में उपस्थित नहीं हो सकता। कृपया मुझे 11 से 14 अक्तूबर तक चार दिन की छुट्टी देकर अनुगृहित करें। अति धन्यवाद सहित।
आपका आज्ञाकारी शिष्य
जयदीप
कक्षा : नौवीं ‘बी’
दिनांक : 10 अक्तूबर, 20XX

JAC Class 9 Hindi रचना पत्र लेखन-औपचारिक पत्र

20. शिक्षा-शुल्क क्षमा करवाने के लिए अपने विद्यालय के प्रधानाचार्य को प्रार्थना-पत्र लिखिए।

प्रधानाचार्य
डी० ए० वी० उच्च विद्यालय
अकोला
विषय – शिक्षा शुल्क क्षमा करवाने के लिए पत्र।
महोदय
सविनय निवेदन यह है कि मैं आपके विद्यालय में कक्षा नौवीं ‘ए’ में पढ़ता हूँ। मैं एक मध्यमवर्गीय परिवार से संबंध रखता हूँ। मेरे पिताजी एक स्थानीय कार्यालय में केवल पाँच हज़ार रुपये मासिक पर कार्य करते हैं। हम परिवार में छह सदस्य हैं। इस महँगाई के समय में इतने कम वेतन में निर्वाह होना अति कठिन है। ऐसी स्थिति में मेरे पिताजी मेरा शुल्क अदा करने में असमर्थ हैं।
मेरी पढ़ाई में विशेष रुचि है। मैं सदा अपनी कक्षा में प्रथम रहता आया हूँ। मैं स्कूल की हॉकी टीम का कप्तान भी हूँ। मेरे सभी अध्यापक मुझसे सर्वथा संतुष्ट हैं। अतः आपसे मेरी विनम्र प्रार्थना है कि आप मेरा शिक्षा शुल्क माफ़ कर मुझे आगे पढ़ने का सुअवसर प्रदान करने की कृपा करें।
मैं आपका सदा आभारी रहूँगा।
आपका आज्ञाकारी शिष्य
प्रमोद कुमार
कक्षा नौवीं ‘ए’
दिनांक : 15 मई, 20XX

21. अपने विद्यालय के प्रधानाचार्य को एक पत्र लिखिए जिसमें स्कूल फंड से पुस्तकें लेकर देने की प्रार्थना की गई हो।

प्रधानाचार्य
केंद्रीय विद्यालय
मदुरै
विषय – स्कूल फंड से पुस्तकें लेकर देने हेतु पत्र।
महोदय
निवेदन है कि मैं आपके विद्यालय में कक्षा नौवीं ‘ए’ का अत्यंत निर्धन विद्यार्थी हूँ। जैसा कि आप जानते हैं कि पिछले वर्ष मेरे पिता की मृत्यु हो गई थी। मेरी माँ पढ़ी-लिखी नहीं है। खेती-बाड़ी या आय का अन्य कोई साधन नहीं है। मेरे अलावा मेरे दो छोटे भाई हैं, जिनमें से एक छठी कक्षा में पढ़ता है। माँ हम सब का मेहनत-मज़दूरी करके पालन-पोषण कर रही है और पढ़ा-लिखा रही है। नवंबर का महीना बीत रहा है और अभी तक मेरे पास एक भी पाठ्य पुस्तक नहीं है। अभी तक मैं साथियों से पुस्तकें माँग कर काम चला रहा हूँ। परंतु परीक्षा के समय मेरे साथी चाहकर भी अपनी पाठ्य-पुस्तकें नहीं दे पाएँगे। यह बात स्मरण करके अपनी बेबसी का बेहद एहसास होता है। वार्षिक परीक्षा दो-तीन महीने बाद शुरू होने वाली है।
अतः आपसे विनम्र प्रार्थना है कि आप मुझे निर्धन छात्र- कोष या पुस्तकालय से सभी आवश्यक पाठ्य-पुस्तकें दिलाने की कृपा करें। आपके इस उपकार का मैं जीवन-भर आभारी रहूँगा।
आपका आज्ञाकारी शिष्य
कमल मोहन
कक्षा नौवीं ‘ए’
दिनांक : 11 मई, 20XX

JAC Class 9 Hindi रचना पत्र लेखन-औपचारिक पत्र

22. अपने विद्यालय के प्रधानाचार्य को विद्यालय-त्याग का प्रमाण-पत्र प्रदान करने के लिए प्रार्थना-पत्र लिखिए।

प्रधानाचार्य
शारदा उच्च विद्यालय
धर्मशाला
विषय – विद्यालय – त्याग प्रमाण पत्र प्रदान करने हेतु।
मान्यवर
सविनय निवेदन है कि मेरे पिताजी का स्थानांतरण शिमला हो गया है। वे कल यहाँ से जा रहे हैं और साथ में परिवार भी जा रहा है। इस अवस्था में मेरा यहाँ अकेला रहना कठिन है। कृपा करके मुझे स्कूल छोड़ने का प्रमाण- पत्र दीजिए ताकि मैं वहाँ जाकर स्कूल में प्रविष्ट हो सकूँ।
कृपा के लिए धन्यवाद।
आपका विनीत शिष्य
सुनील कुमार
कक्षा : नौवीं ‘बी’
दिनांक : 11 फरवरी, 20XX

JAC Class 12 Political Science Important Questions Chapter 6 अंतर्राष्ट्रीय संगठन

Jharkhand Board JAC Class 12 Political Science Important Questions Chapter 6 अंतर्राष्ट्रीय संगठन Important Questions and Answers.

JAC Board Class 12 Political Science Important Questions Chapter 6 अंतर्राष्ट्रीय संगठन

बहुचयनात्मक प्रश्न

1. इजराइल ने लेबनान पर हमला किया।
(अ) फरवरी, 2006
(स) अक्टूबर, 2006
(ब) जून, 2006
(द) जनवरी, 2006
उत्तर:
(ब) जून, 2006

2. संयुक्त राष्ट्रसंघ के दूसरे महासचिव थे
(अ) एंटोनियो गुटेरेस
(ब) यू थांट
(स) बान की मून
(द) डेग हैमरशोल्ड
उत्तर:
(द) डेग हैमरशोल्ड

3. अंतर्राष्ट्रीय मुद्रा कोष (IMF) में कितने सदस्य हैं?
(अ) 193
(ब) 191
(स) 190
(द) 189
उत्तर:
(द) 189

4. अमरीकी राष्ट्रपति रूजवेल्ट और ब्रितानी प्रधानमंत्री चर्चिल ने अटलांटिक चार्टर पर हस्ताक्षर किए-
(अ) फरवरी, 1945
(स) अक्टूबर, 1945
(ब) जून, 1941
(द) अगस्त, 1941
उत्तर:
(द) अगस्त, 1941

5. संयुक्त राष्ट्र संघ की वर्तमान सदस्य संख्या कितनी है
(अ) 193
(ब) 190
(स) 189
(द) 191
उत्तर:
(अ) 193

6. संयुक्त राष्ट्र संघ की स्थापना कब की गई-
(अ) 24 अक्टूबर, 1945
(ब) 1 जनवरी, 1945
(स) 1 जून, 1945
(द) 31 दिसम्बर, 1945
उत्तर:
(अ) 24 अक्टूबर, 1945

JAC Class 12 Political Science Important Questions Chapter 6 अंतर्राष्ट्रीय संगठन

7. अन्तर्राष्ट्रीय न्यायालय का मुख्यालय कहाँ है-
(अ) वाशिंगटन में
(ब) न्यूयार्क में
(स) हेग में
(द) जिनेवा में
उत्तर:
(स) हेग में

8. भारत संयुक्त राष्ट्रसंघ में कब शामिल हुआ?
(अ) 24 अक्टूबर, 1995
(ब) 30 अक्टूबर, 1945
(स) 26 जून, 1945
(द) 15 अक्टूबर, 1945
उत्तर:
(ब) 30 अक्टूबर, 1945

9. संयुक्त राष्ट्र संघ दिवस प्रतिवर्ष मनाया जाता है
(अ) 24 अक्टूबर को
(स) 15 अगस्त को
(ब) 20 जनवरी को
(द) 26 जनवरी को
उत्तर:
(अ) 24 अक्टूबर को

10. अंतर्राष्ट्रीय न्यायालय में न्यायाधीशों की संख्या है।
अ) 11
(ब) 9
(स) 15
(द) 16
उत्तर:
(स) 15

रिक्त स्थानों की पूर्ति कीजिए :

1. संयुक्त राष्ट्रसंघ के प्रतीक चिह्न में ……………………….. की पत्तियाँ हैं, जो कि ……………………. का संकेत करती हैं।
उत्तर:
जैतून, विश्व शांति

2. ………………………. का सबसे प्रभावकारी समाधान वैश्विक तापवृद्धि को रोकना है।
उत्तर:
ग्लोबल वार्मिंग

3. अंतराष्ट्रीय मुद्राकोष में समूह -7 के सदस्यों के पास …………………….. प्रतिशत मत है।
उत्तर:
41.29

4. IMF में अमरीका का मताधिकार ……………………. प्रतिशत है।
उत्तर:
16.52

5. अंतर्राष्ट्रीय न्यायालय में न्यायाधीशों का चुनाव ………………………. तथा ………………………… में पूर्ण बहुमत द्वारा होता है।
उत्तर:
आम सभा, सुरक्षा परिषद्

अतिलघूत्तरात्मक प्रश्न

प्रश्न 1.
दूसरा विश्व युद्ध कब समाप्त हुआ?
उत्तर:
दूसरा विश्व युद्ध 1945 में समाप्त हुआ।

प्रश्न 2.
विश्व बैंक की औपचारिक स्थापना कब हुई?
उत्तर:
द्वितीय विश्व युद्ध के दौरान सन् 1944 में विश्व बैंक की औपचारिक स्थापना हुई।

JAC Class 12 Political Science Important Questions Chapter 6 अंतर्राष्ट्रीय संगठन

प्रश्न 3.
संयुक्त राष्ट्रसंघ का प्रतीक चिह्न क्या है?
उत्तर:
संयुक्त राष्ट्रसंघ के प्रतीक चिह्न में दुनिया का नक्शा बना हुआ है और इसके चारों तरफ जैतून की पत्तियाँ हैं।

प्रश्न 4.
समूह -7 के सदस्य देशों के नाम लिखिए।
उत्तर:
समूह – 7 के सदस्य देशों के नाम- अमरीका, जापान, जर्मनी, फ्रांस, ब्रिटेन, इटली और कनाडा है।

प्रश्न 5.
संयुक्त राष्ट्रसंघ की घोषणा पर हस्ताक्षर कब हुआ?
उत्तर:
धुरी शक्तियों के खिलाफ लड़ रहे 26 मित्र राष्ट्र अटलांटिक चार्टर के समर्थन में वाशिंग्टन में मिले ओर दिसंबर, 1943 में संयुक्त राष्ट्रसंघ की घोषणा पर हस्ताक्षर हुए।

प्रश्न 6.
संयुक्त राष्ट्र संघ के महासचिव की नियुक्ति कौन करता है?
उत्तर:
सुरक्षा परिषद् की सिफारिश के आधार पर महासभा महासचिव को नियुक्त करती है।

प्रश्न 7.
संयुक्त राष्ट्र संघ की स्थापना के समय कुल कितने सदस्य थे?
उत्तर:
संयुक्त राष्ट्र संघ की स्थापना के समय कुल 51 सदस्य थे

प्रश्न 8.
संयुक्त राष्ट्रसंघ के नौवें महासचिव कौन हैं?
उत्तर:
संयुक्त राष्ट्रसंघ के नौवें महासचिव एंटोनियो गुटेरेस हैं। इन्होंने महासचिव का पद 1 जनवरी, 2017 को संभाला।

प्रश्न 9.
विश्व की एकमात्र सुपर पॉवर ( महाशक्ति) का नाम बताइये।
उत्तर:
विश्व की एकमात्र महाशक्ति ‘संयुक्त राज्य अमेरिका’ है।

JAC Class 12 Political Science Important Questions Chapter 6 अंतर्राष्ट्रीय संगठन

प्रश्न 10.
आई. एम. एफ. का पूरा नाम लिखें।
उत्तर:
अन्तर्राष्ट्रीय मुद्रा कोष (International Monetary Fund)

प्रश्न 11.
डब्ल्यू. एच. ओ. (W.H.O.) का पूरा नाम लिखें।
उत्तर:
विश्व स्वास्थ्य संगठन (World Health Organization )

प्रश्न 12.
यूनेस्को (UNESCO) का पूरा नाम लिखें।
उत्तर:
संयुक्त राष्ट्र शैक्षणिक, वैज्ञानिक एवं सांस्कृतिक संगठन (United Nations Educational Scientific Cultural Organization)

प्रश्न 13.
इजरायल ने लेबनान पर कब हमला किया? उसने अपने आक्रमण को क्यों जरूरी बताया?
उत्तर:
इजरायल ने लेबनान पर 2006 में हमला किया। उसने उग्रवादी गुट हिजबुल्लाह पर अंकुश लगाने हेतु आक्रमण को आवश्यक बताया।

प्रश्न 14.
इजरायल के लेबनान पर आक्रमण के प्रति संयुक्त राष्ट्र संघ का क्या दृष्टिकोण रहा?
उत्तर:
इजरायल के लेबनान पर 2006 के आक्रमण के प्रति संयुक्त राष्ट्र संघ का यह दृष्टिकोण रहा कि इजरायली सैन्य बल इस क्षेत्र से वापस जाये।

प्रश्न 15.
अन्तर्राष्ट्रीय मुद्रा कोष का प्रमुख कार्य क्या है?
उत्तर:
अन्तर्राष्ट्रीय मुद्रा कोष मांगे जाने पर सदस्य देशों को वित्तीय एवं तकनीकी मदद उपलब्ध कराता है।

प्रश्न 16.
विश्व व्यापार संगठन की स्थापना कब हुई?
उत्तर:
1 जनवरी, 1995 को।

JAC Class 12 Political Science Important Questions Chapter 6 अंतर्राष्ट्रीय संगठन

प्रश्न 17.
विश्व व्यापार संगठन की क्या भूमिका है?
उत्तर:
विश्व व्यापार संगठन की मुख्य भूमिका विश्व के देशों में समान आर्थिक नियमों को लागू करना एवं मुक्त व्यापार को बढ़ावा देना है।

प्रश्न 18.
विश्व व्यापार संगठन की सदस्य संख्या क्या है?
उत्तर:
विश्व व्यापार संगठन की सदस्य संख्या 150 है।

प्रश्न 19.
विश्व व्यापार संगठन में फैसले कैसे लिये जाते हैं?
उत्तर:
विश्व व्यापार संगठन में फैसले आम सहमति से लिये जाते हैं।

प्रश्न 20.
अन्तर्राष्ट्रीय आणविक ऊर्जा एजेन्सी से क्या अभिप्राय है?
उत्तर:
इस संगठन का मुख्य उद्देश्य परमाणु ऊर्जा के शांतिपूर्ण उपयोग को बढ़ावा देना तथा सैन्य उद्देश्यों में इसके प्रयोग को रोकना है।

प्रश्न 21.
अन्तर्राष्ट्रीय आणविक ऊर्जा एजेन्सी (आई.ए.ई.ए.) का स्थापना वर्ष लिखिये
उत्तर:
सन् 1957 ई.

JAC Class 12 Political Science Important Questions Chapter 6 अंतर्राष्ट्रीय संगठन

प्रश्न 22.
अन्तर्राष्ट्रीय आणविक ऊर्जा एजेन्सी क्या प्रयास करता है?
उत्तर:
यह संगठन परमाणु ऊर्जा के शांतिपूर्ण उपयोग को बढ़ावा देने का कार्य करता है।

प्रश्न 23.
संयुक्त राष्ट्र संघ का मुख्य उद्देश्य क्या है?
उत्तर:
संयुक्त राष्ट्र संघ का मुख्य उद्देश्य शांति और सुरक्षा कायम करना है।

प्रश्न 24.
संयुक्त राष्ट्र संघ का मुख्यालय कहाँ स्थित है?
उत्तर:
संयुक्त राष्ट्र संघ का मुख्यालय न्यूयार्क में स्थित है।

प्रश्न 25.
अमरीका, रूस, ब्रिटेन, फ्रांस और चीन को संयुक्त राष्ट्रसंघ का स्थायी सदस्य क्यों चुना गया?
उत्तर:
दूसरे विश्वयुद्ध के तुरंत बाद के समय में ये देश सबसे ज्यादा ताकतवर थे और इस महायुद्ध के विजेता भी रहे, इसलिए इन्हें स्थायी सदस्य के रूप में चुना गया।

प्रश्न 26.
संयुक्त राष्ट्र संघ का सर्वाधिक महत्त्वपूर्ण अंग कौनसा है?
उत्तर:
संयुक्त राष्ट्र संघ का सर्वाधिक महत्त्वपूर्ण अंग ‘सुरक्षा परिषद्’ है।

प्रश्न 27.
संयुक्त राष्ट्र संघ का दूसरा महत्त्वपूर्ण अंग कौनसा है, जिसके सभी सदस्य राष्ट्र सदस्य हैं?
उत्तर:
आम सभा या महासभा।

प्रश्न 28.
संयुक्त राष्ट्र संघ के मुख्य अंग कितने हैं? उनके नाम लिखिये।
उत्तर:
संयुक्त राष्ट्र संघ के प्रमुख 6 अंग हैं। ये हैं।

  1. महासभा
  2. सुरक्षा परिषद्
  3. आर्थिक और सामाजिक परिषद्
  4. न्यास परिषद्
  5. अन्तर्राष्ट्रीय न्यायालय और
  6. सचिवालय।

प्रश्न 29.
अन्तर्राष्ट्रीय न्यायालय के न्यायाधीशों का कार्यकाल कितने वर्ष का होता है?
उत्तर:
अन्तर्राष्ट्रीय न्यायालय के न्यायाधीशों का कार्यकाल 9 वर्ष का होता है । प्रश्न 30. संयुक्त राष्ट्र संघ के किन्हीं दो विशिष्ट अभिकरणों के नाम लिखिये।
उत्तर:
कृषि एवं खाद्य संगठन, विश्व स्वास्थ्य संगठन

प्रश्न 31.
संयुक्त राष्ट्र सुरक्षा परिषद् के अस्थायी सदस्यों की संख्या कितनी है और उनका कार्यकाल कितना है?
उत्तर:
संयुक्त राष्ट्र सुरक्षा परिषद् के अस्थायी सदस्यों की संख्या 10 है तथा उनका कार्यकाल 4 वर्ष का है।

JAC Class 12 Political Science Important Questions Chapter 6 अंतर्राष्ट्रीय संगठन

प्रश्न 32.
संयुक्त राष्ट्र संघ के दो उद्देश्यों का वर्णन कीजिये।
उत्तर:
संयुक्त राष्ट्र संघ के प्रमुख उद्देश्य ये हैं।

  1. सामूहिक व्यवस्था द्वारा अन्तर्राष्ट्रीय शांति एवं सुरक्षा कायम रखना और आक्रामक प्रवृत्तियों को नियंत्रण में रखना।
  2. अन्तर्राष्ट्रीय विवादों का शांतिपूर्ण समाधान करना।

प्रश्न 33.
संयुक्त राष्ट्र के महासचिव के दो कार्य बताइये।
उत्तर:

  1. साधारण सभा की वार्षिक रिपोर्ट तैयार करना
  2. सुरक्षा सम्बन्धी विवादों को महासभा के समक्ष प्रस्तुत करना।

प्रश्न 34.
अन्तर्राष्ट्रीय स्तर पर कार्य कर रहे किन्हीं दो गैर-सरकारी संगठनों के नाम लिखें।
अथवा
मानवाधिकारों की रक्षा में संलग्न दो गैर सरकारी अन्तर्राष्ट्रीय संगठन कौन से हैं?
उत्तर:

  1. एमनेस्टी इन्टरनेशनल
  2. ह्यमन राइट्स वॉच।

प्रश्न 35.
संयुक्त राष्ट्र संघ के कोई दो कार्य बताइये।
उत्तर:

  1. अन्तर्राष्ट्रीय शांति सुरक्षा बनाये रखना।
  2. अन्तर्राष्ट्रीय कानून सम्बन्धी व्याख्या करना।

प्रश्न 36.
संयुक्त राष्ट्र महासभा द्वारा 1950 में ‘शांति के लिए एकता’ प्रस्ताव की दो विशेषताएँ बताइए।
उत्तर:

  1. महासभा का आपातकालीन अधिवेशन बुलाना
  2. शांति निरीक्षण आयोग की स्थापना करना।

प्रश्न 37.
संयुक्त राष्ट्र संघ की महासभा के दो कार्यों का वर्णन कीजिए।
उत्तर:

  1. संयुक्त राष्ट्र संघ का बजट पारित करना।
  2. सुरक्षा परिषद् तथा अन्य संस्थाओं व संगठनों की रिपोर्ट पर विचार करना।

प्रश्न 38.
संयुक्त राष्ट्र की सुरक्षा परिषद् के स्थायी सदस्यों के नाम लिखिये।
उत्तर:
सुरक्षा परिषद् के 5 स्थायी सदस्य हैं। ये हैं।

  1. संयुक्त राज्य अमेरिका
  2.  ब्रिटेन
  3. फ्रांस
  4. रूस व
  5. चीन।

JAC Class 12 Political Science Important Questions Chapter 6 अंतर्राष्ट्रीय संगठन

प्रश्न 39.
सुरक्षा परिषद् के अस्थायी सदस्यों को कौन निर्वाचित करता है? अस्थायी सदस्यों का कार्यकाल कितना होता है?
उत्तर:
सुरक्षा परिषद् के अस्थायी सदस्यों को महासभा निर्वाचित करती है तथा अस्थायी सदस्यों का कार्यकाल चार वर्ष का होता है।

प्रश्न 40.
वर्तमान में सुरक्षा परिषद् के सदस्यों की कुल संख्या कितनी है?
उत्तर:
वर्तमान में सुरक्षा परिषद् के सदस्यों की कुल संख्या 15 है। इनमें 5 सदस्य स्थायी हैं और 10 अस्थायी हैं । प्रश्न 41. संयुक्त राष्ट्र महासभा की प्रतिष्ठा में वृद्धि के कोई दो कारण बताइये।
उत्तर:

  1. महासभा में प्रायः सभी देशों को प्रतिनिधित्व प्राप्त है।
  2. शांति के लिये एकता प्रस्ताव के कारण।

प्रश्न 42.
जी 8 के सदस्य देशों के नाम लिखिये।
उत्तर:
जी 8 के सदस्य देश हैं।

  1. अमरीका
  2. जापान
  3. जर्मनी
  4. फ्रांस
  5. ब्रिटेन
  6. इटली
  7. कनाडा और
  8. रूस।

प्रश्न 43.
संयुक्त राष्ट्र संघ के वार्षिक बजट में सर्वाधिक योगदान किस देश का है और कितना है?
उत्तर:
संयुक्त राज्य अमेरिका का लगभग 22 प्रतिशत।

प्रश्न 44.
एमनेस्टी इण्टरनेशनल का क्या उत्तरदायित्व है?
उत्तर:
इसका उत्तरदायित्व है। मानवाधिकारों से सम्बद्ध रिपोर्ट तैयार करना तथा उन्हें प्रकाशित करना।

प्रश्न 45.
संयुक्त राष्ट्रसंघ के प्रथम महासचिव कौन थे?
उत्तर:
संयुक्त राष्ट्रसंघ के प्रथम महासचिव ट्राइग्व ली थे।

प्रश्न 46.
संयुक्त राष्ट्रसंघ में अस्थायी सदस्य कितने वर्षों के लिए चुने जाते हैं?
उत्तर:
दो वर्षों के लिए।

प्रश्न 47.
मानवाधिकार परिषद् कब से सक्रिय है?
उत्तर:
मानवाधिकार परिषद् 19 जून, 2006 से सक्रिय है।

प्रश्न 48.
विश्व व्यापार संगठन किसके उत्तराधिकारी के रूप में कार्य करता है?
उत्तर:
जनरल एग्रींट ऑन ट्रेड एंड टैरिफ

प्रश्न 49.
विश्व बैंक के मुख्य कार्य क्या हैं?
उत्तर:
कृषि, ग्रामीण, विकास, पर्यावरण सुरक्षा, सुशासन तथा आधारभूत ढाँचे के लिए काम करता है।

JAC Class 12 Political Science Important Questions Chapter 6 अंतर्राष्ट्रीय संगठन

प्रश्न 50.
अंतर्राष्ट्रीय मुद्राकोष के कार्य लिखिए।
उत्तर:
विश्व स्तर पर वित्त व्यवस्था की देखरेख तथा वित्तीय व तकनीकी सहायता देना।

लघूत्तरात्मक प्रश्न

प्रश्न 1.
सुरक्षा परिषद् के अस्थाई सदस्यों का भौगोलिक वितरण किस प्रकार किया गया है?
उत्तर:
सुरक्षा परिषद् के अस्थायी सदस्यों का भौगोलिक वितरण इस प्रकार है। अफ्रीका और एशिया के पाँच सदस्य, पूर्वी यूरोप का एक सदस्य, लातीन अमेरिका के दो सदस्य, पश्चिमी यूरोप तथा अन्य राज्यों के दो सदस्य।

प्रश्न 2.
वीटो किसे कहते हैं?
उत्तर:
वीटो का शब्दार्थ है, “मैं मना करता हूँ।” सुरक्षा परिषद् का कोई स्थायी सदस्य यदि किसी महत्त्वपूर्ण या सारवान प्रश्न पर असहमति प्रकट करता है तो उसे अस्वीकृत कर दिया जाता है। उसके इस अधिकार/ शक्ति को ही वीटो शक्ति कहा जाता है।

प्रश्न 3.
संयुक्त राष्ट्र संघ के पुनर्गठन या लोकतंत्रीकरण से क्या अभिप्राय है?
उत्तर:
संयुक्त राष्ट्र संघ के पुनर्गठन या लोकतन्त्रीकरण से अभिप्राय है— सुरक्षा परिषद् के स्थायी सदस्यों में विस्तार करना तथा उसमें विकासशील देशों को समुचित प्रतिनिधित्व देना।

प्रश्न 4.
संयुक्त राष्ट्र की सुरक्षा परिषद् में स्थायी सदस्यता हेतु भारत की दावेदारी के समर्थन में दो तर्क लिखिये।
उत्तर:

  1. भारत विश्व का सबसे बड़ा प्रजातंत्रीय देश है।
  2. भारत सैनिक, आर्थिक तथा प्रौद्योगिक दृष्टि से एक सशक्त राष्ट्र- है

प्रश्न 5.
संयुक्त राष्ट्र संघ के संगठन एवं प्रक्रिया में सुधार के लिये कोई दो सुझाव दें।
उत्तर:

  1. वीटो की समाप्ति-सुरक्षा परिषद् की कार्यप्रणाली को व्यवस्थित एवं लोकतांत्रिक बनाने के लिए वीटो की शक्ति को समाप्त कर देना चाहिए।
  2. समान सदस्यता- सुरक्षा परिषद् में अस्थाई सदस्यों की धारणा को समाप्त करके सबको समान स्तर की. सदस्यता प्रदान करनी चाहिए।

प्रश्न 6.
‘ग्लोबल वार्मिंग’ की समस्या को स्पष्ट करें।
उत्तर:
‘ग्लोबल वार्मिंग’ से आशय है। वातावरण में ग्रीनहाउस गैसों के बढ़ने से तापमान में वृद्धि इससे समुद्रतल की ऊँचाई बढ़ने का खतरा है। ऐसा होने पर विश्व के समुद्रतटीय इलाके जलमग्न हो सकते हैं। इस समस्या का सबसे प्रभावकारी समाधान वैश्विक तापवृद्धि को रोकना है।

प्रश्न 7.
‘लीग ऑव नेशंस’ की उत्पत्ति क्यों हुई?
उत्तर:
प्रथम विश्वयुद्ध के पश्चात् विश्व को एक ऐसे अंतर्राष्ट्रीय संगठन की जरूरत का आभास हुआ जो अंतर्राष्ट्रीय झगड़ों का निपटारा बातचीत द्वारा कर सके जिससे युद्ध जैसे विनाशकारी स्थिति को टाला जा सके। इसके परिणामस्वरूप ‘लीग ऑव नेशंस’ का जन्म हुआ।

JAC Class 12 Political Science Important Questions Chapter 6 अंतर्राष्ट्रीय संगठन

प्रश्न 8.
एंटोनियो गुटेरेस के बारे में संक्षेप में लिखिये।
उत्तर:
एंटोनियो गुटेरेस संयुक्त राष्ट्रसंघ के नौवें महासचिव हैं। उन्होंने महासचिव का पद 1 जनवरी, 2017 को ग्रहण किया। ये 1995 से 2002 तक पुर्तगाल के प्रधानमंत्री और 2005 से 2015 तक यूनाइटेड नेशंस हाई कमीशनर फॉर रिफ्यूजीज रहे। 1999-2005 तक ये सोशलिस्ट
इंटरनेशनल के अध्यक्ष भी रहे।

प्रश्न 9.
संयुक्त राष्ट्र संघ की चार विशिष्ट एजेन्सियों के नाम लिखो।
उत्तर:
संयुक्त राष्ट्र संघ की चार विशिष्ट एजेन्सियों के नाम इस प्रकार हैं।

  1. अन्तर्राष्ट्रीय श्रम संगठन (I.L.O.)
  2. संयुक्त राष्ट्र शैक्षिक, वैज्ञानिक एवं सांस्कृतिक संगठन (UNESCO)
  3. विश्व स्वास्थ्य संगठन., (W.H.O.)
  4. खाद्य एवं कृषि संगठन।

प्रश्न 10.
संयुक्त राष्ट्र संघ के कोई तीन सिद्धान्त लिखिए।
उत्तर:

  1. संयुक्त राष्ट्र संघ सदस्यों की संप्रभु समानता पर आधारित है।
  2. संयुक्त राष्ट्र के सभी सदस्य ईमानदारी से घोषणा-पत्र के अन्तर्गत निर्धारित दायित्वों का पालन करेंगे।
  3. संयुक्त राष्ट्र के सभी सदस्य अपने विवादों को शांतिपूर्ण ढंग से निपटायेंगे ।0

प्रश्न 11.
संयुक्त राष्ट्रसंघ के द्वितीय महासचिव कौन थे? उनके द्वारा किए गए कार्यों का विवरण कीजिए।
उत्तर:
संयुक्त राष्ट्रसंघ के द्वितीय महासचिव डेग हैमरशोल्ड थे। ये स्वीडन से थे तथा अर्थशास्त्री और वकील थे इन्होंने स्वेज नहर से जुड़े विवादों को सुलझाने तथा अफ्रीका के अनौपनिवेशीकरण में महत्त्वपूर्ण भूमिका निभाई। कांगो- संकट को सुलझाने की दिशा में किए गए प्रयासों के लिए मरणोपरांत इन्हें नोबेल शांति पुरस्कार दिया गया। हालाँकि सोवियत संघ और फ्रांस ने अफ्रीका में इनकी भूमिका की आलोचना की थी।

प्रश्न 12.
सुरक्षा परिषद् में स्थायी सदस्यों को प्राप्त निषेधाधिकार (Veto) शक्ति से क्या आशय है?
उत्तर:
सुरक्षा परिषद् में स्थायी सदस्यों को प्राप्त निषेधाधिकार की शक्ति का अर्थ है कि यदि पांच स्थायी सदस्यों में से कोई भी एक सदस्य संयुक्त राष्ट्र सुरक्षा परिषद् में रखे गये प्रस्ताव के विरोध में वोट डाल दे तो वह प्रस्ताव पास नहीं होगा।

प्रश्न 13.
संयुक्त राष्ट्रसंघ के समक्ष कौन से दो बुनियादी सुधारों का मसला है?
उत्तर:
संयुक्त राष्ट्रसंघ के समक्ष निम्न दो बुनियादी सुधारों का मसला है।

  1. एक तो इस संगठन की बनावट और इसकी प्रक्रियाओं में सुधार किया जाए।
  2. दूसरा, इस संगठन के न्यायाधिकार में आने वाले मुद्दों की समीक्षा की जाए।

प्रश्न 14.
संयुक्त राष्ट्र संघ की स्थापना कब हुई ? आजकल इसके कितने सदस्य हैं?
उत्तर:
संयुक्त राष्ट्र संघ की विधिवत् स्थापना 24 अक्टूबर, 1945 ई. को हुई थी। इस संस्था का मुख्य कार्यालय न्यूयार्क अमेरिका में है। आजकल संसार के छोटे-बड़े लगभग 193 देश इसके सदस्य हैं। करना।

JAC Class 12 Political Science Important Questions Chapter 6 अंतर्राष्ट्रीय संगठन

प्रश्न 15.
संयुक्त राष्ट्र संघ के प्रमुख उद्देश्यों का उल्लेख कीजिये।
उत्तर:
उद्देश्य-संयुक्त राष्ट्र संघ की स्थापना के मुख्य उद्देश्य निम्न हैं।

  1. अन्तर्राष्ट्रीय शांति और सुरक्षा को बनाये रखना।
  2. भिन्न-भिन्न राष्ट्रों के बीच मैत्रीपूर्ण संबंधों को बढ़ाना।
  3. आपसी सहयोग द्वारा आर्थिक, सामाजिक, सांस्कृतिक तथा मानवीय ढंग की अन्तर्राष्ट्रीय समस्याओं को हल

प्रश्न 16.
संयुक्त राष्ट्र की महासभा के संगठन को संक्षिप्त में बताइये।
उत्तर:
हासभा-महासभा संयुक्त राष्ट्र संघ का सबसे बड़ा अंग है। संयुक्त राष्ट्र के सभी सदस्य इसके सदस्य होते हैं और प्रत्येक सदस्य राष्ट्र इसमें पाँच प्रतिनिधि भेजता है परन्तु उनका मत एक ही होता है। प्राय: वर्ष में इसका एक बार अधिवेशन होता है। वर्तमान में इसके सदस्यों की कुल संख्या 193 है।

प्रश्न 17.
सुरक्षा परिषद् के संगठन पर संक्षिप्त टिप्पणी लिखिये।
उत्तर:
सुरक्षा परिषद् संयुक्त राष्ट्र की कार्यपालिका के समान है। इसके 15 सदस्य होते हैं। जिनमें पाँच स्थाई सदस्य हैं। अमेरिका, इंग्लैंड, फ्रांस, चीन और रूस । इसके अलावा 10 अस्थायी सदस्य दो वर्षों के लिये महासभा द्वारा चुने जाते हैं। सुरक्षा परिषद् के पाँच स्थायी सदस्यों को निषेधाधिकार प्राप्त है।

प्रश्न 18.
अन्तर्राष्ट्रीय आणविक ऊर्जा एजेन्सी क्या है?
उत्तर:
अन्तर्राष्ट्रीय आणविक ऊर्जा एजेन्सी परमाणविक ऊर्जा के शांतिपूर्ण उपयोग को बढ़ावा देने और सैन्य उद्देश्यों में इसके प्रयोग को रोकने की कोशिश करता है। इसके अधिकारी विश्व की परमाणविक सुविधाओं की जाँच करते हैं ताकि नागरिक परमाणु संयंत्रों का इस्तेमाल सैन्य उद्देश्यों के लिए न हो।

प्रश्न 19.
एमनेस्टी इंटरनेशनल क्या है?
उत्तर:
एमनेस्टी इन्टरनेशनल
एमनेस्टी इंटरनेशनल एक स्वयंसेवी संगठन है। यह पूरे विश्व में मानवाधिकारों की रक्षा के लिए अभियान चलाता है। यह संगठन मानवाधिकारों से जुड़ी रिपोर्ट तैयार और प्रकाशित करता है। ये रिपोर्टें मानवाधिकारों से संबंधित अनुसंधान और तरफदारी में बड़ी महत्त्वपूर्ण भूमिका निभाती हैं।

प्रश्न 20.
ह्यूमन राइट वाच क्या है?
उत्तर:
ह्यूमन राइट वाच (Human Right Watch):
ह्यमन राइट वाच मानवाधिकारों की वकालत और उनसे संबंधित अनुसंधान करने वाला एक अन्तर्राष्ट्रीय स्वयंसेवी संगठन है। यह अमरीका का सबसे बड़ा अन्तर्राष्ट्रीय मानवाधिकार संगठन है। यह विश्व की मीडिया का ध्यान मानवाधिकारों के उल्लंघन की ओर खींचता है।

प्रश्न 21.
विश्व व्यापार संगठन क्या है?
उत्तर:
विश्व व्यापार संगठन (WTO):
‘विश्व व्यापार संगठन’ (World Trade Organization) एक अन्तर्राष्ट्रीय संगठन है। यह वैश्विक व्यापार के नियमों को तय करता है। इसकी स्थापना सन् 1995 में हुई। इसके सदस्यों की संख्या 164 है।

JAC Class 12 Political Science Important Questions Chapter 6 अंतर्राष्ट्रीय संगठन

प्रश्न 22.
सन् 2018 तक स्थायी सदस्यों द्वारा वीटो पॉवर का इस्तेमाल कितनी-कितनी बार किया गया है? उत्तर-सन् 2018 तक स्थायी सदस्यों द्वारा वीटो पॉवर का इस्तेमाल इस प्रकार किया गया है।

  1. सोवियत संघ / रूस द्वारा 135 बार
  2. अमरीका द्वारा 84 बार
  3. ब्रिटेन द्वारा 32 बार
  4. फ्रांस द्वारा 18 बार
  5. चीन द्वारा 11 बार।

प्रश्न 23.
सन् 1992 में संयुक्त राष्ट्रसंघ की आमसभा में किन तीन शिकायतों का प्रस्ताव स्वीकृत हुआ था?
उत्तर:
सन् 1992 में संयुक्त राष्ट्रसंघ की आम सभा में निम्न तीन शिकायतों का प्रस्ताव स्वीकृत हुआ था।

  1. सुरक्षा परिषद् राजनीतिक वास्तविकताओं की नुमाइंदगी नहीं करती।
  2. इनके फैसलों पर पश्चिमी देशों के मूल्यों और हितों की छाप होती है और इन फैसलों पर चंद देशों का दबदबा होता है।
  3. सुरक्षा परिषद् में बराबर का प्रतिनिधित्व नहीं है।

प्रश्न 24.
सुरक्षा परिषद् के पांच स्थायी सदस्यों को वीटो का अधिकार क्यों दिया गया?
उत्तर:
सुरक्षा परिषद् के पांच स्थायी सदस्यों को वीटो का अधिकार इसलिये दिया गया क्योंकि-

  1. ये देश द्वितीय विश्व युद्ध के विजेता थे।
  2. राजनीतिक मामलों में इनकी सहमति सर्वाधिक महत्त्वपूर्ण थी।
  3. वीटो का अधिकार न देने पर संभवतः ये समस्याओं में अधिक रुचि नहीं लेते।

प्रश्न 25.
मानवाधिकार परिषद् के संस्थापक कौन थे? इनके कुछ प्रमुख उपलब्धियों का वर्णन कीजिए।
उत्तर:
मानवाधिकार परिषद् के आयोग की स्थापना कोफी ए. अन्नान ने की। इनकी कुछ उपलब्धियाँ निम्नलिखित

  1. ये (1997 से 2006 तक) संयुक्त राष्ट्र संघ के महासचिव रहे।
  2. इन्होंने एड्स, टीबी और मलेरिया से लड़ने के लिए एक वैश्विक कोष बनाया।
  3. इन्होंने शान्ति संस्थापक आयोग की स्थापना 2005 में की।
  4. महासचिव के रूप में किए गए कार्यों के कारण 2001 में इनको नोबेल शान्ति पुरस्कार दिया गया।

JAC Class 12 Political Science Important Questions Chapter 6 अंतर्राष्ट्रीय संगठन

प्रश्न 26.
” सोवियत संघ की गैर मौजूदगी में अमरीका एक भाग महाशक्ति है।” इस कथन का सत्यापन
कीजिए।
उत्तर:
सोवियत संघ की गैर मौजूदगी में अमरीका एकमात्र महाशक्ति है। क्योंकि

  1. अमरीका की ताकत पर आसानी से अंकुश नहीं लगाया जा सकता।
  2. अपनी सैन्य और आर्थिक ताकत के सहारे अमरीका संयुक्त राष्ट्रसंघ या किसी अन्य अंतर्राष्ट्रीय संगठन की अनदेखी कर सकता है।
  3. संयुक्त राष्ट्रसंघ के भीतर अमरीका का खास प्रभाव है क्योंकि वह संयुक्त राष्ट्रसंघ के बजट में सबसे ज्यादा योगदान करता है। संयुक्त राष्ट्रसंघ अमरीकी भू-क्षेत्र में स्थित है। संयुक्त राष्ट्रसंघ के कई नौकरशाह अमरीका के नागरिक हैं।

प्रश्न 27.
“अंतर्राष्ट्रीय मुद्रा कोष के हर सदस्य की राय का वजन बराबर नहीं है।” स्पष्ट कीजिए।
उत्तर:
अंतराष्ट्रीय मुद्राकोष में 189 सदस्य हैं लेकिन हर सदस्य की राय का वजन बराबर नहीं है। क्योंकि

  1. समूह – 7 के सदस्य ( अमरीका, जापान, जर्मनी, फ्रांस, ब्रिटेन, इटली और कनाडा) के पास 41.29% मत हैं। अकेले अमरीका के पास 16.52% मताधिकार है।
  2. अन्य अग्रणी सदस्यों में चीन ( 6.09%), भारत (2.64%), रूस (2.59% ), ब्राजील (2.22%) और सऊदी अरब (0.02%) है।

प्रश्न 28.
अंतर्राष्ट्रीय संगठन की आवश्यकता क्यों है?
उत्तर:
अंतर्राष्ट्रीय संगठन की आवश्यकता निम्न कारणों की वजह से है-

  1. समस्याओं का शांतिपूर्ण समाधान हेतु।
  2. ये संगठन सहयोग के उपाय जुटाने मेंसहायक होते हैं।
  3. ये संगठन नियमों तथा नौकरशाही की रूपरेखा तैयार करते हैं।
  4. ये शांति और प्रगति के प्रति मानवता की आशा का प्रतीक़ होते हैं

प्रश्न 29.
संयुक्त राष्ट्र के सिद्धान्तों का वर्णन कीजिये।
उत्तर:
संयुक्त राष्ट्र के सिद्धान्त-संयुक्त राष्ट्र संघ अपने उद्देश्यों की पूर्ति के लिये निम्नलिखित सिद्धान्तों के. अनुसार कार्य करता है।

  1. संयुक्त राष्ट्र · सभी सदस्यों की प्रभुसत्ता और समानता के सिद्धान्त में विश्वास रखता है।
  2. सभी राष्ट्रों की अखण्डता और राजनीतिक स्वाधीनता का आदर करें।
  3. संयुक्त राष्ट्र चार्टर के अनुसार कोई कार्यवाही करें तो सभी सदस्य राष्ट्रों को उसकी सहायता करनी चाहिये।
  4. सभी राष्ट्र अपने विवादों को शांतिपूर्ण तरीकों से निपटायें।
  5. संयुक्त राष्ट्र उन मामलों में हस्तक्षेप नहीं करेगा जो अनिवार्य रूप से किसी राज्य के आंतरिक अधिकार क्षेत्र में आते हैं।

JAC Class 12 Political Science Important Questions Chapter 6 अंतर्राष्ट्रीय संगठन

प्रश्न 30.
क्या संयुक्त राष्ट्र अमरीकी प्रभुत्व के खिलाफ संतुलन के रूप में कार्य कर सकता है?
उत्तर:
संयुक्त राष्ट्रसंघ का अमरीकी प्रभुत्व के खिलाफ संतुलन के रूप में कार्य न कर पाने की निम्न वजहें हैं।

  1. अमरीका अपने वीटो पावर के द्वारा ऐसे किसी भी कदम को रोक सकती है जिससे उसका हित ना सधता हो।
  2. 1991 के बाद से अमरीका एकमात्र महाशक्ति बन कर उभरा है जो अपनी आर्थिक और सैन्य ताकत से किसी भी अंतर्राष्ट्रीय संगठनों की अनदेखी कर सकता है।
  3. अपनी ताकत और निषेधाधिकार के कारण संयुक्त राष्ट्रसंघ के महासचिव के चयन में भी अमरीका की बात बहुत वजन रखती है।

प्रश्न 31.
” भारत संयुक्त राष्ट्र सुरक्षा परिषद् का स्थायी सदस्य बनना चाहता है।” क्या इस कथन से आप सहमत हैं? तर्कों के माध्यम से स्पष्ट कीजिए।
उत्तर:
भारत संयुक्त राष्ट्र सुरक्षा परिषद् का स्थायी सदस्य बनना चाहता है। इस कथन से मैं सहमत हूँ। निम्न कथनों के द्वारा इस कथन को स्पष्ट किया जा सकता है-

  1. भारत विश्व में आबादी वाला दूसरा बड़ा देश है।
  2. भारत में विश्व की कुछ जनसंख्या का 1/5 वाँ हिस्सा निवास करता है।
  3. भारत विश्व का सबसे बड़ा लोकतंत्र है।
  4. भारत ने संयुक्त राष्ट्र की लगभग सभी पहलकदमियों में भाग लिया है।
  5. संयुक्त राष्ट्रसंघ के शांति बहाल करने के प्रयासों में भारत लंबे समय से ठोस भूमिका निभाता आ रहा है।
  6. भारत तेजी से अंतर्राष्ट्रीय फलक पर आर्थिक शक्ति बनकर उभर रहा है।
  7. भारत ने संयुक्त राष्ट्रसंघ के बजट में नियमित रूप से अपना योगदान दिया है कभी भी यह अपने भुगतान से चुका नहीं है।

प्रश्न 32.
संयुक्त राष्ट्र सुरक्षा परिषद् के 5 स्थायी सदस्यों के नाम बताइये तथा इसकी कार्यप्रणाली की कमियाँ बताइये।
उत्तर:

  • संयुक्त राष्ट्र सुरक्षा परिषद् के 5 स्थायी सदस्य हैं।
    1. अमेरिका
    2. फ्रांस
    3. ब्रिटेन
    4. रूस
    5. चीन।
  • संयुक्त राष्ट्र सुरक्षा परिषद् की कार्यप्रणाली की कमियाँ ये हैं।
    1. सुरक्षा परिषद् अब राजनीतिक वास्तविकताओं की नुमाइंदगी नहीं करती।
    2. इसके फैसलों पर पश्चिमी मूल्यों और हितों की छाप तथा चंद देशों का दबदबा होता है।
    3. इसमें बराबर का प्रतिनिधित्व नहीं है।

प्रश्न 33.
एक ध्रुवीय विश्व में संयुक्त राष्ट्रसंघ की प्रासंगिकता को स्पष्ट कीजिए।
उत्तर:

  1. संयुक्त राष्ट्रसंघ क्षेत्रीय या विश्वव्यापी विवाद के मुद्दों पर सभी देशों का बातचीत करके हल निकालने का मंच देता है।
  2. अमरीकी सरकार यह जानती है कि झगड़ों और सामाजिक आर्थिक विकास के मसलों पर संयुक्त राष्ट्रसंघ के माध्यम से 193 राष्ट्रों को एकत्रित किया जा सकता है।
  3. शेष विश्व के लिए संयुक्त राष्ट्रसंघ एक ऐसा मंच है जहाँ अमरीकी रवैये और नीतियों पर अंकुश लगाया जा सकता है।

JAC Class 12 Political Science Important Questions Chapter 6 अंतर्राष्ट्रीय संगठन

प्रश्न 34.
एक ध्रुवीय विश्व में संयुक्त राष्ट्र संघ की क्या भूमिका है? व्याख्या कीजिये।
अथवा
क्या आप इस बात से सहमत हैं कि संयुक्त राष्ट्र संघ जैसा अन्तर्राष्ट्रीय संगठन पूरे विश्व के लिए अपरिहार्य है? अपने उत्तर के पक्ष में कोई चार कारण स्पष्ट कीजिए। एक ध्रुवीय विश्व में संयुक्त राष्ट्र संघ की प्रासंगिकता को स्पष्ट कीजिये।
उत्तर:
संयुक्त राष्ट्र संघ जैसा अन्तर्राष्ट्रीय संगठन पूरे विश्व के लिए अपरिहार्य है। एक ध्रुवीय विश्व में संयुक्त राष्ट्र संघ की प्रासंगिकता या अपरिहार्यता को निम्न प्रकार स्पष्ट किया जा सकता है।

  1. संयुक्त राष्ट्र संघ अमरीका और शेष विश्व के बीच विभिन्न मुद्दों पर बातचीत कायम कर सकता है।
  2. झगड़ों और सामाजिक-आर्थिक विकास के मुद्दे पर संयुक्त राष्ट्र संघ के जरिये 193 देशों को एक साथ किया जा सकता है।
  3. शेष विश्व संयुक्त राष्ट्र संघ के मंच के माध्यम से अमरीकी रवैये और नीतियों पर कुछ न कुछ अंकुश लगा सकता है।
  4. आज विभिन्न समाजों और मुद्दों के बीच आपसी तार जुड़ते जा रहे हैं। आने वाले दिनों में पारस्परिक निर्भरता बढ़ती जायेगी । इसलिये संयुक्त राष्ट्र संघ का महत्त्व भी निरन्तर बढ़ेगा ।

प्रश्न 35.
अन्तर्राष्ट्रीय मुद्राकोष पर संक्षिप्त टिप्पणी लिखिये।
अथवा
अन्तर्राष्ट्रीय मुद्राकोष क्या है? कितने देश इसके सदस्य हैं?
उत्तर:
अन्तर्राष्ट्रीय मुद्रा कोष: यह संगठन वैश्विक स्तर की वित्त व्यवस्था की देखरेख करता है और मांगे जाने. पर वित्तीय तथा तकनीकी सहायता मुहैया करता है। वैश्विक स्तर की वित्त व्यवस्था का आशय अन्तर्राष्ट्रीय स्तर पर काम करने वाली वित्तीय संस्थाओं और लागू होने वाले नियमों से है। वर्तमान में 189 देश अन्तर्राष्ट्रीय मुद्रा कोष के सदस्य हैं लेकिन हर सदस्य की राय का वजन बराबर नहीं है। समूह -7 के सदस्य (अमरीका, जापान, जर्मनी, फ्रांस, ब्रिटेन, इटली और कनाडा) के पास 41.29 प्रतिशत मत है। अन्य अग्रणी सदस्यों में चीन ( 6.09%), भारत (2.64%), रूस (2.59%), ब्राजील. (2.22%) और सऊदी अरब (2.02%) है। अकेले अमरीका के पास 16.52% मताधिकार है।

प्रश्न 36.
विश्व बैंक की स्थापना कब हुई? इसके प्रमुख कार्यों को संक्षेप में बताइये।
उत्तर:
विश्व बैंक – दूसरे विश्व युद्ध के तुरन्त बाद सन् 1945 में विश्व बैंक की औपचारिक स्थापना हुई। विश्व बैंक के कार्य-विश्व बैंक की गतिविधियाँ प्रमुख रूप से विकासशील देशों से संबंधित हैं। यथा

  1. यह बैंक मानवीय विकास ( शिक्षा, स्वास्थ्य), कृषि और ग्रामीण विकास (सिंचाई, ग्रामीण सेवाएँ), पर्यावरण सुरक्षा (प्रदूषण में कमी, नियमों का निर्माण और उन्हें लागू करना), आधारभूत ढाँचा (सड़क, शहरी विकास, बिजली) तथा सुशासन ( कदाचार का विरोध, विधिक संस्थाओं का विकास) के लिए काम करता है ।
  2. यह अपने सदस्य देशों को आसान ऋण और अनुदान देता है । ज्यादा गरीब देशों को ये अनुदान वापिस नहीं चुकाने पड़ते। इस अर्थ में यह संस्था समकालीन वैश्विक अर्थव्यवस्था को भी प्रभावित करती है।

प्रश्न 37.
अन्तर्राष्ट्रीय संगठन की आवश्यकता के कोई दो बिन्दु बताइये।
उत्तर:
अन्तर्राष्ट्रीय संगठन की आवश्यकता – अन्तर्राष्ट्रीय संगठन की आवश्यकता को निम्नलिखित बिन्दुओं के अन्तर्गत स्पष्ट किया गया है।

  1. समस्याओं के शांतिपूर्ण समाधान के लिए आवश्यक – बातचीत के माध्यम से दो या अधिक देशों के मध्य के झगड़ों और विभेदों को बिना युद्ध के हल करने की दृष्टि से अन्तर्राष्ट्रीय संगठनों की भूमिका महत्त्वपूर्ण होती है। अन्तर्राष्ट्रीय संगठन समस्याओं के शांतिपूर्ण समाधान में सदस्य देशों की मदद करते हैं।
  2. चुनौतीपूर्ण समस्याओं को निपटाने में विभिन्न देशों को मिलकर कार्य करने में सहायता करना- अन्तर्राष्ट्रीय संगठन ऐसी चुनौतीपूर्ण समस्याओं को निपटाने के लिए आवश्यक है, जिनसे निपटने के लिए विभिन्न देशों को मिलाकर सहयोग करना आवश्यक होता है।

प्रश्न 38.
1991 के बाद वैश्विक – व्यवस्था में क्या प्रमुख परिवर्तन आये हैं?’
उत्तर:
1991 के बाद विश्व की राजनीतिक और आर्थिक स्थितियों में निम्नलिखित परिवर्तन आए हैं।

  1. 1991 के बाद शीत युद्ध काल की दो महाशक्तियों – सोवियत संघ और संयुक्त राज्य अमेरिका में से एक महाशक्ति सोवियत संघ का पतन हो गया है तथा वह बिखर गया है।
  2. सोवियत संघ के उत्तराधिकारी राज्य रूस और अमरीका के बीच अब संबंध कहीं ज्यादा सहयोगात्मक हैं।
  3. चीन बड़ी तेजी से एक महाशक्ति के रूप में उभर रहा है।
  4. सोवियत संघ से स्वतंत्र हुए अनेक नये देश संयुक्त राष्ट्र संघ में शामिल हुए हैं।
  5. वर्तमान में विश्व के सामने नयी चुनौतियों की एक पूरी कड़ी विद्यमान है। ये चुनौतियाँ हैं। जनसंहार, गृहयुद्ध, जातीय संघर्ष, आतंकवाद, परंमाणविक प्रसार, जलवायु में बदलाव, पर्यावरण की हानि, महामारी आदि।

प्रश्न 39.
संयुक्त राष्ट्र संघ को अधिक प्रासंगिक बनाने के लिए हाल ही में क्या कदम उठाये गये हैं?
उत्तर:
संयुक्त राष्ट्र संघ को अधिक प्रासंगिक बनाने के लिए हाल ही में निम्नलिखित कदम उठाये गये हैं।

  1. शांति संस्थापक आयोग का गठन किया गया है।
  2. यदि कोई राष्ट्र अपने नागरिकों को अत्याचारों से बचाने में असफल हो जाए तो विश्व बिरादरी इसका उत्तरदायित्व ले- इस बात की स्वीकृति।
  3. मानवाधिकार परिषद की स्थापना
  4. सहस्राब्दि विकास लक्ष्य को प्राप्त करने पर सहमति
  5. हर रूप रीति के आतंकवाद की निंदा करना
  6. एक लोकतंत्र कोष का गठन।

JAC Class 12 Political Science Important Questions Chapter 6 अंतर्राष्ट्रीय संगठन

प्रश्न 40.
सुरक्षा परिषद् के स्थायी व अस्थायी सदस्यों के लिए सुझाये गये कोई चार मानदण्ड बताइये।
उत्तर:
सुरक्षा परिषद् के स्थायी व अस्थायी सदस्यों के लिए चार आवश्यक मानदण्ड ये होने चाहिए:

  1. वह देश क्षेत्र तथा जनसंख्या की दृष्टि से बड़ा देश हो।
  2. वह एक लोकतांत्रिक देश हो।
  3. वह आर्थिक तथा सैनिक शक्ति के रूप में उभर रहा हो तथा संयुक्त राष्ट्र संघ के बजट में अपना नियमित योगदान देता आ रहा हो।
  4. वह संयुक्त राष्ट्र संघ के शांति बहाल करने के प्रयासों में निरन्तर अपनी प्रभावी भूमिका निभाता आ रहा हो।

प्रश्न 41.
संयुक्त राष्ट्र संघ के उद्देश्यों का वर्णन कीजिए।
उत्तर:
संयुक्त राष्ट्र संघ के उद्देश्य: संयुक्त राष्ट्र संघ के प्रमुख उद्देश्य निम्नलिखित हैं।

  1. अन्तर्राष्ट्रीय शांति और सुरक्षा बनाए रखना।
  2. सभी राष्ट्रों के बीच लोगों के समान अधिकार एवं स्वतंत्रता के सिद्धान्तों पर आधारित मैत्रीपूर्ण सम्बन्धों पर विकास करना और विश्व शान्ति को सुदृढ़ बनाने के लिए उचित उपाय करना।
  3. सामाजिक, आर्थिक, सांस्कृतिक और मानवीय क्षेत्रों में अन्तर्राष्ट्रीय सहयोग को प्रोत्साहित व मजबूत करना तथा सबके लिए मानवाधिकारों और मौलिक स्वतंत्रताओं के सम्मान को बढ़ाना और प्रोत्साहन देना।
  4. उपर्युक्त सामूहिक उद्देश्यों की प्राप्ति में राष्ट्रों के कार्यों में समन्वय बनाने के लिए संयुक्त राष्ट्र संघ को केन्द्र बनाना।

प्रश्न 42.
एक विश्व संस्था के रूप में संयुक्त राष्ट्रसंघ की असफलताओं पर प्रकाश डालिये।
उत्तर:
एक विश्व संस्था के रूप में संयुक्त राष्ट्रसंघ की असफलताएँ निम्न हैं-

  1. कुछ मसलों पर जैसे निःशस्त्रीकरण, विकसित और विकासशील देशों के बीच आर्थिक असन्तुलन कम करने में आंशिक सफलता ही मिली।
  2. संयुक्त राष्ट्रसंघ अनेक विवादों को सुलझाने तथा महाशक्तियों की मनमानी को रोकने में असफल रहा है।

प्रश्न 43.
‘पारस्परिक निर्भरता’ से आप क्या समझते हैं?
उत्तर:
पारस्परिक निर्भरता – हालाँकि संयुक्त राष्ट्रसंघ में थोड़ी कमियाँ हैं लेकिन इसका अस्तित्व आवश्यक है क्योंकि इसके बिना दुनिया और बदहाल हो जाएगी। संयुक्त राष्ट्रसंघ विश्व के सात अरब लोगों को एक साथ रहने के लिए मददगार है। आज विभिन्न समाजों और मसलों के बीच तार जुड़ते जा रहे हैं, इसे ही ‘पारस्परिक निर्भरता’ का नाम दिया गया है।

प्रश्न 44.
संयुक्त राष्ट्रसंघ की आर्थिक ओर सामाजिक परिषद् के ऊपर संक्षेप में टिप्पणी लिखिए।
उत्तर:
संयुक्त राष्ट्रसंघ की आर्थिक और सामाजिक परिषद् में कुल 54 सदस्य होते हैं जिनका चुनाव महासभा 2/3 बहुमत से तीन वर्ष के लिए करती है। इसके 1/3 सदस्य प्रतिवर्ष अवकाश ग्रहण करते हैं । इस प्रकार यह परिषद् एक स्थायी संस्था है।

प्रश्न 45.
संयुक्त राष्ट्रसंघ की सफलताओं पर प्रकाश डालिए।
उत्तर:
संयुक्त राष्ट्रसंघ की सफलताओं को निम्न बिन्दुओं द्वारा स्पष्ट किया जा सकता है।

  1. आतंकवाद का विरोध करने में।
  2. निःशस्त्रीकरण।
  3. शांति एवं सुरक्षा की स्थापना करने में।
  4. अन्तर्राष्ट्रीय भावना का विकास करने में।
  5. मानवाधिकारों का संरक्षण करने में।

निबन्धात्मक प्रश्न

प्रश्न 1.
हमें अन्तर्राष्ट्रीय संगठन क्यों चाहिए? स्पष्ट कीजिए।
उत्तर:
अन्तर्राष्ट्रीय संगठन की आवश्यकता हमें अन्तर्राष्ट्रीय संगठन की आवश्यकता है। इसके प्रमुख कारण निम्नलिखित हैं।

  1. समस्याओं के शांतिपूर्ण समाधान हेतु: अन्तर्राष्ट्रीय संगठन देशों की समस्याओं के शांतिपूर्ण समाधान में सदस्य देशों की सहायता करते हैं। अन्तर्राष्ट्रीय संगठन का निर्माण विभिन्न राज्य ही करते हैं और यह उनके मामलों के लिए जवाबदेह होता है। एक बार इसका निर्माण हो जाने के बाद ये सदस्य देशों की समस्याओं के शांतिपूर्ण समाधान में सहायता करते हैं।
  2. सहयोग के उपाय जुटाने में सहायक: एक अन्तर्राष्ट्रीय संगठन सहयोग करने के उपाय तथा सूचनाएँ एकत्रित करने में मदद कर सकता है।
  3. नियमों तथा नौकरशाही की रूपरेखा: एक अन्तर्राष्ट्रीय संगठन नियमों और नौकरशाही की एक रूपरेखा दे सकता है ताकि सदस्यों को यह विश्वास हो कि आने वाली लागत में सबकी समुचित साझेदारी होगी, लाभ का बँटवारा न्यायोचित होगा और यदि कोई सदस्य उस समझौते में शामिल हो जाता है तो वह इस समझौते के नियम और शर्तों का पालन करेगा।
  4. शांति और प्रगति के प्रति मानवता की आशा का प्रतीक: एक अन्तर्राष्ट्रीय संगठन शांति और प्रगति प्रति मानवता की आशा का प्रतीक होता है। वर्तमान में संयुक्त राष्ट्र संघ इसका प्रतीक है। इस संगठन को विश्व भर के अधिकांश लोग एक अनिवार्य संगठन मानते हैं।

JAC Class 12 Political Science Important Questions Chapter 6 अंतर्राष्ट्रीय संगठन

प्रश्न 2.
संयुक्त राष्ट्र संघ के विकास क्रम एवं उसकी स्थापना का वर्णन कीजिये।
उत्तर:
संयुक्त राष्ट्र संघ का विकास क्रम एवं स्थापना: संयुक्त राष्ट्र संघ के विकास क्रम को निम्न प्रकार स्पष्ट किया गया है।

  1. अटलांटिक चार्टर ( अगस्त, 1941 ): 14 अगस्त, 1941 को ब्रिटेन के प्रधानमंत्री चर्चिल और अमरीकी राष्ट्रपति रूजवेल्ट ने विश्व शांति की स्थापना के आधारभूत सिद्धान्तों की व्यवस्था की। इस पर दोनों देशों के अध्यक्षों ने हस्ताक्षर किये। इसे अटलांटिक चार्टर के नाम से जाना जाता है।
  2. संयुक्त राष्ट्र घोषणा-पत्र: धुरी शक्तियों के खिलाफ लड़ रहे 26 मित्र राष्ट्र अटलांटिक चार्टर के समर्थन में वाशिंगटन में मिले और दिसम्बर 1943 में संयुक्त राष्ट्र संघ के घोषणा-पत्र पर हस्ताक्षर किये।
  3. याल्टा सम्मेलन ( फरवरी, 1945 ): तीन बड़े नेताओं ( रूजवेल्ट, स्टालिन और चर्चिल) ने याल्टा सम्मेलन में संयुक्त राष्ट्र संघ के गठन, उसकी सदस्यता एवं प्रकृति पर विचार किया तथा एक सम्मेलन करने का निर्णय किया।
  4. सेनफ्रांसिस्को सम्मेलन (अप्रेल-मई 1945): सेन फ्रांसिस्को सम्मेलन में 50 देशों के प्रतिनिधियों ने भाग लिया। इसमें संयुक्त राष्ट्र संघ का चार्टर तैयार किया गया । 26 जून, 1945 को इस घोषणा पत्र (चार्टर) पर 50 देशों के प्रतिनिधियों ने तथा पोलैंड ने 15 अक्टूबर, 1945 को हस्ताक्षर किये। इस तरह संयुक्त राष्ट्र संघ में 51 मूल संस्थापक सदस्य हैं।
  5. संयुक्त राष्ट्र संघ की स्थापना: 24 अक्टूबर, 1945 को संयुक्त राष्ट्र संघ की स्थापना हुई । इसीलिए 24 अक्टूबर को संयुक्त राष्ट्र संघ का स्थापना दिवस मनाया जाता है। 10 फरवरी, 1946 को संयुक्त राष्ट्र संघ के विभिन्न पदाधिकारियों के चुनाव हुए।

प्रश्न 3.
संयुक्त राष्ट्र संघ के संगठन पर एक निबन्ध लिखिये।
उत्तर:
संयुक्त राष्ट्र संघ का संगठन: संयुक्त राष्ट्र संघ के संगठन का विवेचन निम्नलिखित बिन्दुओं के अन्तर्गत किया गया है।

  • संयुक्त राष्ट्र संघ की सदस्यता: वर्तमान में संयुक्त राष्ट्र संघ के सदस्यों की संख्या 193 है।
  • अधिकारिक भाषाएँ तथा मुख्यालय: सामान्य तौर पर संयुक्त राष्ट्र संघ की कार्यवाही की दो भाषाएँ हैं- अंग्रेजी व फ्रेंच। इनके अतिरिक्त चीनी, अरबी, स्पेनिश तथा रशियन भाषाओं को भी मान्यता प्राप्त है। इसका मुख्यालय न्यूयार्क शहर के मैनहट्टन द्वीप में है।
  • संयुक्त राष्ट्र संघ के मुख्य अंग ( निकाय ) संयुक्त राष्ट्र संघ के प्रमुख 6 अंग हैं। यथा
    1. महासभा (आमसभा ) महासभा संयुक्त राष्ट्र का मुख्य विचार-विनिमय निकाय है। इसमें सभी सदस्य देशों के प्रतिनिधि होते हैं।
    2. सुरक्षा परिषद् सुरक्षा परिषद् में कुल 15 सदस्य होते हैं। इसमें 5 स्थायी तथा 10 अस्थायी होते हैं। 5 स्थायी सदस्य देश हैं। अमरीका, चीन, रूस, ब्रिटेन और फ्रांस पाँचों स्थायी सदस्यों को ‘वीटो’ की शक्ति दी गई है।
    3. आर्थिक व सामाजिक परिषद्: आर्थिक-सामाजिक परिषद् के 54 सदस्य हैं, जिनका चुनाव महासभा 2/3 बहुमत से तीन वर्ष के लिए करती है। इसके 1/3 सदस्य प्रतिवर्ष अवकाश ग्रहण करते हैं। इस प्रकार यह परिषद् एक स्थायी संस्था है।
    4. ट्रस्टीशिप परिषद्: इस परिषद् का उद्देश्य 11 ट्रस्ट क्षेत्रों के प्रशासन की देखभाल करना था। 1994 तक ये . सभी ट्रस्ट स्वतंत्र हो चुके हैं।
    5. अन्तर्राष्ट्रीय न्यायालय: अन्तर्राष्ट्रीय न्यायालय में 15 न्यायाधीश होते हैं जिनका चयन महासभा और सुरक्षा परिषद् द्वारा एक साथ ही किया जाता है। न्यायाधीशों का कार्यकाल 9 वर्ष का होता है। इसका मुख्यालय हेग में है।
    6. सचिवालय: सचिवालय में महासचिव तथा संघ की आवश्यकता के अनुसार कर्मचारी रहते हैं।

प्रश्न 4.
संयुक्त राष्ट्र संघ के सुधारों में भारत की भूमिका का वर्णन करें।
उत्तर:
संयुक्त राष्ट्र संघ के सुधारों में भारत की भूमिका: भारत ने संयुक्त राष्ट्र संघ के ढांचे में सुधार के मुद्दे को निम्नलिखित आधारों पर समर्थन दिया है।

  • (1) संयुक्त राष्ट्र संघ की मजबूती पर बल: बदलते हुए विश्व में संयुक्त राष्ट्र संघ की मजबूती और दृढ़ता जरूरी है।
  • (2) विकास के मुद्दे पर बल: संयुक्त राष्ट्र संघ विभिन्न देशों के बीच सहयोग बढ़ाने और विकास को बढ़ावा देने में ज्यादा बड़ी भूमिका निभाए ।
  • (3) सुरक्षा परिषद् की संरचना में सुधार किया जाये:  इस संदर्भ में भारत के प्रमुख तर्क अग्रलिखित हैं।
    1. सुरक्षा परिषद् की संरचना प्रतिनिधित्वमूलक हो:  भारत का तर्क है कि सुरक्षा परिषद् का विस्तार करने पर वह ज्यादा प्रतिनिधिमूलक होगी तथा उसे विश्व – बिरादरी का अधिक समर्थन मिलेगा।
    2. सुरक्षा परिषद् में विकासशील देशों की संख्या बढ़ायी जाये – संयुक्त राष्ट्र संघ की आम सभा में ज्यादातर विकासशील सदस्य देश हैं। इसलिए सुरक्षा परिषद् में उनका यथोचित प्रतिनिधित्व होना चाहिए।
    3. सुरक्षा परिषद् की गतिविधियों का दायरा बढ़ा है: सुरक्षा परिषद् के काम-काज की सफलता विश्व- बिरादरी के समर्थन पर निर्भर है। इस कारण सुरक्षा परिषद् के पुनर्गठन की कोई योजना व्यापक धरातल पर बननी चाहिए।
    4. भारत को सुरक्षा परिषद् का स्थायी सदस्य बनाया जाये:  भारत स्वयं भी पुनर्गठित सुरक्षा परिषद् में एक स्थायी सदस्य बनना चाहता है।

JAC Class 12 Political Science Important Questions Chapter 6 अंतर्राष्ट्रीय संगठन

प्रश्न 5.
संयुक्त राष्ट्रसंघ के महासचिव के पद पर बैठने वाले दो एशियाई व्यक्तियों के नाम लिखिए। उनके कार्यकाल के दौरान उनके कार्यों का उल्लेख कीजिए।
उत्तर:
संयुक्त राष्ट्रसंघ के महासचिव के पद पर बैठने वाले दो एशियाई व्यक्ति हैं।
1. यू थांट-यू थांट 1961 से 1971 तक संयुक्त राष्ट्रसंघ के महासचिव के पद पर बने रहे। ये बर्मा (म्यांमार) से थे। पेशे से ये शिक्षक और राजनयिक थे। अपने महासचिव के कार्यकाल के दौरान इन्होंने क्यूबा के मिसाइल – संकट के समाधान और कांगो के संकट की समाप्ति के लिए प्रयास किए। साइप्रस में संयुक्त राष्ट्रसंघ की सेना बहाल की। वियतनाम युद्ध के दौरान अमरीका की आलोचना की।

2. बान की मून-बान की मून कोरिया गणराज्य से थे और 2007 से 2016 तक संयुक्त राष्ट्रसंघ के महासचिव रहे। ये संयुक्त राष्ट्रसंघ के आठवें महासचिव थे। महासचिव के पद पर बैठने वाले ये दूसरे एशियाई हैं। अपने कार्यकाल के दौरान इन्होंने जलवायु परिवर्तन पर विश्व का ध्यान खींचा। इनका ध्यान सहस्राब्दि विकास लक्ष्य और सतत विकास लक्ष्य की तरफ रहा। इन्होंने यूएन वीमेन के निर्माण में महत्त्वपूर्ण भूमिका निभाई । युद्ध वियोजन और परमाणु निरस्त्रीकरण पर जोर दिया।

प्रश्न 6.
1997 के बाद के सालों में सुरक्षा परिषद् की स्थायी और अस्थायी सदस्यता हेतु नए मानदंड सुझाए · गए, इनकी संक्षेप में व्याख्या कीजिए।
उत्तर:
1997 के बाद के सालों में सुरक्षा परिषद् की स्थायी और अस्थायी सदस्यता हेतु नये मानदंड सुझाए गए। इस सुझावों के अनुसार नए सदस्य को-

  1. बड़ी आर्थिक ताकत होना चाहिए।
  2. बड़ी सैन्य ताकत होना चाहिए।
  3. संयुक्त राष्ट्रसंघ के बजट में ऐसे देशों का योगदान ज्यादा होना चाहिए।
  4. आबादी के लिहाज से वह राष्ट्र बड़ा होना चाहिए।
  5. वह देश लोकतंत्र और मानवाधिकार का सम्मान करता हो।
  6. वह देश ऐसा हो जो अपने भूगोल, अर्थव्यवस्था और संस्कृति के लिहाज से विश्व की विविधता में नुमाइंदगी करता हो।

प्रश्न 7.
एक विश्व संस्था के रूप में संयुक्त राष्ट्र संघ की सफलताओं और असफलताओं पर एक लेख लिखिए।
उत्तर:
एक विश्व संस्था के रूप में संयुक्त राष्ट्र संघ की प्रमुख सफलताएँ व असफलताएँ निम्नलिखित हैं- संयुक्त राष्ट्र संघ की सफलताएँ

  1. शांति एवं सुरक्षा की स्थापना: संयुक्त राष्ट्र संघ ने अनेक अन्तर्राष्ट्रीय विवादों, मतभेदों व तनावों को अनेक बार युद्ध में परिणत होने से बचाया है।
  2. आतंकवाद का विरोध: संयुक्त राष्ट्र महासभा ने सब प्रकार के आतंकवाद के विरुद्ध एक प्रस्ताव पारित कर विश्व – -समुदाय से आतंकवाद की चुनौती को मिलकर सामना करने का आग्रह किया है।
  3. निःशस्त्रीकरण: संयुक्त राष्ट्र संघ ने निःशस्त्रीकरण के लिए अनेक प्रयास किये हैं।
  4. साम्राज्यवाद और उपनिवेशवाद का विरोध: संयुक्त राष्ट्र संघ ने साम्राज्यवाद और उपनिवेशवाद को समाप्त करने में बहुत सहायता दी है।
  5. आर्थिक एवं सामाजिक क्षेत्र में सफलताएँ – आर्थिक व सामाजिक असन्तोष, भूख, दरिद्रता, निरक्षरता आदि को दूर करने में संयुक्त राष्ट्र संघ ने निरन्तर प्रयत्न किये हैं।
  6. अन्तर्राष्ट्रीय भावना का विकास-संयुक्त राष्ट्र संघ निरन्तर अन्तर्राष्ट्रीय सहयोग, सद्भावना और सहअस्तित्व की भावना का विकास कर रहा है।
  7. मानवाधिकारों का संरक्षक: मानवाधिकार आयोग के माध्यम से संयुक्त राष्ट्र संघ विश्व में मानवाधिकारों के संरक्षक की भूमिका निभा रहा है।

संयुक्त राष्ट्र संघ की असफलताएँ

  1. आंशिक सफलताएँ: संयुक्त राष्ट्र संघ को निःशस्त्रीकरण, हिन्द महासागर को शांति क्षेत्र बनाने, विकसित और विकासशील देशों के बीच आर्थिक असन्तुलन को कम करने, समुद्री सम्पदा के उचित दोहन आदि मामलों में आंशिक सफलता ही मिली है।
  2. पूर्णतः असफलताएँ: संयुक्त राष्ट्र संघ अनेक विवादों को सुलझाने तथा महाशक्तियों की मनमानी रोकने में असफल रहा है।

प्रश्न 8.
एक ध्रुवीय विश्व में संयुक्त राष्ट्र संघ क्या अमरीका की मनमानी को रोक सकता है? यदि नहीं तो क्यों? एक ध्रुवीय विश्व में संयुक्त राष्ट्र संघ की प्रासंगिकता को स्पष्ट कीजिये।
उत्तर:
एक – ध्रुवीय विश्व में संयुक्त राष्ट्र संघ एक-ध्रुवीय विश्व में संयुक्त राष्ट्र अमरीका की मनमानी को नहीं रोक सकता क्योंकि

  1. अपनी सैन्य और आर्थिक ताकत के बल पर अमरीका संयुक्त राष्ट्र संघ या किसी अन्य अन्तर्राष्ट्रीय संगठन की अनदेखी कर सकता है।
  2. संयुक्त राष्ट्र संघ के भीतर उसके बजट, अमरीकी भू क्षेत्र में संयुक्त राष्ट्र संघ की स्थिति आदि के कारण अमरीका का अत्यधिक प्रभाव है।
  3. सुरक्षा परिषद् का वह स्थायी सदस्य है तथा उसके पास वीटो की शक्ति है।
  4. अमरीका अपनी ताकत और निषेधाधिकार के कारण संयुक्त राष्ट्र संघ के महासचिव के चयन में भी दखल रखता है।
  5. अमरीका अपनी सैन्य और आर्थिक शक्ति के बल पर विश्व बिरादरी में फूट डाल सकता है।

एक – ध्रुवीय विश्व में संयुक्त राष्ट्र की प्रासंगिकता: एक – ध्रुवीय विश्व में संयुक्त राष्ट्र संघ की प्रासंगिकता को निम्न प्रकार बताया जा सकता है।

  1. संयुक्त राष्ट्र संघ क्षेत्रीय या विश्वव्यापी विवाद के मुद्दों पर सभी देशों के बीच बातचीत का एक मंच प्रस्तुत करता है।
  2. अमरीकी नेता समझते हैं कि झगड़ों और सामाजिक-आर्थिक विकास के मुद्दे पर संयुक्त राष्ट्र संघ के जरिये 193 राष्ट्रों को एक साथ किया जा सकता है।
  3. शेष विश्व के लिए संयुक्त राष्ट्र संघ एक ऐसा मंच है जहाँ अमरीकी रवैये और नीतियों पर कुछ अंकुश लगाया जा सकता है।

JAC Class 12 Political Science Important Questions Chapter 6 अंतर्राष्ट्रीय संगठन

प्रश्न 9.
‘विश्व व्यापार संगठन’ पर टिप्पणी लिखिए।
उत्तर:
विश्व व्यापार संगठन (WTO):
यह एक अंतर्राष्ट्रीय संगठन है जो कि वैश्विक व्यापार के नियमों को तय करता है। इस संगठन की स्थापना 1995 में हुई थी। यह संगठन ‘जनरल एग्रीमेंट ऑन ट्रेड एंड टैरिफ के उत्तराधिकारी के रूप में काम करता है जो कि दूसरे विश्वयुद्ध के बाद अस्तित्व में आया था । 29 जुलाई, 2016 तक इसके सदस्यों की संख्या 164 थी।

इस संगठन में हर फैसला सभी सदस्यों की सहमति से किया जाता है लेकिन अमरीका, यूरोपीय संघ तथा जापान जैसी बड़ी आर्थिक शक्तियाँ विश्व व्यापार संगठन में व्यापार के नियमों को इस तरह बना दिया है जिससे उनका हित सधता हो। विकासशील देशों की अक्सर शिकायत रहती है इस संगठन की कार्यविधि पारदर्शी नहीं है और बड़ी आर्थिक ताकतें इन देशों को पीछे ढकेलती है।’

प्रश्न 10.
संयुक्त राष्ट्रसंघ के सुरक्षा परिषद् की संरचना का वर्णन कीजिए। इसके स्थायी और अस्थायी सदस्यों को दिए गए विशेषाधिकारों में क्या अंतर है।
उत्तर:
सुरक्षा परिषद् में पाँच स्थायी और दस अस्थायी सदस्य हैं।

  1. स्थायी सदस्य: दूसरे विश्वयुद्ध के बाद दुनिया में स्थिरता कायम करने के लिए संयुक्त राष्ट्रसंघ के घोषणा पत्र में पाँच स्थायी सदस्यों को विशेष हैसियत दी गई। पाँच स्थायी सदस्यों की सदस्यता स्थायी होगी और उन्हें ‘वीटो ‘ का अधिकार होगा।
  2. अस्थायी सदस्य: अस्थायी सदस्य दो वर्षों के लिए चुने जाते हैं और इस अवधि के बाद उनकी जगह नए सदस्यों का चयन होता है। दो साल की अवधि तक अस्थायी सदस्य रहने के तत्काल बाद किसी देश को फिर से इस पद के लिए नहीं चुना जा सकता अस्थायी सदस्यों का निर्वाचन इस प्रकार होता है कि विश्व के सभी महादेशों का प्रतिनिधित्व हो सके अस्थायी देशों को वीटो का अधिकार नहीं है।

प्रश्न 11.
कुछ देश संयुक्त राष्ट्र सुरक्षा परिषद् के स्थायी सदस्य के रूप में भारत को शामिल करने के मुद्दे पर सवाल क्यों उठाते हैं? व्याख्या कीजिए।
अथवा
भारत या किसी और देश के लिए भविष्य में संयुक्त राष्ट्रसंघ की सुरक्षा परिषद् का स्थायी सदस्य बन पाना मुश्किल लगता है। क्यों?
उत्तर:
यद्यपि भारत चाहता है कि वह संयुक्त राष्ट्रसंघ की सुरक्षा परिषद् का स्थायी सदस्य बने तथा उसे भी निषेधाधिकार प्राप्त हो लेकिन कुछ देश सुरक्षा परिषद् में भारत की स्थायी सदस्यता का विरोध करते हैं। यथा।

  1. सिर्फ पड़ोसी पाकिस्तान ही नहीं, जिनके साथ भारत के संबंध दिक्कततलब रहे हैं, बल्कि कुछ और देश भी चाहते हैं कि भारत को सुरक्षा परिषद् में वीटोधारी सदस्य के रूप में ना शामिल किया जाए।
  2. कुछ देश भारत के परमाणु हथियारों को लेकर चिंतित हैं।
  3. कुछ देशों का मानना है कि पाकिस्तान के साथ संबंधों में कठिनाई के कारण भारत स्थायी सदस्य के रूप में प्रभावी नहीं रहेगा।
  4. कुछ अन्य देशों की राय है कि उभरती हुई ताकत के रूप में अन्य देशों जैसे कि -ब्राजील, जर्मनी, जापान और शायद दक्षिण अफ्रीका को भी शामिल करना पड़ेगा जिसका ये देश विरोध करते हैं।
  5. कुछ देशों का मत है कि यदि सुरक्षा परिषद् में किसी तरह का विस्तार होता है तो अफ्रीका और दक्षिणी अमरीका को जरूर प्रतिनिधित्व मिलना चाहिए क्योंकि मौजूदा सुरक्षा परिषद् में इन्हीं महादेशों की नुमाइंदगी नहीं है। उपर्युक्त वजहों को देखते हुए यह अनुमान लगाया जा सकता है कि भारत या किसी और देश के लिए निकट भविष्य में संयुक्त राष्ट्रसंघ की सुरक्षा परिषद् का स्थायी सदस्य बन पाना मुश्किल है।

प्रश्न 12.
“संयुक्त राष्ट्रसंघ एक अपरिहार्य संगठन है।” स्पष्ट कीजिए।
उत्तर:
संयुक्त राष्ट्रसंघ एक अपरिहार्य संगठन है। इस कथन को निम्न तर्कों से स्पष्ट किया जा सकता है।

  1. संयुक्त राष्ट्रसंघ ही वो माध्यम है जिसके द्वारा अमरीका जो महाशक्ति है और बाकी विभिन्न देशों को विभिन्न मुद्दों पर चर्चा करने के लिए एकत्रित किया जा सकता है।
  2. संयुक्त राष्ट्रसंघ ने संघर्ष और आर्थिक विकास से निपटने के लिए 190 से अधिक देशों को एक साथ लाने का कार्य किया है।
  3. संयुक्त राष्ट्रसंघ अमरीका के अलावा बाकी देशों को बातचीत करने का मंच प्रदान करवाता है जहाँ अमरीकी दृष्टिकोण और नीतियों को संशोधित करना संभव हो। हालाँकि अमरीका के खिलाफ देशों को शायद ही एकजुट किया जाता है। संयुक्त राष्ट्र एक ऐसा मंच है जहाँ अमरीकी दृष्टिकोण तथा नीतियों के खिलाफ बहस सुनी जाती है और उसके अनुरूप बदलाव किया जा सकता है।
  4. वैश्वीकरण को इस आधुनिक दुनिया में संयुक्त राष्ट्रसंघ में कमियों के बावजूद इसका अस्तित्व आवश्यक है। आज़ विभिन्न समाजों और मसलों के बीच तार जुड़ते जा रहे हैं। इसे ‘पारस्परिक निर्भरता’ का नाम दिया गया है। प्रौद्योगिकी यह सिद्ध कर रही है कि आने वाले समय में विश्व में पारस्परिक निर्भरता बढ़ती जाएगी। ऐसी स्थिति में संयुक्त राष्ट्रसंघ उन तरीकों को खोजने में मददगार होगा जो अंतर्राष्ट्रीय समुदाय के हितों के अनुरूप होगा।

JAC Class 12 Political Science Important Questions Chapter 5 समकालीन दक्षिण एशिया

Jharkhand Board JAC Class 12 Political Science Important Questions Chapter 5 समकालीन दक्षिण एशिया Important Questions and Answers.

JAC Board Class 12 Political Science Important Questions Chapter 5 समकालीन दक्षिण एशिया

बहुचयनात्मक प्रश्न

1. दक्षिण एशिया में सबसे बड़ा देश है।
(अ) भारत
(ब) पाकिस्तान
(स) श्रीलंका
(द) बांग्लादेश
उत्तर:
(अ) भारत

2. दक्षिण एशियाई सहयोग संगठन (सार्क) का गठन किया गया।
(अ) 1985 में
(ब) 1986 में
(स) 1990 में
(द) 1991 में
उत्तर:
(अ) 1985 में

3. कश्मीर को लेकर भारत और पाकिस्तान के बीच लड़ाई कब हुई ?
(अ) 1954
(ब) 1947
(स) 1966
(द) 1955
उत्तर:
(ब) 1947

4. ‘साफ्टा’ सम्बन्धित है।
(अ) आसियान से
(ब) सार्क से
(स) हिमतेक्ष से
(द) ओपेक से
उत्तर:
(ब) सार्क से

5. आजादी के बाद से श्रीलंका की राजनीति पर किस समुदाय का दबदबा रहा?
(अ) तमिल
(ब) हिन्दू
(स) मुस्लिम
(द) सिंहली
उत्तर:
(द) सिंहली

JAC Class 12 Political Science Important Questions Chapter 5 समकालीन दक्षिण एशिया

6. भारत और पाकिस्तान के बीच 1948 में हुए युद्ध के फलस्वरूप किस प्रांत के दो हिस्से हुए?
(अ) बांग्लादेश
(ब) बंगाल
(स) कश्मीर
(द) असम
उत्तर:
(स) कश्मीर

7. 1998 में पाकिस्तान ने कहाँ पर परमाणु परीक्षण किया?
(अ) कश्मीर
(ब) पश्चिमी पाकिस्तान
(स) चगाई पहाड़ी
(द) पोखरण
उत्तर:
(स) चगाई पहाड़ी

8. बांग्लादेश एक स्वतन्त्र सम्प्रभु राष्ट्र बना।
(अ) 1947 में
(ब) 1948 में
(स) 1949 में
(द) 1971 में
उत्तर:
(द) 1971 में

रिक्त स्थानों की पूर्ति कीजिए

1. 1998 में भारत ने …………………………. में परमाणु परीक्षण किया।
उत्तर:
पोखरण

2. …………………… के दशक में भारत तथा पाकिस्तान ने परमाणु हथियार और ऐसे हथियारों को एक-दूसरे पर दागने ………………………की क्षमता वाले मिसाइल हासिल कर लिए।
उत्तर:
1990

3. 1960 में विश्व बैंक की मदद से भारत और पाकिस्तान ने …………………… पर दस्तखत किए।
उत्तर:
सिंधु जल संधि

4. भूटान ………………………. में संवैधानिक राजतंत्र बना।
उत्तर:
2008

5. मालदीव ………………………. में गणतंत्र बना और यहाँ शासन की ……………………… प्रणाली अपनायी गयी।
उत्तर:
1968, अध्यक्षात्मक

6. …………………………. से ………………………………. तक बांग्लादेश पाकिस्तान का अंग था।
उत्तर:
1947, 1971

अतिलघूत्तरात्मक प्रश्न

प्रश्न 1.
स्वतंत्रता के बाद दक्षिण एशिया के किन दो देशों में आज तक लोकतांत्रिक व्यवस्था कायम है?
उत्तर:
भारत और श्रीलंका में।

प्रश्न 2.
भारत में स्वतंत्रता के बाद किस प्रकार की शासन प्रणाली अपनाई गई?
उत्तर:
लोकतांत्रिक शासन प्रणाली।

प्रश्न 3.
भारत और पाकिस्तान के बीच हुए 1971 के युद्ध का क्या परिणाम निकला?
उत्तर:
1971 में भारत और पाकिस्तान के बीच हुए युद्ध से पूर्वी पाकिस्तान ‘बांग्लादेश’ के रूप में नया स्वतंत्र देश

प्रश्न 4.
भारत के पड़ौसी देश कौन-कौन से हैं?
उत्तर:
भारत के पड़ौसी देश हैं।

  1. पाकिस्तान,
  2. नेपाल,
  3. भूटान,
  4. बांग्लादेश,
  5. श्रीलंका,
  6. चीन आदि।

प्रश्न 5.
भारत और पाकिस्तान के बीच शिमला समझौता कब हुआ?
उत्तर:
जुलाई, 1972 में भारत-पाक के बीच शिमला समझौता हुआ।

JAC Class 12 Political Science Important Questions Chapter 5 समकालीन दक्षिण एशिया

प्रश्न 6.
श्रीलंका को आजादी के बाद से ही किस कठिन चुनौती का सामना करना पड़ रहा है?
उत्तर:
जातीय संघर्ष का।

प्रश्न 7.
दक्षिण एशियायी देशों द्वारा बहुस्तरीय साधनों से आपस में सहयोग के लिए उठाया गया कदम कौनसा है?’ – यह बड़ा कदम है। दक्षेस (सार्क) की स्थापना का।
उत्तर:

प्रश्न 8.
12वें दक्षेस सम्मेलन में किस संधि पर हस्ताक्षर किये गये?
उत्तर:
मुक्त व्यापार संधि पर।

प्रश्न 9.
दक्षेस के देशों ने साफ्टा (SAFTA) पर कब हस्ताक्षर किये?
उत्तर:
दक्षेस के देशों ने सन् 2004 में साफ्टा पर हस्ताक्षर किये।

प्रश्न 10.
साफ्टा में क्या वायदा किया गया है?
उत्तर:
‘साफ्टा’ में पूरे दक्षिण एशिया के लिए मुक्त व्यापार क्षेत्र बनाने का वायदा है।

प्रश्न 11.
सार्क का पूरा नाम लिखिये।
उत्तर:
सार्क का पूरा नाम है। साउथ एशियन एसोसियेशन फॉर रीजनल कोऑपरेशन (दक्षिण एशियाई क्षेत्रीय सहयोग संघ)।

प्रश्न 12.
सार्क में कितने देश हैं?
उत्तर:
सार्क में कुल आठ देश हैं। ये हैं- भारत, पाकिस्तान, नेपाल, बांग्लादेश, श्रीलंका, भूटान, मालदीव और अफगानिस्तान।

JAC Class 12 Political Science Important Questions Chapter 5 समकालीन दक्षिण एशिया

प्रश्न 13.
दक्षिण एशिया का इलाका एक विशिष्ट प्राकृतिक क्षेत्र के रूप में नजर आता है। क्यों?
उत्तर:
उत्तर की विशाल हिमालय पर्वत श्रृंखला, दक्षिण का हिन्द महासागर, पश्चिम का अरब सागर और पूरब में मौजूद बंगाल की खाड़ी से यह इलाका एक विशिष्ट प्राकृतिक क्षेत्र के रूप में नजर आता है। उत्तर- ‘साफ्टा’ का पूरा नाम है।’ दक्षिण एशियाई मुक्त व्यापार क्षेत्र समझौता’ (साउथ एशियन फ्री ट्रेड एरिया)।

प्रश्न 14.
दक्षेस को अधिक प्रभावी बनाने हेतु कोई दो सुझाव दीजिये।
उत्तर:

  1. महाशक्तियों को इस क्षेत्र से दूर रखा जाये।
  2. अन्तर्राष्ट्रीय मंचों पर दक्षेस देशों द्वारा सर्वसम्मत दृष्टिकोण अपनाया जाये।

प्रश्न 15.
ताशकंद समझौता कब और किसके मध्य हुआ?
उत्तर:
सन् 1966 में भारत और पाकिस्तान के मध्य ताशकंद समझौता हुआ।

प्रश्न 16.
दो समझौतों के नाम लिखिये जिन्हें भारत ने पाकिस्तान के साथ किये।
उत्तर:

  1. 1965 में ताशकंद समझौता
  2. 1971 में शिमला समझौता।

प्रश्न 17.
शिमला समझौते के दो प्रावधान लिखिये।
उत्तर:

  1. दोनों देश अपने मतभेदों को द्विपक्षीय वार्ता द्वारा शांतिपूर्ण ढंग से हल करने का प्रयास करेंगे।
  2. दोनों देश एक-दूसरे के विरुद्ध बल का प्रयोग नहीं करेंगे।

प्रश्न 18.
भारत और बांग्लादेश के बीच तनाव के दो मुख्य बिन्दु बताइये।
उत्तर:

  1. कमा शरणार्थियों की समस्या।
  2. गंगा – जल के वितरण की समस्या।

प्रश्न 19.
भारत-पाक मैत्री के मार्ग में आने वाली किन्हीं दो बाधाओं का उल्लेख कीजिये।
उत्तर:
ये दो बाधायें हैं।

  1. चीन का पाकिस्तान को सामरिक सहयोग तथा
  2. भारतीय आतंकवादियों को प्रशिक्षण व सहायता देना।

JAC Class 12 Political Science Important Questions Chapter 5 समकालीन दक्षिण एशिया

प्रश्न 20.
भारत को किन दो कठिनाइयों का सामना पड़ौसियों के साथ अच्छे सम्बन्ध बनाने में करना पड़ता है?
उत्तर:

  1. पड़ौसी देशों द्वारा भारतीय आतंकवादियों को संरक्षण, प्रशिक्षण व सहयोग देना।
  2. सीमा विवाद तथा शरणार्थियों की समस्या।

प्रश्न 21.
राजनीतिक प्रणाली की दृष्टि से भारत और श्रीलंका की किसी एक समानता को बताइये।
उत्तर:
भारत और श्रीलंका में ब्रिटेन से आजाद होने के बाद, लोकतांत्रिक व्यवस्था सफलतापूर्वक कायम है। एक राष्ट्र के रूप में भारत और श्रीलंका हमेशा लोकतांत्रिक रहे हैं।

प्रश्न 22.
पाकिस्तान में शीत युद्ध के बाद के सालों में कौनसी दो लोकतांत्रिक सरकारें बनीं?
उत्तर:
पाकिस्तान में शीत युद्ध के बाद के सालों में लगातार ये दो लोकतांत्रिक सरकारें बनीं

  1. बेनजीर भुट्टो के नेतृत्व में बनी सरकार और
  2. नवाज शरीफ के नेतृत्व में बनी सरकार।

प्रश्न 23.
दक्षिण एशिया के दो सबसे छोटे देशों के नाम लिखिये।
उत्तर:
दक्षिण एशिया के दो सबसे छोटे देश हैं। मालदीव और भूटान।

प्रश्न 24.
मालदीव में 1968 के बाद कौनसी शासन प्रणाली अपनाई गई?
अथवा
मालदीव में कैसी शासन प्रणाली है?
उत्तर:
1968 से मालदीव में लोकतन्त्र की अध्यक्षात्मक शासन प्रणाली अपनाई गई है।

प्रश्न 25.
दक्षिण एशियायी देशों की जनता किस प्रकार के शासन को वरीयता देती है?
उत्तर:
दक्षिण एशियायी देशों के लोग लोकतंत्र को वरीयता देते हैं।

प्रश्न 26.
दक्षेस का कार्यालय कहाँ स्थित है?
उत्तर:
दक्षेस का कार्यालय नेपाल की राजधानी काठमाण्डू में स्थित है।

प्रश्न 27.
भारत के उत्तर में किन्हीं दो सदस्य देशों का उल्लेख कीजिये जो सार्क में सम्मिलित हैं।
उत्तर:
भारत के उत्तर में स्थित नेपाल और भूटान सार्क के सदस्य देश हैं।

प्रश्न 28.
भारत के दक्षिण में सार्क के किन्हीं दो सदस्य देशों का उल्लेख कीजिये।
उत्तर:
श्रीलंका और मालदीव

JAC Class 12 Political Science Important Questions Chapter 5 समकालीन दक्षिण एशिया

प्रश्न 29.
आर्थिक वैश्वीकरण का दक्षिण एशिया पर क्या सकारात्मक प्रभाव पड़ा?
उत्तर:
आर्थिक वैश्वीकरण से दक्षिण एशिया में मुक्त व्यापार को बढ़ावा मिला तथा आर्थिक सुविधाओं में गुणात्मक सुधार हुआ।

प्रश्न 30.
नेपाल को धर्मनिरपेक्ष राज्य में कब परिवर्तित किया गया?
उत्तर:
नेपाल को 17 मई, 2006 को एक धर्मनिरपेक्ष राज्य में परिवर्तित किया गया।

प्रश्न 31.
श्रीलंका में जातीय संघर्ष के कोई दो कारण बताइये।
उत्तर:

  1. श्रीलंका में सिंहली जाति बहुसंख्या में है।
  2. उसमें बहुसंख्यकवाद की भावना है जो अल्पसंख्यक तमिलों की उपेक्षा करती है।

प्रश्न 32
भारत द्वारा शांति सेना कब और किस देश में भेजी गई?
उत्तर:
1987 में श्रीलंका में।

प्रश्न 33.
श्रीलंका के जातीय समुदायों के नाम लिखिये।
उत्तर:
श्रीलंका के जातीय समुदाय ये हैं

  1. सिंहली
  2. श्रीलंकाई तमिल
  3. भारतवंशी तमिल तथा
  4. मुसलमान।

JAC Class 12 Political Science Important Questions Chapter 5 समकालीन दक्षिण एशिया

प्रश्न 34.
सियाचीन की समस्या किन देशों के बीच मतभेद का प्रमुख कारण है?
उत्तर:
भारत और पाकिस्तान के बीच।

प्रश्न 35.
फरक्का संधि पर हस्ताक्षर कब और किनके बीच हुआ था?
उत्तर:
फरक्का संधि पर हस्ताक्षर दिसंबर, 1996 में भारत और बांग्लादेश के बीच हुआ था।

प्रश्न 36.
1975 में बांग्लादेश के संविधान में क्या संशोधन किया गया?
उत्तर:
1975 में शेख मुजीबुर्रहमान ने संविधान में संशोधन कराया और संसदीय प्रणाली की जगह अध्यक्षात्मक शासन प्रणाली को मान्यता मिली।

प्रश्न 37.
श्रीलंका में पैदा हुए जान्ति संघर्ष का निवारण करने हेतु किन दो स्कैंडिनेवियाई देशों ने मध्यस्थ की भूमिका निभाई?
उत्तर:
नार्वे और आइसलैंड।

लघूत्तरात्मक प्रश्न

प्रश्न 1.
दक्षिण एशिया से क्या आशय है?
उत्तर:
सामान्यतः दक्षिण एशिया प्रदेश का प्रयोग सात देशों

  1. बांग्लादेश,
  2. भूटान,
  3. भारत,
  4. मालदीव,
  5. नेपाल,
  6. पाकिस्तान और
  7. श्रीलंका के लिए किया जाता है। इसमें जब-तब अफगानिस्तान और म्यांमार को भी शामिल किया जाता है।

प्रश्न 2.
पाकिस्तान में लोकतंत्र के स्थायी न बन पाने के क्या कारण हैं?
उत्तर:
पाकिस्तान में लोकतंत्र के स्थायी न बन पाने के प्रमुख कारण ये हैं।

  1. यहाँ सेना, धर्मगुरु और भू-स्वामी अभिजनों का सामाजिक दबदबा है।
  2. भारत के साथ निरन्तरं तनातनी रहने के कारण सेना समर्थक समूह अधिक मजबूत है।
  3. अमरीका तथा अन्य पश्चिमी देशों ने अपने स्वार्थ पूर्ति हेतु पाकिस्तान में सैनिक शासन को बढ़ावा दिया।

प्रश्न 3.
पूर्वी पाकिस्तान के लोग मूलतः पश्चिमी पाकिस्तान के विरोधी क्यों थे?
उत्तर:
पूर्वी पाकिस्तान क्षेत्र के लोग पश्चिमी पाकिस्तान के दबदबे और अपने ऊपर उर्दू भाषा को लादने के खिलाफ थे। पाकिस्तान के निर्माण के तुरंत बाद ही यहाँ के लोगों ने बंगाली संस्कृति और भाषा के साथ किये जा रहे दुर्व्यवहार के खिलाफ विरोध जताना शुरू कर दिया था ।

प्रश्न 4.
किन कारणों की वजह से अमरीका तथा अन्य पश्चिमी देश पाकिस्तान को ‘पश्चिम’ तथा दक्षिण एशिया में पश्चिमी हितों को रखवाला मानते हैं?
उत्तर:
पाकिस्तान में सेना, धर्मगुरु और भूस्वामी अभिजनों का सामाजिक दबदबा है। परिणामतः कई बार निर्वाचित सरकारों को गिराकर सैनिक शासन कायम हुआ । यद्यपि पाकिस्तान में लोकतंत्र का जज्बा मजबूती के साथ कायम रहा है तथापि लोकतंत्र पूरी तरह से सफल नहीं हो पाया। लोकतांत्रिक शासन के लिए पाकिस्तान को कोई खास अंतर्राष्ट्रीय समर्थन नहीं मिलता। इस वजह से भी सेना को अपना प्रभुत्व कायम करने के लिए बढ़ावा दिया।

अमरीका तथा अन्य पश्चिमी देशों ने अपने-अपने स्वार्थों की वजह से भी पाकिस्तान में सैनिक शासन को प्रोत्साहन दिया। इन देशों को विश्वव्यापी इस्लामी आतंकवाद से भय लगता है कि पाकिस्तान के परमाण्विक हथियार आतंकवादी समूहों के हाथ न लग जाएँ। उपरोक्त बातों के मद्देनजर पाकिस्तान को ये देश ‘पश्चिम’ तथा दक्षिण एशिया में पश्चिमी हितों का रखवाला मानते हैं।

JAC Class 12 Political Science Important Questions Chapter 5 समकालीन दक्षिण एशिया

प्रश्न 5.
दक्षेस से आप क्या समझते हैं?
उत्तर:
दक्षेस से आशय है। दक्षिण एशिया क्षेत्रीय सहयोग संगठन। यह दक्षिण एशिया के आठ देशों:

  1. भारत,
  2. पाकिस्तान,
  3. बांग्लादेश,
  4. श्रीलंका,
  5. नेपाल,
  6. भूटान,
  7. मालदीव,
  8. अफगानिस्तान- का एक क्षेत्रीय आर्थिक संगठन है जिसकी स्थापना इन देशों ने आपसी सहयोग बढ़ाने के उद्देश्य से की है।

प्रश्न 6.
लिट्टे क्या है?
उत्तर:
लिट्टे का पूरा नाम है- लिबरेशन टाइगर्स ऑफ तमिल ईलम । यह 1983 के बाद से श्रीलंका में सक्रिय एक उग्र तमिल संगठन है जो श्रीलंका के तमिलों के लिए एक अलग देश की मांग को लेकर श्रीलंकाई सेना के साथ सशस्त्र संघर्ष कर रहा है।

प्रश्न 7.
श्रीलंका की प्रमुख सफलताएँ क्या हैं?
उत्तर:
श्रीलंका की प्रमुख सफलताएँ निम्न हैं।

  1. श्रीलंका ने अच्छी आर्थिक वृद्धि और विकास के उच्च स्तर को हासिल किया है।
  2. इसने जनसंख्या की वृद्धि दर पर सफलतापूर्वक नियंत्रण स्थापित किया है।
  3. दक्षिण एशियायी देशों में सबसे पहले श्रीलंका ने ही आर्थिक उदारीकरण किया।
  4. श्रीलंका में निरन्तर लोकतांत्रिक व्यवस्था कायम रही है।

प्रश्न 8.
भारत-श्रीलंका की दो समान विशेषताएँ लिखिये।
उत्तर:

  1. भारत तथा श्रीलंका दोनों ही ब्रिटेन के अधीन रहे तथा कुछ माह के अन्तराल से इन्हें स्वतंत्रता मिली – भारत 1947 में तथा श्रीलंका 1948 में स्वतंत्र हुआ।
  2. स्वतंत्रता के बाद से दोनों ही देशों में लोकतांत्रिक शासन चला आ रहा है।

प्रश्न 9.
‘बांग्लादेश’ के निर्माण पर संक्षिप्त टिप्पणी लिखिए।
उत्तर:
जनरल याहिया खान के सैनिक शासन में पश्चिमी पाकिस्तानी सेना ने पूर्वी पाकिस्तान के बंगाली जनता के आंदोलन को कुचलने का प्रयत्न किया। हजारों लोगों को मौत के घाट उतार दिया गया। इस वजह से पूर्वी पाकिस्तान से बड़ी संख्या में लोग भारत पलायन कर गए। परिणामस्वरूप इन शरणार्थियों को संभालने की समस्या भारत के सामने खड़ी हो गई। अतः भारतीय सरकार ने पूर्वी पाकिस्तान की आजादी का समर्थन किया। इस कारण 1971 में भारत और पाकिस्तान के बीच युद्ध आरंभ हो गया। युद्ध की समाप्ति पूर्वी पाकिस्तान में पाकिस्तनी सेना के आत्मसमर्पण तथा एक स्वतंत्र राष्ट्र ‘बांग्लादेश’ के निर्माण के साथ हुई।

प्रश्न 10.
दक्षिण एशिया के देशों में मुख्य रूप से कौन-कौन सी समस्यायें हैं?
उत्तर:
दक्षिण एशिया के देशों में प्रमुख समस्यायें ये हैं।

  1. निर्धनता और बेरोजगारी की समस्या
  2. अन्तर्राष्ट्रीय आतंकवाद की समस्या
  3. जातीय संघर्ष
  4. संसाधनों के बंटवारे में होने वाले झगड़े
  5. सीमा एवं नदी जल बंटवारे से संबंधित विवाद

प्रश्न 11.
दक्षिण एशिया क्षेत्र की विशेषताओं को स्पष्ट कीजिये।
उत्तर:

  1. दक्षिण एशिया एक ऐसा क्षेत्र है, जहाँ सद्भाव और शत्रुता, आशा और निराशा तथा पारस्परिक शंका और विश्वास साथ-साथ बसते हैं।
  2. सामान्यतः बांग्लादेश, भूटान, भारत, मालदीव, नेपाल, पाकिस्तान और श्रीलंका को इंगित करने के लिए ‘दक्षिण एशिया’ पद का व्यवहार किया जाता है। इस क्षेत्र में जब-तब अफगानिस्तान और म्यांमार को भी शामिल किया जाता है।
  3. उत्तर की विशाल हिमालय पर्वत श्रृंखला, दक्षिण का हिंद महासागर, पश्चिम का अरब सागर और पूरब में मौजूद बंगाल की खाड़ी से यह क्षेत्र एक विशिष्ट प्राकृतिक क्षेत्र के रूप में नजर आता है 1
  4. दक्षिण एशिया विविधताओं से भरा-पूरा इलाका है फिर भी भू-राजनैतिक धरातल पर यह एक क्षेत्र है।

प्रश्न 12.
दक्षिण एशिया के देशों की राजनैतिक प्रणाली किस प्रकार की है?
उत्तर:
दक्षिण एशिया के विभिन्न देशों में एक सी राजनैतिक प्रणाली नहीं है। यथा।

  1. इस क्षेत्र के दो देशों, भारत और श्रीलंका में ब्रिटेन से आजाद होने के बाद, सफलतापूर्वक कायम है। लोकतांत्रिक व्यवस्था
  2. पाकिस्तान और बांग्लादेश में लोकतांत्रिक और सैनिक दोनों तरह की शासन व्यवस्थाएँ बदलती रही हैं। वर्तमान में दोनों में लोकतांत्रिक शासन व्यवस्था है।
  3. नेपाल में 2006 तक संवैधानिक राजतंत्र था। अप्रेल, 2006 से वहाँ लोकतंत्र की स्थापना हुई है।
  4. भूटान में अब भी राजतंत्र है लेकिन यहाँ के राजा ने भूटान में लोकतंत्र स्थापित करने की योजना की शुरुआत . कर दी है।
  5. मालदीव में 1968 तक राजतंत्र (सल्तनत) शासन था। 1968 में यह एक गणतंत्र बना।

JAC Class 12 Political Science Important Questions Chapter 5 समकालीन दक्षिण एशिया

प्रश्न 13.
” दक्षिण एशियायी देशों की जनता लोकतंत्र की आकांक्षाओं में सहभागी है।” इस कथन की व्याख्या कीजिये।
उत्तर:
दक्षिण एशिया में लोकतंत्र का रिकार्ड मिला-जुला रहा है। इसके बावजूद इस क्षेत्र के देशों की जनता लोकतंत्र की आकांक्षाओं में सहभागी है अर्थात् वह लोकतंत्र को अन्य शासन प्रणालियों से अच्छा समझती है। इस क्षेत्र के पांच बड़े देशों – बांग्लादेश, नेपाल, भारत, पाकिस्तान और श्रीलंका में हाल ही में एक सर्वेक्षण किया गया था जिसमें यह बात स्पष्ट हुई कि इन पांच देशों में लोकतंत्र को व्यापक जन-समर्थन हासिल है। इन देशों में हर वर्ग और धर्म के आम नागरिक लोकतंत्र को अच्छा मानते हैं और प्रतिनिधिमूलक लोकतंत्र की संस्थाओं का समर्थन करते हैं। इन देशों के लोग शासन की किसी और प्रणाली की अपेक्षा लोकतंत्र को वरीयता देते हैं और मानते हैं कि उनके देश के लिए लोकतंत्र ही ठीक है।

प्रश्न 14.
बांग्लादेश में बहुदलीय चुनावों पर आधारित प्रतिनिधि मूलक लोकतंत्र कब से कायम है?
उत्तर:
शेख मुजीब की हत्या के बाद जियाउर्रहमान ने बांग्लादेश में सैनिक शासन की पुनर्स्थापना की तथा बांग्लादेश नेशनल पार्टी बनायी। जियाउर्रहमान की हत्या के बाद लेफ्टिनेंट जनरल एच एम इरशाद ने सैन्य शासन की बागडोर संभाली। परन्तु बांलादेश की जनता लोकतंत्र की माँग करने लगी। इस आंदोलन के फलस्वरूप जनरल इरशाद को राजनीतिक गतिविधियों में छूट देनी पड़ी। इसके बाद जनरल इरशाद पाँच सालों के लिए राष्ट्रपति निर्वाचित हुए। परन्तु 1990 में जनता के विरोध के आगे जनरल इरशाद को राष्ट्रपति पद छोड़ना पड़ा। और 1991 में पुन: चुनाव हुए। इसके बाद से बांगलादेश में बहुदलीय चुनावों पर आधारित प्रतिनिधि मूलक लोकतंत्र कायम है।

प्रश्न 15.
नेपाल में लोकतंत्र की राह कैसे अवरुद्ध हुई?
उत्तर:
नेपाल अतीत में एक हिन्दू-राज्य था फिर आधुनिक काल में कई सालों तक संवैधानिक राजतंत्र रहा। संवैधानिक राजतंत्र के समय में नेपाल की राजनीतिक पार्टियाँ और आम जनता ज्यादा खुले और उत्तरदायी शासन की आवाज उठाते रहे । परन्तु राजा ने सेना की सहायता से शासन पर पूरा नियंत्रण कर लिया और इस प्रकार नेपाल में लोकतंत्र की राह अवरुद्ध हो गई।

प्रश्न 16.
बांग्लादेश का निर्माण क्यों और कैसे हुआ?
उत्तर:
1947 से 1971 तक बांग्लादेश पाकिस्तान का एक अंग था, जिसे ‘पूर्वी पाकिस्तान’ नाम से जाना जाता था। इस क्षेत्र के लोग पश्चिमी पाकिस्तानी दबदबे और अपने ऊपर उर्दू भाषा को लादने के खिलाफ थे।

  1. शेख मुजीब की गिरफ्तारी:
    जब 1970 के चुनावों में शेख मुजीब के नेतृत्व वाली अवामी लीग को संपूर्ण पाकिस्तान के लिए प्रस्तावित संविधान सभा में बहुमत हासिल हो गया तो सरकार ने सभा को आहूत करने से इन्कार कर दिया। शेख मुजीब को गिरफ्तार कर लिया गया। इसकी प्रतिक्रिया में पूर्वी पाकिस्तान क्षेत्र में शासन के विरुद्ध जन- आंदोलन फैल गया।
  2. भारत-पाक युद्ध और बांग्लादेश का निर्माण: जन-आंदोलन के दौरान पूर्वी पाकिस्तान से बड़ी संख्या में लोग भारत शरणार्थी के रूप में आये भारत सरकार ने लोगों के जन-आंदोलन का समर्थन किया परिणामस्वरूप 1971 में भारत-पाक युद्ध हुआ जिसकी समाप्ति पूर्वी पाकिस्तान के एक स्वतंत्र राष्ट्र ‘बांग्लादेश’ के निर्माण के साथ हुई।

प्रश्न 17.
1975 में बांग्लादेश में सेना ने शासन के खिलाफ बगावत क्यों की?
उत्तर;
स्वतंत्रता के बाद बांग्लादेश ने अपना संविधान बनाकर उसमें अपने को एक धर्मनिरपेक्ष, लोकतांत्रिक तथा समाजवादी देश घोषित किया। लेकिन, 1975 में शेख मुजीबुर्रहमान ने संविधान में संशोधन कराया और संसदीय प्रणाली की जगह अध्यक्षात्मक शासन प्रणाली को मान्यता दी गई। शेख मुजीब ने अपनी पार्टी अवामी लीग को छोड़कर अन्य सभी पार्टियों को समाप्त कर दिया। इससे तनाव और संघर्ष की स्थिति पैदा हुई। अगस्त, 1975 में सेना ने शेख मुजीब के शासन के खिलाफ बगावत कर दी।

प्रश्न 18.
श्रीलंका की जातीय समस्या पर एक टिप्पणी लिखिये।
उत्तर:
श्रीलंका की जातीय समस्या: श्रीलंका की राजनीति पर बहुसंख्यक सिंहली समुदाय के हितों की नुमाइंदगी करने वालों का दबदबा रहा है। ये लोग भारत से आकर श्रीलंका में बसे तमिलों के खिलाफ हैं। सिंहली राष्ट्रवादियों का मानना है कि श्रीलंका सिर्फ सिंहली लोगों का है। श्रीलंका में तमिलों के साथ कोई रियायत नहीं बरती जानी चाहिए। सिंहलियों के तमिलों के प्रति इस उपेक्षा भरे व्यवहार से श्रीलंका में एक उग्र तमिल राष्ट्रवाद की आवाज बुलंद हुई। 1983 के बाद से उग्र तमिल संगठन ‘लिबरेशन टाइगर्स ऑव तमिल ईलम’ (लिट्टे) श्रीलंका सेना के साथ सशस्त्र संघर्ष कर रहा है। इसने ‘तमिल ईलम’ यानी श्रीलंका के तमिलों के लिए एक अलग देश की माँग की है। इस प्रकार श्रीलंका को जातीय संघर्ष का सामना करना पड़ रहा है जिसकी मांग है कि श्रीलंका के एक क्षेत्र को अलग राष्ट्र बनाया जाये।

प्रश्न 19.
भारत सरकार श्रीलंका के तमिल मुद्दे पर प्रत्यक्ष रूप से कब और किस रूप में शामिल हुई?
उत्तर:
भारत की तमिल जनता का भारतीय सरकार पर भारी दबाव रहा कि वह श्रीलंकाई तमिलों के हितों की रक्षा करे। भारत सरकार समय-समय पर इस संदर्भ में श्रीलंका सरकार से बातचीत करने की कोशिश करती रही। लेकिन 1987 में भारतीय सरकार श्रीलंका के तमिल मुद्दे पर प्रत्यक्ष रूप से शामिल हुई। सन् 1987 में भारत की सरकार ने श्रीलंका सरकार से तमिल मुद्दे को सुलझाने के लिए एक समझौता किया तथा श्रीलंका सरकार और तमिलों के बीच रिश्ते सामान्य करने के लिए भारत ने शांति सेना को भेजा। लेकिन शांति सेना अन्ततः लिट्टे के साथ संघर्ष में फंस गई। भारतीय सेना की उपस्थिति को श्रीलंका की जनता ने श्रीलंका के अंदरूनी . मामलों में भारत की दखलंदाजी समझा । परिणामस्वरूप ‘शांति सेना’ अपना लक्ष्य हासिल किए बिना वापस बुला ली गई।

प्रश्न 20.
श्रीलंका की आर्थिक व्यवस्था पर एक टिप्पणी लिखिए।
उत्तर:
श्रीलंका ने अच्छी आर्थिक वृद्धि और विकास के उच्च स्तर को हासिल किया है। उसने जनसंख्या की वृद्धि दर पर सफलतापूर्वक नियंत्रण किया है। दक्षिण एशिया के देशों में सबसे पहले श्रीलंका ने ही अपनी अर्थव्यवस्था का उदारीकरण किया । जातीय संघर्ष से गुजरने के बावजूद कई सालों से इस देश का प्रति व्यक्ति सकल घरेलू उत्पाद दक्षिण एशिया में सबसे ज्यादा है।

प्रश्न 21.
सार्क की स्थापना के क्या उद्देश्य थे?
उत्तर:
सार्क की स्थापना के मुख्य उद्देश्य निम्नलिखित थे।

  1. दक्षिण एशिया के लोगों के कल्याण करने की कामना तथा उनके जीवन स्तर को सुधारना।
  2. दक्षिण एशिया क्षेत्र में आर्थिक विकास, सामाजिक प्रगति तथा सांस्कृतिक उन्नति को प्राप्त करना तथा इस क्षेत्र के सभी व्यक्तियों के लिए प्रतिष्ठा के अवसर प्रदान करना।
  3. दक्षिण एशिया के देशों में सामूहिक आत्मविश्वास और आत्मनिर्भरता पैदा करना तथा उसको बढ़ावा देना।
  4. एक-दूसरे की समस्याओं को पारस्परिक विश्वास, सूझ-बूझ तथा अभिमूल्यन की दृष्टि से देखना।

JAC Class 12 Political Science Important Questions Chapter 5 समकालीन दक्षिण एशिया

प्रश्न 22.
क्षेत्रीय सहयोग के साधन के रूप में सार्क की प्रमुख उपलब्धियों का वर्णन कीजिये।
उत्तर:
दक्षिण एशिया क्षेत्र में सहयोग के साधन के रूप में सार्क की प्रमुख उपलब्धियाँ निम्नलिखित हैं।

  1. इस क्षेत्र में सातों देश एक-दूसरे के काफी नजदीक आए हैं तथा इससे उनमें दिखाई देने वाला तनाव कम हुआ है।
  2. दक्षेस (सार्क) के कारण इस क्षेत्र के देशों के छोटे-मोटे मतभेद अपने-आप आसानी से सुलझ रहे हैं और इन देशों में अपनापन विकसित हुआ है।
  3. सार्क के माध्यम से इस क्षेत्र में विदेशी शक्तियों का इस क्षेत्र में प्रभाव कम हुआ है और ये देश अपने को अधिक स्वतंत्र महसूस करने लगे हैं।
  4. सार्क ने एक संरक्षित अन्न भंडार की स्थापना की है जो इस क्षेत्र के देशों की आत्मनिर्भरता की भावना के प्रबल होने का सूचक है।

प्रश्न 23.
भारत और बांग्लादेश के बीच मतभेद के कारणों पर एक टिप्पणी लिखिए।
उत्तर:
भारत और बांग्लादेश के बीच मतभेद के मुद्दे: भारत और बांग्लादेश के बीच मतभेद के प्रमुख मुद्दे इस प्रकार हैं।
(अ) भारत के बांग्लादेश से अप्रसन्नता के कारण – भारतीय सरकारों के बांग्लादेश से नाखुश होने के कारण हैं।

  1. भारत में अवैध आप्रवास पर ढाका द्वारा खंडन करना।
  2. बांग्लादेश सरकार द्वारा भारत विरोधी इस्लामी कट्टरपंथी जमातों को समर्थन देना।
  3. भारतीय सेना को पूर्वोत्तर भारत में जाने के लिए अपने इलाके से रास्ता देने से बांग्लादेश का इन्कार करना।
  4. म्यांमार को बांग्लादेशी इलाके से भारत को प्राकृतिक गैस निर्यात न करने देना।

(ब) बांग्लादेश भारत पर निम्न कारणों से अप्रसन्न है।

  1. भारतीय सरकार गंगा और ब्रह्मपुत्र नदी – जल में हिस्सेदारी के प्रश्न पर इलाके के दादा की तरह बरताव करती है।
  2. भारत की सरकार चटगाँव पर्वतीय क्षेत्र में विद्रोह को हवा दे रही है।
  3. भारत उसकी प्राकृतिक गैस में सेंधमारी कर रहा है तथा व्यापार में बेईमानी बरत रहा है।

प्रश्न 24.
भारत और बांग्लादेश के सहयोग के मुद्दों को स्पष्ट कीजिये।
उत्तर:
भारत और बांग्लादेश के बीच सहयोग के मुद्दे भारत और बांग्लादेश निम्नलिखित मुद्दों पर आपस में सहयोग करते हैं।

  1.  पिछले बीस वर्षों के दौरान दोनों देशों के बीच आर्थिक सम्बन्धं ज्यादा बेहतर हुए हैं।
  2. बांग्लादेश भारत के ‘लुक ईस्ट’ और 2014 से ‘एक्ट ईस्ट’ की नीति का हिस्सा है। इस नीति के अन्तर्गत स्यांमार के जरिए दक्षिण-पूर्व एशिया से सम्पर्क साधने की बात है ।
  3. आपदा प्रबंधन और पर्यावरण के मसले पर भी दोनों देशों ने निरन्तर सहयोग किया है।
  4. इस बात के भी प्रयास किये जा रहे हैं कि साझे खतरों को पहचान कर तथा एक-दूसरे की जरूरतों के प्रति ज्यादा संवेदनशीलता बरत कर सहयोग के दायरे को बढ़ाया जाये।

प्रश्न 25.
भारत-नेपाल के सम्बन्धों के बीच कड़वाहट के मुद्दों पर एक टिप्पणी लिखिये । भारत-नेपाल के सम्बन्धों के बीच तनाव के मुद्दे
उत्तर:
भारत-नेपाल के मधुर सम्बन्धों के बीच निम्नलिखित मुद्दे मनमुटाव पैदा करते रहे हैं।

  1. नेपाल की चीन के साथ मित्रता को लेकर भारत सरकार ने अक्सर अपनी अप्रसन्नता जतायी है।
  2. नेपाल सरकार भारत विरोधी तत्त्वों के विरुद्ध आवश्यक कदम नहीं उठाती है। इससे भी भारत नाखुश है।
  3. भारत की सुरक्षा एजेंसियां नेपाल में चल रहे माओवादी आंदोलन को अपनी सुरक्षा के लिए खतरा मानती हैं।
  4. नेपाल के लोगों की यह सोच है कि भारत की सरकार नेपाल के अंदरूनी मामलों में दखल दे रही है और उसके नदी जल तथा पन बिजली पर आँख गड़ाए हुए है।
  5. नेपाल को यह भी लगता है कि भारत उसको अपने भू-क्षेत्र से होकर समुद्र तक पहुँचने से रोकता है।

प्रश्न 26.
” भारत-नेपाल के बीच मधुर संबंध हैं ।” स्पष्ट कीजिए।
उत्तर:
भारत और नेपाल के बीच मधुर संबंध हैं, इसे हम निम्न उदाहरणों से स्पष्ट कर सकते हैं।
1. भारत और नेपाल दोनों देशों के बीच पारगमन संधि है। इस संधि के तहत दोनों देशों के नागरिक एक-दूसरे के देश में बिना पासपोर्ट (पारपत्र) और वीजा के आ-जा सकते हैं और काम कर सकते हैं।
2. दोनों देश व्यापार, वैज्ञानिक सहयोग, साझे प्राकृतिक संसाधन, बिजली उत्पादन और जल प्रबंधन ग्रिड के मसले पर एक साथ हैं।
3. नेपाल में लोकतंत्र की बहाली से दोनों देशों के बीच संबंधों के और मजबूत होने की संभावना है। प्रश्न 27. भारत-श्रीलंका के सम्बन्धों पर एक टिप्पणी लिखिये।
उत्तर:
भारत और श्रीलंका के सम्बन्ध

  1. यद्यपि श्रीलंका और भारत की सरकारों के संबंधों में तनाव इस द्वीप में जारी जातीय संघर्ष को लेकर है तथापि 1987 के सैन्य हस्तक्षेप के बाद से भारतीय सरकार श्रीलंका के अंदरूनी मामलों में असंलग्नता की नीति अपनाये हुए है।
  2. दोनों देशों की सरकारों ने परस्पर एक मुक्त व्यापार समझौते पर हस्ताक्षर कर अपने सम्बन्धों को और मजबूत किया है।
  3. श्रीलंका में ‘सुनामी’ से हुई तबाही के बाद के पुनर्निर्माण कार्यों में भारतीय मदद से भी दोनों देश एक-दूसरे के नजदीक आये हैं।

JAC Class 12 Political Science Important Questions Chapter 5 समकालीन दक्षिण एशिया

प्रश्न 28.
भूटान और मालदीव के साथ भारत के सम्बन्धों पर एक टिप्पणी लिखिये।
उत्तर:

  • भारत-भूटान सम्बन्ध – भारत के भूटान के साथ बहुत अच्छे सम्बन्ध हैं तथा भूटानी सरकार के साथ भारत का कोई बड़ा झगड़ा नहीं है।
    1. भूटान से अपने काम का संचालन कर रहे पूर्वोत्तर भारत के उग्रवादियों और गुरिल्लों को भूटान ने अपने क्षेत्र से खदेड़ भगाया। भूटान के इस कदम से भारत को बड़ी सहायता मिली है।
    2. भारत भूटान में पन बिजली की बड़ी परियोजनाओं में हाथ बँटा रहा है
    3. भूटान के विकास कार्यों के लिए सबसे ज्यादा अनुदान भारत से हासिल होता है।
  • भारत और मालदीव सम्बन्ध:
    1. मालदीव के साथ भारत के संबंध सौहार्द्रपूर्ण तथा गर्मजोशी से भरे हैं।
    2. भारत ने मालदीव के आर्थिक विकास, पर्यटन और मत्स्य उद्योग में भी मदद की है।

प्रश्न 29.
‘साफ्टा’ क्या है?
उत्तर:
साफ्टा (SAFTA ) साफ्टा’ का पूरा नाम है। दक्षिण एशिया मुक्त व्यापार क्षेत्र (South Asia Free Trade Area)! दक्षेस के सदस्य देशों ने सन् 2002 में ‘दक्षिण एशियाई मुक्त व्यापार क्षेत्र समझौते’ पर दस्तखत करने का वायदा किया। इसमें पूरे दक्षिण एशिया के लिए मुक्त व्यापार क्षेत्र बनाने का वायदा है। इस समझौते पर 2004 में हस्ताक्षर हुए और यह समझौता 1 जनवरी, 2006 से प्रभावी हो गया है। इस समझौते का लक्ष्य है कि इन देशों के बीच आपसी व्यापार में लगने वाले सीमा शुल्क को 2007 तक 20 प्रतिशत कम कर दिया जाये।

प्रश्न 30.
भारत-पाकिस्तान के बीच सियाचिन की समस्या को संक्षेप में बताइये।
उत्तर:
भारतीय नक्शे में शीर्ष पर स्थित सियाचिन हिमनाद को लेकर भारत पाकिस्तान के बीच विवाद बना हुआ है। सियाचिन सैनिक तथा सामरिक दृष्टि महत्त्वपूर्ण है क्योंकि यहाँ से पाक अधिकृत कश्मीर, चीन के सिक्यांग प्रान्त, रूसी राष्ट्रकुल के देशों तथा अफगानिस्तान पर पैनी दृष्टि रखी जा सकती है। इसलिए इसे भारत और पाकिस्तान दोनों अपने-अपने अधिकार में लेना चाहते हैं। इसे लेकर दोनों देशों में अनेक मुठभेड़ें हो चुकी हैं। वर्तमान में दोनों ही देश इस समस्या के स्थायी हल के लिए वार्ताएँ कर रहे हैं।

प्रश्न 31.
“श्रीलंका की समस्या भारतवंशी लोगों से जुड़ी है।” इस कथन को स्पष्ट कीजिए।
उत्तर:
भारत की तमिल जनता का भारतीय सरकार पर भारतीय सरकार पर भारी दबाव है कि वह श्रीलंकाई तमिलों के हितों की रक्षा करे भारतीय सरकार ने समय-समय पर तमिलों के सवाल पर श्रीलंका की सरकार से बातचीत की कोशिश की है। 1987 में भारतीय सरकार श्रीलंका के तमिल मसले में प्रत्यक्ष रूप से शामिल हुई। भारतीय सरकार ने श्रीलंका से समझौता किया तथा श्रीलंका और तमिलों के बीच रिश्ता सामान्य करने के लिए भारतीय सेना भेजा अतः हम कह सकते हैं कि श्रीलंका की समस्या भारतवंशी लोगों से जुड़ी है।

प्रश्न 32.
1989 में भारत को अपनी ‘शांति सेना’ वापस बुलानी पड़ी।
उत्तर:
भारतीय सरकार ने श्रीलंका से समझौता किया और इसके फलस्वरूप भारतीय सरकार ने श्रीलंका सरकार और तमिलों के बीच रिश्ते सामान्य करने के लिए भारतीय सेना भेजी सेना लिट्टे के साथ संघर्ष में फँस गई। भारतीय सेना की उपस्थिति का श्रीलंका की जनता ने विरोध किया। वहाँ की जनता ने यह समझा कि भारतीय सेना उनके अंदरूनी मामलों में हस्तक्षेप कर रही है। फलस्वरूप 1989 में भारत ने अपनी ‘शांति सेना’ लक्ष्य हासिल किए बिना वापस बुला ली।

JAC Class 12 Political Science Important Questions Chapter 5 समकालीन दक्षिण एशिया

प्रश्न 33.
कश्मीर के दो हिस्सों में विभाजित होने के कारणों को स्पष्ट कीजिए।
उत्तर:
कश्मीर के दो हिस्सों में विभाजित होने के कारण निम्नलिखित हैं।

  1. दक्षिण एशिया में स्थित देशों के आपसी संघर्षों में सबसे प्रमुख और सर्वग्रासी संघर्षों में सबसे प्रमुख और सर्वग्रासी संघर्ष भारत और पाकिस्तान के बीच का संघर्ष है।
  2. भारत और पाकिस्तान के विभाजन के तुरंत बाद ही दोनों देश कश्मीर के मसले पर लड़ पड़े।
  3. पाकिस्तान सरकार कश्मीर पर अपना दावा कर रही थी।
  4. पाकिस्तान सरकार कश्मीर पर अपना दावा कर रही थी।
  5. 1948 के युद्ध के फलस्वरूप कश्मीर के दो हिस्से हो गए।

प्रश्न 34.
“दक्षिण एशिया के सारे झगड़े सिर्फ भारत और उसके पड़ोसी देशों के बीच ही नहीं है।” इस कथन को स्पष्ट कीजिए।
उत्तर:
दक्षिण एशिया के सारे झगड़े सिर्फ भारत और उसके पड़ोसी देशों के बीच ही नहीं है क्योंकि।

  1. नेपाल – भूटान तथा बांग्लादेश – म्यांमार के बीच जातीय मूल के नेपालियों के भूटान आप्रवास तथा रोहिंग्या लोगों म्यांमार में आप्रवास के मामलों पर मतभेद रहे हैं।
  2. बांग्लादेश और नेपाल के बीच हिमालयी नदियों के जल की हिस्सेदारी को लेकर खटपट है।

प्रश्न 35.
‘साफ्टा’ के उद्देश्य पर भारत और अन्य देश एकमत नहीं है। इस कथन की संक्षेप में व्याख्या कीजिए।
उत्तर:
दक्षिण एशिया के सभी देशों के बीच 2004 में ‘साफ्टा’ पर हस्ताक्षर हुआ और यह समझौता 1 जनवरी, 2006 से प्रभावी हो गया। यद्यपि इस समझौते का मकसद इन क्षेत्रों के सीमा रेखा मुक्त व्यापार पर सहमत होना था। परंतु कुछ देशों का मानना था कि ‘साफ्टा’ की ओट लेकर भारत उनके बाजार में सेंध मारना चाहता है ओर व्यावसायिक उद्यम तथा व्यावसायिक मौजूदगी जरिये उनके समाज ओर राजनीति पर असर डालना चाहता है।

जबकि भारत सोचता है कि ‘साफ्टा’ के जरिये इस क्षेत्र के हर देश का फायदा होगा और क्षेत्र में मुक्तव्यापार बढ़ने से राजनीतिक मसलों पर सहयोग बेहतर होगा। भारत में कुछ लोगों का मानना है कि भारत को साफ्टा पर ज्यादा ध्यान नहीं देना चाहिए क्योंकि भारत भूटान, नेपाल और श्रीलंका से पहले ही द्विपक्षीय समझौता कर चुका है।

प्रश्न 36.
दक्षिण एशिया के क्षेत्रों की सुरक्षा और शांति के भविष्य से अमरीका के हित बंधे हुए हैं। इस कथन की पुष्टि कीजिए।
उत्तर:
शीतयुद्ध के बाद से दक्षिण एशिया में अमरीकी प्रभाव तेजी से बढ़ा है। अमरीका ने शीतयुद्ध के बाद भारत और पाकिस्तान दोनों देशों से अपने संबंध सुधारे हैं। आवश्यकता पड़ने पर अमरीका ने भारत-पाक के बीच मध्यस्थता की भूमिका भी निभाई है। दोनों देशों में आर्थिक सुधार हुए हैं और उदार नीतियाँ अपनाई गई हैं। इस वजह से दक्षिण एशिया में अमरीकी भागीदारी ज्यादा गहरी हुई है। अमरीका में दक्षिण एशियाई मूल के लोगों की जनसंख्या अच्छी-खासी है। दक्षिण एशिया की जनसंख्या और बाजार भी भारी भरकम है। इस कारण इस क्षेत्र की सुरक्षा और शांति के भविष्य से अमरीका के हित भी बंधे हुए हैं।

प्रश्न 37.
नेपाल में लोकतंत्र की बहाली किस प्रकार हुई ? संक्षेप में लिखिए।
उत्तर:
नेपाल में अप्रेल, 2006 में लोकतंत्र के लिए देशव्यापी आंदोलन हुए। लोकतंत्र के समर्थन में जगह-जगह प्रदर्शन किए गए। संघर्षरत लोकतंत्र – समर्थकों को जीत हासिल हुई और नेपाल के राजा को बाध्य होकर संसद को चलने की अनुमति दे दी इस प्रकार नेपाल में लोकतंत्र की बहाली हुई।

निबन्धात्मक प्रश्न

प्रश्न 1.
दक्षिण एशिया से आप क्या समझते हैं? शीत युद्ध के बाद के काल में दक्षिण एशिया के देशों में राजनीतिक प्रणाली की मुख्य प्रवृत्ति क्या रही है?
उत्तर:
दक्षिण एशिया से आशय सामान्य रूप से बांग्लादेश, भूटान, भारत, मालदीव, नेपाल, पाकिस्तान और श्रीलंका इन सात देशों को इंगित करने के लिए दक्षिण एशिया पद का इस्तेमाल किया जाता है। इस क्षेत्र की चर्चा में जब-तब अफगानिस्तान और म्यांमार को भी शामिल किया जाता है। दक्षिण एशिया के देशों में राजनीतिक प्रणाली शीत युद्ध के बाद के काल में इस क्षेत्र के देशों में तथा यहाँ के लोगों में मुख्य प्रवृत्ति लोकतांत्रिक राजनीतिक प्रणाली को अपनाने की रही है। यथा

  1. भारत, श्रीलंका और बांग्लादेश: भारत और श्रीलंका में ब्रिटेन से स्वतंत्र होने के बाद से आज तक लोकतांत्रिक व्यवस्था सफलतापूर्वक कायम है तथा शीत युद्ध के बाद से बांग्लादेश में भी लोकतांत्रिक सरकारें कायम हैं।
  2. पाकिस्तान: पाकिस्तान में शीत युद्ध के बाद के सालों में लगातार दो लोकतांत्रिक सरकारें बनीं। 1999 से 2007 तक सैनिक तख्ता पलट के कारण पाकिस्तान में गैर लोकतांत्रिक सरकार रही लेकिन 2008 से आज तक वहां लोकतांत्रिक सरकार कार्यरत है।
  3. नेपाल: अप्रेल, 2006 तक नेपाल में संवैधानिक राजतंत्र था, लेकिन अप्रेल 2006 से आज तक नेपाल में भी लोकतांत्रिक राजनीतिक व्यवस्था कायम है।
  4. भूटान: भूटान में 2007 तक राजतंत्र शासन रहा। लेकिन मार्च, 2008 से यहाँ भी लोकतांत्रिक व्यवस्था कायम है।
  5.  मालदीव: मालदीव में 1968 तक राजतंत्र शासन व्यवस्था रही। 1968 से यहाँ भी लोकतांत्रिक व्यवस्था कायम है। स्पष्ट है कि दक्षिण एशिया के देशों में लोकतंत्र को अपनाने की मुख्य प्रवृत्ति रही है।

JAC Class 12 Political Science Important Questions Chapter 5 समकालीन दक्षिण एशिया

प्रश्न 2.
पाकिस्तान में लोकतंत्र और उसके सैनिक तंत्र में बदलाव तथा पुनः लोकतंत्र की स्थापना की प्रक्रिया पर एक निबंध लिखिये।
उत्तर:
14 अगस्त, 1947 को भारत का विभाजन करके पाकिस्तान की स्थापना हुई। पाकिस्तान की राजनीतिक व्यवस्था मुस्लिम लीग के अध्यक्ष मुहम्मद अली जिन्ना पाकिस्तान के प्रथम गवर्नर जनरल बने और लियाकत अली खां ने प्रधानमंत्री का पद संभाला। लेकिन 1948 में जिन्ना की मृत्यु हो गई और पाकिस्तान के शासन की बागडोर लियाकत अली खां के हाथों में पूरी तरह से आ गई। अक्टूबर, 1951 में लियाकत अली खां की हत्या कर दी गई। इस घटना के बाद ख्वाजा निजामुद्दीन को प्रधानमंत्री तथा गुलाम मोहम्मद को गवर्नर जनरल के पद पर नियुक्त किया गया। 7 अप्रैल, 1953 को गवर्नर जनरल गुलाम मोहम्मद ने निजामुद्दीन मंत्रिमंडल को भंग कर दिया।

अमरीकी प्रभाव में अब मुहम्मद अली को प्रधानमंत्री बनाया गया। सैनिक तानाशाही: 7 अक्टूबर, 1958 को प्रधान सेनापति जनरल अयूब खां के नेतृत्व में सेना ने सरकार के विरुद्ध विद्रोह कर दिया तथा सत्ता प्राप्त कर ली। तब से लेकर 1971 तक तथा पुनः 5 जुलाई, 1977 से 1988 तक पाकिस्तान में सैनिक शासन रहा।

लोकतंत्र: 1971 में बांग्लादेश के निर्माण के बाद पाकिस्तान में जुल्फिकार अली भुट्टो के नेतृत्व में लोकतांत्रिक सरकार बनी जो 1977 तक कायम रही। इसके बाद 1989 से 1999 तक पाकिस्तान में पुनः लोकतान्त्रिक सरकारों ने काम किया।

  1. पुनः सैनिक शासन ( 1999 से 2007 तक ): सन् 1999 में सेना प्रमुख जनरल परवेज मुशर्रफ ने प्रधानमंत्री नवाज शरीफ को पदच्युत करके अपने आपको वहां का शासक बना लिया जो 2007 तक सत्ता पर काबिज रहे।
  2. पुनः लोकतंत्र की वापसी (2008 से वर्तमान तक ): जन आंदोलनों और जनमत के दबाव में 2008 में पाकिस्तान में चुनाव हुए और तब से वर्तमान तक वहाँ लोकतांत्रिक सरकारें चली आ रही हैं।

प्रश्न 3.
बांग्लादेश का निर्माण क्यों और किस प्रकार हुआ?
उत्तर-
बांग्लादेश का निर्माण क्यों और कैसे?
1947 से 1971 तक बांग्लादेश पाकिस्तान का अंग था, जिसे पूर्वी पाकिस्तान के नाम से जाना जाता था।

  1. बंगाली संस्कृति की उपेक्षा: पूर्वी पाकिस्तान के लोग पश्चिमी पाकिस्तान के दबदबे और अपने ऊपर उर्दू भाषा को लादने के खिलाफ थे। पाकिस्तान के निर्माण के बाद से ही यहां के लोगों ने बंगाली संस्कृति और भाषा के साथ किये जा रहे दुर्व्यवहार के खिलाफ विरोध जताना शुरू कर दिया था।
  2. प्रशासन तथा राजनीतिक सत्ता में भागीदारी की माँग- इस क्षेत्र की जनता ने प्रशासन में अपने न्यायोचित प्रतिनिधित्व तथा राजनीतिक सत्ता में समुचित हिस्सेदारी तथा पूर्वी क्षेत्र के लिए स्वायत्तता की मांग उठायी।
  3. 1970 के आम चुनाव और शेख मुजीब की अवामी लीग को बहुमत मिलना:1970 के आम चुनाव में शेख मुजीबुर्रहमान के नेतृत्व वाली अवामी लीग को सम्पूर्ण पाकिस्तान के लिए प्रस्तावित संविधान सभा में बहुमत हासिल हो गया। लेकिन सरकार पर पश्चिमी पाकिस्तान के नेताओं का दबदबा था और सरकार ने इस सभा को आहूत करने से इन्कार कर दिया। शेख मुजीब को गिरफ्तार कर लिया गया।
  4. भारत-पाक युद्ध 1971: पाकिस्तानी सेना ने पूर्वी पाकिस्तान की जनता के आंदोलन को कुचलने हेतु हजारों लोगों को मौत के मुँह सुला दिया। वहाँ के लोगों ने भारत में पलायन किया। इससे भारत में शरणार्थी की समस्या खड़ी हो गई। फलतः भारत-पाक युद्ध 1971 हुआ ।
  5. बांग्लादेश का निर्माण: युद्ध समाप्त होने से पहले ही पूर्वी पाकिस्तान में पाकिस्तानी सेना ने आत्मसमर्पण कर दिया और अन्त में अप्रेल, 1971 में बांग्लादेश का निर्माण हुआ।

JAC Class 12 Political Science Important Questions Chapter 5 समकालीन दक्षिण एशिया

प्रश्न 4.
बांग्लादेश में लोकतंत्र के समक्ष चुनौतियों की व्याख्या कीजिए। (कोई चार )
अथवा
बांग्लादेश में लोकतंत्र की स्थापना की चर्चा कीजिए।
अथवा
बांग्लादेश में लोकतंत्र की स्थापना की प्रक्रिया को संक्षेप में बताइये।
उत्तर:
बांग्लादेश में लोकतंत्रीय शासन स्थापना की प्रक्रिया: बांग्लादेश में लोकतंत्र की स्थापना निम्न प्रकार हुई।

1. संसदीय लोकतंत्र की स्थापना:
12 अप्रेल, 1971 को स्वतंत्र बांग्लादेश सरकार का गठन हुआ। बांग्लादेश ने अपना एक नवीन संविधान बनाया जिसमें इसे संसदात्मक, धर्मनिरपेक्ष, लोकतांत्रिक तथा समाजवादी देश घोषित किया गया।

2. संसदीय लोकतंत्र के स्थान पर अध्यक्षीय लोकतंत्र:
19075 में शेख मुजीब ने संसदीय लोकतंत्र के स्थान पर अध्यक्षात्मक प्रणाली को मान्यता दी। इसके विरोध में सेना ने बगावत कर दी तथा शेख मुजीब की हत्या कर दी।

3. सैनिक शासन:
शेख मुजीब की हत्या के बाद 1975 में सैनिक शासके जियाउर्रहमान ने बांग्लादेश नेशनल पार्टी बनाई तथा 1979 के चुनाव में विजयी रही। लेकिन जियाउर्रहमान की हत्या हुई। अब लेफ्टिनेंट जनरल एम. एम. इरशाद के नेतृत्व में एक और सैनिक शासन ने बागडोर संभाली तथा 1990 तक सत्ता में बने रहे।

4. पुनः लोकतंत्र की स्थापना: जनता के व्यापक विरोध के समक्ष झुकते हुए ले. जनरल इरशाद को राष्ट्रपति का पद 1990 में छोड़ना पड़ा और 1991 में चुनाव हुए। इसके बाद से बांग्लादेश में बहुदलीय लोकतांत्रिक व्यवस्था कायम है।

प्रश्न 5.
नेपाल में लोकतंत्रीकरण की प्रक्रिया की एक समीक्षा प्रस्तुत कीजिये।
उत्तर:
नेपाल में संवैधानिक राजतंत्र: 1947 से 1960 तक नेपाल में संवैधानिक राजतंत्र रहा। संवैधानिक राजतंत्र के दौर में नेपाल की राजनीतिक पार्टियाँ और आम जनता खुले और उत्तरदायी शासन की माँग उठाते रहे। विवश होकर राजा ने 1990 में नए लोकतान्त्रिक संविधान की माँग मान ली। इस प्रकार नेपाल में 1990 से लोकतांत्रिक सरकार का गठन हुआ। लेकिन 1990 के दशक में ही 5-6 जिलों में माओवादियों की समानान्तर शासन व्यवस्था को नियंत्रित करने की प्रक्रिया में शाही सेना व माओवादियों के बीच हिंसक लड़ाई छिड़ गई।

कुछ समय तक राजा की सेना, लोकतंत्र – समर्थकों और माओवादियों के बीच त्रिकोणीय संघर्ष हुआ। अन्त में सन् 2002 में राजा ने संसद को भंग कर दिया और सरकार को गिरा दिया। लोकतंत्र की बहाली ( 2006 ) – अप्रेल, 2006 में नेपाल में देशव्यापी लोकतंत्र समर्थक प्रदर्शन हुए तथा 24 अप्रेल, 2006 को राजा ज्ञानेन्द्र ने बाध्य होकर संसद को बहाल किया।

प्रश्न 6.
भारत के नेपाल व श्रीलंका के साथ सम्बन्धों की विवेचना कीजिये।
अथवा
भारत-श्रीलंका सम्बन्धों का विस्तार से वर्णन कीजिए।
उत्तर:
भारत-नेपाल सम्बन्ध: भारत के नेपाल के साथ सम्बन्धों को निम्न प्रकार स्पष्ट किया गया है।

  1. दोनों देशों के बीच बहुत नजदीकी सांस्कृतिक सम्बन्ध रहे हैं।
  2. 1950 से नेपाल और भारत के बीच पारगमन और व्यापार संधि रही है।
  3. नेपाल के राजनीतिक विकास और लोकतंत्रीकरण में भारत की महत्त्वपूर्ण भूमिका रही है।
  4. भारत ने नेपाल में शिक्षण संस्थाओं के विकास, हवाई सेवा के विकास, भारतीय सशस्त्र बलों से सेवानिवृत्त नेपाली लोगों के कल्याण आदि में नेपाल को सहयोग दिया है।
  5. दोनों देश व्यापार, वैज्ञानिक सहयोग, साझे प्राकृतिक संसाधन, बिजली उत्पादन और जल प्रबंधन ग्रिड के मुद्दों पर एक साथ हैं।
  6. दोनों देशों के बीच पारगमन संधि पर नेपाल के संदेह, नेपाल के चीन से बढ़ते रिश्ते, कालापानी के मुद्दे, जल- विवाद आदि के सम्बन्ध में मतभेद उभरते रहे हैं। उक्त विवादों के होते हुए दोनों देशों के सम्बन्ध मैत्रीपूर्ण बने हुए हैं।

भारत-श्रीलंका सम्बन्ध: भारत और श्रीलंका दोनों पड़ौसी देश हैं। दोनों के बीच घनिष्ठ मैत्री सम्बन्ध हैं। यथा।

  1. दक्षिण एशिया में भारत और श्रीलंका दोनों ही स्वतंत्रता के बाद से लोकतांत्रिक देश रहे हैं।
  2. भारत और श्रीलंका के बीच केवल समुद्र है और दोनों देशों के बीच समुद्री सीमा विषयक समझौता है।
  3. भारत ने समय-समय पर आर्थिक सहयोग दिया है। दोनों गुटनिरपेक्ष देश हैं। दोनों के बीच मुक्त व्यापार समझौता है। यथा।
  4. दोनों देशों के बीच श्रीलंका तमिल समस्या विवाद का विषय रही। उक्त विवेचन से स्पष्ट है कि भारत-श्रीलंका के बीच परस्पर अच्छे पड़ौसी के सम्बन्ध कायम हैं।

प्रश्न 7.
भारत और नेपाल के आपसी सम्बन्धों में विवाद और सहयोग के मुख्य मुद्दों की विवेचना कीजिये भारत और नेपाल सम्बन्ध
उत्तर:
भारत व नेपाल के बीच सम्बन्धों में विवाद और सहयोग दोनों का सुगुंफन है। यथा।
I. भारत और नेपाल के सम्बन्धों में सहयोग के मुद्दे – भारत-नेपाल के बीच सहयोग के प्रमुख मुद्दे निम्नलिखित

  1. दोनों देशों के बीच बहुत नजदीकी सांस्कृतिक सम्बन्ध रहे हैं।
  2. 1950 से नेपाल और भारत के बीच पारगमन और व्यापार संधि रही है।
  3. नेपाल के राजनीतिक विकास और लोकतंत्रीकरण में भारत की महत्त्वपूर्ण भूमिका रही है।
  4. भारत ने नेपाल में शिक्षा के क्षेत्र में भी अनेक संस्थाओं और संगठनों के विकास में योगदान दिया है।
  5. दोनों देशों के बीच हवाई सेवा सम्बन्धी समझौता है।

II. भारत-नेपाल के बीच विवाद के मुद्दे – भारत – नेपाल के बीच अनेक मुद्दों पर विवाद उभरता रहा है।

  1. नेपाल सरकार भारत-विरोधी तत्त्वों के खिलाफ कदम नहीं उठाती।
  2. भारत की सुरक्षा एजेंसियाँ नेपाल में चल रहे माओवादी आंदोलन को अपनी सुरक्षा के लिए खतरा मानती है क्योंकि वे भारत विरोधी गतिविधियों में शामिल हैं।
  3. नेपाल के चीन से बढ़ते रिश्तों के कारण भारत की चिंता बढ़ी है।
  4. नेपाल के बहुत से लोग सोचते हैं कि भारत सरकार नेपाल के अंदरूनी मामलों में दखल दे रही है और उसके नदी जल तथा पन बिजली पर आँख गड़ाए हुए है।
  5. नेपाल को यह संशय है कि भारत उसको अपने भूक्षेत्र से होकर समुद्र तक पहुँचने से रोकता है।

JAC Class 12 Political Science Important Questions Chapter 5 समकालीन दक्षिण एशिया

प्रश्न 8.
भारत-पाक के बीच प्रमुख विवादग्रस्त मुद्दे क्या हैं? प्रकाश डालिये।
अथवा
भारत-पाकिस्तान के बीच तनाव के प्रमुख मुद्दों का विवेचन कीजिये । भारत – पाकिस्तान के बीच तनाव के प्रमुख मुद्दे
उत्तर;
भारत-पाकिस्तान के बीच तनाव के प्रमुख मुद्दे निम्नलिखित हैं।’
1. कश्मीर का मुद्दा: विभाजन के तुरन्त बाद दोनों देश कश्मीर के मुद्दे पर लड़ पड़े। पाकिस्तान की सरकार का दावा था कि कश्मीर पाकिस्तान का है जबकि भारत का कहना है कि कश्मीर भारत का अंग है। दोनों देशों के अपने-अपने तर्क हैं। इस मुद्दे को लेकर भारत पाकिस्तान के बीच 1947-48 तथा 1965 का युद्ध हो चुका है, लेकिन इन युद्धों से इस मसले का समाधान नहीं हो पाया है।

2. सियाचिन ग्लेशियर पर नियंत्रण का मुद्दा:
हिमालय में भारत – पाक-चीन सीमा पर स्थित सियाचिन ग्लेशियर उचित सीमा निर्धारण नहीं किये जा सकने के कारण भारत-पाक के बीच विवाद का मुद्दा बना हुआ है। सामरिक दृष्टि से इस क्षेत्र का अत्यधिक महत्त्व होने के कारण दोनों देश इस पर अपना अधिकार स्थापित करना चाहते हैं।

3. हथियारों की होड़ का मुद्दा: हथियारों की होड़ को लेकर भी भारत और पाकिस्तान के बीच तनातनी रहती है। 1998 में दोनों ने परमाणु परीक्षण किये तथा दोनों परमाणु अस्त्रों से लैस हैं।

4. एक-दूसरे पर संदेह तथा आरोप-प्रत्यारोप: दोनों देशों की सरकारें लगातार एक-दूसरे को संदेह की नजर से देखती हैं। उग्रवाद, आतंकवाद, जासूसी आदि के लिए एक-दूसरे पर आरोप प्रत्यारोप करती रहती है।

5. नदी – जल बँटवारे पर विवाद: भारत और पाकिस्तान के बीच सिंधु जल संधि की व्याख्या और नदी – जल के इस्तेमाल को लेकर विवाद बना हुआ है।

6. सरक्रीक की समस्या: कच्छ के रन में सरक्रीक की सीमा रेखा को लेकर दोनों देशों के बीच मतभेद हैं।

प्रश्न 9.
पाकिस्तान के साथ भारत के सम्बन्धों को किस प्रकार सुधारा जा सकता है?
अथवा
भारत-पाक सम्बन्धों को सुधारने हेतु सुझाव दीजिये।
उत्तर:
भारत-पाक सम्बन्धों को सुधार हेतु सुझाव: भारत और पाकिस्तान के बीच सम्बन्धों को सुधारने के लिए निम्नलिखित सुझाव दिये जा सकते हैं।

  1. राजनीतिक स्तर पर बातचीत एवं विश्वास बहाली के प्रयास: भारत और पाकिस्तान दोनों राजनीतिक स्तर पर प्रयास करके आपसी विवादों को बातचीत और समझौतों के द्वारा दूर कर सकते हैं।
  2. आर्थिक स्तर पर प्रयास: दोनों देशों को आपसी सम्बन्ध सुधारने के लिए आर्थिक स्तर पर ‘मुक्त व्यापार संधि’ तथा एक-दूसरे की आर्थिक जरूरतों को पूरा करके सम्बन्धों में सुधार के प्रयत्न करने चाहिए।
  3. सांस्कृतिक स्तर पर प्रयास: सांस्कृतिक स्तर पर दोनों देशों को साहित्य, कला और खेल गतिविधियों के आदान-प्रदान, वीजा सुविधा तथा सिनेमा के द्वारा सहयोग बढ़ाना चाहिए।
  4. सामाजिक स्तर पर प्रयास: भारत और पाकिस्तान को अपने सम्बन्ध सुधारने के लिए समय पर इन लोगों को आपस में मिलने की सुविधा प्रदान करें।समय
  5. तकनीकी तथा चिकित्सा सेवा का आदान-प्रदान- दोनों देश तकनीकी ज्ञान तथा चिकित्सा के क्षेत्र में भी साथ काम करके आपसी सम्बन्ध सुधार सकते हैं। गया है।
  6. शिमला समझौते का पालन- दोनों देशों को शिमला समझौते की शर्तों का पालन करना चाहिए।

JAC Class 12 Political Science Important Questions Chapter 5 समकालीन दक्षिण एशिया

प्रश्न 10.
भारत और बांग्लादेश के मध्य तनाव तथा सहयोग के मुद्दों का विवेचन कीजिये।
उत्तर:
भारत और बांग्लादेश के मध्य तनाव और सहयोग के मुद्दे भारत और बांग्लादेश के बीच तनाव तथा सहयोग के मुद्दों का विवेचन निम्नलिखित बिन्दुओं के अन्तर्गत किया दोनों देशों के मध्य तनाव के मुद्दे भारत और बांग्लादेश के मध्य तनाव के अग्रलिखित प्रमुख मुद्दे हैं।
(अ) भारत सरकार निम्न कारणों से बांग्लादेश से नाराज है।

  1. भारत में अवैध आप्रवास (बांग्लादेशी शरणार्थियों की भारत में घुसपैठ ) पर ढाका द्वारा खंडन करना।
  2. बांग्लादेश सरकार का भारत विरोधी इस्लामी कट्टरपंथी जमातों को समर्थन देना।
  3. भारतीय सेना को पूर्वोत्तर भारत में जाने के लिए अपने इलाके से रास्ता देने से बांग्लादेश का इन्कार करना।
  4. ढाका का भारत को प्राकृतिक गैस निर्यात न करने का फैसला।
  5. म्यांमार को बांग्लादेशी इलाके से होकर भारत को प्राकृतिक गैस निर्यात नहीं करने देना।

(ब) बांग्लादेश सरकार निम्न कारणों से भारत सरकार से नाखुश है।

  • बांग्लादेश सरकार नदी – जल में हिस्सेदारी के सवाल पर भारत सरकार से नाखुश है।
  • भारत की सरकार पर चटगांव पर्वतीय क्षेत्र में विद्रोह को हवा देने, बांग्लादेश के प्राकृतिक गैस में सेंधमारी करने के भी आरोप हैं।
    1. पिछले दस वर्षों के भारत और बांग्लादेश के मध्य सहयोग के मुद्दे. दौरान दोनों देशों के बीच आर्थिक सम्बन्ध ज्यादा बेहतर हुए हैं।
    2. आपदा प्रबंधन और पर्यावरण के मसले पर भी दोनों देशों ने निरंतर सहयोग किया है।
    3.  इस बात के भी प्रयास किये जा रहें हैं कि साझे खतरों को पहचान कर तथा एक-दूसरे की जरूरतों के प्रति ज्यादा संवेदनशीलता बरतकर सहयोग के दायरे को बढ़ाया जाये ।

प्रश्न 11.
दक्षिण एशियायी क्षेत्रीय सहयोग के क्षेत्र में सार्क की स्थापना के उद्देश्यों को स्पष्ट कीजिये तथा उसके संगठन पर प्रकाश डालिये।
उत्तर:
‘सार्क सार्क की स्थापना: दक्षिण एशियाई क्षेत्रीय सहयोग संगठन (सार्क) की स्थापना दिसम्बर, 1985 में हुई । वर्तमान में, सार्क में आठ सदस्य राष्ट्र हैं। सार्क के उद्देश्य – सार्क के प्रमुख उद्देश्य निम्नलिखित हैं।

  1. दक्षिण एशिया के लोगों के कल्याण में वृद्धि तथा उनके जीवन स्तर में उन्नति लाना।
  2. इस क्षेत्र में आर्थिक विकास, सामाजिक प्रगति तथा सांस्कृतिक विकास लाना।
  3. दक्षिण एशिया के देशों के बीच सामूहिक आत्मविश्वास को विकसित करने का प्रयास करना।
  4. एक-दूसरे की समस्याओं को समझने, सुलझाने तथा परस्पर विश्वास को लाने में योगदान करना।
  5. आर्थिक, सामाजिक, सांस्कृतिक, तकनीकी तथा वैज्ञानिक क्षेत्रों में परस्पर सहयोग करना।
  6. दूसरे विकासशील देशों के साथ पारस्परिक सहयोग में वद्धि करना।
  7. समान हितों के मामलों में अन्तर्राष्ट्रीय आधारों पर परस्पर सहयोग में वृद्धि करना।
  8. समान उद्देश्यों वाले क्षेत्रीय और अन्तर्राष्ट्रीय संगठनों के साथ सहयोग करना।

सार्क का संगठन: सार्क की प्रमुख संस्थाएँ निम्नलिखित हैं।

  1. शिखर सम्मेलन: सार्क देशों का प्रतिवर्ष एक शिखर सम्मेलन आयोजित किया जाता है जिसमें सदस्य देशों के शासनाध्यक्ष भाग लेते हैं।
  2. मंत्रिपरिषद्: सार्क के सभी राष्ट्रों के विदेशमंत्रियों से मिलकर एक मंत्रिपरिषद् का निर्माण किया गया है जो नीतियों का निर्माण करती है ।
  3. स्थायी समिति: सार्क की एक स्थायी समिति है जो परिषद् की योजनाओं की स्वीकृति देती है तथा उनका वित्तीय प्रबन्ध करती है।
  4. तकनीकी समिति: सार्क की तकनीकी समिति क्षेत्रीय सहयोग के विस्तार, योजनाओं का निर्माण व उनके कार्यान्वयन का मूल्यांकन आदि कार्य करती है।
  5. सचिवालय: सार्क का एक सचिवालय है। इसका एक महासचिव होता है जिसका कार्यकाल 2 वर्ष रखा गया है।
  6. वित्तीय व्यवस्था: सार्क के चार्टर के अनुच्छेद 9 में वित्तीय व्यवस्थाओं का प्रावधान किया गया है।

प्रश्न 12.
दक्षिण एशिया में शांति एवं सहयोग हेतु किये गये प्रयासों की विवेचना कीजिए।
अथवा
दक्षिण (सार्क) क्या है? दक्षिण एशिया की शांति व सहयोग में इसका क्या योगदान है।?
उत्तर:
सार्क (दक्षेस) से आशय: सार्क (दक्षेस) दक्षिण एशिया के देशों का इस क्षेत्र में शांति एवं सहयोग की स्थापना हेतु स्थापित किया गया एक क्षेत्रीय संगठन है। इसकी स्थापना 1985 में हुई। वर्तमान में इसके सदस्य देश हैं- भारत, बांग्लादेश, नेपाल, श्रीलंका, पाकिस्तान, भूटान, मालदीव और अफगानिस्तान। दक्षिण एशिया के देशों में शांति एवं सहयोग के कदम दक्षिण एशिया के देशों ने दक्षिण एशिया में शांति एवं सहयोग की स्थापना की दिशा में निम्न प्रमुख कदम उठाये हैं

(1) सार्क (दक्षेस) की स्थापना तथा उसके कार्य:
सार्क के विभिन्न शिखर सम्मेलनों में दक्षिण एशिया के देशों में नशीले पदार्थों की तस्करी रोकने, पर्यटन के विकास, आपदा प्रबन्ध, खाद्य सुरक्षा भण्डार की स्थापना, पर्यावरण की सुरक्षा, संयुक्त उद्यम स्थापित करने, क्षेत्रीय परियोजनाओं हेतु सामूहिक कोष गठित करने आदि के निर्णय लिये गये

(2) दक्षिण एशिया मुक्त व्यापार क्षेत्र (South Asia Free Trade Area)-SAFTA ): इन देशों ने साफ्टा (SAFTA) लागू करना स्वीकार कर लिया। सन् 2004 में दक्षिण एशिया मुक्त व्यापार क्षेत्र के समझौते पर सार्क के देशों ने हस्ताक्षर किये तथा यह समझौता 2006 से प्रभावी हो गया। इसके अतिरिक्त इन सम्मेलनों में एक गरीबी उन्मूलन कोष गठित करने; दोहरे करारोपण से बचाव, वीसा नियमों में उदारता तथा दक्षेस संचार विषयों पर सहमति हुई है।

(3) भारत-पाकिस्तान सहयोग के प्रयास: भारत और पाकिस्तान के बीच तनाव को कम करने और शांति बहाल करने के लिए सार्क सम्मेलनों में लगातार प्रयास हुए हैं।

प्रश्न 13
श्रीलंका के जातीय संघर्ष का वर्णन कीजिए।
उत्तर: श्रीलंका का जातीय संघर्ष श्रीलंका 1948 में स्वतंत्र हुआ और स्वतंत्रता के बाद से अब तक वहाँ लोकतंत्र कायम है। लेकिन स्वतंत्रता के बाद से ही श्रीलंका को जातीय संघर्ष का सामना करना पड़ रहा है।

  1. सिंहली समुदाय व उसका दृष्टिकोण: श्रीलंका की राजनीति में सिंहली समुदाय बहुसंख्यक है। तथा सिंहली समुदाय का ही वहाँ दबदबा रहा है। सिंहलियों का मानना है कि श्रीलंका सिर्फ सिंहली लोगों का है। अतः तमिलों को कोई रियायत नहीं दी जाए।
  2. तमिल समुदाय: श्रीलंका के तमिल समुदाय की माँग रही है कि श्रीलंका के एक क्षेत्र को तमिलों के लिए एक अलग राष्ट्र बनाया जाये। लेकिन सिंहलियों के तमिल अल्पसंख्यकों के उक्त उपेक्षा भरे व्यवहार के कारण वहाँ उग्र- तमिल राष्ट्रवाद की आवाज बुलंद हुई।
  3. जातीय संघर्ष: 1983 के बाद श्रीलंका के एक उग्रवादी तमिल संगठन लिट्टे ने श्रीलंका की सेना के साथ एक पृथक तमिल राष्ट्र की स्थापना के लिए सशस्त्र संघर्ष प्रारंभ कर दिया। श्रीलंका का यह हिंसक संघर्ष लगातार चलता आ रहा है। अन्तर्राष्ट्रीय मध्यस्थ के रूप में नार्वे और आइसलैंड जैसे देश युद्धरत दोनों पक्षों को बातचीत करने के लिए राजी कर रहे हैं। लेकिन यह संभव नहीं हो पाया है। अंदरूनी संघर्ष के झंझावतों को झेलकर भी श्रीलंका ने लोकतांत्रिक राज व्यवस्था कायम रखी है।

प्रश्न 14.
पूर्वी पाकिस्तान से बड़ी संख्या में लोग पलायन करके भारत आ गए। कारण सहित स्पष्ट कीजिए।
उत्तर:
अंग्रेजी शासन के समय बंगाल और असम के विभाजित हिस्सों से पूर्वी पाकिस्तान बना था। भारत और पाकिस्तान के विभाजन के बाद से ही पूर्वी पाकिस्तान के लोगों ने बंगाली संस्कृति और भाषा के साथ किए जा रहे दुर्व्यवहार के खिलाफ विरोध का प्रदर्शन करने लगे। इस क्षेत्र की जनता ने प्रशासन में अपने न्यायोचित प्रतिनिधित्व तथा राजनीतिक सत्ता में हिस्सेदारी की माँग की। पश्चिमी पाकिस्तान के प्रभुत्व के खिलाफ लोगों ने आंदोलन किया तथा पूर्वी क्षेत्र के लिए स्वायत्तता की माँग की।

परिणामस्वरूप शेख मुजीबुर्रहमान के नेतृत्व में आवामी लीग को 1970 में पूर्वी पाकिस्तान की सारी सीटों पर विजय मिली। लेकिन सरकार पर पश्चिमी पाकिस्तान के नेताओं का दबदबा था। जनरल याहिया खान के सैनिक शासन में पाकिस्तानी सेना ने बंगाली जनता के आंदोलन को कुचलने की कोशिश की। हजारों लोग पाकिस्तानी सेना के हाथों मारे गए। इस वजह से पूर्वी पाकिस्तान से बड़ी संख्या में लोग भारत पलायन कर गए।

JAC Class 9 Hindi व्याकरण शब्द विचार

Jharkhand Board JAC Class 9 Hindi Solutions Vyakaran शब्द विचार Questions and Answers, Notes Pdf.

JAC Board Class 9 Hindi Vyakaran शब्द विचार

परिभाषा-श्रुतिसम अर्थात सुनने में समान। जो शब्द पढ़ने और सुनने में लगभग एक समान लमते है, परंतु अर्थ की दृष्टि से भिन्नत पाई जाती हैं, वे श्रुतिसम-भिन्नर्थक शब्द कहलाते हैं। इन शब्दों में स्वर, मात्रा अथवा व्यंजन में थोड़-सी भिन्नता पाई जाती हैं। ये बोलने में लगभग एक जैसे लगते हैं, परंतु उनके अर्थ में भिन्नता अवश्य होती है। यही शब्द ‘ ग्रुतिसम भिन्नार्थक शब्द’ होते हैं।
जैसे – घन और धन दोनों के उच्चारण में कोई खास भिन्नता महसूस नहीं होती परंतु अर्थ में भिन्नता है।

घन = बादल
धन = रुपया-पैसा या संपत्ति

हिंदी भाषा में ऐसे बहुत से शब्द हैं, जिनमें से कुछ की सूची नीचे दी जा रही है :

JAC Class 9 Hindi व्याकरण शब्द विचार 1

पमासनाही पालन

जिन शब्दों के अर्थ में समानता होती है, उन्हें पर्यायवाची या समानार्थी शब्द कहते हैं। यद्यपि पर्यायवाची शब्दों के अर्थ में समानता तो होती है, परंतु प्रत्येक शब्द की अपनी विशेषता होती है। भावगत भिन्नता पाई जाती है। इनके प्रयोग में हमें सावधानी बरतनी होती है।

  • अच्छा – सुष्ठु, शुभ, श्रेष्ठ, शोभन, सुंदर
  • अनुरूप – अनुकूल, संगत, अनुसार
  • अनुपम – अद्भुत, अद्वितीय, अनोखा, अपूर्व, निराला, अनूठा
  • अधम – पतित, भ्रष्ट, नीच, निकृष्ट, खल, पामर, दुर्जन
  • अपमान – अनादर, उपेक्षा, तिरस्कार, निरादर
  • अंकुश – रोक, दबाव, प्रतिबंध
  • अमृत – सुधा, पीयूष, अमिय, सोम
  • अरिन – हुताशन, वहनि, अनल, पावक, आग, दहन, ज्वाला
  • अलि – भ्रमर, भौरा, मधुकर, मधुप, मिलिंद
  • असुर – राक्षस, दैत्य, दानव, दनुज, निशाचर
  • ईश्वर – ईश, परमात्मा, परमेश्वर, प्रभु, भगवान, जगदीश
  • उषा – प्रभात, सवेरा, अरुणोदय, निशांत
  • उन्नति – उदय, वृद्धि, विकास, उत्कर्ष, उत्थान, अभ्युदय, प्रगति, उन्नयन
  • अंकुर – कोंपल, अँखुआ, कलिका
  • अँधेरा – तम, तिमिर, अंधकार, अँधियारा
  • आँसू – अश्रु, नयनजल, नेत्रनीर
  • अहंकार – अभिमान, गर्व, घमंड, मद, दर्प
  • आनंद – मोद, प्रमोद, हर्ष, आमोद, प्रसन्नता, उल्लास
  • आँख – नेत्र, चक्षु, नयन, लोचन, अक्षि
  • आकाश – व्योम, गगन, अंबर, नभ, आसमान, अनंत
  • अतिधि – अभ्यागत, मेहमान, आगंतुक, पाहुन
  • अश्व – घोड़ा, हय, बाजी, घोटक, तुरंग
  • आभूषण – अलंकार, भूषण, गहना
  • इनाम – पुरस्कार, पारितोषिक, प्रीतिकर, आनंदकर
  • इच्छा – अभिलाषा, चाह, मनोरथ, कामना, आकांक्षा, लालसा
  • इंद्र – देवेंद्र, सुरेंद्र, सुरपति, पुरंदर, देवराज, शचीपति
  • उद्देश्य – अभिग्राय, आशय, लक्ष्य, ध्येय, इष्ट, तात्पर्य
  • उपकार – हित, भलाई, नेकी, भला
  • कपड़ा – वस्त्र, अंबर, चीर, पट, वसन, परिधान
  • किरण – रश्मि, अंशु, मरीची, मयूख, कर
  • केश – बाल, अलक, कच, कुंतल
  • कोयल – पिक, कोकिल, श्यामा, कलकंठी, कोकिला, वसंतदूत
  • कृपा – अनुग्रह, मेहरबानी, दया, अनुकंपा
  • कमल – अरविंद, जलज, नलिन, पंकज, सरोज, राजीव, नीरज, अंबुज
  • कान – कर्ण, श्रोत्र, श्रवण
  • किनारा – तट, तीर, कूल, पुलिन
  • कृष्ण – वासुदेव, गोपाल, गिरधर, केशव
  • क्रोध – गुस्सा, रोष, कोप, आमर्ष
  • खल – अधम, कुटिल, दुष्ट, शठ
  • खून – रक्त, लहू, शोणित
  • गणेश – गणपति, भालचंद्र, लंबोदर, गजानन, विनायक
  • गंगा – भागीरथी, देवनदी, सुरसरी, मंदाकिनी, नदीश्वरी
  • गौ – गाय, सुरभि, धेनु, गऊ, दुधा
  • गोद – अंक, क्रोड, उत्सर्ग
  • गुफा – गुहा, विवर, कंदरा
  • जल – वारि, नीर, पानी, पय, तोय, सलिल, अंबु, उदक
  • जंगल – विपिन, कानन, वन, अरण्य
  • घर – गृह, सदन, निकेतन, भवन, आवास, आलय, धाम, गेह
  • चंद्रमा – शशि, विधु, चंद्र, राकेश, इंदु, चाँद, सोम, सुधाकर
  • चतुर – दक्ष, प्रवीण, निपुण, कुशल, योग्य, होशियार
  • चाँदनी – ज्योत्स्ना, चंद्रिका, कौमुदी, चंद्रमरीची
  • झंडा – ध्वज, पताका, निशान, केतु
  • तलवार – खड्ग, कृपाण, असि, शमशीर, करवाल
  • तरंग – लहर, उर्मि, वीचि, हिलोर
  • जीभ – जिह्वा, रसना, रसज्ञा, रसा
  • दूध – दुग्ध, पय, गोरस, क्षीर
  • तालाब – सर, सरोवर, तड़ाग, जलाशय, ताल, पोखर
  • तारा – नक्षत्र, तारक, नखत
  • तीर – बाण, शर, सायक, इषु, नाराच, शिलिमुख
  • दास – भृत्य, नौकर, सेवक, अनुचर, परिचारक
  • दिवस – दिन, वार, वासर, दिव, अहर
  • देवता – सुर, अमर, देव, अजर
  • दुर्गा – भवानी, देवी, कालिका, अंबा, चंडिका
  • दुख – पीड़ा, कष्ट, व्यथा, विषाद, यातना, वेदना
  • धनुष – धनु, कोदंड, चाप, कमान, शरासन, पिनाक, कार्मुक
  • धन – द्रव्य, अर्थ, वित्त, संपत्ति, लक्ष्मी
  • नदी – सरिता, तरंगिणी, सरित, नद, तटिनी
  • दंत – दाँत, दशन, रद, द्विज, दंश
  • नमस्कार – नमः, प्रणाम, अभिवादन, नमस्ते
  • नरक – दुर्गति, संघात, यमपुर, यमलोक, यमालय
  • पर्वत – गिरी, पहाड़, शैल, नग, अचल, महीधर
  • पुत्र – सुत, बेटा, लड़का, पूत, तनय
  • पुत्री – सुता, बेटी, लड़की, नंदिनी, तनया
  • नारी – स्त्री, कामिनी, महिला, अबला, वनिता, भामिनी, ललना
  • नागर – नगरी, पत्तन, पुर, पुरी
  • निशा – यामिनी, रात, रात्रि, रजनी, शर्वरी
  • निर्मल – शुद्ध, स्वच्छ, विमल, पवित्र
  • नौका – नाव, तरिणी, जलयान, बेड़ा, पतंग, तरी
  • पंडित – विद्वान, प्राज्ञ, बुद्धिमान, धीमान, धीर, सुधी
  • पवन – हवा, वायु, समीर, बयार, मारूत, अनिल
  • पक्षी – खग, नभचर, विहग, पंछी, विहंग, पतंग
  • पत्नी – वधु, गृहिणी, स्त्री, प्राणप्रिया, अर्धांगिनी, भार्या
  • पति – स्वामी, नाथ, वर, कांत, प्राणनाथ, आर्य, ईश
  • पत्ता – पात, दल, पत्र, पल्लव
  • पराग – पुष्पराज, कुसुमरज, पुष्पधूलि
  • पत्थर – पाषाण, वज्र, पाहन, उपल, शिला
  • पुष्प – कुसुम, सुमन, फूल, मंजरी, प्रसून
  • पृथ्वी – भू, भूमि, धरणी, धरती, वसुधा, धरा
  • भयानक – डरावना, भयजनक, विकट, गंभीर
  • भिक्षा – भीख, याचना, माँगना, खैरात
  • प्रकाश – उजाला, ज्योति, प्रभा, विभा, आलोक
  • प्रतीक – चिहन, प्रतिमा, निशान, प्रतिभूति
  • बाग – बगीचा, वाटिका, उपवन, उद्यान, आराम
  • बिजली – विद्युत, तड़ित, दामिनी, चंचला, चपला
  • बंदर – कपि, वानर, शाखामृग, हरि
  • मदिरा – शराब, सुरा, मद्य, वारुणी
  • मित्र – सखा, सहचर, साथी, मीत, दोस्त
  • माता – जननी, माँ, मात, मैया, अंबा
  • मुख – मुँह, मुखड़ा, आनन, वदन
  • भूख – क्षुधा, बुभुक्षा, अन्नलिप्सा, आहारेच्छा
  • मछली – अंडज, मीन, मत्स्य, शफरी
  • मृत्यु – निधन, देहांत, अंत, मौत
  • मनुष्य – मनुज, नर, आदमी, मानव, पुरुष
  • मेघ – जलद, बादल, नीरद, घन, वारिद
  • मोर – मयूर, शिखी, शिखंडी, कलापी
  • मूर्ख – अज, अबोध, गंवार, मूढ़
  • मल्लाह – नाविक, माँझी, खेवट, कर्णधार, केवट
  • यत्न – उद्यम, प्रयास, उद्योग, प्रयत्त, पुरुषार्थ
  • लक्ष्मी – इंदिरा, कमला, रमा, हरिप्रिया, श्री
  • वन – विपिन, जंगल, कानन, अटवी, अरण्य
  • यौवन – जवानी, युवावस्था, जीवन, शिरोमणि, तरुणाई
  • युवक – जवान, युवा, तरुण
  • युद्ध – समर, लड़ाई, रण, संग्राम
  • राजा – नृप, भूपति, भूप, नरेश, सम्राट, नरेंद्र, नरपति
  • रण – संग्राम, सम, युद्ध, लड़ाई
  • शत्रु – रिपु, दुश्मन, विपक्षी, अरि, वैरी
  • शरीर – देह, काया, कलेवर, तन, वपु, अंग, गात
  • शिव – शंकर, महादेव, रुद्र, भूतनाथ, नीलकंठ, पिनाकी
  • वृक्ष – तरु, पादप, विटप, द्रुम
  • वर्ष – अब्द, साल, बरस, संवत्
  • शोभा – छवि, दीप्ति, कांति, सुषमा, आभा, छटा
  • समुद्र – सागर, उदधि, रत्नाकर, जलधि, सिंधु
  • सरस्वती – शारदा, भारती, गिरा, वाणी
  • सेना – सैन्य, दल, कटक, वाहिनी
  • सिंह – शेर, केसरी, मृंगंद्र, वनराज, मृगराज
  • स्वर्ण – कंचन, कनक, कुंदन, सोना
  • हाथ – कर, पाणि, हस्त
  • हिरन – कुरंग, मृग, सारंग, कृष्णसार
  • सूर्य – रवि, प्रभाकर, भास्कर, दिवाकर, आदित्य, सविता, दिनकर
  • सर्प – भुजंग, नाग, विषधर, व्याल
  • संसार – दुनिया, जगत्, विश्व, जग, भवन, लोक
  • संतान – संतति, अपत्य, प्रजा, प्रसूति, औलाद
  • हंस – मराल, कलहंस, मानसौकस
  • हनुमान – महावीर, पवनसुत, मारूत, कपीश, बजरंगबली, आंजनेय
  • हाथी – गज, कुंजर, गयंद, हस्ती, मतंग, करी

JAC Class 9 Hindi व्याकरण शब्द विचार

विलोम शब्द

किसी शब्द के उलटे अर्थ को व्यक्त करने वाले शब्द को विलोम शब्द कहते है। भाषा में भावों-विचारों की स्पष्टता के लिए विलोम शब्द महत्वपूर्ण भूमिका निभाते हैं।

शब्द – विलोम

  • अंतरंग – बहिरंग
  • अक्षत – विक्षत
  • अस्तित्व – अनस्तित्व
  • अवर – प्रवर
  • अति – अल्प
  • अर्थ – अनर्थ
  • अथ – इति
  • अंत – आरंभ
  • अंधकार – प्रकाश
  • आमिष – निरामिष
  • हिंसा – अहिंसा
  • इच्छा – अनिच्छा
  • ईश – अनीश
  • उद्यम – आलस्य
  • उचित – अनुचित
  • उपेक्षा – अपेक्षा
  • उत्कर्ष – अपकर्ष
  • अल्पज्ञ – सर्वज्ञ
  • आज्ञा – अवज्ञा
  • आदि – अंत
  • आस्तिक – नास्तिक
  • आय – व्यय
  • आदर – निरादर
  • आयात – निर्यात
  • आरोह – अवरोह
  • आगत – अनागत
  • आर्य – अनार्य
  • आंतरिक – बाह्य
  • अन्याय – न्याय
  • अनुज – अग्रज
  • अनुराग – विराग
  • अभिमान – नम्रता
  • अज्ञ – प्राज्ञ
  • उपकार – अपकार
  • अवनति – उन्नति
  • अनुकूल – प्रतिकूल
  • अपेक्षा – उपेक्षा
  • अनिवार्य – ऐच्छिक
  • अमृत – विष
  • उन्नति – अवनति
  • उन्नयन – पलायन
  • आना – जाना
  • उर्वरा – ऊसर
  • आस्था – अनास्था
  • उतार – चढ़ाव
  • उत्थान – पतन
  • उदार – कृपण
  • आकाश – पाताल
  • कुरूप – सुरूप
  • खल – सज्जन
  • खरा – खोटा
  • गुण – अवगुण
  • लौकिक – अलौकिक
  • लोक – परलोक
  • लेन – देन
  • प्राचीन – नवीन
  • वादी – प्रतिवादी
  • आरंभ – अंत
  • आदान – प्रदान
  • उत्कृष्ट – निकृष्ट
  • उत्तीर्ण – अनुत्तीर्ण
  • उपस्थित – अनुपस्थित
  • उत्तर – प्रश्न
  • उत्तम – अधम
  • उपयुक्त – अनुपयुक्त
  • एक – अनेक
  • आलस्य – स्फूर्ति
  • आचार – अनाचार
  • आशा – निराशा
  • एकता – अनेकता
  • ऐच्छिक – आवश्यक
  • कृतज्ञ – कृतघ्न
  • क्रय – विक्रय
  • कायर – साहसी
  • धर्म – अधर्म
  • ज्येष्ठ – कनिष्ठ
  • जल – स्थल
  • उदय – अस्त
  • कीर्ति – अपकीर्ति
  • कोमल – कठोर
  • जीवित – मृत
  • जन्म – मृत्यु
  • जय – पराजय
  • जीत – हार
  • ऊपर – नीचे
  • कर्कश – मधुर
  • कनिष्ठ – ज्येष्ठ
  • कुमति – सुमति
  • कठिन – सरल
  • कपटी – निष्कपट
  • सम – विषम
  • चतुर – मूर्ख
  • काला – सफ़ेद
  • कृत्रिम – स्वाभाविक
  • उष्ण – शीत
  • गुप्त – प्रकट
  • ऋण – उत्रण
  • घटिया – बढ़िया
  • ठौर – कुठौर
  • चर – अचर
  • चेतन – जड़
  • चल – अचल
  • चंचल – स्थिर
  • उधार – नकद
  • उजाला – अँधेरा
  • उपयोगी – अनुपयोगी
  • उत्पत्ति – विनाश
  • दु:शील – सुशील
  • दुर्बल – बलवान
  • दोष – गुण
  • दायाँ – बायाँ
  • शिक्षित – अशिक्षित
  • शकुन – अपशकुन
  • वीर – कायर
  • शयन – जागरण
  • नया – पुराना
  • बुढ़ापा – यौवन
  • जटिल – सरल
  • झूठ – सच
  • दुर्जन – सज्जन
  • डर – निडर
  • डरपोक – निर्भीक
  • निकट – दूर
  • तृष्णा – संतोष
  • त्यागी – स्वार्थी
  • दानी – कृपण
  • दिन – रात
  • दयालु – निर्दयी
  • दुर्गंध – सुगंध
  • देश – विदेश
  • देव – दानव
  • धीर – अधीर
  • धनवान – निर्धन
  • स्वर्ग – नरक
  • सुरूप – कुरूप
  • सामान्य – विशेष
  • भला – बुरा
  • भारी – हलका
  • नेकी – बदी
  • विधवा – सधवा
  • विरोध – समर्थन
  • विरोधी – समर्थक
  • शांत – अशांत
  • नश्वर – अनश्वर
  • गहरा – छिछला
  • नास्तिक – आस्तिक
  • नत – उन्नत
  • नूतन – पुरातन
  • निद्रा – जागरण
  • निर्गुण – सगुण
  • रात्रि – प्रातः
  • स्वदेश – विदेश
  • स्तुति – निंदा
  • शीत – उष्ण
  • निर्यात – आयात
  • निश्चय – अनिश्चय
  • निंदा – स्तुति
  • सुख – दु:ख
  • साकार – निराकार
  • सामान्य – असामान्य
  • सदुपयोग – दुरुपयोग
  • निंदनीय – प्रशंसनीय
  • निगलना – उगलना
  • पाप – पुण्य
  • प्रत्यक्ष – परोक्ष
  • पंडित – मूर्ख
  • पराधीन – स्वाधीन
  • शुष्क – आर्द्र
  • शाप – वरदान
  • सुगंध – दुर्गध
  • सार्थक – निर्थक
  • सरल – जटिल
  • सुगम – दुर्गम
  • सकाम – निष्काम
  • स्वार्थ – परमार्थ
  • सभ्य – असभ्य
  • योगी – भोगी
  • विक्रय – क्रय
  • विद्वान – मूर्ख
  • शुद्ध – अशुद्ध
  • शुभ – अशुभ
  • शांति – अशांति
  • परतंत्र – स्वतंत्र
  • पूर्व – पश्चिम
  • पूर्ण – अपूर्ण
  • पवित्र – अपवित्र
  • सबल – निर्बल
  • प्रीति – वैर
  • परिश्रम – आलस्य
  • पक्षपात – निष्पक्ष
  • प्रसन्न – अप्रसन्न
  • पक्ष – विपक्ष
  • प्रश्न – उत्तर
  • पुरस्कार – तिरस्कार
  • बुराई – भलाई
  • बाहर – अंदर
  • सदाचार – दुराचार
  • विजय – पराजय
  • वृष्टि – अनावृष्टि
  • सुपुत्र – कुपुत्र
  • सुबोध – दुर्बोध
  • भयभीत – निर्भय
  • रुग्ण – स्वस्थ
  • स्थूल – सूक्ष्म
  • रक्षक – भक्षक
  • हर्ष – विषाद
  • प्रीति – दोष
  • सौम्य – भीषण
  • सुयोग – कुयोग
  • मधुर – कटु
  • मान – अपमान
  • राग – दूवेष
  • लघुता – गुरुता
  • लाभ – हानि
  • हार – जीत
  • स्वाधीन – पराधीन
  • स्थिर – चंचल
  • सधवा – विधवा
  • सरस – नीरस
  • सौभाग्य – दुर्भाग्य
  • योग्य – अयोग्य
  • राजा – रंक
  • मित्र – शत्रु
  • मानव – दानव
  • मृदु – कठोर
  • मूक – वाचाल
  • संधि – विग्रह
  • मलिन – निर्मल
  • मिथ्या – सत्य
  • मुख्य – गौण
  • यश – अपयश
  • सफल – असफल
  • संयोग – वियोग
  • सुलभ – दुर्लभ
  • ज्ञान – अज्ञान
  • ज्ञानी – मूर्ख
  • क्षमा – दंड
  • क्षय – अक्षय
  • संकल्प – विकल्प
  • सुर – असुर
  • सुरक्षित – असुरक्षित
  • आत्मीय – अनात्मीय

JAC Class 9 Hindi व्याकरण अनुस्वार एवं अनुनासिक

Jharkhand Board JAC Class 9 Hindi Solutions Vyakaran अनुस्वार एवं अनुनासिक Questions and Answers, Notes Pdf.

JAC Board Class 9 Hindi Vyakaran अनुस्वार एवं अनुनासिक

अनुस्वार, अनुनासिक –

हिंदी में अनुस्वार और अनुनासिक का प्रायः प्रयोग किया जाता है।
अनुस्वार का उच्चारण इ इ, ण्, च, म् के समान होता है। जैसे – कंघी (कड्घी), गंजा (गज्जा), घंटी (घण्टी), संत (सल), पंप (पम्प) अनुस्वार के साथ मिलती-जुलती एक ध्वनि अनुनासिक भी है, जिसका रूप ‘ज’ है। इन दोनों में भेद केवल यह है कि जनस्वार की ध्वनि कठोर होती है और अनुनासिक की कोमल। अनुस्वार का उच्चारण नाक से होता है और अनुनासिक का उच्चारण मुख और नासिका दोनों से होता है। जैसे-हँस और हंस। हँस का अर्थ है-हँसना और हंस का अर्थ है-एक प्रकार का पक्षी।

हिंदी मानक लिपि में बिंदु –

(क) पंचमाक्षों (ङ, उ, ण, न, म) का जब अपने वर्ग के वर्णों के साथ संयोग होता है तब उनके स्थान पर अनुस्वार (-लिखा जाता है । जैसे –

  • दन्त = दंत
  • चज्वल = चंचल
  • ठण्डा = ठंडा
  • कड्यी = कंघी
  • पम्प = पंप
  • कन्द्रा = कंधा

JAC Class 9 Hindi व्याकरण अनुस्वार एवं अनुनासिक

(ख) य, र, ल, व, श, ष, स, ह के साथ अनुस्वार ही लगता है। जैसे -संयोग, संरक्षण, संसार, संशय, शंका, लंपट, बंदना, संबाद. हुंकार, हंता, रंक, रंज, लंका आदि।
(ग) यदि वर्ण के ऊपर मात्रा लगी हो, तो अनुनासिक (-) का उच्चारण होने पर भी अनुस्वार की बिंदी ही लगेरी। जैंसे-मैं, हैं, बेंमल्ला, सौंफ आदि।
अनुस्वार के प्रयोग संबंधी अन्य उदाहरण –

पुराना रूप – मानक रूप

  • चन्दन – चंदन
  • प्रशान्त – प्रशांत
  • किन्तु – किंतु
  • हिन्दी – हिंदी
  • अन्त – अंत
  • पण्डा – पंडा
  • इन्द्र – इंद्र
  • प्रबन्ध – प्रबंध
  • अन्तर – अंतर
  • सम्बन्धी – संबंधी
  • निरन्तर – निरंतर
  • हिन्दू – हिंदू
  • चिह्न – चिह्न
  • गन्दा – गंदा
  • क्रन्दन – क्रंदन
  • सन्ध्या – संध्या
  • सुन्दर – सुंदर
  • दन्त – दंत
  • अन्तर – अंतर
  • केन्द्र – कैंद्र
  • निन्दनीय – निंदनीय
  • आरम्भ – आरंभ
  • आनन्द – आनंद
  • ठण्डक – एंडक
  • अभिनन्दन – अभिनंद्न
  • सम्भावना – संभावन
  • इप्टरनेशनल – इंटरनेशनल
  • सम्पादक मण्डल – संपादक मंडल
  • प्रेमचन्द – प्रेमचंद
  • सन्दिग्ध – संदिन्ध
  • सन्देह – संदेह
  • अन्दर – अंदर
  • तुरन्त – तुरंत
  • इन्तज़ार – इंतजार
  • सम्भव – संभव
  • सन्देहास्पद – संदेहास्पद्
  • शान्ति – शांति
  • स्वतन्त्र – स्वतंत्र
  • गणतन्त्र – गणतंत्र
  • अम्बाला – अंबाला
  • चण्डीगढ़ – चंडीगढ़
  • मुम्बई – मुंबई
  • हिन्दुस्तान – हिंदुस्तान
  • गान्धी – गांधी
  • सन्दीपन – संदीपन
  • क्रान्ति – क्रांति
  • श्रद्धार्जलि – श्रद्धाजाल
  • दुर्गन्ध – दुर्गंध
  • सन्धि – संधि
  • सन्तोष – संतोष
  • अम्बर – अंबर
  • अझ्ग – अंग
  • कुन्तल – कुंतल
  • अम्भोज – अंभोज
  • निशान्त – निशांत
  • उत्कण्ठा – उत्कंठा
  • पउ्चबाण – पंचबाण
  • लम्बोदर – लंबोदर
  • चन्द्रमा – चंद्रमा
  • कालिन्दी – कालिंदी
  • दन्तच्छद – दंतच्छद
  • नरेन्द्र – नरेंद्र
  • इन्दिरा – इंदिरा
  • सन्तान – संतान
  • वसुन्धरा – वसुंधरा
  • बन्दर – बंदर
  • देहान्त – देहांत
  • कुन्दन – कुंदन
  • किंवदन्ती – किंवर्दंती
  • दम्पति – दंपति
  • पन्द्रह – पंद्रह
  • सम्पत्ति – संपत्ति
  • अनन्त – अनंत
  • इन्द्रियां – इंद्रियाँ
  • पाण्डव – पांडव
  • छन्द – छंद
  • तम्बू – तंबू
  • मन्दिर – मंदिर
  • क्रान्ति – क्रांति
  • अनुकम्पा – अनुकंपा
  • दयानन्द – द्यानंद
  • कवीन्द्र – कवींद्र
  • सम्पदाय – संप्रदाय
  • सम्भावित – संभावित
  • पीताम्बर – पीतांबर
  • परमानन्द – परमानंद्
  • मन्दाकिनी – मंदाकिनी

JAC Class 9 Hindi व्याकरण अनुस्वार एवं अनुनासिक

अनुनासिकता के अशुद्ध प्रयोग संबंधी कुछ शुद्ध उदाहरण –

अशुद्ध – शुद्ध

  • आंगन – आँगन
  • उंगली – उँगली
  • कांटा – काँटा
  • पांव – पाँव
  • चांद – चाँद
  • गांव – गाँव
  • बहुएं – बहुएँ
  • अशुद्ध – शुद्ध
  • खांसी – खाँसी
  • टांक – टाँक
  • फंसा – फँसा
  • संभल – सँभल
  • सांप – साँप
  • नदियां – नदियाँ

JAC Class 9 Hindi व्याकरण अनुस्वार एवं अनुनासिक

आगत ध्वनियाँ :

अर्धचंद्राकार –

आजकल अंग्रेज़ी भाषा के प्रभाव से हिंदी में ‘आ’ आगत ध्वनि का प्रयोग किया जाने लगा है। जो ‘आ’ और ‘ओ’ के बीच की ध्वनि है। जैसे –

JAC Class 9 Hindi व्याकरण अनुस्वार एवं अनुनासिक 1

यदि हिंदी में ‘ऑ’ आगत ध्वनि का प्रयोग न किया जाए तब शब्दों की अभिव्यक्ति में दोष उत्पन्न होने की पूरी संभावना हो सकती है। जैसे –

  • हाल – हालचाल
  • बाल – शरीर के बाल
  • काल – समय
  • काफ़ी – पर्याप्त
  • डाल – डालना
  • हॉल – एक बड़ा कमरा
  • बॉल – गेंद
  • कॉल – बुलाना
  • कॉफ़ी – एक पेय
  • डॉल – गुड़िया

JAC Class 9 Hindi व्याकरण शब्द और पद

Jharkhand Board JAC Class 9 Hindi Solutions Vyakaran शब्द और पद Questions and Answers, Notes Pdf.

JAC Board Class 9 Hindi Vyakaran शब्द और पद

शब्द भाषा की स्वतंत्र और सार्थक इकाई है। शब्द को व्याकरण के नियमों के अनुसार किसी वाक्य में प्रयोग करने पर वह पद बन जाता है। एक से अधिक पद जुड़कर एक ही व्याकरणिक इकाई का काम करने पर पदबंध कहलाते हैं।

शब्द का वर्गीकरण इस प्रकार है –

JAC Class 9 Hindi व्याकरण शब्द और पद 1

प्रश्न 1.
शब्द किसे कहते हैं ?
उत्तर :
शब्द वर्णों के मेल से बनाई गई भाषा की स्वतंत्र और सार्थक इकाई है। जैसे – राम, रावण, मारना।

प्रश्न 2.
शब्द का प्रत्यक्ष रूप किसे कहते हैं ?
उत्तर :
भाषा में जब शब्द कर्ता में बिना परसर्ग के आता है तो वह अपने मूल रूप में आता है। इसे शब्द का प्रत्यक्ष रूप कहते हैं। जैसे – वह, लड़का, मैं।

JAC Class 9 Hindi व्याकरण शब्द और पद

प्रश्न 3.
कोशीय शब्द किसे कहते हैं ?
उत्तर :
जिस शब्द का अर्थ शब्दकोश से प्राप्त हो जाए, उसे कोशीय शब्द कहते हैं। जैसे- मनुष्य, घोड़ा।

प्रश्न 4.
व्याकरणिक शब्द किसे कहते हैं ?
उत्तर :
व्याकरणिक शब्द उस शब्द को कहते हैं, जो व्याकरणिक कार्य करता है। जैसे-‘मुझसे आजकल विद्यालय नहीं जाया जाता।’ इस वाक्य में ‘जाता’ शब्द व्याकरणिक शब्द है।

प्रश्न 5.
पद किसे कहते हैं ?
अथवा
शब्द वाक्य में प्रयुक्त होने पर क्या कहलाता है ?
उत्तर :
जब किसी शब्द को व्याकरण के नियमों के अनुसार किसी वाक्य में प्रयोग किया जाता है तब वह पद बन जाता है। जैसे- राम, रावण, मारा शब्द है। इनमें विभक्तियों, परसर्ग, प्रत्यय आदि को जोड़कर पद बन जाता है। जैसे- श्रीराम ने रावण को मारा।

प्रश्न 6.
पद के कितने और कौन-कौन से भेद हैं ?
उत्तर :
पद के पाँच भेद संज्ञा, सर्वनाम, विशेषण, क्रिया और अव्यय हैं।

JAC Class 9 Hindi व्याकरण शब्द और पद

प्रश्न 7.
शब्द और पद में क्या अंतर है ?
उत्तर :
शब्द भाषा की स्वतंत्र और सार्थक इकाई है और वाक्य के बाहर रहता है, परंतु जब शब्द वाक्य के अंग के रूप में प्रयोग किया जाता है तो इसे पद कहते हैं। जैसे- ‘लड़का, मैदान, खेलना’ शब्द हैं। इन शब्दों से यह वाक्य बनाने पर – ‘लड़के मैदान में खेलते हैं।’- ये पद बन जाते हैं।

प्रश्न 8.
पदबंध किसे कहते हैं ?
उत्तर :
पदबंध का शाब्दिक अर्थ है – पदों में बँधा हुआ। जब एक से अधिक पद मिलकर एक इकाई के रूप में व्याकरणिक कार्य करते हैं तो वे पदबंध कहलाते हैं। जैसे चिड़िया सोने के पिंजरे में बंद है। इस वाक्य में सोने का पिंजरा पदबंध है। पदबंध वाक्यांश मात्र होते हैं, काव्य नहीं। पदबंध में एक से अधिक पदों का योग होता है और ये पद आपस में जुड़े होते हैं। शब्द के सभी भेद स्पष्ट कीजिए।

प्रश्न 9.
शब्द और शब्दावली वर्गीकरण निम्नलिखित दृष्टियों से किया जा सकता है –
उत्तर :
(क) अर्थ की दृष्टि से वर्गीकरण –
अर्थ की दृष्टि से निम्नलिखित चार भेद किए जाते हैं –
(i) एकार्थी – जिन शब्दों का प्रयोग केवल एक अर्थ में ही होता है, उन्हें एकार्थी शब्द कहते हैं। जैसे –
पुस्तक, पेड़, घर, घोड़ा, पत्थर आदि।
(ii) अनेकार्थी – जो शब्द एक से अधिक अर्थ बताने में समर्थ हैं, उन्हें अनेकार्थी शब्द कहते हैं। इन शब्दों का प्रयोग जिस संदर्भ में किया जाएगा, ये उसी के अनुसार अर्थ देंगे। जैसे –
काल – समय, मृत्यु।
अर्क – सूर्य, आक का पौधा।
(iii) पर्यायवाची या समानार्थी – जिन शब्दों के अर्थों में समानता हो, उन्हें पर्यायवाची या समानार्थी शब्द कहते हैं
जैसे – कमल – जलज, नीरज, अंबुज, सरोज।
आदमी – नर, मनुष्य, मानव।
(iv) विपरीतार्थी – विपरीत अर्थ प्रकट करने वाले शब्दों को विपरीतार्थी अथवा विलोम शब्द कहते हैं। जैसे –
आशा-निराशा हँसना – रोना

JAC Class 9 Hindi व्याकरण शब्द और पद

(ख) उत्पत्ति की दृष्टि से वर्गीकरण –
इतिहास की दृष्टि से शब्दों के पाँच भेद हैं –
(i) तत्सम – तत्सम शब्द का अर्थ है – तत् (उसके) + सम (समान) अर्थात उसके समान जो शब्द संस्कृत के मूल रूपों के समान ही हिंदी में प्रयुक्त होते हैं, उन्हें तत्सम कहते हैं। इनका प्रयोग हिंदी में भी उसी रूप में किया जाता है, जिस रूप में संस्कृत में किया जाता है। जैसे- नेत्र, जल, पवन, सूर्य, आत्मा, माता, भवन, नयन, आशा, सर्प, पुत्र, हास, कार्य, यदि आदि।

(ii) तद्भव – तद्भव शब्द का अर्थ है तत् ( उससे) + भव ( पैदा हुआ) जो शब्द संस्कृत के मूल रूपों से बिगड़ कर हिंदी में प्रयुक्त होते हैं, उन्हें तद्भव कहते हैं, जैसे- दुध से दुग्ध, घोटक से घोड़ा, आम्र से आम, अंधे से अंधा, कर्म से काम, माता से माँ, सर्प से साँप, सप्त से सात, रत्न से रतन, भक्त से भगत।

(iii) देशज – देशज का अर्थ होता है देश + ज अर्थात देश में जन्मा। लोकभाषाओं से आए हुए शब्द देशज कहलाते हैं। कदाचित ये शब्द बोलचाल से बने हैं। जैसे – पेड़, खिड़की, अटकल, तेंदुआ, लोटा, डिबिया, जूता, खोट, फुनगी आदि।

(iv) विदेशज – जो शब्द विदेशी भाषाओं से लिए गए हैं, उन्हें विदेशज कहते हैं।
अंग्रेज़ी – डॉक्टर, नर्स, स्टेशन, प्लेटफ़ॉर्म, पेंसिल, बटन, फ़ीस, मोटर, कॉलेज, ट्रेन, ट्रक, कार, बस, स्कूटर, फ्रीज़ आदि।
फ़ारसी – दुकान, ईमान, ज़हर, किशमिश, उम्मीद, फ़र्श, जहाज़, कागज़, ज़मींदार, बीमार, सब्ज़ी, दीवार आदि।
अरबी – कीमत, फ़ैसला, कायदा, तरफ़, नहर, कसरत, नशा, वकील, वज़न, कानून, तकदीर, खराब, कत्ल, फौज़ नज़र, खत आदि।
तुर्की – तगमा, तोप, लाश, चाकू, उर्दू, कैंची, बेग़म, गलीचा, बावर्ची, बहादुर, चम्मच, कैंची, कुली, कुरता आदि।
पुर्तगाली – तंबाकू, पेड़ा, गिरिजा, कमीज़, तौलिया, बालटी, मेज़, कमरा, अलमारी, संतरा, साबुन, चाबी, आलपीन, कप्तान आदि।
फ्रांसीसी – कारतूस, कूपन, अंग्रेज़।

(v) संकर – दो भाषाओं के शब्दों के मिश्रण से बने शब्द संकर शब्द कहलाते हैं।
यथा – जाँचकर्ता – जाँच (हिंदी) कर्ता (संस्कृत)
सज़ाप्राप्त – सज़ा (फ़ारसी) प्राप्त (संस्कृत)
रेलगाड़ी – रेल (अंग्रेज़ी) गाड़ी (हिंदी)
उद्गम के आधार पर शब्दों की एक और कोटि हिंदी शब्दावली में पाई जाती है जिसे अनुकरणात्मक या ध्वन्यात्मक कहते हैं।
यथा – हिनहिनाना, चहचहाना, खड़खड़ाना, भिनभिनाना आदि।

(ग) रचना की दृष्टि से वर्गीकरण –
प्रयोग के आधार पर शब्दों के भेद तीन प्रकार के होते हैं –

(i) मूल अथवा रूढ़ शब्द – जो शब्द किसी अन्य शब्द के संयोग से नहीं बनते हैं और अपने आप में पूर्ण होते हैं उन्हें रूढ़ अथवा मूल शब्द कहते हैं। ये शब्द किसी विशेष अर्थ के लिए रूढ़ या प्रसिद्ध हो जाते हैं। इनके खंड या टुकड़े नहीं किए जा सकते।
जैसे – घड़ा, घोड़ा, काला, जल, कमल, कपड़ा, घास, दिन, घर, किताब, मुँह।

(ii) यौगिक – दो शब्दों के संयोग से जो सार्थक शब्द बनते हैं, उन्हें यौगिक कहते हैं। दूसरे शब्दों में यौगिक शब्द वे शब्द होते हैं जिनमें रूढ़ शब्द के अतिरिक्त प्रत्यय, उपसर्ग या एक अन्य रूढ़ शब्द अवश्य होता है अर्थात ये दो अंशों को जोड़ने से बनते हैं, जिनमें एक शब्द आवश्यक रूप से रूढ़ होता है। जैसे – नमकीन – इसमें नमक रूढ़ तथा ईन प्रत्यय है।
जैसे – गतिमान, विचारवान, पाठशाला, विद्यालय, प्रधानमंत्री, गाड़ीवान, पानवाला, घुड़सवार, बैलगाड़ी, स्नानघर, अनपढ़, बदचलन।

(iii) योगरूढ़ – दो शब्दों के योग से बनने पर भी किसी एक निश्चित अर्थ में रूढ़ हो जाने वाले शब्द योगरूढ़ कहलाते हैं।
जैसे – चारपाई, तिपाई, जलज (जल + ज = कमल), दशानन (दस + आनन = रावण), जलद (जल + द = बादल), जलधि (जल + धि समुद्र)।

JAC Class 9 Hindi व्याकरण शब्द और पद

(घ) रूपांतरण की दृष्टि से शब्दों के भेद –

(i) विकारी शब्द – जिन शब्दों के रूप में विकार (परिवर्तन) उत्पन्न हो जाता है, उन्हें विकारी शब्द कहते हैं। संज्ञा, सर्वनाम, विशेषण और क्रिया – ये चार प्रकार के शब्द विकारी कहलाते हैं। इनमें लिंग, वचन एवं कारक आदि के कारण विकारी उत्पन्न हो जाता है।

(ii) अविकारी शब्द – जिन शब्दों के रूप में विकार (परिवर्तन) उत्पन्न नहीं होता और जो अपने मूल में बने रहते हैं, उन्हें अविकारी शब्द कहते हैं।

अविकारी शब्द को अव्यय भी कहा जाता है। क्रिया-विशेषण, समुच्चयबोधक, संबंधसूचक तथा विस्मयादिबोधक – ये चार प्रकार के शब्द अविकारी कहलाते हैं। क्रिया-विशेषण इधर समुच्चयबोधक और संबंधसूचक के ऊपर विस्मयादिबोधक ओह।

(ङ) प्रयोग की दृष्टि से –
प्रयोग के आधार पर शब्दों का वर्गीकरण निम्नलिखित दो वर्गों में किया जाता है –
(i) सामान्य शब्द – आम जन-जीवन में प्रयोग होने वाले शब्द सामान्य शब्द कहलाते हैं; जैसे- दाल, भात, खाट, लोटा, सुबह, हाथ, पाँव, घर, मैदान।

(ii) पारिभाषिक शब्द – जो शब्द ज्ञान – विज्ञान अथवा विभिन्न व्यवसायों में विशेष अर्थों में प्रयोग किए जाते हैं, पारिभाषिक शब्द कहलाते हैं।
इन्हें तकनीकी शब्द भी कहते हैं, जैसे- संज्ञा, सर्वनाम, रसायन, समाजशास्त्र, अधीक्षक।

JAC Class 9 Hindi व्याकरण शब्द और पद

प्रश्न 10.
शब्द पद कब बन जाता है ? उदाहरण देकर तर्कसंगत उत्तर दीजिए।
उत्तर :
शब्द को जब व्याकरण के नियमों के अनुसार वाक्य में प्रयोग करते हैं तब वह पद बन जाता है, जैसे- राम, रावण, मारा शब्द हैं। इनसे बना वाक्य – राम ने रावण को मारा- पद है।

JAC Class 12 History Important Questions Chapter 5 यात्रियों के नज़रिए : समाज के बारे में उनकी समझ

Jharkhand Board JAC Class 12 History Important Questions Chapter 5 विचारक, विश्वास और इमारतें : सांस्कृतिक विकास Important Questions and Answers.

JAC Board Class 12 History Important Questions Chapter 5 यात्रियों के नज़रिए : समाज के बारे में उनकी समझ

बहुविकल्पीय प्रश्न (Multiple Choice Questions)

1. इनबतूता का जन्म हुआ था-
(अ) भारत
(ब) ओमान
(स) तुर्की
(द) तैंजियर
उत्तर:
(द) तैंजियर

2. इलबतूता अनेक देशों की यात्रा करने के बाद स्वदेश वापस पहुँचा-
(अ) 1333 ई.
(ब) 1432 ई.
(स) 1354 ई.
(द) 1454 ई.
उत्तर:
(स) 1354 ई.

3. ‘रिहला’ का लेखक कौन था?
(अ) अल-बिरुनी
(ब) हसननिजामी
(स) फिरदौसी
(द) इलबतूता
उत्तर:
(द) इलबतूता

JAC Class 12 History Important Questions Chapter 5 यात्रियों के नज़रिए : समाज के बारे में उनकी समझ

4. फ्रांस्वा बर्नियर कहाँ का निवासी था?
(अ) ब्रिटेन
(ब) फ्रांस
(स) जर्मनी
(द) इटली
उत्तर:
(ब) फ्रांस

5. फ्रांस्वा बर्नियर भारत आया था-
(अ) सोलहवीं सदी
(ब) पन्द्रहवीं सदी
(स) सत्रहवीं सदी
(द) अठारहर्वीं सदी
उत्तर:
(स) सत्रहवीं सदी

6. इन्नबतूता के पाठक किससे पूरी तरह से अपरिचित थे-
(अ) खजूर
(ब) नारियल
(स) केला
(द) अंगूर
उत्तर:
(ब) नारियल

7. इबबतूता ने किस शहर को भारत में सबसे बड़ा बताया है-
(अ) आगरा
(ब) इलाहाबाद
(स) जौनपुर
(द) दिल्ली
उत्तर:
(द) दिल्ली

8. इलबतूता भारत की कौनसी प्रणाली की कार्यकुशलता को देखकर आशचर्यचकित हो गया था-
(अ) जल-निकास प्रणाली
(ब) डाक प्रणाली
(स) गुप्तचर प्रणाली
(द) सुरक्ष प्रणाली
उत्तर:
(ब) डाक प्रणाली

JAC Class 12 History Important Questions Chapter 5 यात्रियों के नज़रिए : समाज के बारे में उनकी समझ

9. ‘ट्रैवल्स इन द मुगल एम्पायर’ नामक ग्रन्थ का रचयिता था-
(अ) मनूची
(ब) सर टॉमस रो
(स) बर्नियर
(द) विलियम हॉकिंस
उत्तर:
(स) बर्नियर

10. किताब-उल-हिन्द के लेखक कौन हैं?
(अ) इब्नबतूता
(ब) बर्नियर
(स) अल-बिरुनी
(द) अब्दुर रज्जाक
उत्तर:
(स) अल-बिरुनी

11. ख्वारिज्म में अल-बिरुनी का जन्म हुआ-
(अ) 1071 ई.
(ब) 933 ई.
(स) 1023 ई.
(द) 973 ई.
उत्तर:
(द) 973 ई.

12. अल-बिरुनी किसके साथ भारत आया?
(अ) मोहम्मद गौरी
(ब) मोहम्मद बिन कासिम
(स) महमूद गजनवी
(द) इनमें से कोई नहीं
उत्तर:
(स) महमूद गजनवी

13. मोहम्मद तुगलक के कहने पर इबबतूता किस देश की यात्रा पर गया?
(अ) अफगानिस्तान
(ब) रूस
(स) नेपाल
(द) चीन
उत्तर:
(द) चीन

14. बर्नियर पेशे से क्या थे?
(अ) तोपची
(ब) चिकित्सक
(स) सुनार
(द) वैज्ञानिक
उत्तर:
(ब) चिकित्सक

JAC Class 12 History Important Questions Chapter 5 यात्रियों के नज़रिए : समाज के बारे में उनकी समझ

15. इब्नबतूता ने अपना यात्रा-वृत्तान्त किस भाषा में लिखा?
(अ) अरबी
(ब) फारसी
(स) हित्रू
(द) उर्दू
उत्तर:
(अ) अरबी

रिक्त स्थानों की पूर्ति कीजिए :
1. यूक्लिड यूनानी ……………..
2. ख्वारिज्म ……………… में स्थित है।
3. सुल्तान महमूद ने ख्वारिज्म पर आक्रमण ……………. ई. में किया।
4. इनबतूता ……………. में स्थलमार्ग से …………. पहुँचा।
5. भारत में पुर्तगालियों का आगमन लगभग ……………….. ई. में हुआ।
6. बर्नियर ने अपनी प्रमुख कृति को फ्रांस के शासक ……………. को समर्पित किया था।
उत्तर:
1. गणितज्ञ
2 . उज्बेकिस्तान
3.1017
4. मध्य एशिया, सिन्ध
5. 1500
6. लुई

अतिलघूत्तरात्मक प्रश्न

प्रश्न 1.
फ्रांस्वा बर्नियर कौन था?
उत्तर:
फ्रांस्वा बर्नियर एक चिकित्सक, राजनीतिक, दार्शानिक और इतिहासकार था।

प्रश्न 2.
अल-बिरुनी का जन्म कब और कहाँ हुआ था?
उत्तर:
अल-बिरुनी का जन्म ख्वारिज्म में सन् 973 में हुआ था।

प्रश्न 3.
15 वीं सदी में भारत की यात्रा करने वाले फारस के दूत का क्या नाम था?
उत्तर:
अब्दुर रज्जाक।

प्रश्न 4.
दसवीं शताब्दी से सत्रहवीं सदी तक भारत की यात्रा करने वाले तीन विदेशी यात्रियों के नाम लिखिए।
(1) अल बिरूनी
(2) इब्नबतूता
(3) फ्रांस्वा
उत्तर:
बर्नियर।

JAC Class 12 History Important Questions Chapter 5 यात्रियों के नज़रिए : समाज के बारे में उनकी समझ

प्रश्न 5.
‘किताब-उल-हिन्द’ का रचयिता कौन था? यह ग्रन्थ किस भाषा में लिखा गया है?
उत्तर:
(1) अल-विरुनी
(2) अरबी भाषा।

प्रश्न 6.
अल-विरुनी द्वारा जिन दो ग्रन्थों का संस्कृत में अनुवाद किया गया, उनके नाम लिखिए।
उत्तर:
(1) पतंजलि का व्याकरण
(2) यूक्लिड के कार्य।

प्रश्न 7.
इब्नबतूता कहाँ का निवासी था?
उत्तर:
इब्नबतूता मोरक्को का निवासी था।

प्रश्न 8.
इब्नबतूता के भारत पहुँचने पर किस सुल्तान ने किस पद पर नियुक्त किया था?
उत्तर:
(1) मुहम्मद बिन तुगलक ने
(2) दिल्ली के काजी (न्यायाधीश) के पद पर

प्रश्न 9.
इब्नबतूता भारत कब पहुँचा और किस मार्ग से पहुंचा?
उत्तर:
इब्नबतूता 1333 ई. में स्थल मार्ग से सिन्ध पहुँचा।

प्रश्न 10.
इब्नबतूता ने अपना यात्रा वृखन्त किस भाषा में लिखा? यह यात्रा वृत्तान्त किस नाम से प्रसिद्ध है?
उत्तर:
(1) अरबी भाषा में
(2) रिहला।

प्रश्न 11.
अब्दुरक समरवन्दी ने भारत में किस भाग की यात्रा की थी और कब?
उत्तर:
1440 के दशक में अब्दुररज्जाक ने दक्षिण भारत की यात्रा की।

प्रश्न 12.
फ्रांस्वा बर्नियर भारत में कितने वर्ष रहा था ?
उत्तर:
फ्रांस्वा बर्नियर 12 वर्ष (1656-1668 ई.) तक भारत में रहा।

प्रश्न 13.
भारतीय समाज को समझने में अल-बिरुनी को कौनसी बाधाओं का सामना करना पड़ा ?
उत्तर:
(1) संस्कृत भाषा की कठिनाई
(2) धार्मिक अवस्था, प्रथाओं में भिन्नता
(3) अभिमान

प्रश्न 14.
इब्नबतूता ने कौनसी दो वानस्पतिक उपजों का रोचक वर्णन किया है, जिनसे उसके पाठक पूरी तरह से अपरिचित थे?
उत्तर:
(1) नारियल
(2) पान।

JAC Class 12 History Important Questions Chapter 5 यात्रियों के नज़रिए : समाज के बारे में उनकी समझ

प्रश्न 15.
इब्नबतूता के अनुसार भारत के दो बड़े शहर कौन से थे?
उत्तर:
(1) दिल्ली
(2) दौलताबाद।

प्रश्न 16.
इब्नबतूता ने भारत की किस प्रणाली की कुशलता का उल्लेख किया है?
उत्तर:
डाक प्रणाली का

प्रश्न 17.
अब्दुररज्जाक ने किस शहर के मन्दिर के शिल्प और कारीगरी को अद्भुत बताया था ?
उत्तर:
मंगलौर शहर से 9 मील के भीतर स्थित मन्दिर

प्रश्न 18.
ऐसे तीन विदेशी यात्रियों के नाम लिखिए जिन्होंने अल बिरूनी और इब्नबतूता के पदचिन्हों का अनुसरण किया।
उत्तर:
(1) अब्दुर रज्जाक
(2) महमूद वली बल्छी
(3) शेख अली हाजिन।

प्रश्न 19.
पेलसर्ट ने भारत की किस सामाजिक समस्या की ओर ध्यान आकृष्ट किया?
उत्तर:
भारत की व्यापक तथा दुःखद गरीबी की समस्या।

प्रश्न 20.
बर्नियर के अनुसार भारत और यूरोप के बीच एक प्रमुख मूल भिन्नता बताइये।
उत्तर:
भारत में निजी भू-स्वामित्व का अभाव।

प्रश्न 21.
अल-विरुनी ने अपनी पुस्तक ‘किताब-उल- हिन्द’ किस भाषा में लिखी?
उत्तर:
अरबी में।

प्रश्न 22.
इब्नबतूता का जन्म कहाँ हुआ था?
उत्तर:
तैंजियर में।

प्रश्न 23.
शरिया का क्या अर्थ है?
उत्तर:
इस्लामी कानून।

प्रश्न 24.
इब्नबतूता ने भारत के लिए कब प्रस्थान
उत्तर:
1332-33 ई. में।

प्रश्न 25.
इब्नबतूता अपने देश वापस कब पहुँचा ?
उत्तर:
1354 ई. में

प्रश्न 26.
1600 ई. के बाद भारत आने वाले दो यूरोपीय यात्रियों के नाम लिखिए।
उत्तर:
(1) ज्यॉँ-बैप्टिस्ट तैर्नियर
(2) मनूकी।

JAC Class 12 History Important Questions Chapter 5 यात्रियों के नज़रिए : समाज के बारे में उनकी समझ

प्रश्न 27.
बर्नियर का यात्रा वृत्तान्त कहाँ और कब प्रकाशित हुआ?
उत्तर:
फ्रांस में 1670-71 में

प्रश्न 28.
जाति व्यवस्था के सम्बन्ध में ब्राह्मणवादी व्याख्या को मानने के बावजूद अल बिरूनी ने किस मान्यता को अस्वीकार किया ?
उत्तर:
अपवित्रता की मान्यता।

प्रश्न 29.
अल बिरूनी ने भारत में प्रचलित वर्ण- व्यवस्था के अन्तर्गत किन चार प्रमुख वर्णों का उल्लेख किया है?
उत्तर:
(1) ब्राह्मण
(2) क्षत्रिय
(3) वैश्य
(4) शूद्र।

प्रश्न 30.
जातिव्यवस्था के विषय में अल-विरुनी का विवरण किन ग्रन्थों पर आधारित था?
उत्तर:
संस्कृत ग्रन्थों पर।

प्रश्न 31.
इब्नबतूता के अनुसार दिल्ली शहर में कितने दरवाजे थे? इनमें से सबसे विशाल दरवाजा कौनसा था ?
उत्तर:
(1) 28 दरवाजे
(2) बदायूँ दरवाजा।

प्रश्न 32.
अब्दुररज्जाक ने किसे ‘विचित्र देश’ बताया था?
उत्तर:
कालीकट बन्दरगाह पर बसे हुए लोगों को।

प्रश्न 33.
बर्नियर द्वारा रचित ग्रन्थ ‘ट्रेवल्स इन द मुगल एम्पायर’ की अपनी किन विशेषताओं के लिए विख्यात है?
अथवा
‘ट्रेवल्स इन द मुगल एम्पायर’ क्या है?
उत्तर:
(1) गहन चिन्तन
(2) गहन प्रेक्षण
(3) आलोचनात्मक अन्तर्दृष्टि

प्रश्न 34.
बर्नियर के अनुसार सत्रहवीं शताब्दी में भारत में जनसंख्या का कितने प्रतिशत भाग नगरों में रहता था?
उत्तर:
लगभग पन्द्रह प्रतिशत।

प्रश्न 35.
इब्नबतूता के अनुसार सुल्तान मुहम्मद बिन तुगलक अमीरों की गतिविधियों की जानकारी प्राप्त करने के लिए किन्हें नियुक्त करता था?
उत्तर:
दासियों को।

प्रश्न 36.
अल बिरूनी द्वारा अपनी कृतियों में जिस विशिष्ट शैली का प्रयोग किया गया, उसे स्पष्ट कीजिये।
उत्तर:
आरम्भ में एक प्रश्न, फिर संस्कृत परम्पराओं पर आधारित वर्णन।

JAC Class 12 History Important Questions Chapter 5 यात्रियों के नज़रिए : समाज के बारे में उनकी समझ

प्रश्न 37.
वर्नियर के विवरणों ने किन दो पश्चिमी विचारकों को प्रभावित किया?
उत्तर:
(1) मॉन्टेस्क्यू
(2) कार्ल मार्क्स।

प्रश्न 38.
वर्नियर ने मुगलकालीन नगरों को क्या कहा है?
अथवा
बर्नियर भारतीय नगरों को किस रूप में देखता है?
उत्तर:
बर्नियर ने मुगलकालीन नगरों को ‘शिविर नगर कहा है।

प्रश्न 39.
बर्नियर ने मुगलकालीन नगरों को शिविर नगर क्यों कहा है?
उत्तर:
क्योंकि ये नगर राजकीय शिविर पर निर्भर थे।

प्रश्न 40.
मुगलकालीन भारत में कौन-कौन से प्रकार के नगर अस्तित्व में थे?
उत्तर:
मुगलकालीन भारत में उत्पादन केन्द्र, व्यापारिक नगर, बन्दरगाह नगर, धार्मिक केन्द्र तीर्थ स्थान आदि नगर अस्तित्व में थे।

प्रश्न 41.
इब्नबतूता के अनुसार भारत में कितने प्रकार की डाक व्यवस्था प्रचलित थी?
उत्तर:
दो प्रकार की डाक व्यवस्था –
(1) अश्व डाक व्यवस्था (उलुक) तथा
(2) पैदल डाक व्यवस्था (दावा)।

प्रश्न 42.
इब्नबतूता के अनुसार ‘ताराबबाद’ क्या था?
उत्तर:
दौलताबाद में पुरुष और महिला गायकों के लिए एक बाजार था, जिसे ‘तारावबाद’ कहते थे।

प्रश्न 43.
इब्नबतूता के अनुसार भारत का सबसे बड़ा शहर कौनसा था ?
उत्तर:
इब्नबतूता के अनुसार दिल्ली भारत का सबसे बड़ा शहर था।

प्रश्न 44.
अल-विरुनी के अनुसार भारत में वर्ण- व्यवस्था का उद्भव किस प्रकार से हुआ?
उत्तर:
अल बिरुनी के अनुसार ब्राह्मण ब्रह्मन् के सिर से, क्षत्रिय कन्धों और हाथों से वैश्य जंघाओं से तथा शूद्र चरणों से उत्पन्न हुए।

प्रश्न 45.
अल बिरुनी किन भाषाओं का ज्ञाता था?
उत्तर:
अल बिरूनी संस्कृत, सीरियाई, फारसी, हिब्रू नामक भाषाओं का ज्ञाता था।

प्रश्न 46.
अल बिरुनी तथा इब्नबतूता किन देशों से और कब भारत आए ?
उत्तर:
अल बिरुनी 11वीं शताब्दी में उज्बेकिस्तान तथा इब्नबतूता 14वीं शताब्दी में मोरक्को से भारत आए थे।

JAC Class 12 History Important Questions Chapter 5 यात्रियों के नज़रिए : समाज के बारे में उनकी समझ

प्रश्न 47.
अल बिरूनी ने अपनी पुस्तक ‘किताब- उल-हिन्द’ में किन विषयों का विवेचन किया है?
अथवा
‘किताब-उल-हिन्द’ पर सक्षिप्त टिप्पणी लिखिए।
अथवा
‘किताब-उल-हिन्द’ क्या है?
उत्तर:
धर्म और दर्शन, त्यौहारों, खगोल विज्ञान, कीमिया, रीति-रिवाजों तथा प्रथाओं, सामाजिक जीवन, माप- तौल, मूर्तिकला, कानून तथा मापतन्त्र विज्ञान ।

प्रश्न 48.
इब्नबतूता के अनुसार किन देशों में किस भारतीय माल की अत्यधिक मांग थी?
उत्तर:
मध्य एशिया तथा दक्षिण-पूर्व एशिया में भारत के सूती कपड़े, महीन मलमल, रेशम, जरी तथा सदन की अत्यधिक मांग थी।

प्रश्न 49.
किस डच यात्री ने भारतीय उपमहाद्वीप की यात्रा की थी और कब?
उत्तर:
(1) पेलसर्ट
(2) सत्रहवीं शताब्दी के आरम्भिक दशकों में

प्रश्न 50.
फ्रांस्वा बर्नियर के अनुसार भारत में किस स्थिति के लोग नहीं थे?
उत्तर;
फ्रांस्वा बर्नियर के अनुसार भारत में मध्य की स्थिति के लोग नहीं थे।

प्रश्न 51.
बर्नियर ने भारत के किस नगर अल्पवयस्क विधवा को सती होते हुए देखा था? उसे किनकी सहायता से चिता स्थल की ओर ले जाया गया?
उत्तर:
(1) लाहौर में
(2) तीन या चार ब्राह्मणाँ तथा एक वृद्ध महिला की सहायता से।

प्रश्न 52.
बर्नियर के अनुसार कौनसे शिल्प भारत में प्रचलित थे?
उत्तर:
गलीचे बनाना, जरी कसीदाकारी कढ़ाई, सोने और चाँदी के वस्त्रों, रेशमी तथा सूती वस्त्रों का निर्माण।

प्रश्न 53.
अल बिरूनी के अनुसार फारस में समाज किन चार वर्गों में विभाजित था?
उत्तर:

  • घुड़सवार और शासक वर्ग
  • भिक्षु, आनुष्ठानिक पुरोहित तथा चिकित्सक
  • खगोलशास्वी तथा अन्य वैज्ञानिक
  • कृषक तथा शिल्पकार।

प्रश्न 54.
दुआ बरबोसा कौन था?
उत्तर:
दुआर्ते बरबोसा एक प्रसिद्ध यूरोपीय लेखक था जिसने दक्षिण भारत में व्यापार और समाज का एक विस्तृत विवरण लिखा ।

प्रश्न 55.
यूरोप के दो यात्रियों के नाम लिखिए जिन्होंने भारतीय कृषकों की गरीबी का वर्णन किया है।
उत्तर:
(1) पेलसर्ट
(2) बर्नियर

JAC Class 12 History Important Questions Chapter 5 यात्रियों के नज़रिए : समाज के बारे में उनकी समझ

प्रश्न 56.
इब्नबतूता भारतीय डाक प्रणाली की कार्यकुशलता देखकर क्यों चकित हुआ? उल्लेख कीजिए।
उत्तर:
डाक व्यवस्था की कार्यकुशलता के कारण व्यापारियों के लिए न केवल लम्बी दूरी तक सूचना और उधार भेजना सम्भव हुआ, बल्कि अल्प सूचना पर माल भेजना भी सम्भव हो गया।

प्रश्न 57.
इब्नबतूता ने मोरक्को जाने से पूर्व किन देशों की यात्रा की थी?
उत्तर:
उत्तरी अफ्रीका, पश्चिम एशिया, मध्य एशिया कुछ भागों, भारतीय उपमहाद्वीप तथा चीन

प्रश्न 58.
1400 से 1800 के बीच भारत की यात्रा करने वाले विदेशी यात्रियों के नाम लिखिए जिन्होंने फारसी में अपने यात्रा-वृत्तान्त लिखे।
उत्तर:
अब्दुररजाक समरकंदी, महमूद वली बल्खी, शेख अली हाजिन

प्रश्न 59.
इब्नबतूता के अनुसार भारत की डाक- प्रणाली क्यों लाभप्रद थी?
उत्तर:
डाक प्रणाली से व्यापारियों के लिए लम्बी दूरी तक सूचना भेजना, उधार भेजना और अल्प सूचना पर माल भेजना सम्भव हो गया।

प्रश्न 60.
पेलसर्ट कौन था?
उत्तर:
पेलसर्ट एक डच यात्री था जिसने सत्रहवीं शताब्दी में भारत की यात्रा की थी।

प्रश्न 61.
बर्नियर के अनुसार मुगल साम्राज्य के स्वरूप की दो त्रुटियों का उल्लेख कीजिए।
उत्तर:
(1) मुगल साइट भिखारियों’ और ‘क्रूर लोगों’ का राजा था
(2) इसके शहर विनष्ट तथा खराब हवा से दूषित थे।

प्रश्न 62.
बर्नियर ने किस जटिल सामाजिक सच्चाई का उल्लेख किया है?
उत्तर:
(1) सम्पूर्ण विश्व से बड़ी मात्रा में बहुमूल्य धातुओं का भारत में आना
(2) भारत में एक समृद्ध व्यापारिक समुदाय का अस्तित्व।

प्रश्न 63.
बर्नियर ने भारत की कृषि की किन दो विशेषताओं का उल्लेख किया है?
उत्तर:
(1) देश के विस्तृत भू-भाग का अधिकांश भाग अत्यधिक उपजाऊ था
(2) भूमि पर खेती अच्छी होती थी।

प्रश्न 64.
बर्नियर ने मुगलकालीन नगरों को किसकी संज्ञा दी है और क्यों?
उत्तर:
(1) शिविर नगर
(2) क्योंकि ये नगर अपने अस्तित्व के लिए राजकीय शिविर पर निर्भर थे।

प्रश्न 65.
बर्नियर के अनुसार पश्चिमी भारत में व्यापारियों के समूह क्या कहलाते थे? उनके मुखिया को क्या कहते घे?
उत्तर:
(1) पश्चिमी भारत में व्यापारियों के समूह महाजन कहलाते थे।
(2) उनके मुखिया सेठ कहलाते थे।

प्रश्न 66.
बर्नियर के अनुसार अन्य शहरी समूहों में व्यावसायिक वर्ग में कौन-कौन लोग सम्मिलित थे?
उत्तर:

  1. चिकित्सक
  2. अध्यापक
  3. अधिवक्ता
  4. चित्रकार
  5. वास्तुविद
  6. संगीतकार
  7. सुलेखक।

प्रश्न 67.
इब्नबतूता के अनुसार दासों की सेवाओं को विशेष रूप से किस कार्य में उपयोग किया जाता था?
उत्तर:
दास पालकी या डोले में पुरुषों और महिलाओं को ले जाने का कार्य करते थे।

प्रश्न 68.
इब्नबतूता के अनुसार अधिकांश दासियाँ अ किस प्रकार प्राप्त की जाती थीं?
उत्तर:
अधिकांश दासियों को आक्रमणों और अभियानों के दौरान बलपूर्वक प्राप्त किया जाता था

JAC Class 12 History Important Questions Chapter 5 यात्रियों के नज़रिए : समाज के बारे में उनकी समझ

प्रश्न 69.
विदेशी यात्री अब्दुरज्जाक ने कालीकट बन्दरगाह पर बसे हुए लोगों को क्या बताया था?
उत्तर:
अब्दुरम्नांक ने कालीकट बन्दरगाह पर बसे हुए लोगों को एक विचित्र देश’ बताया था।

प्रश्न 70.
“कृषकों को इतना निचोड़ा जाता है कि पेट भरने के लिए उनके पास सूखी रोटी भी मुश्किल से बचती है।” यह कथन किसका है?
उत्तर:
यह कथन पेलसर्ट नामक एक डच यात्री का

प्रश्न 71.
बर्नियर ने भारत में पाई जाने वाली सती प्रथा का विवरण क्यों दिया?
उत्तर:
क्योंकि महिलाओं से किया जाने वाला बर्ताव प्रायः पश्चिमी तथा पूर्वी समाजों के बीच भिन्नता का प्रतीक माना जाता था।

प्रश्न 72.
मुहम्मद बिन तुगलक के दूत के रूप में किस विदेशी यात्री को मंगोल शासक के पास चीन जाने का आदेश दिया गया और कब दिया गया?
उत्तर:
(1) इब्नबतूता को
(2) 1342 ई. में

प्रश्न 73.
अल बिरूनी ने संस्कृत भाषा की किन विशेषताओं का उल्लेख किया।
उत्तर:
(1) शब्दों तथा विभक्तियों दोनों में संस्कृति की पहुँच विस्तृत है।
(2) एक ही वस्तु के लिए कई शब्द प्रयुक्त होते हैं।

प्रश्न 74.
बर्नियर ने अपने वृतान्त में भारत को किसके रूप में दिखाया है?
उत्तर:
बर्नियर ने भारत को यूरोप के प्रतिलोम के रूप में अथवा फिर यूरोप का विपरीत जैसा दिखाया है।

प्रश्न 75.
दासों को सामान्यतः किस कार्य के लिए प्रयुक्त किया जाता था?
उत्तर:
दासों को सामान्यतः घरेलू श्रम के लिए ही प्रयुक्त किया जाता था।

प्रश्न 76.
बर्नियर ने सती प्रथा के बारे में क्या लिखा है?
उत्तर:
कुछ महिलाएँ प्रसन्नतापूर्वक मृत्यु को गले लगा लेती थीं, अन्यों को मरने के लिए बाध्य किया जाता था।

प्रश्न 77.
बर्नियर के अनुसार ‘शिविर नगर’ क्या थे?
उत्तर:
बर्नियर के अनुसार ‘शिविर नगर’ वे थे जो अपने अस्तित्व और बने रहने के लिए राजकीय शिविर पर निर्भर थे।

JAC Class 12 History Important Questions Chapter 5 यात्रियों के नज़रिए : समाज के बारे में उनकी समझ

प्रश्न 78.
बर्नियर के अनुसार भारत और यूरोप के बीच मूल भिन्नता क्या थी?
उत्तर:
बर्नियर के अनुसार भारत और यूरोप के बीच मूल भिन्नता भारत में निजी भू-स्वामित्व का अभाव था।

प्रश्न 79.
किस सुल्तान ने नसीरुद्दीन नामक धर्मोपदेशक से प्रसन्न होकर उसे एक लाख के तथा दो सौ दाम दिये ?
उत्तर:
मुहम्मद बिन तुगलक ने।

प्रश्न 80.
इब्नबतूता ने भारत की किन बातों का विशेष रूप से वर्णन किया है?
उत्तर:
इब्नबतूता ने डाक व्यवस्था, पान तथा नारियल का विशेष रूप से वर्णन अपने ग्रन्थ ‘रिहला’ में किया है।

प्रश्न 81.
इब्नबतूता ने भारत के किस शहर को सबसे बड़ा कहा है?
उत्तर:
दिल्ली।

प्रश्न 82.
बर्नियर के ग्रन्थ का क्या नाम है?
उत्तर:
ट्रेवल्स इन द मुगल एम्पावर।

प्रश्न 83.
ताराबबाद किसे कहा जाता है?
उत्तर:
दौलताबाद में पुरुष तथा महिला गायकों के लिए बाजार होता था; जिसे तारावबाद कहा जाता था।

प्रश्न 84.
बर्नियर ने मुगल सेना के साथ कहाँ की न यात्रा की थी?
उत्तर:
कश्मीर

प्रश्न 85.
बर्नियर जब भारत आया उस समय यूरोप में कौनसा युग गतिमान था?
उत्तर:
बर्नियर भारत में सत्रहवीं शताब्दी में आया था, कि उस समय लगभग सम्पूर्ण यूरोप में पुनर्जागरण का काल था।

प्रश्न 86.
किस यात्री ने सुल्तान मुहम्मद तुगलक को भेंट में देने के लिए घोड़े, अँट तथा दास खरीदे ?
उत्तर:
मोरक्को निवासी इब्नबतूता ने।

प्रश्न 87.
यह तर्क किसने दिया कि भारत में ही उपनिवेशवाद से पहले अधिशेष का अधिग्रहण राज्य द्वारा होता था?
उत्तर:
कार्ल मार्क्स।

लघुत्तरात्मक प्रश्न

प्रश्न 1.
लगभग दसवीं सदी से सत्रहवीं सदी तक के काल में लोगों के यात्राएँ करने के क्या उद्देश्य थे?
उत्तर:
लगभग दसवीं सदी से सत्रहवीं सदी तक के काल में महिलाओं और पुरुषों के यात्राएं करने के निम्नलिखित उद्देश्य थे –

  1. कार्य की तलाश में
  2. आपदाओं से बचाव के लिए
  3. व्यापारियों सैनिकों, पुरोहितों और तीर्थयात्राओं के रूप में
  4. साहस की भावना से प्रेरित होकर।

प्रश्न 2.
विदेशी यात्रियों की कौनसी बात उनके यात्रा-वृत्तान्तों को अधिक रोचक बनाती है?
उत्तर:
पूर्ण रूप से भिन्न सामाजिक तथा सांस्कृतिक पृष्ठभूमि से आने के कारण ये विदेशी यात्री दैनिक गतिविधियों तथा प्रथाओं के प्रति अधिक सावधान रहते थे। देशज लेखकों के लिए ये सभी विषय सामान्य थे, जो वृत्तान्तों में उल्लिखित करने योग्य नहीं थे दृष्टिकोण में यही भिन्नता ही उनके यात्रा वृत्तान्तों को अधिक रोचक बनाती है।

JAC Class 12 History Important Questions Chapter 5 यात्रियों के नज़रिए : समाज के बारे में उनकी समझ

प्रश्न 3.
अल-बिरुनी के यात्रा-वृत्तान्त लिखने के उद्देश्य लिखिए।
उत्तर:
अल- विरुनी के यात्रा-वृत्तान्त लिखने के निम्नलिखित उद्देश्य थे –
(1) उन लोगों की सहायता करना जो हिन्दुओं से धार्मिक विषयों पर चर्चा करना चाहते थे।
(2) ऐसे लोगों के लिए सूचना का हिन्दुओं के साथ सम्बद्ध होना चाहते थे।

प्रश्न 4.
अल-विरुनी ग्रन्थों का अनुवाद करने में क्यों सक्षम था? स्पष्ट कीजिए उसके द्वारा अनुवादित ग्रन्थों के नाम लिखिए।
उत्तर:
अल बिरूनी कई भाषाओं में दक्ष था जिनमें सीरियाई, फारसी, हिब्रू तथा संस्कृत शामिल हैं। इसलिए वह भाषाओं की तुलना तथा ग्रन्थों का अनुवाद करने में सक्षम रहा। उसने अनेक संस्कृत ग्रन्थों का अरबी में अनुवाद किया। संग्रह करना जो उसने पतंजलि के व्याकरण ग्रन्थ का भी अरबी भाषा में अनुवाद किया। उसने अपने ब्राह्मण मित्रों के लिए यूनानी गणित यूक्लिड के कार्यों का संस्कृत में अनुवाद किया।

प्रश्न 5.
किताब-उल-हिन्द’ के बारे में आप क्या जानते हैं ?
उत्तर:
अल बिरूनी ने अरबी भाषा में अपनी पुस्तक ‘किताब-उल-हिन्द’ लिखी। इसकी भाषा सरल और स्पष्ट है। यह एक विस्तृत ग्रन्थ है जो अस्सी अध्यायों में विभाजित है। इस ग्रन्थ में भारतीय धर्म और दर्शन त्योहारों, खगोल- विज्ञान, कीमिया, रीति-रिवाजों तथा प्रथाओं, सामाजिक- जीवन, भार-तौल तथा मापन विधियों, मूर्तिकला, कानून, मापतन्त्र विज्ञान आदि विषयों का विवेचन किया गया है।

प्रश्न 6.
इब्नबतूता अकेला ही विश्व यात्रा पर क्यों निकल पड़ा? उस समय उसकी क्या आयु थी? वह अपने घर वापस कब पहुँचा ?
उत्तर:
इब्नबतूता के वृत्तान्त से ज्ञात होता है कि वह अपने जन्म स्थान जियर से अकेला ही अपनी यात्रा पर निकल पड़ा। उसके मन में लम्बे समय से प्रसिद्ध पुण्य स्थानों को देखने की तीव्र इच्छा थी इसलिए उसने किसी कारणों में शामिल होने की प्रतीक्षा नहीं की और अकेला ही घर से निकल पड़ा। उस समय इब्नबतूता की आयु बाईस वर्ष थी। वह 1354 में अपने घर वापस पहुँच गया।

प्रश्न 7.
अल बिरूनी को भारत का यात्रा-वृत्तान्त लिखने में किन कठिनाइयों का सामना करना पड़ा?
उत्तर:
(1) अल बिरूनी के अनुसार पहली कठिनाई भाषा थी। उसके अनुसार संस्कृत, अरबी और फारसी से इतनी भिन्न थी कि विचारों और सिद्धान्तों को एक ही भाषा से दूसरी में अनुवादित करना सरल नहीं था।
(2) दूसरी कठिनाई धार्मिक अवस्था और प्रथाओं में भिन्नता थी उसे इन्हें समझने के लिए वेदों, पुराणों आदि की सहायता लेनी पड़ी।
(3) अल-बिरुनी के अनुसार तौसरी कठिनाई भारतीयों का जातीय अभिमान था।

प्रश्न 8.
अपनी श्रेणी के अन्य यात्रियों से इब्नबतूता किन बातों में अलग था?
उत्तर:
इब्नबतूता पुस्तकों के स्थान पर यात्राओं से प्राप्त अनुभव को अपनी जानकारी का अधिक महत्त्वपूर्ण स्त्रोत मानता था। उसे यात्राएँ करने का बड़ा शौक था और उसने नये-नये देशों तथा लोगों के विषय में जानने के लिए | दूर-दूर के क्षेत्रों तक की यात्रा की 1332-33 ई. में भारत के लिए प्रस्थान करने से पूर्व वह मक्का, सीरिया, इराक, फारस, यमन, ओमान तथा पूर्वी अफ्रीका के कई तटीय व्यापारिक बन्दरगाहों की यात्राएँ कर चुका था।

प्रश्न 9.
इब्नबतूता के तत्कालीन सुल्तान मुहम्मद- बिन तुगलक के साथ सम्बन्धों का वर्णन कीजिए।
उत्तर:
1333 में इब्नबतूता दिल्ली पहुंचा। मुहम्मद- बिन तुगलक इब्नबतूता की विद्वता से बझ प्रभावित हुआ और उसे दिल्ली का काली अथवा न्यायाधीश नियुक्त किया। उसने इस पद पर कई वर्षों तक कार्य किया। कुछ कारणों से सुल्तान इब्नबतूता से नाराज हो गया और उसे कारागार में कैद कर दिया गया। परन्तु कुछ समय बाद सुल्तान की नाराजगी दूर हो गई और उसने 1342 में इब्नबतूता को अपने दूत के रूप में चीन के शासक के पास भेजा।

प्रश्न 10.
“इब्नबतूता एक हठीला यात्री था।” व्याख्या कीजिए।
उत्तर:
इब्नबतूता अपनी सुन का पक्का था उसे यात्राएँ करने का बहुत शौक था। वह लम्बी यात्राओं के दौरान होने वाली कठिनाइयों से हतोत्साहित नहीं होता था। उसने उत्तर-पश्चिमी अफ्रीका में अपने निवास स्थान मोरक्को जाने से पूर्व कई वर्ष उत्तरी अफ्रीका, पश्चिमी एशिया, मध्य एशिया के भागों, भारतीय उपमहाद्वीप तथा चीन की यात्रा की थी। उसके वापिस लौटने पर मोरक्को के शासक ने उसकी कहानियों को दर्ज करने के निर्देश दिए।

JAC Class 12 History Important Questions Chapter 5 यात्रियों के नज़रिए : समाज के बारे में उनकी समझ

प्रश्न 11.
“बर्नियर द्वारा प्रस्तुत ग्रामीण समाज का चित्रण सच्चाई से बहुत दूर था।” स्पष्ट कीजिए।
उत्तर:
बर्नियर द्वारा प्रस्तुत ग्रामीण समाज का चित्रण सच्चाई से बहुत दूर था सोलहवीं तथा सत्रहवीं शताब्दी में ग्रामीण समाज में चारित्रिक रूप से बड़े पैमाने पर सामाजिक और आर्थिक विभेद था। एक ओर बड़े जमींदार थे और दूसरी ओर ‘अस्पृश्य’ भूमि विहीन श्रमिक इन दोनों के बीच में बड़ा किसान था जो किराए के श्रम का प्रयोग करता था और माल उत्पादन में जुटा रहता था कुछ छोटे किसान भी थे, जो बड़ी कठिनाई से गुजरे योग्य उत्पादन कर पाते थे।

प्रश्न 12.
बर्नियर ने शहरी समूहों में किन व्यावसायिक वर्गों का उल्लेख किया है?
उत्तर:
बर्नियर के अनुसार शहरी समूहों में चिकित्सक (हकीम अथवा वैद्य), अध्यापक (पंडित या मुल्ला), अधिवक्ता (वकील), चित्रकार, वास्तुविद्, संगीतकार, सुलेखक आदि व्यावसायिक वर्ग थे। कई लोग राजकीय संरक्षण पर आश्रित थे तथा कई अन्य लोग संरक्षकों या भीड़-भाड़ वाले बाजार में सामान्य लोगों की सेवा द्वारा अपना जीवनयापन करते थे।

प्रश्न 13.
इब्नबतूता के विवरण के अनुसार दासों में काफी विभेद था। स्पष्ट कीजिए।
उत्तर:
इब्नबतूता के विवरण से ज्ञात होता है कि दाखों में काफी विभेद था। सुल्तान की सेवा में कार्यरत कुछ दासियाँ संगीत और गायन में निपुण थीं। इब्नबतूता सुल्तान की बहिन की शादी के अवसर पर उनके प्रदर्शन से बड़ा आनन्दित हुआ। सुल्तान अपने अमीरों की गतिविधियों पर नजर रखने के लिए दासियों को भी नियुक्त करता था। अधिकतर दासों का प्रयोग घरेलू श्रम के लिए किया जाता था घरेलू श्रम करने वाले दासों, दासियों की कीमत बहुत कम होती थी।

प्रश्न 14.
इब्नबतूता के अनुसार सुल्तान किस प्रकार अमीरों पर दासियों द्वारा नजर रखता था?
उत्तर:
इब्नबतूता के अनुसार सुल्तान की यह आदत थी कि हर बड़े या छोटे अमीर के साथ एक दास को रखता जो उनकी मुखबिरी करता था। वह इन अमीरों के घरों में महिला सफाई कर्मचारियों की भी नियुक्ति करता था। दासियों के पास जो भी सूचनाएँ होती थीं, वे इन महिला सफाई कर्मचारियों को दे देती थीं। अधिकांश दासियों को हमलों और अभियानों के दौरान बलपूर्वक प्राप्त किया जाता था।

प्रश्न 15.
अल बिरूनी के प्रारम्भिक जीवन का वर्णन कीजिए।
अथवा
अल-विरुनी के विषय में आप क्या जानते हैं ? उत्तर- अल बिरूनी का जन्म आधुनिक उज्बेकिस्तान में स्थित ख्वारिज्म में सन् 973 में हुआ था। अल बिरुनी ने उच्च कोटि की शिक्षा प्राप्त की। वह सीरियाई, फारसी, हिब्रू, संस्कृत आदि कई भाषाओं का ज्ञाता था। 1017 ई. में महमूद गजनवी ने ख्वारिज्म पर आक्रमण किया और अल बिरूनी सहित यहाँ के कई विद्वानों को अपने साथ अपनी राजधानी गजनी ले गया। उसने अपना शेष जीवन गजनी में ही बिताया। 70 वर्ष की आयु में उसकी मृत्यु हो गई।

प्रश्न 16.
गजनी में रहते हुए अल बिरूनी की भारत के प्रति रुचि कैसे विकसित हुई?
उत्तर:
आठवीं शताब्दी से ही संस्कृत में रचित खगोल- विज्ञान, गणित और चिकित्सा सम्बन्धी कार्यों का अरबी भाषा में अनुवाद होने लगा था। पंजाब के गजनवी साम्राज्य का भाग बन जाने के पश्चात् स्थानीय लोगों से हुए सम्पर्को से आपसी विश्वास और समझ का वातावरण बना। अल- विरुनी ने ब्राह्मण पुरोहितों तथा विद्वानों के साथ कई वर्ष व्यतीत किए और संस्कृत, धर्म तथा दर्शन का ज्ञान प्राप्त किया जिससे अल बिरुनी की भारत के प्रति रुचि विकसित हुई।

प्रश्न 17.
‘हिन्दू’, ‘हिन्दुस्तान’ तथा ‘हिन्दवी’ शब्दों का प्रचलन किस प्रकार हुआ?
उत्तर:
‘हिन्दू’ शब्द लगभग छठी पांचवीं शताब्दी ईसा पूर्व में प्रयुक्त होने वाले एक प्राचीन फारसी शब्द से निकला था, जिसका प्रयोग सिन्धु नदी (Indus) के पूर्व के क्षेत्र के लिए होता था। अरबी लोगों ने इस फारसी शब्द का प्रयोग करना जारी रखा। इस क्षेत्र को ‘अल-हिन्द’ तथा यहाँ के निवासियों को ‘हिन्दी’ कहा। कालान्तर में तुर्की ने सिन्धु से पूर्व में रहने वाले लोगों को ‘हिन्दू’, उनके निवास क्षेत्र को ‘हिन्दुस्तान’ तथा उनकी भाषा को ‘हिन्दवी’ की संज्ञा दी।

JAC Class 12 History Important Questions Chapter 5 यात्रियों के नज़रिए : समाज के बारे में उनकी समझ

प्रश्न 18.
अल बिरुनी के लेखन कार्य की विशेषताओं का विवेचन कीजिए।
उत्तर:

  1. अल बिरुनी ने लेखन में अरबी भाषा का प्रयोग किया था।
  2. अल बिरूनी ने सम्भवतः अपने ग्रन्थ उपमहाद्वीप के सीमान्त क्षेत्रों में रहने वाले लोगों के लिए लिखे थे।
  3. वह संस्कृत, पालि तथा प्राकृत ग्रन्थों के अरबी भाषा में हुए अनुवादों से परिचित था।
  4. इन ग्रन्थों की लेखन सामग्री शैली के विषय में अल बिरूनी का दृष्टिकोण आलोचनात्मक था वह उनमें सुधार करना चाहता था।

प्रश्न 19.
“इब्नबतूता की यात्राएँ कठिन तथा जोखिम भरी हुई थीं।” उदाहरण देते हुए स्पष्ट कीजिए।
उत्तर:
इब्नबतूता के अनुसार चौदहवीं शताब्दी में यात्रा करना अधिक कठिन, जोखिम भरा कार्य और असुरक्षित था इब्नबतूता को कई बार डाकुओं के समूहों के आक्रमणों का सामना करना पड़ा था। फिर भी राजमार्ग असुरक्षित थे। जब वह मुल्तान से दिल्ली की यात्रा कर रहा था, डाकुओं ने उसके कारवाँ पर आक्रमण किया जिसके फलस्वरूप उसके कई साथी यात्री मारे गए। जो यात्री बच गए थे, वे भी बुरी तरह से घायल हो गए थे। इनमें इब्नबतूता भी सम्मिलित था।

प्रश्न 20.
इब्नबतूता के श्रुतलेखों को लिखने के लिए नियुक्त किए गए इब्नजुजाई ने अपनी प्रस्तावना में क्या वर्णन किया है?
उत्तर:
इब्नजुजाई ने अपनी प्रस्तावना में लिखा है कि राजा ने इब्नबतूता को निर्देश दिया कि वह अपनी यात्रा में देखे गए शहरों का तथा रोचक घटनाओं का वृतान्त लिखवाएँ। इसके साथ ही वह विभिन्न देशों के जिन शासकों से मिले, उनके महान साहित्यकारों के तथा उनके धर्मनिष्ठ सन्तों के विषय में भी बताएँ। इस आदेश के अनुसार इब्नबतूता ने इन सभी विषयों पर एक कथानक लिखवाया। इसके अतिरिक्त इब्नबतूता ने कई प्रकार के असाधारण विवरण भी दिए।

प्रश्न 21.
अल बिरूनी और इब्नबतूता के पदचिन्हों का अनुसरण करने वाले यात्रियों का वर्णन कीजिए।
उत्तर:
1400 से 1800 के बीच भारत आने वाले अनेक यात्रियों ने अल बिरूनी और इब्नबतूता के पदचिन्हों का अनुसरण किया। इनमें अब्दुररज्जाक समरकंदी, महमूद वली बल्खी तथा शेख अली हानि उल्लेखनीय हैं। अब्दुरस्ज्जाक ने 1440 के दशक में दक्षिण भारत की यात्रा की तथा महमूद वली बल्खी ने 1620 के दशक में व्यापक रूप से यात्राएं की थीं। शेख अली हाजिन ने 1740 के दशक में उत्तर भारत की यात्रा की थी।

प्रश्न 22.
बर्नियर ने भारत में जो देखा, उसकी तुलना यूरोप से की। इसका मूल्यांकन कीजिए।
अथवा
बर्नियर द्वारा दी गई पूर्व और पश्चिम की तुलना का वर्णन कीजिए।
अथवा
“बर्नियर प्रायः भारत में जो देखता था, उसकी तुलना यूरोपीय स्थिति से ही करता था।” स्पष्ट कीजिए।
उत्तर:
बर्नियर प्राय: भारत में जो देखता था, उसकी तुलना यूरोपीय स्थिति से करता था वह यूरोप की सर्वश्रेष्ठता प्रतिपादित करना चाहता था यूरोपीय स्थितियों के मुकाबले में वह भारत की स्थितियों को दयनीय दर्शाना चाहता था। यही कारण है कि लगभग प्रत्येक दृष्टान्त में बर्नियर ने भारत की स्थिति को यूरोप में हुए विकास की तुलना में दयनीय बताया। यद्यपि उसका आकलन हम्मेसटीक नहीं था, फिर भी जब उसके कार्य प्रकाशित हुए). (बर्नियर के वृत्तांत अत्यधिक प्रसिद्ध हुए।

प्रश्न 23.
बर्नियर द्वारा मुगल सेना के कश्मीर कूच का वर्णन कीजिए। वह अपने साथ कौनसी वस्तुएँ ले गया था? उससे क्या अपेक्षा की जाती थी?
उत्तर:
बर्नियर मुगल सेना के कश्मीर कूच के सम्बन्ध में लिखता है कि इस देश की प्रथा के अनुसार उससे दो अच्छे तुर्कमान घोड़े देखने की अपेक्षा की जाती थी। वह अपने साथ एक शक्तिशाली पारसी ऊँट तथा चालक, अपने घोड़ों के लिए एक साईस, एक खानसामा तथा एक सेवक भी रखता था। उसे एक तम्बू एक दरी, एक छोटा बिस्तर, एक तकिया, एक विछौना, चमड़े के मेजपोश कुछ अंगोछे, झोले, जाल आदि वस्तुएँ दी गई थीं उसने चावल, मीठी रोटी, नींबू, चीनी आदि वस्तुएँ अपने साथ रखी थीं।

प्रश्न 24.
संस्कृत भाषा के विषय में अल बिरूनी के विचार व्यक्त कीजिए।
उत्तर:
अल बिरूनी के अनुसार संस्कृत भाषा को सीखना एक कठिन कार्य है क्योंकि अरबी भाषा की भाँति ही, शब्दों तथा विभक्तियों, दोनों में ही संस्कृत भाषा की पहुँच बहुत विस्तृत है। इसमें एक ही वस्तु के लिए कई शब्द, मूल तथा व्युत्पन्न दोनों प्रयुक्त होते हैं। इसमें एक ही शब्द का प्रयोग कई वस्तुओं के लिए होता है, जिन्हें अच्छी तरह से समझने के लिए विभिन्न विशेषक संकेत पदों के माध्यम से एक-दूसरे से पृथक किया जाना आवश्यक है।

प्रश्न 25.
अल बिरूनी द्वारा वर्णित फारस के चार सामाजिक वर्गों का उल्लेख कीजिए।
उत्तर:
अल बिरूनी के अनुसार प्राचीन फारस में समाज चार वर्गों में विभाजित था ये चार वर्ग थे –
(1) घुड़सवार और शासक वर्ग
(2) भिक्षु, आनुष्ठानिक पुरोहित
(3) चिकित्सक, खगोलशास्त्री तथा अन्य वैज्ञानिक और
(4) कृषक तथा शिल्पकार अल बिरुनी यह दर्शाना चाहता था कि ये सामाजिक वर्ग केवल भारत तक ही सीमित नहीं थे, बल्कि ये अन्य देशों में भी थे।

JAC Class 12 History Important Questions Chapter 5 यात्रियों के नज़रिए : समाज के बारे में उनकी समझ

प्रश्न 26.
“ जाति-व्यवस्था के सम्बन्ध में ब्राह्मणवादी व्याख्या को स्वीकार करने के बावजूद, अल बिरूनी ने अपवित्रता की मान्यता को अस्वीकार कर दिया।” स्पष्ट कीजिए।
उत्तर:
अल बिरूनी ने लिखा है कि प्रत्येक वह वस्तु जो अपवित्र हो जाती है, अपनी पवित्रता की मूल स्थिति को पुनः प्राप्त करने का प्रयास करती है और सफल होती है। सूर्य वायु को स्वच्छ करता है और समुद्र में नमक पानी को गंदा होने से बचाता है अल बिरुनी जोर देकर कहता है कि यदि ऐसा नहीं होता, तो पृथ्वी पर जीवन असम्भव हो जाता। उसके अनुसार जाति-व्यवस्था में शामिल अपवित्रता की अवधारणा प्रकृति के नियमों के विरुद्ध थी।

प्रश्न 27.
” जाति व्यवस्था के विषय में अल-बिरुनी का विवरण संस्कृत ग्रन्थों पर आधारित था।” स्पष्ट कीजिए।
उत्तर:
जाति व्यवस्था के विषय में अल बिरुनी का विवरण संस्कृत ग्रन्थों के अध्ययन से पूर्ण रूप से प्रभावित था। इन ग्रन्थों में ब्राह्मणों के दृष्टिकोण से जाति-व्यवस्था को संचालित करने वाले नियमों का प्रतिपादन किया गया था परन्तु वास्तविक जीवन में यह व्यवस्था इतनी कठोर नहीं थी। उदाहरणार्थ, अन्त्यजों (जाति-व्यवस्था से परे रहने वाले लोग) से प्रायः यह अपेक्षा की जाती थी कि वे किसानों और जमींदारों के लिए सस्ता श्रम प्रदान करें ये आर्थिक तन्त्र में सम्मिलित थे।

प्रश्न 28.
अल बिरूनी द्वारा उल्लिखित भारत की वर्ण-व्यवस्था का वर्णन कीजिए।
उत्तर:

  1. ब्राह्मण-ब्राह्मणों की जाति सबसे ऊँची थी। ब्राह्मण ब्रह्मन् के सिर से उत्पन्न हुए थे। इन्हें सबसे उत्तम माना जाता था।
  2. क्षत्रिय ब्राह्मणों के बाद दूसरी जाति क्षत्रियों की थी जिनका जन्म ब्रह्मन् के कन्धों और हाथों से हुआ • था। उनका दर्जा ब्राह्मणों से अधिक नीचा नहीं था।
  3. वैश्य क्षत्रियों के बाद वैश्य आते हैं। इनका जन्म ब्रह्मन् की जंघाओं से हुआ था।
  4. शूद्र इनका उद्भव ब्रह्मन् के चरणों से हुआ था।

प्रश्न 29.
“इब्नबतूता में अनजाने को जानने की लालसा कूट-कूटकर भरी हुई थी।” स्पष्ट कीजिए । अपरिचित को रेखांकित करने का उसका क्या उद्देश्य था?
उत्तर:
इब्नबतूता ने भारत में व्यापक यात्राएँ कीं और अपरिचित को जानने का भरसक प्रयास किया। उसके वृत्तान्तों में धर्मनिष्ठ लोगों, क्रूर और दयालु शासकों, सामान्य पुरुषों तथा महिलाओं और उनके जीवन की कहानियाँ शामिल थीं। इब्नबतूता को इन कहानियों में जो भी कुछ अपरिचित लगा था, उसे उसने विशेष रूप से रेखांकित किया ताकि श्रोता अथवा पाठक सुदूर देशों के वृत्तान्तों से पूर्ण रूप से प्रभावित हो सकें।

प्रश्न 30.
इब्नबतूता ने नारियल का वर्णन किस प्रकार किया है?.
उत्तर:
इब्नबतूता के अनुसार नारियल के वृक्ष स्वरूप में सबसे अनोखे और प्रकृति में सबसे आश्चर्यजनक वृक्षों में से हैं। ये बिल्कुल खजूर के वृक्ष जैसे दिखते हैं। इनमें केवल एक अन्तर है कि नारियल से काष्ठफल प्राप्त होता तथा दूसरे से खजूर नारियल मानव सिर से मेल खाता हैं क्योंकि इसमें भी दो आँखें तथा एक मुख है और अन्दर का भाग हरा होने पर मस्तिष्क जैसा दिखता है। इससे जुड़ा रेशा बालों जैसा दिखाई देता है वे इससे रस्सी बनाते हैं।

प्रश्न 31.
इब्नबतूता द्वारा पान का वर्णन किस प्रकार किया गया है? उसने पान का वर्णन क्यों किया? पान का किस प्रकार प्रयोग किया जाता था?
उत्तर:
इब्नबतूत ने पान का वर्णन इसलिए किया क्योंकि इससे उसके पाठक पूरी तरह से अपरिचित थे। इब्नबतूता के अनुसार पान का कोई फल नहीं होता और इसे केवल इसकी पत्तियों के लिए ही उगाया जाता था पान को अंगूर लता की तरह ही उगाया जाता था। पान के प्रयोग करने की विधि यह थी कि इसे खाने से पहले सुपारी ली जाती थी। इसके छोटे- छोटे टुकड़ों को मुंह में रखकर चबाया जाता था। इसके पश्चात् पान की पत्तियों के साथ इन्हें चबाया जाता था।

प्रश्न 32.
इब्नबतूता ने भारतीय शहरों के सम्बन्ध में जो लिखा है, उस पर प्रकाश डालिए।
अथवा
इब्नबतूता ने भारतीय शहरों का किस प्रकार वर्णन किया है?
उत्तर:
इब्नबतूता के अनुसार भारतीय शहर पनी आबादी वाले तथा समृद्ध थे, परन्तु ये कभी-कभी युद्धों तथा अभियानों में विनष्ट हो जाते थे। अधिकांश शहरों में भीड़-भीड़ वाली सड़कें तथा चमक-दमक वाले और रंगीन बाजार थे जो विविध प्रकार की वस्तुओं से भरे रहते थे। दिल्ली एक बड़ा शहर था जिसकी आबादी बहुत अधिक थी तथा यह भारत में सबसे बड़ा शहर था। दौलताबाद (महाराष्ट्र में ) भी कम नहीं था और आकार में दिल्ली को चुनौती देता था।

प्रश्न 33.
इब्नबतूता के अनुसार भारतीय शहरों के बाजारों की क्या विशेषताएँ थीं?
उत्तर:
इब्नबतूता के अनुसार भारतीय शहरों के बाजार चमक-दमक वाले तथा रंगीन थे जहाँ विविध प्रकार की वस्तुएँ उपलब्ध रहती थीं ये बाजार केवल आर्थिक विनिमय के स्थान ही नहीं थे बल्कि ये सामाजिक तथा आर्थिक गतिविधियों के केन्द्र भी थे। अधिकांश बाजारों में एक मस्जिद तथा एक मन्दिर होता था और उनमें से कम से कम कुछ में तो नर्तकों, संगीतकारों एवं गायकों के सार्वजनिक प्रदर्शन के स्थान भी निर्धारित थे।

JAC Class 12 History Important Questions Chapter 5 यात्रियों के नज़रिए : समाज के बारे में उनकी समझ

प्रश्न 34.
इब्नबतूता द्वारा उल्लिखित दिल्ली ( देहली) शहर की विशेषताओं का वर्णन कीजिए।
उत्तर:
दिल्ली बड़े क्षेत्र में फैला पनी जनसंख्या वाला शहर था। शहर के चारों ओर बनी प्राचीर अतुलनीय थी। इसके अन्दर रात्रि के पहरेदार तथा द्वारपालों के कक्ष थे। प्राचीरों के अन्दर अनेक भंडार गृह बने हुए थे इस शहर के अट्ठाईस द्वार थे जिन्हें ‘दरवाजा’ कहा जाता है। इनमें से ‘बदायूँ दरवाजा’ सबसे विशाल था मांडवी दरवाजे के भीतर एक अनाज मंडी थी तथा गुल दरवाजे की बगल में एक फलों का बगीचा था।

प्रश्न 35.
इब्नबतूता के वृत्तान्त से भारतीय कृषि, व्यापार और वाणिज्य के बारे में क्या जानकारी मिलती है?
उत्तर:
इब्नबतूता के अनुसार भारतीय कृषि के अत्यधिक उत्पादनकारी होने का कारण भूमि का उपजाऊपन था। इस वजह से किसान वर्ष में दो फसलें उगाते थे। भारतीय उपमहाद्वीप व्यापार तथा वाणिज्य के अन्तर एशियाई तन्त्रों से अच्छी तरह से जुड़ा हुआ था भारतीय माल की मध्य तथा दक्षिण-पूर्वी एशिया, दोनों में बहुत मांग थी, जिससे शिल्पकार तथा व्यापारी बहुत लाभ कमाते थे भारतीय सूती कपड़े, महीन मलमल, रेशम, जी तथा साटन की बहुत अधिक माँग थीं।

प्रश्न 36.
इब्नबतूता ने दौलताबाद के संगीत बाजार का क्या वृत्तान्त दिया है? अपने वर्णन में इब्नबतूता ने इन गतिविधियों को उजागर क्यों किया?
उत्तर:
इब्नबतूता के अनुसार दौलताबाद में पुरुष और महिला गायकों के लिए एक बाजार था जिसे ‘ताराववाद’ कहते थे। यहाँ अनेक दुकानें थीं जिन्हें कालीनों से सजाया गया था। दुकान के मध्य में एक झूला था, जिस पर गायिका बैठती थी। बाजार के मध्य में एक विशाल गुम्बद खड़ा था, जिसमें कालीन बिछाए गए थे। इस बाजार में इबादत के लिए मस्जिदें बनी हुई थीं।

प्रश्न 37.
अपने वर्णन में इब्नबतूता ने इन गतिविधियों का उल्लेख क्यों किया?
उत्तर:
हमारे विचार में इब्नबतूता ने अपने विवरण में इन गतिविधियों का वर्णन इसलिए किया था क्योंकि इस प्रकार के संगीत के बाजारों से उसके पाठक अपरिचित थे। अतः उसने इन गतिविधियों का वर्णन किया ताकि उसके पाठक इन वृत्तान्तों से प्रभावित हो सकें।

प्रश्न 38.
अब्दुररज्जाक ने अपने यात्रा-वृत्तान्त में दक्षिण भारत का किस प्रकार वर्णन किया है?
उत्तर:
1440 के दशक में लिखा गया अब्दुरज्जाक का यात्रा-वृत्तान्त संवेगों और अवबोधनों का एक रोचक मिश्रण है। उसने केरल में कालीकट (आधुनिक कोलीकोड) बन्दरगाह पर जो देखा, उसे प्रशंसनीय नहीं माना। उसने लिखा है कि “यहाँ ऐसे लोग बसे हुए थे, जिनकी कल्पना उसने कभी भी नहीं की थी।” इन लोगों को उसने एक ‘विचित्र देश’ बताया।

प्रश्न 39.
अब्दुररज्जाक द्वारा वर्णित मंगलौर के मन्दिर का विवरण प्रस्तुत कीजिए।
उत्तर:
अब्दुररज्याक ने लिखा है कि मंगलौर से 9 मील के भीतर ही उसने एक ऐसा पूजा स्थल देखा जो सम्पूर्ण विश्व में अतुलनीय है। यह वर्गाकार था तथा चार द्वार मंडपों के साथ काँसे से ढका हुआ था प्रवेश-द्वार के द्वार-मंडप में सोने की बनी एक मूर्ति थी जो मानव- आकृति जैसी तथा आदमकद थी इसकी दोनों आँखों में काले रंग के माणिक इतनी चतुराई से लगाए गए थे कि ऐसा लगता था मानो वह देख सकती हों। यह शिल्प और कारीगरी अद्भुत थी।

प्रश्न 40.
इब्नबतूता ने भारत की डाक व्यवस्था को संचार की एक अनूठी प्रणाली क्यों बताया है?
अथवा
मध्यकालीन भारत में डाक प्रणाली पर प्रकाश डालिए।
उत्तर:
इब्नबतूता भारत की डाक प्रणाली की कार्यकुशलता देखकर बड़ा आश्चर्यचकित हुआ। इससे व्यापारियों के लिए न केवल लम्बी दूरी तक सूचना और उधार भेजना सम्भव हुआ बल्कि अल्प सूचना पर माल भेजना भी सम्भव हो गया। डाक प्रणाली इतनी कुशल थी कि जहाँ सिन्ध से दिल्ली की यात्रा में पचास दिन लगते थे, वहीं गुप्तचरों की सूचनाएँ सुल्तान तक इस डाक- व्यवस्था के द्वारा केवल पाँच दिनों में पहुँच जाती थीं।

JAC Class 12 History Important Questions Chapter 5 यात्रियों के नज़रिए : समाज के बारे में उनकी समझ

प्रश्न 41.
सोलहवीं तथा सत्रहवीं शताब्दियों में भारत में आने वाले तीन यूरोपीय यात्रियों का उल्लेख कीजिए।
उत्तर:

  1. सोलहवीं शताब्दी में दुआर्ते बरबोसा नामक यूरोपीय यात्री ने दक्षिण भारत में व्यापार और समाज का एक विस्तृत विवरण लिखा।
  2. सत्रहवीं शताब्दी में ज्यों वैप्टिस्ट तैवर्नियर नामक एक फ्रांसीसी जौहरी ने भारत की कम से कम 6 बार यात्रा की वह भारत की व्यापारिक स्थितियों से बड़ा प्रभावित था।
  3. सत्रहवीं शताब्दी में इतालवी चिकित्सक मनूकी भारत आए और यहीं बस गए।

प्रश्न 42.
फ्रांस्वा बर्नियर का संक्षिप्त परिचय दीजिए।
उत्तर:
फ्रांस्वा बर्नियर फ्रांस का निवासी था। यह एक चिकित्सक, राजनीतिक दार्शनिक तथा एक इतिहासकार था। यह मुगल साम्राज्य में अवसरों की तलाश में भारत आया था। वह 1656 से 1668 ई. तक भारत में बारह वर्ष तक रहा और मुगल दरबार से घनिष्ठ सम्बन्ध बनाए रखे। प्रारम्भ में उसने मुगल सम्राट शाहजहाँ के ज्येष्ठ पुत्र दाराशिकोह के चिकित्सक के रूप में कार्य किया तथा बाद में एक मुगल अमीर दानिशमन्द खान के साथ कार्य किया।

प्रश्न 43.
डच यात्री पेलसर्ट ने भारत में व्याप्त व्यापक गरीबी का किस प्रकार वर्णन किया है?
उत्तर:
पेलसर्ट नामक एक हच यात्री भारत के लोगों में व्याप्त गरीबी को देखकर बड़ा आश्चर्यचकित था। उसने लिखा है कि “लोग इतनी अधिक और दुःखद गरीबी में रहते थे कि उनके जीवन को मात्र नितान्त अभाव के घर और कठोर कष्ट दुर्भाग्य के आवास के रूप में चित्रित किया जा सकता है।” पेलसर्ट के अनुसार, “कृषकों को इतना अधिक निचोड़ा जाता था कि पेट भरने के लिए उनके पास सूखी रोटी भी कठिनाई से बचती थी।”

प्रश्न 44.
“बर्नियर का उद्देश्य यूरोप की श्रेष्ठता को दर्शाना तथा भारतीय स्थितियों को दयनीय बताना था।” स्पष्ट कीजिए।
उत्तर:
बर्नियर निरन्तर मुगलकालीन भारत की तुलना तत्कालीन यूरोप से करता रहा और प्रायः यूरोप की श्रेष्ठता को दर्शाता रहा। वह यूरोपीय प्रथाओं, रीति-रिवाज और प्रशासनिक व्यवस्था को श्रेष्ठ दर्शाना चाहता था तथा भारतीय परिस्थितियों को दयनीय बताना चाहता था। उसने भारत को यूरोप के प्रतिलोम के रूप में दिखाया है। उसने भारत में जो भिताएँ अनुभव की उन्हें भी पदानुक्रम के अनुसार क्रमबद्ध किया ताकि भारत पश्चिमी संसार को निम्न कोटि का लगे।

प्रश्न 45.
भूस्वामित्व के सम्बन्ध में वर्नियर के विचारों को संक्षेप में स्पष्ट कीजिए।
अथवा
बर्नियर के अनुसार भूमि स्वामित्व के प्रश्न पर भारत और यूरोप के बीच क्या भिन्नता थी? राजकीय भूस्वामित्व राज्य तथा उसके निवासियों के लिए क्यों हानिकारक था?
उत्तर:
बर्नियर के अनुसार भारत और यूरोप के बीच मूल भिन्नताओं में से एक भारत में निजी भूस्वामित्व का अभाव था। बर्नियर निजी स्वामित्व का समर्थक था। उसके अनुसार भूमि पर राजकीय स्वामित्व राज्य तथा उसके निवासियों, दोनों के लिए हानिकारक था उसने यह महसूस किया कि मुगल साम्राज्य में सम्राट सम्पूर्ण भूमि का स्वामी था जो इसे अपने अमीरों में बांटता था और इसके अर्थव्यवस्था तथा समाज के लिए विनाशकारी परिणाम होते थे।

प्रश्न 46.
बर्नियर के अनुसार राजकीय भूस्वामित्व राज्य के निवासियों के लिए क्यों विनाशकारी था?
उत्तर:
अनियर के अनुसार राजकीय भूस्वामित्व के कारण, भूधारक अपने बच्चों को भूमि नहीं दे सकते थे। इसलिए वे उत्पादन के स्तर को बनाए रखने और उसमें वृद्धि के लिए प्रयास नहीं करते थे। निजी भूस्वामित्व के अभाव ने बेहतर भूधारकों को पनपने से रोका। इसी वजह से कृषि का विनाश हुआ, किसानों को अत्यधिक उत्पीड़न का सामना करना पड़ा तथा समाज के सभी वर्गों के जीवन स्तर में लगातार पतन की स्थिति उत्पन्न हुई।

प्रश्न 47.
बर्नियर द्वारा वर्णित भारतीय ग्रामीण क्षेत्रों के कृषकों की दशा का वृत्तान्त प्रस्तुत कीजिये।
उत्तर:
बर्नियर के अनुसार भारत के कई ग्रामीण अंचल रेतीली भूमियाँ या बंजर पर्वत थे। यहाँ की खेती अच्छी नहीं थी और इन क्षेत्रों की आबादी भी कम थी। यहाँ श्रमिकों के अभाव में कृषि योग्य भूमि का एक बड़ा भाग कृषि विहीन रह जाता था। कई श्रमिक गवर्नरों द्वारा किये गए अत्याचारों के फलस्वरूप मर जाते थे। गरीबों को न केवल जीवन निर्वहन के साधनों से वंचित कर दिया जाता था, बल्कि उनके बच्चों को दास बना लिया जाता था।

JAC Class 12 History Important Questions Chapter 5 यात्रियों के नज़रिए : समाज के बारे में उनकी समझ

प्रश्न 48.
“बर्नियर भारतीय समाज को दरिद्र लोगों के जनसमूह से बना वर्णित करता है।” स्पष्ट कीजिए। उत्तर बर्नियर के अनुसार भारतीय समाज दरिद्र लोगों के समरूप जनसमूह से बना था। यह वर्ग एक अत्यन्त अमीर तथा शक्तिशाली शासक वर्ग के द्वारा अधीन बनाया जाता था। शासक वर्ग के लोग अल्पसंख्यक होते थे गरीबों में सबसे गरीब तथा अमीरों में सबसे अमीर व्यक्ति के बीच नाममात्र को भी कोई सामाजिक समूह या वर्ग नहीं था बर्नियर दृढ़तापूर्वक कहता है कि “भारत में मध्या की स्थिति के लोग नहीं हैं।”

प्रश्न 49.
बर्नियर ने मुगल साम्राज्य को जिस रूप में देखा, उसका वर्णन कीजिए।
उत्तर:
बर्नियर के अनुसार मुगल साम्राज्य का राजा भिखारियों तथा क्रूर लोगों का राजा था मुगल साम्राज्य के शहर और नगर विनष्ट तथा ‘खराब वायु’ से दूषित थे और इसके खेत ‘शाड़ीदार’ तथा ‘घातक दलदल’ से परिपूर्ण।। इसका केवल एक ही कारण था-राजकीय भूस्वामित्व।

प्रश्न 50.
बर्नियर ने मुगल साम्राज्य को भूमि का एकमात्र स्वामी बताया है। क्या इसकी पुष्टि मुगल साक्ष्यों धे से होती है?
उत्तर:
एक भी सरकारी मुगल दस्तावेज यह नहीं दर्शाता कि राज्य ही भूमि का एकमात्र स्वामी था। उदाहरण के लिए अकबर के काल के सरकारी इतिहासकार अबुल फजल ने भूमि राजस्व को ‘राजत्व का पारिश्रमिक’ बताया है जो राजा द्वारा अपनी प्रजा को सुरक्षा प्रदान करने के बदले की गई माँग लगती है, न कि अपने स्वामित्व वाली भूमि पर लगान कुछ यूरोपीय यात्री ऐसी मांगों को लगान मानते थे परन्तु वास्तव में यह न तो लगान था, न ही भूमिकर, बल्कि उपज पर लगने वाला कर था।

प्रश्न 51.
“बर्नियर द्वारा प्रस्तुत भारतीय ग्रामीण समाज का चित्रण सच्चाई से बहुत दूर था।” स्पष्ट कीजिए।
उत्तर:
बर्नियर द्वारा प्रस्तुत भारतीय ग्रामीण समाज का चित्रण सच्चाई से बहुत दूर था सोलहवीं तथा सहव शताब्दी में ग्रामीण समाज में चारित्रिक रूप से बड़े पैमाने पर सामाजिक और आर्थिक विभेद था। एक ओर बड़े जमींदार थे, जो भूमि पर उच्चाधिकारों का उपभोग करते थे तथा दूसरी ओर अस्पृश्य भूमिहीन श्रमिक (बलाहार) थे। इन दोनों के बीच में बड़ा किसान था तथा साथ ही कुछ छोटे किसान भी थे, जो बड़ी कठिनाई से अपने गुजारे लायक उत्पादन कर पाते थे।

प्रश्न 52.
बर्नियर ने अपने वृत्तान्त में किस अधिक जटिल सामाजिक सच्चाई का उल्लेख किया है?
उत्तर:
बर्नियर लिखता है कि शिल्पकारों को अपने उत्पादों की वृद्धि के लिए कोई प्रोत्साहन नहीं दिया जाता था क्योंकि समस्त लाभ राज्य द्वारा ही प्राप्त कर लिया जाता था। इसलिए उत्पादन सर्वत्र पतनोन्मुख था। इसके साथ ही बनियर ने यह भी स्वीकार किया है कि सम्पूर्ण विश्व से बड़ी मात्रा में बहुमूल्य धातुएँ भारत में आती थीं क्योंकि उत्पादों का सोने और चांदी के बदले निर्यात होता था बर्नियर ने एक समृद्ध व्यापारिक समुदाय के अस्तित्व का भी उल्लेख किया है।

प्रश्न 53.
बर्नियर ने यूरोपीय राजाओं को मुगल ढाँचे का अनुसरण करने पर क्या चेतावनी दी है? उसने सर्वनाश के दृश्य का चित्रण किस प्रकार किया है?
उत्तर:
बर्नियर के अनुसार यूरोपीय राज्य इस प्रकार अच्छी तरह से जुते और बसे हुए इतनी अच्छी प्रकार से निर्मित तथा इतने समृद्ध नहीं रह जायेंगे, जैसा कि लोग उन्हें देखते हैं वे शीघ्र ही रेगिस्तान तथा निर्जन स्थानों के, भिखारियों तथा क्रूर लोगों के राजा बनकर रह जायेंगे जैसे | कि मुगल शासक हम उन महान शहरों और नगरों को खराब वायु के कारण न रहने योग्य अवस्था में पाएँगे। विनाश की स्थिति में टीले और झाड़ियाँ अथवा पातक दलदल से भरे खेत ही रह जायेंगे।

प्रश्न 54.
बर्नियर ने अपने वृत्तान्त में भारतीय कृषि तथा शिल्प उत्पादन की उन्नत स्थिति का किस प्रकार वर्णन किया है?
उत्तर:
बर्नियर ने लिखा है कि देश का अधिकांश भू- भाग अत्यधिक उपजाऊ है। उदाहरण के लिए बंगाल राज्य चावल, मकई, रेशम कपास तथा नील के उत्पादन में से आगे है। यहाँ के शिल्पकार आलसी होते हुए भी गलीचों, जरी, कसीदाकारी कढ़ाई, सोने और चाँदी के स्वों तथा विभिन्न प्रकार के रेशमी एवं सूती वस्त्रों निर्माण का कार्य करने में संलग्न रहते हैं विश्व के सभी भागों में संचलन के बाद सोना और चाँदी कुछ सीमा तक खो जाता है।

प्रश्न 55.
बर्नियर द्वारा उल्लिखित राजकीय कारखानों की कार्यप्रणाली का वर्णन कीजिए।
उत्तर:
बर्नियर के अनुसार कई स्थानों पर बड़े कक्ष दिखाई देते थे, जिन्हें कारखाना अथवा शिल्पकारों की कार्यशाला कहते थे एक कक्ष में कसीदाकार एक मास्टर के निरीक्षण में कार्यरत रहते थे। एक अन्य कक्ष में सुनार कार्यरत थे। तीसरे कक्ष में चित्रकार तथा चौधे में प्रलाक्षा रस का रोगन लगाने वाले कार्यरत थे।

पाँचवें कक्ष में बढ़ई, खरादी, दर्जी तथा जूते बनाने वाले तथा हठे कक्ष में रेशम, जरी तथा बारीक मलमल का काम करने वाले कार्यरत थे। प्रश्न 56. बर्नियर के अनुसार ‘मुगलकालीन शहर’ ‘शिविर नगर’ थे। स्पष्ट कीजिए। जाता था। शासक वर्ग के लोग अल्पसंख्यक होते थे गरीबों में सबसे गरीब तथा अमीरों में सबसे अमीर व्यक्ति के बीच नाममात्र को भी कोई सामाजिक समूह या नहीं था। बर्नियर दृढ़तापूर्वक कहता है कि “भारत में की स्थिति के लोग नहीं हैं।”

JAC Class 12 History Important Questions Chapter 5 यात्रियों के नज़रिए : समाज के बारे में उनकी समझ

प्रश्न 49.
बर्नियर ने मुगल साम्राज्य को जिस रूप देखा, उसका वर्णन कीजिए।
उत्तर:
बर्नियर के अनुसार मुगल साम्राज्य का राज भिखारियों तथा क्रूर लोगों का राजा था मुगल साम्राज्य के शहर और नगर विनष्ट तथा ‘खराब वायु’ से दूषित और इसके खेत ‘शाड़ीदार’ तथा ‘घातक दलदल से परिपूर्ण थे। इसका केवल एक ही कारण था राजकीय भूस्वामित्व।

प्रश्न 50.
बर्नियर ने मुगल साम्राज्य को भूमि का एकमात्र स्वामी बताया है। क्या इसकी पुष्टि मुगल साक्ष्यों से होती है?
उत्तर:
एक भी सरकारी मुगल दस्तावेज यह नहीं दर्शाता कि राज्य ही भूमि का एकमात्र स्वामी था। उदाहरण के लिए अकबर के काल के सरकारी इतिहासकार अबुल फजल ने भूमि राजस्व को ‘राजत्व का पारिश्रमिक’ बताया है जो राजा द्वारा अपनी प्रजा को सुरक्षा प्रदान करने के बदले की गई माँग लगती है, न कि अपने स्वामित्व वाली भूमि पर लगान कुछ यूरोपीय यात्री ऐसी माँगों को लगान मानते थे परन्तु वास्तव में यह न तो लगान था, न ही भूमिकर, बल्कि उपज पर लगने वाला कर था।

प्रश्न 51.
” बर्नियर द्वारा प्रस्तुत भारतीय ग्रामीण समाज का चित्रण सच्चाई से बहुत दूर था।” स्पष्ट कीजिए।
उत्तर:
बर्नियर द्वारा प्रस्तुत भारतीय ग्रामीण समाज का चित्रण सच्चाई से बहुत दूर था सोलहवीं तथा सत्रहवीं शताब्दी में ग्रामीण समाज में चारित्रिक रूप से बड़े पैमाने पर सामाजिक और आर्थिक विभेद था। एक ओर बड़े जमींदार थे, जो भूमि पर उच्चाधिकारों का उपभोग करते थे तथा दूसरी ओर अस्पृश्य भूमिहीन श्रमिक (बलाहार) थे। इन दोनों के बीच में बड़ा किसान था तथा साथ ही कुछ छोटे किसान भी थे, जो बड़ी कठिनाई से अपने गुजारे लायक उत्पादन कर पाते थे।

प्रश्न 52.
बर्नियर ने अपने वृत्तान्त में किस अधिक जटिल सामाजिक सच्चाई का उल्लेख किया है?
उत्तर:
बर्नियर लिखता है कि शिल्पकारों को अपने उत्पादों की वृद्धि के लिए कोई प्रोत्साहन नहीं दिया जाता था क्योंकि समस्त लाभ राज्य द्वारा ही प्राप्त कर लिया जाता था। इसलिए उत्पादन सर्वत्र पतनोन्मुख था। इसके साथ ही बनियर ने यह भी स्वीकार किया है कि सम्पूर्ण विश्व से बड़ी मात्रा में बहुमूल्य धातुएँ भारत में आती थीं क्योंकि उत्पादों का सोने और चांदी के बदले निर्यात होता था बर्नियर ने एक समृद्ध व्यापारिक समुदाय के अस्तित्व का भी उल्लेख किया है।

प्रश्न 53.
बर्नियर ने यूरोपीय राजाओं को मुगल ढाँचे का अनुसरण करने पर क्या चेतावनी दी है? उसने सर्वनाश के दृश्य का चित्रण किस प्रकार किया है?
उत्तर:
बर्नियर के अनुसार यूरोपीय राज्य इस प्रकार अच्छी तरह से जुते और बसे हुए इतनी अच्छी प्रकार से निर्मित तथा इतने समृद्ध नहीं रह जायेंगे, जैसा कि लोग उन्हें देखते हैं वे शीघ्र ही रेगिस्तान तथा निर्जन स्थानों के, भिखारियों तथा क्रूर लोगों के राजा बनकर रह जायेंगे जैसे कि मुगल शासक हम उन महान शहरों और नगरों को खराब बायु के कारण न रहने योग्य अवस्था में पाएँगे। विनाश की स्थिति में टीले और झाड़ियाँ अथवा पातक दलदल से भरे खेत ही रह जायेंगे।

प्रश्न 54.
बर्नियर ने अपने वृत्तान्त में भारतीय कृषि तथा शिल्प उत्पादन की उन्नत स्थिति का किस प्रकार वर्णन किया है?
उत्तर:
बर्नियर ने लिखा है कि देश का अधिकांश भू- भाग अत्यधिक उपजाऊ है। उदाहरण के लिए बंगाल राज्य चावल, मकई, रेशम कपास तथा नील के उत्पादन में मिल से आगे है। यहाँ के शिल्पकार आलसी होते हुए भी गलीचों, जरी कसीदाकारी कढ़ाई, सोने और चांदी के यस्व तथा विभिन्न प्रकार के रेशमी एवं सूती वस्त्रों के निर्माण का कार्य करने में संलग्न रहते हैं। विश्व के सभी भागों में संचलन के बाद सोना और चाँदी भारत में आकर कुछ सीमा तक खो जाता है।

प्रश्न 55.
बर्नियर द्वारा उल्लिखित राजकीय कारखानों की कार्यप्रणाली का वर्णन कीजिए।
उत्तर:
बर्नियर के अनुसार कई स्थानों पर बड़े कक्ष दिखाई देते थे, जिन्हें कारखाना अथवा शिल्पकारों की कार्यशाला कहते थे। एक कक्ष में कसीदाकार एक मास्टर के निरीक्षण में कार्यरत रहते थे। एक अन्य कक्ष में सुनार कार्यरत थे। तीसरे कक्ष में चित्रकार तथा पौधे में प्रलाक्षा रस का रोगन लगाने वाले कार्यरत थे। पाँचवें कक्ष में बढ़ई, खरादी, दर्जी तथा जूते बनाने वाले तथा हठे कक्ष में रेशम, जरी तथा बारीक मलमल का काम करने वाले कार्यरत थे।

प्रश्न 56.
बर्नियर के अनुसार ‘मुगलकालीन शहर’ “शिविर नगर’ थे स्पष्ट कीजिए।
अथवा
बर्नियर के वृत्तान्त से उभरने वाले शहरी केन्द्रों के चित्र पर चर्चा कीजिये। बर्नियर भारतीय शहरों को किस रूप में देखता है?
उत्तर:
बनिंवर ने ‘मुगलकालीन शहरों’ को ‘शिविर नगर’ कहा है। शिविर नगरों से उसका अभिप्राय उन नगरों से था, जो अपने अस्तित्व और बने रहने के लिए राजकीय शिविरों पर निर्भर थे। उसका विचार था कि ये नगर राजकीय दरबार के आगमन के साथ अस्तित्व में आते थे तथा दरबार के कहीं और चले जाने के बाद ये तेजी से विलुप्त हो जाते थे। उसके अनुसार इन नगरों की सामाजिक और आर्थिक नींव व्यावहारिक नहीं होती थी और ये राजकीय संरक्षण पर आश्रित रहते थे।

प्रश्न 57.
बर्नियर के अनुसार “मुगलकालीन व्यापारी प्रायः सुदृढ़ सामुदायिक अथवा बन्धुत्व के सम्बन्धों से जुड़े होते थे।” स्पष्ट कीजिए।
उत्तर:
बर्नियर के अनुसार मुगलकाल में व्यापार प्रायः सुदृद् सामुदायिक अथवा वन्धुत्व के सम्बन्धों में जुड़े होते थे और अपनी जाति तथा व्यावसायिक संस्थाओं के माध्यम से संगठित रहते थे। पश्चिमी भारत में ऐसे समूहों को ‘महाजन’ कहा जाता था और उनका मुखिया ‘सेठ’ कहलाता था। अहमदाबाद जैसे शहरी केन्द्रों में सभी महाजनों का सामूहिक प्रतिनिधित्व व्यापारिक समुदाय के मुखिया द्वारा होता था, जिसे ‘नगर सेठ’ कहा जाता था।

JAC Class 12 History Important Questions Chapter 5 यात्रियों के नज़रिए : समाज के बारे में उनकी समझ

प्रश्न 58.
इब्नबतूता ने तत्कालीन दास-दासियों की स्थिति का क्या विवरण प्रस्तुत किया है?
अथवा
इब्नबतूता द्वारा दास प्रथा के सम्बन्ध में दिए गए साक्ष्यों का विवेचन कीजिये।
उत्तर:
इब्नबतूता के अनुसार पुरुष तथा महिला दास बाजारों में खुले आम बेचे जाते थे और नियमित रूप से भेंट में दिए जाते थे। दासों का प्रायः घरेलू श्रम के लिए ही प्रयोग किया जाता था। ये लोग और महिलाओं को ले जाते थे पालकी या डोले में पुरुषों घरेलू श्रम में लगे हुए दासों का मूल्य बहुत कम होता था इसलिए अधिकांश परिवार कम से कम एक या दो दासों को रख पाने में समर्थ थे। सुल्तान की सेवा में लगी हुई कुछ दासियाँ संगीत और गायन में निपुण थीं।

प्रश्न 59.
“भारतीय महिलाओं का जीवन सती प्रथा के अलावा कई और चीजों के चारों ओर घूमता था। ” स्पष्ट कीजिए।
उत्तर:
महिलाओं का जीवन सती प्रथा के अलावा कई और चीजों के चारों ओर घूमता भी था उनका श्रम कृषि तथा कृषि के अलावा होने वाले उत्पादन, दोनों में महत्त्वपूर्ण था। व्यापारिक घरानों की महिलाएँ व्यापारिक गतिविधियों में भाग लेती थीं। वे कभी-कभी वाणिज्यिक विवादों को न्यायालय के सामने भी ले जाती थीं। अतः यह सम्भव नहीं लगता है कि महिलाओं को उनके घरों के विशेष स्थानों तक परिसीमित कर रखा जाता था।

प्रश्न 60.
‘बर्नियर द्वारा किया गया भारत का चित्रण द्वि-विपरीतता के नमूने पर आधारित है।” स्पष्ट कीजिये।
उत्तर:
बर्नियर द्वारा किया गया भारत का चित्रण द्वि-विपरीतता के नमूने पर आधारित है। उसने भारत को यूरोप के प्रतिलोम के रूप में दिखाया है या फिर यूरोप के विपरीत देश के रूप में दर्शाया है। उसने जो भिन्नताएँ महसूस कीं, उन्हें भी पदानुक्रम के अनुसार क्रमबद्ध किया जिससे भारत यूरोपीय देशों को निम्न कोटि का प्रतीत हो।

प्रश्न 61.
बर्नियर ने सती प्रथा का विस्तृत विवरण देना क्यों उचित समझा ?
उत्तर:
बर्नियर भारत में प्रचलित उन सभी प्रथाओं का वर्णन करना चाहता था जिससे भारत यूरोपीय देशों की तुलना में एक निम्न कोटि का देश प्रतीत होता हो उस समय सभी समकालीन यूरोपीय यात्रियों तथा लेखकों के लिए महिलाओं से किया जाने वाला व्यवहार प्रायः पश्चिमी तथा पूर्वी देशों के बीच भिन्नता का एक महत्त्वपूर्ण संकेतक माना जाता था इसलिए बर्नियर ने सती प्रथा का वर्णन करना उचित समझा।

प्रश्न 62.
बर्नियर ने लाहौर में एक अल्पवयस्क विधवा के सती होने का किस प्रकार विवरण दिया है?
उत्तर:
लाहौर में बर्नियर ने एक अत्यन्त सुन्दर अल्पवयस्क विधवा को सती होते हुए देखा। यह बालिका बुरी तरह से रो रही थी परन्तु उसे कुछ ब्राह्मण लोगों तथा एक वृद्ध महिला की सहायता से उस अनिच्छुक पीड़िता को बलपूर्वक सती-स्थल की ओर ले जाया गया। उसे लकड़ियों पर बिठाया गया, उसके हाथ और पैर बांध दिए गए ताकि वह भाग न जाए और इस स्थिति में उस निर्दोष विधवा को जीवित जला दिया गया।

प्रश्न 63.
बर्नियर के विवरणों ने फ्रांसीसी दार्शनिक मान्टेस्क्यू को किस प्रकार प्रभावित किया?
उत्तर:
बर्नियर के विवरणों से प्रभावित होकर फ्रांसीसी दार्शनिक मॉन्टेस्क्यू ने वर्नियर के वृतान्त का प्रयोग प्राच्य निरंकुशवाद के सिद्धान्त को विकसित करने में किया। इस सिद्धान्त के अनुसार एशिया (प्राच्य अथवा पूर्व) में शासक अपनी प्रजा के ऊपर अपने प्रभुत्व का उपभोग करते थे, जिसे दासता तथा गरीबी की स्थितियों में रखा जाता था। इस तर्क का आधार यह था कि सम्पूर्ण भूमि पर राजा का स्वामित्व होता था।

प्रश्न 64.
बर्नियर के विवरणों से कार्ल मार्क्स किस प्रकार प्रभावित हुआ?
उत्तर:
कार्ल मार्क्स ने लिखा कि भारत ( तथा अन्य एशियाई देशों में ) उपनिवेशवाद से पहले अधिशेष का अधिग्रहण राज्य द्वारा होता था। इससे एक ऐसे समाज का उद्भव हुआ जो बड़ी संख्या में स्वतन्त्र तथा आन्तरिक रूप से समतावादी ग्रामीण समुदायों से बना था। इन ग्रामीण समुदायों पर राजकीय दरबार का नियंत्रण होता था और जब तक अधिशेष की आपूर्ति बिना किसी बाधा के जारी रहती थी इनकी स्वायत्तता का सम्मान किया जाता था।

प्रश्न 65.
इब्नबतूता भारत की डाक प्रणाली को देखकर चकित क्यों हो गया?
उत्तर:
इब्नबतूता भारत की डांक प्रणाली को देखकर चकित हो गया क्योंकि भारत की डाक प्रणाली इतनी कुशल थी कि जहाँ सिन्ध से दिल्ली यात्रा में पचास दिन लगाते थे, वहीं सुल्तान तक गुप्तचरों की खबर मात्र पाँच दिनों में ही पहुँच जाती थी। इसके अतिरिक्त इससे व्यापारियों के लिए न केवल लम्बी दूरी तक सूचना भेजी जा सकती श्री बल्कि अल्प सूचना पर माल भी भेजा जा सकता था।

JAC Class 12 History Important Questions Chapter 5 यात्रियों के नज़रिए : समाज के बारे में उनकी समझ

प्रश्न 66.
‘ट्रैवल्स इन द मुगल एम्पायर’ नामक ग्रन्थ में बर्नियर भारत को पश्चिमी जगत की तुलना में अल्प- विकसित व निम्न श्रेणी का दर्शाना चाहता था। स्पष्ट कीजिए।
उत्तर:
‘ट्रैवल्स इन द मुगल एम्पायर में बनिंबर की भारतीय उपमहाद्वीप की यात्राओं का वर्णन है। यह ग्रन्थ वर्नियर की गहन आलोचनात्मक चिन्तन दृष्टि का उदाहरण है, लेकिन बर्नियर का यह दृष्टिकोण पूर्वाग्रह से प्रेरित हैं तथा एकपक्षीय है। बर्नियर ने मुगलकालीन इतिहास को भारत की भौगोलिक, सामाजिक, आर्थिक परिस्थितियों को ध्यान में न रखकर एक वैश्विक ढाँचे में ढालने का प्रयास किया। वह यूरोप के परिप्रेक्ष्य में मुगलकालीन इतिहास की तुलना निरन्तर करने का प्रयास करता रहा। बर्नियर के अनुसार यूरोप की प्रशासनिक व्यवस्था, यूरोप की सामाजिक- आर्थिक स्थिति भारत से कहीं बेहतर है। बास्तव में उसका भारत चित्रण पूरी तरह प्रतिकूलता पर आधारित है।

प्रश्न 67.
इब्नबतूता ने भारतीय शहरों में क्या विशेषताएं देखीं? संक्षिप्त टिप्पणी लिखिए।
उत्तर:
इब्नबतूता ने भारतीय शहरों को उन्नत अवस्था में पाया। उसके अनुसार जो लोग जीवन को समग्रता से जीने की आकांक्षा रखते हैं, जिनके पास कला-कौशल, दृढ़ इच्छा-शक्ति और पर्याप्त साधन हैं, उनके लिए आगे बढ़ने हेतु भारत के शहरों में व्यापक अवसर मौजूद थे। भारतीय शहरों की आबादी धनी थी व शहरों में समृद्धि झलकती थी। परन्तु युद्ध आदि की विभीषिका का दुष्परिणाम शहरों को कभी-कभी भुगतना पड़ता था।

इब्नबतूता के अनुसार शहर के बाजार चहल-पहल तथा चमक-दमक से भरपूर विभिन्न प्रकार की व्यापारिक वस्तुओं से भरे रहते थे। भारत में दिल्ली एक विपुल आबादी वाला सबसे बड़ा शहर था महाराष्ट्र में स्थित दौलताबाद भी दिल्ली से किसी प्रकार कमतर नहीं था। बाजारों में केवल व्यापारिक गतिविधियाँ ही नहीं अन्य धार्मिक तथा सामाजिक गतिविधियाँ भी होती थीं। इब्नबतूता का यह वर्णन भारतीय शहरों की विकसित समृद्धि का व्यापक उदाहरण है।

प्रश्न 68.
अब्दुर रज्जाक द्वारा लिखित यात्रा वृत्तान्त संवेगों और अवबोधनों का एक रोचक मिश्रण है, कैसे?
उत्तर:
1440 के दशक में लिखा गया अब्दुर रजाक का यात्रा वृत्तान्त उसके शब्दों में, “यहाँ ऐसे लोग बसे हुए थे जिनकी कल्पना मैंने कभी नहीं की।” 1440 के दशक में अब्दुर रजाक केरल के कालीकट बन्दरगाह पर पहुँचा। इन लोगों को देखकर उसके मुँह से ‘विचित्र देश का सम्बोधन निकला। लेकिन कालान्तर में अपनी पुनः भारत यात्रा के दौरान वह मंगलौर आया और यहाँ पश्चिमी पाट को पार कर उसने एक मन्दिर देखा, जिसे देखकर वह बहुत अधिक प्रभावित हुआ। मन्दिर की स्थापत्य कला को देखकर वह मन्त्रमुग्ध रह गया और ‘विचित्र देश’ की छवि उसके मस्तिष्क से निकल गई।

प्रश्न 69.
पेलसर्ट नामक डच यात्री की भारत यात्रा का वर्णन संक्षेप में कीजिए।
उत्तर:
पेलसर्ट नामक एक डच यात्री ने सत्रहवीं शताब्दी के आरम्भिक दशकों में भारतीय उपमहाद्वीप की यात्रा की थी। बर्नियर की ही तरह वह भी लोगों में व्यापक गरीबी देखकर अचम्भित था “लोग इतनी अधिक तथा दुःखद गरीबी में रहते हैं कि इनके जीवन को मात्र नितान्त अभाव के पर तथा कठोर कष्ट दुर्भाग्य के आभास के रूप में चित्रित अथवा ठीक प्रकार से वर्णित किया जा सकता है।” राज्य को उत्तरदायी ठहराते हुए वह कहता है: ‘कृषकों को इतना अधिक निचोड़ा जाता है कि पेट भरने के लिए उनके पास सूखी रोटी भी मुश्किल से बचती है।”

प्रश्न 70.
भारतीय महिलाओं की मध्यकालीन सामाजिक स्थिति के बारे में यूरोपीय लेखकों के विचार क्या थे?
उत्तर:
विभिन्न यूरोपीय यात्री जो भारत आये उन्होंने सामाजिक परिस्थितियों के अन्तर्गत मध्यकालीन भारतीय महिलाओं की स्थिति पर अपने विचार व्यक्त किए हैं। इन विचारकों ने पाया कि भारतीय महिलाओं से किया जाने वाला व्यवहार पश्चिमी समाजों से सर्वथा भिन्न था। बर्नियर ने इस सन्दर्भ में भारत की क्रूरतम अमानवीय सती प्रथा का उदाहरण दिया है जो भारत के अतिरिक्त विश्व के किसी भी देश में नहीं थी।

पस्तु सती प्रथा के अतिरिक्त यूरोपीय लेखकों ने महिलाओं के सामाजिक जीवन के अन्य पक्षों पर भी अपना ध्यान केन्द्रित किया है। वे केवल घरों की चारदीवारी तक ही सीमित नहीं रहती थीं। कृषि तथा कृषि से सम्बन्धित कार्यों जैसे पशुपालन में उनके श्रम का महत्त्व था। व्यापारिक परिवारों की महिलाएँ व्यापारिक गतिविधियों में भागीदारी रखती थीं। कुछ महिलाएँ प्रशासनिक कार्यों में भी भाग लेती थीं।

JAC Class 12 History Important Questions Chapter 5 यात्रियों के नज़रिए : समाज के बारे में उनकी समझ

निबन्धात्मक प्रश्न

प्रश्न 1.
भारत के 10वीं से 17वीं सदी तक के इतिहास के पुनर्निर्माण में विदेशी यात्रियों के विवरणों का क्या योगदान है ? उदाहरण सहित स्पष्ट कीजिए।
उत्तर:
विदेशी यात्रियों के वृत्तान्त निश्चित रूप से इतिहास के निर्माण में सहायक होते हैं। यही स्थिति हम 10वीं से 17वीं शताब्दी के मध्य भी देखते हैं। इस काल में अनगिनत यात्री भारत आए तथा अपनी समझ के अनुसार भारत का विवरण प्रस्तुत किया। इस तथ्य को हम निम्न विन्दुओं के माध्यम से समझ सकते हैं-
(1) अधिकांश यात्री भिन्न-भिन्न देशों, भिन्न-भिन्न आर्थिक तथा सामाजिक परिदृश्य से आए थे।

(2) स्थानीय लेखक भारत की तत्कालीन परिस्थितियों का विवरण करने में रुचि नहीं रखते थे; जबकि इन विदेशी लेखकों ने भारत की तत्कालीन आर्थिक, सामाजिक तथा सांस्कृतिक गतिविधियों पर विशेष विवरण दिए हैं।

(3) 10वीं शताब्दी के आस-पास भारतीय विद्वान विदेशों के साथ सम्बन्ध बनाने तथा उनके विषय में जानने .में अधिक रूचि नहीं रखते थे। इसके फलस्वरूप ये तुलनात्मक अध्ययन करने में असमर्थ थे।

(4) विदेशी यात्रियों ने अपने विवरण में उन तथ्यों को अधिक महत्त्व दिया है; जो उन्हें विचित्र जान पड़ते थे। इससे उनके विवरण में रोचकता आ जाती है।

(5) विदेशी यात्रियों के विवरण तत्कालीन राजदरबार के क्रियाकलापों, धार्मिक विश्वास तथा स्थापना की प्रमुख विशेषताओं पर प्रकाश डालते हैं। इससे इतिहास निर्माण में अत्यधिक सहायता प्राप्त होती है। उपर्युक्त बिन्दुओं को हम पृथक्-पृथक् यारियों के विवरण के साथ समझ सकते हैं।

ये विवरण निम्नलिखित –
1. अल बिरूनी का विवरण अल-विरुनी का विवरण निम्नलिखित बिन्दुओं के अन्तर्गत किया गया है –

  • अल-विरुनी ने अपनी रचनाओं में भारत में व्याप्त जाति-व्यवस्था तथा वर्ण व्यवस्था का वर्णन किया है।
  • अल बिरूनी ने भारतीय समाज की रूढ़िवादिता को भी अपनी किताबों में दर्शाया है।
  • अल बिरूनी ने भारत के ज्योतिष, खगोल विज्ञान तथा गणित की विस्तारपूर्वक चर्चा की है।

2. इब्नबतूता का विवरण इससे सम्बन्धित मुख्य बिन्दु निम्न प्रकार है –

  • इब्नबतूता ने भारतीय डाक प्रणाली की प्रशंसा की है तथा उससे हमें परिचित भी कराया।
  • इब्नबतूता की रचनाओं द्वारा हमें यह पता चलता है कि उस समय भारत में नारियल तथा पान की खेती का व्यापक प्रचलन था।
  • इब्नबतूता ने दासों का भी विस्तारपूर्वक विवरण दिया है; जिससे यह ज्ञात होता है कि तत्कालीन समाज तथा राजनैतिक व्यवस्था में दासों की महत्वपूर्ण भूमिका थी।
  • इब्नबतूता की रचनाओं में भारतीय बाजारों तथा व्यापारियों की सम्पन्नता का उल्लेख भी मिलता है। जिससे यह पता चलता है कि तत्कालीन भारत आर्थिक रूप से समृद्ध था।

3. बर्नियर का विवरण बर्नियर द्वारा रचित ‘ट्रैवल्स इन द मुगल एम्पायर’ में दिया गया विवरण निम्न बिन्दुओं द्वारा किया जा सकता है-

  • बर्नियर के अनुसार भारतीयों को निजी भू-स्वामित्व का अधिकार प्राप्त नहीं है।
  • बर्नियर की रचनाओं में सती प्रथा का विवरण मिलता है।
  • बर्नियर ने मुगल सेना के साथ कश्मीर यात्रा की थी तथा उस यात्रा का विस्तृत विवरण दिया है। उस विवरण से यह ज्ञात होता है कि उस समय यात्रा में किन सामानों की आवश्यकता होती थी। इस प्रकार स्पष्ट होता है कि समकालीन यात्रियों के विवरण इतिहास को समझने तथा उसका निर्माण करने में अत्यधिक सहायक होते हैं।

प्रश्न 2.
अल बिरूनी के प्रारम्भिक जीवन का वर्णन करते हुए ‘किताब-उल-हिन्द’ की विशेषताओं पर प्रकाश डालिए।
उत्तर:
अल बिरूनी का प्रारम्भिक जीवन अल बिरूनी का जन्म आधुनिक उज्बेकिस्तान में स्थिता ख्वारिज्म में सन् 973 में हुआ था। ख्वारिज्म शिक्षा का एक प्रसिद्ध केन्द्र था। अल बिरूनी ने उस समय उपलब्ध सबसे अच्छी शिक्षा प्राप्त की। वह एक उच्च कोटि का विद्वान था तथा सीरियाई फारसी, हिब्रू और संस्कृत भाषाओं का ज्ञाता था। यद्यपि वह यूनानी भाषा का जानकार नहीं था, फिर भी वह प्लेटो तथा अन्य यूनानी दार्शनिकों की रचनाओं से परिचित था जिनका उसने अरबी अनुवादों के माध्यम से अध्ययन किया था। 1017 ई. में महमूद गजनवी ने ख्वारिज्म पर आक्रमण किया और यहाँ से कई विद्वानों तथा कवियों को अपने साथ अपनी राजधानी गजनी ले गया।

अल बिरुनी भी उनमें से एक था। वह एक बन्धक के रूप में गजनी आया था, परन्तु धीरे-धीरे उसकी गजनी शहर में रुचि बढ़ने लगी। उसने अपना शेष जीवन गजनी में ही बिताया। 70 वर्ष की आयु में गजनी में अल बिरुनी की मृत्यु हो गयी। भारत के प्रति रुचि बढ़ना-गजनी में रहते हुए अल- बिरनी की भारत के प्रति रुचि बढ़ने लगी। पंजाब को गलनी साम्राज्य में सम्मिलित करने के बाद स्थानीय लोगों से सम्पर्कों से आपसी विश्वास तथा समझ का वातावरण बना। अल-विरुनी ने ब्राह्मण पुरोहितों तथा विद्वानों के साथ कई वर्ष व्यतीत किए और संस्कृत, धर्म तथा दर्शन का ज्ञान प्राप्त किया।

JAC Class 12 History Important Questions Chapter 5 यात्रियों के नज़रिए : समाज के बारे में उनकी समझ

‘किताब-उल-हिन्द’ की विशेषताएं –
(1) विविध विषयों का विवेचन-किताब-उल- हिन्द अरबी में लिखी गई अल बिरूनी की प्रसिद्ध रचना है इसकी भाषा सरल और स्पष्ट है। यह एक विस्तृत ग्रन्थ है जिसमें धर्म और दर्शन, त्यौहारों, खगोल विज्ञान, कीमिया, रीति-रिवाजों तथा प्रथाओं, सामाजिक-जीवन, भार तौल, मापन विधियों, मूर्तिकला, कानून, मापतन्त्र विज्ञान आदि विषयों का विस्तृत विवेचन किया गया है। यह ग्रन्थ अस्सी अध्यायों में विभाजित है।

(2) विशिष्ट शैली अल-बिहनी ने प्रत्येक अध्याय में एक विशिष्ट शैली का प्रयोग किया है, जिसमें आरम्भ में एक प्रश्न होता था, फिर संस्कृतवादी परम्पराओं पर आधारित वर्णन था और अन्त में अन्य संस्कृतियों के साथ तुलना की गई थी। यह लगभग एक ज्यामितीय संरचना है।

(3) स्पष्टता तथा पूर्वानुमेयता यह ग्रन्थ अपनी स्पष्टता तथा पूर्वानुमेयता के लिए प्रसिद्ध है।

(4) भारतीय ग्रन्थों के अरबी भाषा में अनुवादों से परिचित अल बिरूनी ने सम्भवतः अपनी रचनाएँ भारतीय उपमहाद्वीप के सीमान्त क्षेत्रों में रहने वाले लोगों के लिए लिखी थीं। अल बिरुनी संस्कृत, पालि तथा प्राकृत ग्रन्थों के अरबी भाषा में अनुवादों तथा रूपान्तरणों से परिचित था।

(5) समालोचनात्मक दृष्टिकोण- इन ग्रन्थों की लेखन सामग्री तथा शैली के विषय में अल-बिरुनी का दृष्टिकोण समालोचनात्मक था और निश्चित रूप से वह उनमें सुधार करना चाहता था।

प्रश्न 3.
इब्नबतूता के प्रारम्भिक जीवन एवं उसकी यात्राओं का वर्णन कीजिए।
उत्तर:
इब्नबतूता का प्रारम्भिक जीवन इब्नबतूता मोरक्को का निवासी था उसका जन्म 1304 में तैंजियर नामक नगर के एक सम्मानित एवं शिक्षित परिवार में हुआ था। उसका परिवार इस्लामी कानून अथवा शरिया पर अपनी विशेषज्ञता के लिए प्रसिद्ध था अपने परिवार की परम्परा के अनुसार इब्नबतूता ने कम आयु में ही साहित्यिक तथा शास्वारूढ़ शिक्षा प्राप्त की। उसका यात्रा वृत्तान्त ‘रिला’ के नाम से प्रसिद्ध है।

(1) इब्नबतूता की यात्राएँ – अन्य विदेशी यात्रियों के विपरीत, इब्नबतूता पुस्तकों के स्थान पर यात्राओं से प्राप्त अनुभव को ज्ञान का अधिक महत्त्वपूर्ण स्रोत मानता था। इब्नबतूता को यात्राएँ करने का बहुत शौक था इसलिए वह नए-नए देशों तथा लोगों के विषय में जानकारी प्राप्त करने के लिए दूर-दूर के प्रदेशों तक में गया। 1332-33 ई. में भारत के लिए प्रस्थान करने से पहले वह मक्का की तीर्थ यात्राएँ और सीरिया, इराक, फारस, यमन, ओमान तथा पूर्वी अफ्रीका के अनेक तटीय व्यापारिक बन्दरगाहों की यात्राएँ कर चुका था।

(2) भारत की यात्रा – मध्य एशिया के मार्ग से होते हुए इब्नबतूता 1333 ई. में स्थल मार्ग से सिन्ध पहुँचा। उसने दिल्ली के सुल्तान मुहम्मद बिन तुगलक के बारे में सुन रखा था कि वह कला और साहित्य का उदार संरक्षक है अतः उसकी ख्याति से आकर्षित हो इब्नबतूता ने मुल्तान और कच्छ होते हुए दिल्ली को ओर प्रस्थान किया। मुहम्मद बिन तुगलक इब्नबतूता की विद्वता से बड़ा प्रभावित हुआ और उसे दिल्ली का काजी अथवा न्यायाधीश नियुक्त किया। वह इस पद पर कई वर्ष तक रहा।

(3) चीन के राजदूत के रूप में इब्नबतूता की नियुक्ति 1342 ई. में इब्नबतूता को मंगोल शासक के पास सुल्तान के दूत के रूप में चीन जाने का आदेश दिया गया। उसने दिल्ली से चीन के लिए प्रस्थान किया और मध्य भारत के रास्ते मालाबार तट की ओर बढ़ा। मालाबार से वह मालद्वीप गया तथा वहाँ वह अठारह महीनों तक काजी के पद पर रहा। अन्ततः उसने लंका जाने का निश्चय किया। बाद में वह एक बार पुनः मालाबार तट तथा मालद्वीप गया। चीन जाने के अपने कार्य को पुनः शुरू करने से भा पहले वह बंगाल तथा असम भी गया।

प्रश्न 4.
लगभग 1500 ई. के बाद भारत की यात्रा करने वाले यूरोपीय लेखकों का वर्णन कीजिए।
उत्तर:
लगभग 1500 ई. के बाद भारत की यात्रा करने वाले यूरोपीय लेखक लगभग 1500 ई. में भारत में पुर्तगालियों के आगमन के बाद उनमें से अनेक लोगों ने भारतीय सामाजिक रीति- रिवाजों तथा धार्मिक प्रथाओं के विषय में विस्तृत वृत्तान्त लिखे। जेसुइट राबट नोबिली भी एक ऐसा ही लेखक था जिसने भारतीय ग्रन्थों का यूरोपीय भाषाओं में अनुवाद भी किया।

JAC Class 12 History Important Questions Chapter 5 यात्रियों के नज़रिए : समाज के बारे में उनकी समझ

(1) दुआतें बरबोसा- दुआर्ते बरबोसा यूरोप का एक प्रसिद्ध लेखक था जिसने दक्षिण भारत में व्यापार और समाज का एक विस्तृत विवरण लिखा 1600 ई. के बाद भारत में आने वाले डच अंग्रेज और फ्रांसीसी यात्रियों की संख्या बढ़ने लगी थी।

(2) फ्रांसिस्को पेलसर्ट-फ्रांसिस्को पेलसर्ट एक डच यात्री था जिसने सत्रहवीं शताब्दी के आरम्भिक दशकों में उपमहाद्वीप की यात्रा की थी। वह यहाँ के लोगों में व्यापक गरीबी देखकर आश्चर्यचकित था। उसने कृषकों की अत्यन्त दयनीय दशा को मार्मिक चित्रण किया है।

(3) ज्यों-बैप्टिस्ट तैवर्नियर ज्यों-बैप्टिस्ट तैवर्नियर एक फ्रांसीसी जौहरी था जिसने कम से कम छः बार भारत की यात्रा की यह विशेष रूप से भारत की व्यापारिक स्थितियों से बहुत प्रभावित था। उसने भारत की तुलना ईरान और ओटोमन साम्राज्य से की।

(4) मनूकी मनूकी एक इतालवी चिकित्सक था। वह कभी भी यूरोप वापस नहीं गया और भारत में ही बस गया।

(5) फ्रांस्वा बर्नियर-फ्रांस का निवासी फ्रांस्वा बनियर एक चिकित्सक, राजनीतिक दार्शनिक तथा एक इतिहासकार था। वह अन्य लोगों की भांति मुगल साम्राज्य में अवसरों की तलाश में आया था। वह 1656 से 1668 तक भारत में बारह वर्ष तक रहा और मुगल दरबार से निकटता से जुड़ा रहा-पहले सम्राट शाहजहाँ के ज्येष्ठ पुत्र दारा शिकोह के चिकित्सक के रूप में तथा बाद में मुगल दरबार के एक आर्मीनियाई अमीर दानिशमंद खान के साथ एक बुद्धिजीवी तथा वैज्ञानिक के रूप में।

प्रश्न 5.
अल बिरुनी के यात्रा-वृत्तान्त का आलोचनात्मक वर्णन कीजिए।
उत्तर:
अल बिरूनी का यात्रा वृत्तान्त अल बिरूनी एक उच्च कोटि का विद्वान था। वह भारत में कई वर्षों तक रहा। उसने ब्राह्मण पुरोहितों तथा विद्वानों के साथ कई वर्ष बिताये और संस्कृत, धर्म तथा दर्शन का ज्ञान प्राप्त किया। अल बिरुनी की लेखन शैली की विशेषताएँ-अल- बिरुनी की लेखन शैली की प्रमुख विशेषताएँ निम्नलिखित किया।

  1. अल बिरूनी ने लेखन में अरबी भाषा का प्रयोग
  2. उसने सम्भवतः अपनी कृतियाँ उपमहाद्वीप के सीमान्त क्षेत्रों में रहने वाले लोगों के लिए लिखी थीं।
  3. वह संस्कृत, पालि तथा प्राकृत ग्रन्थों के अरबी भाषा में अनुवादों तथा रूपान्तरणों से परिचित था।
  4. इन ग्रन्थों की लेखन सामग्री शैली के विषय में उसका दृष्टिकोण आलोचनात्मक था और निश्चित रूप से वह उनमें सुधार करना चाहता था।

अल बिरूनी द्वारा अपवित्रता की मान्यता को स्वीकार करना-यद्यपि अल बिरुनी जाति-व्यवस्था के सम्बन्ध में ब्राह्मणवादी व्याख्या को मानता था, फिर भी उसने अपवित्रता की मान्यता को अस्वीकार कर दिया। उसने लिखा कि प्रत्येक यह वस्तु जो अपवित्र हो जाती है, अपनी पवित्रता की मूल स्थिति को पुनः प्राप्त करने का प्रयास करती है और सफल होती है। उसका कहना था कि जाति व्यवस्था में संलग्न अपवित्रता की अवधारणा प्रकृति के नियमों के विरुद्ध थी। भारत में प्रचलित वर्ण-व्यवस्था का विवरण अल- बिरूनी ने भारत में प्रचलित वर्ण व्यवस्था का उल्लेख अग्र प्रकार से किया है –

  • ब्राह्मण-ब्राह्मणों की जाति सबसे ऊँची थी। हिन्दू ग्रन्थों के अनुसार ब्राह्मण ब्रह्मन् के सिर से उत्पन्न हुए थे। हिन्दू ब्राह्मणों को मानव जाति में सबसे उत्तम मानते हैं।
  • क्षत्रिय अल-विरुनी के अनुसार ऐसी मान्यता थी कि क्षत्रिय ब्रह्मन् के कंधों और हाथों से उत्पन्न हुए थे। उनका दर्जा ब्राह्मणों से अधिक नीचा नहीं है।
  • वैश्य क्षत्रियों के बाद वैश्य आते हैं। वैश्य ब्रह्मन् की जंघाओं से उत्पन्न हुए थे।
  • शूद्र इनका जन्म ब्रह्मन् के चरणों से हुआ था।

अल बिरुनी के अनुसार अन्तिम दो वर्णों में अधिक अन्तर नहीं है। परन्तु इन वर्गों के बीच भिन्नता होने पर भी ये शहरों और गाँवों में मिल-जुलकर रहते हैं। जाति व्यवस्था के बारे में अल बिरूनी का विवरण संस्कृत ग्रन्थों पर आधारित होना जाति व्यवस्था के बारे में अल बिरुनी का विवरण संस्कृत ग्रन्थों के अध्ययन से पूर्णतया प्रभावित था। इन ग्रन्थों में ब्राह्मणों के दृष्टिकोण से जाति व्यवस्था का संचालन करने वाले नियमों का प्रतिपादन किया गया था। परन्तु वास्तविक जीवन में यह व्यवस्था इतनी कठोर नहीं थी।

प्रश्न 6.
इब्नबतूता द्वारा किए गए दिल्ली तथा दौलताबाद के वर्णन प्रस्तुत कीजिए।
अथवा
इब्नबतूता द्वारा वर्णित दिल्ली का संक्षिप्त विवरण दीजिये।
अथवा
दिल्ली के विशेष संदर्भ में भारतीय नगरों के बारे में इब्नबतूता के वृत्तान्तों की व्याख्या कीजिये।
उत्तर:
1. इब्नबतूता द्वारा किया गया दिल्ली का वर्णन –
(1) दिल्ली की बनावट इनवतृत के अनुसार दिल्ली बड़े क्षेत्र में फैला पनी जनसंख्या वाला शहर है। शहर के चारों ओर बनी प्राचीर अतुलनीय है दीवार की चौड़ाई ग्यारह हाथ (एक हाथ लगभग 20 इंच के बराबर) है और इसके भीतर रात्रि के पहरेदार तथा द्वारपालों के कक्ष हैं।

प्राचीरों के अन्दर खाद्य सामग्री, हथियार, बारूद, प्रक्षेपास्त्र तथा पेरेबन्दी में प्रयुक्त होने वाली मशीनों के संग्रह के लिए भंडार गृह बने हुए थे। प्राचीर में खिड़कियाँ बनी हैं जो शहर की ओर खुलती हैं तथा इन्हीं खिड़कियों के द्वारा प्रकाश भीतर आता है। प्राचीर का निचला भाग पत्थर से बना है तथा ऊपरी भाग ईंटों से निर्मित है।

(2) शहर के द्वार दिल्ली शहर के 28 द्वार हैं जिन्हें दरवाजा कहा जाता है और इनमें से बदायूँ दरवाजा सबसे विशाल है। मांडवी दरवाजे के भीतर एक अनाज मंडी है, गुल-दरवाजे की बगल में एक फलों का बगीचा है।

(3) कब्रगाह दिल्ली शहर में एक उत्तम कब्रगाह है, जिसमें बनी कब्रों के ऊपर गुम्बद बनाई गई है और गुम्बद विहीन कब्रों पर मेहराब बने हुए हैं। कब्रगाह में कदाकार चमेली तथा जंगली गुलाब जैसे फूल उगाए जाते हैं और फूल सभी ऋतुओं में खिले रहते हैं।

JAC Class 12 History Important Questions Chapter 5 यात्रियों के नज़रिए : समाज के बारे में उनकी समझ

2. इब्नबतूता द्वारा दौलताबाद का विवरण
(1) पुरुष और महिला गायकों का बाजार- इब्नबतूता के अनुसार दौलताबाद में पुरुष और महिला गायकों के लिए एक बाजार है, जिसे ‘तारामबाद’ कहते हैं यहाँ बहुत-सी दुकानें हैं और प्रत्येक दुकान में एक ऐसा दरवाजा है, जो मालिक के आवास में खुलता है। दुकानें कालीनों से सुसज्जित हैं और दुकान के मध्य में झूला है, जिस पर गायिका बैठती है।

(2) विशाल गुम्बद बाजार के मध्य में एक विशाल गुम्बद खड़ा है जिसमें कालीन बिछे हुए हैं और यह खूब सजाया गया है। इसमें प्रत्येक गुरुवार प्रातः काल की उपासना के बाद संगीतकारों के प्रमुख अपने सेवकों और दासों के साथ स्थान ग्रहण करते हैं। गायिकाएँ एक के बाद एक झुंडों में उनके समक्ष आकर सूर्यास्त का गीत गाती हैं और नृत्य करती हैं जिसके पश्चात् वे चले जाते हैं।

(3) मस्जिदें इब्नबतूता के अनुसार इस बाजार में इबादत के लिए मस्जिदें बनी हुई हैं जब भी कोई हिन्दू शासक इस बाजार से गुजरता था, वह गुम्बद में उत्तर कर आता था और गायिकाएँ उसके समक्ष गान प्रस्तुत करती थीं। यहाँ तक कि अनेक मुस्लिम शासक भी ऐसा ही करते थे।

प्रश्न 7.
इब्नबतूता द्वारा वर्णित भारत की डाक व्यवस्था का वर्णन कीजिए।
उत्तर:
इब्नबतूता द्वारा वर्णित भारत की डाक व्यवस्था इब्नबतूता के अनुसार व्यापारियों को प्रोत्साहित करने के लिए राज्य विशेष उपाय करता था। लगभग सभी व्यापारिक मार्गों पर सराय तथा विश्राम गृह स्थापित किए गए थे। इब्नबतूता भारत की डाक व्यवस्था की कार्यकुशलता देखकर बड़ा चकित हुआ।

इससे व्यापारियों के लिए न केवल लम्बी दूरी तक सूचना भेजना और उधार भेजना सम्भव हुआ, बल्कि इससे अल्पसूचना पर माल भेजना भी आसान हो गया। इब्नबतूता के अनुसार डाक प्रणाली इतनी कुशल थी कि जहाँ सिन्ध से दिल्ली की यात्रा में पचास दिन लगते थे, वहीं गुप्तचरों की सूचनाएं सुल्तान तक इस डाक व्यवस्था के द्वारा केवल पाँच दिनों में पहुँच जाती थीं।

डाक व्यवस्था इब्नबतूता के अनुसार भारत में दो प्रकार की डाक व्यवस्था थी –
(1) अश्व डाक व्यवस्था तथा
(2) पैदल डाक व्यवस्था।
(1) अश्व डाक व्यवस्था अश्व डाक व्यवस्था को ‘उल्लुक’ कहा जाता था। यह हर चार मील की दूरी पर स्थापित राजकीय घोड़ों द्वारा चालित होती थी।
(2) पैदल डाक व्यवस्था पैदल डाक व्यवस्था में प्रति मील तीन चौकियाँ होती थीं, जिन्हें ‘दावा’ कहा जाता था। यह एक मील का एक तिहाई होता था। हर | तीन मील पर घनी आबादी वाला एक गाँव होता था, जिसके बाहर तीन मण्डप होते थे, जिनमें लोग कार्य शुरू करने के लिए तैयार बैठे रहते थे। उनमें से प्रत्येक के पास दो हाथ लम्बी एक छड़ी होती थी, जिसके ऊपर ताँबे की घंटियाँ लगी होती थीं।

जब सन्देशवाहक शहर से यात्रा आरम्भ करता था, तो एक हाथ में पत्र तथा दूसरे में घटियाँ वाली छड़ लिए वह यथाशक्ति तेज भागता था जब मंडप में बैठे लोग घंटियों की आवाज सुनते थे, तो तैयार हो जाते थे। जैसे ही सन्देशवाहक उनके निकट पहुँचता था, उनमें से एक पत्र ले लेता था और वह छड़ी हिलाते हुए पूरी शक्ति से दौड़ता था, जब तक वह अगले दावा तक नहीं पहुँच जाता। पत्र के अपने गन्तव्य स्थान तक पहुँचने तक यही प्रक्रिया चलती रहती थी यह पैदल डाक व्यवस्था अश्व डाक व्यवस्था से अधिक तीव्र होती थी। इसका प्रयोग प्रायः खुरासान के फलों के परिवहन के लिए होता था, जिन्हें भारत में बहुत पसन्द किया जाता था।

प्रश्न 8.
इब्नबतूता द्वारा वर्णित भारतीय यात्रा- वृत्तांत का विवेचन कीजिए।
उत्तर:
इब्नबतूता का भारतीय यात्रा वृत्तांत इब्नबतूता के भारतीय यात्रा वृत्तांत का विवेचन निम्नलिखित बिन्दुओं के अन्तर्गत किया जा सकता है –
(1) नारियल – इब्नबतूता के अनुसार नारियल एक अनोखा तथा विस्मयकारी वृक्ष था। यह खजूर के वृक्ष जैसा दिखता था। इन दोनों वृक्षों में एक ही अन्तर था-नारियल से काष्ठफल प्राप्त होता था तथा दूसरे से खजूर नारियल के वृक्ष का फल मानव सिर से मेल खाता था। लोग नारियल के रेशे से रस्सी बनाते थे।

(2) पान इब्नबतूता के अनुसार पान को अंगूर लता की तरह ही उगाया जाता था। पान का कोई वृक्ष नहीं होता था और इसे केवल इसकी पत्तियों के लिए ही उगाया जाता था। इसे
खाने से पहले सुपारी ली जाती थी। इसके छोटे- छोटे टुकड़ों को मुँह में रखकर चनाया जाता था। इसके बाद पान की पत्तियों के साथ इन्हें चबाया जाता था।

(3) भारतीय शहर इब्नबतूता के अनुसार भारतीय शहर घनी आबादी वाले तथा समृद्ध थे परन्तु कभी-कभी युद्धों तथा अभियानों के दौरान नष्ट हो जाते थे। अधिकांश शहरों में भीड़-भाड़ वाली सड़कें तथा चमक-दमक वाले और रंगीन बाजार थे। ये बाजार विभिन्न प्रकार की वस्तुओं से भरे रहते थे। इब्नबतूता के अनुसार दिल्ली बहुत अधिक आबादी वाला शहर था। वह भारत में सबसे बड़ा शहर था दौलताबाद (महाराष्ट्र में भी कम नहीं था तथा आकार में दिल्ली को चुनौती देता था।’ शहरों में वस्त्र उद्योग उन्नत अवस्था में था। विदेशों में भारतीय सूती कपड़े, महीन मलमल, रेशम, जरी तथा साटन की अत्यधिक मांग थी।

(4) डाक व्यवस्था इब्नबतूता के अनुसार भारतीय डाक प्रणाली इतनी कुशल थी कि जहाँ सिन्ध से दिल्ली की यात्रा में पचास दिन लगते थे, वहीं गुप्तचरों की सूचनाएँ सुल्तान तक इस डाक व्यवस्था के द्वारा केवल पाँच दिनों में ही पहुँच जाती थीं। इब्नबतूता के अनुसार भारत में दो प्रकार की डाक व्यवस्था थी –

  • अश्व डाक व्यवस्था तथा
  • पैदल डाक व्यवस्था।

(5) महिलाएँ दासियाँ सती तथा अमिक- इब्नबतूता के अनुसार बाजारों में दास अन्य वस्तुओं की तरह खुले आम बेचे जाते थे और नियमित रूप से भेंट स्वरूप दिए जाते थे। दासों में विभेद – इब्नबतूता के अनुसार दासों में काफी विभेद था। सुल्तान की सेवा में कार्यरत कुछ दासियाँ संगीत और गायन में निपुण थीं।

इसके अतिरिक्त सुल्तान अपने अमीरों पर नजर रखने के लिए दासियों को भी नियुक्त करता था। घरेलू श्रम के लिए दासों का प्रयोग इब्नबतूता के अनुसार दासों का प्रायः घरेलू श्रम के लिए प्रयोग किया जाता था। ये लोग पालकी या डोले में पुरुषों और महिलाओं को ले जाते थे। घरेलू श्रम के काम में लगे हुए दासों एवं दासियों की कीमत बहुत कम होती थी। अतः अधिकांश परिवार एक-दो दास तो रखते ही थे।

JAC Class 12 History Important Questions Chapter 5 यात्रियों के नज़रिए : समाज के बारे में उनकी समझ

प्रश्न 9.
विदेशी यात्रियों द्वारा वर्णित भारतीय महिलाओं, दास-दासियों एवं सती प्रथा का विवरण प्रस्तुत कीजिये।
अथवा
इब्नबतूता एवं बर्नियर द्वारा वर्णित भारतीय महिलाओं, दास-दासियों एवं सती प्रथा का विवरण प्रस्तुत कीजिये।
उत्तर:
विदेशी यात्रियों द्वारा वर्णित दास- दासियों एवं सती प्रथा का विवरण –
(1) दास-दासियाँ – इब्नबतूता के अनुसार भारतीय बाजारों में दास अन्य वस्तुओं की भाँति खुले आम बेचे जाते थे और नियमित रूप से भेंटस्वरूप दिए जाते थे। सिन्ध पहुँचने पर इब्नबतूता ने सुल्तान मुहम्मद बिन तुगलक के लिए भेंट स्वरूप घोड़े, ऊँट तथा दास खरीदे। मुल्तान पहुँचने पर उसने गवर्नर को किशमिश तथा बादाम के साथ एक दास और घोड़ा भेंट के रूप में दिए। इब्नबतूता के अनुसार मुहम्मद बिन तुगलक ने नसीरुदीन नामक धर्मोपदेशक के प्रवचन से प्रसन्न होकर उसे एक लाख टके तथा तथा दो सौ दास दिए थे।

(2) दासों में विभेद इब्नबतूता के अनुसार दासों में काफी विभेद था। सुल्तान की सेवा में कार्यरत कुछ दासियाँ संगीत और गायन में निपुण थीं। इब्नबतूता सुल्तान की बहन की शादी के अवसर पर उनके प्रदर्शन से खूब आनन्दित हुआ।

(3) अमीरों की गतिविधियों की जानकारी के लिए दासियों की नियुक्ति इनवता के अनुसार सुल्तान मुहम्मद बिन तुगलक अपने अमीरों की गतिविधियों पर नजर रखने के लिए दासियों को नियुक्त करता था।

(4) दासों का घरेलू श्रम के लिए प्रयोग करना- इब्नबतूता के अनुसार दासों को सामान्यतः घरेलू श्रम के लिए ही प्रयुक्त किया जाता था। ये लोग पालकी या डोले में महिलाओं को ले जाते थे। दासों का मूल्य, विशेष रूप से उन दासियों का मूल्य, जिनका प्रयोग घरेलू श्रम के लिए किया जाता था, बहुत कम होता था। इसी वजह से अधिकांश परिवार कम से कम एक या दो दास तो रखते ही थे।

(5) सती प्रथा बर्नियर के अनुसार भारत में सती प्रथा प्रचलित थी। यद्यपि कुछ महिलाएँ प्रसन्नता से चिता में जल कर मर जाती थीं, परन्तु अनेक विधवाओं को मरने के लिए बाध्य किया जाता है। बर्नियर ने लाहौर में एक बारह वर्षीय विधवा की बलि का उल्लेख करते हुए लिखा है कि “इस बालिका को उसकी इच्छा के विरुद्ध जीवित जला दिया था। यह बालिका काँपते हुए बुरी तरह रो रही थी परन्तु तीन या चार ब्राह्मणों तथा एक बूढ़ी महिला की सहायता से उस अनिच्छुक बालिका को जबरन चिता स्थल की ओर ले जाया गया, उसे लकड़ियों पर बिठाया गया और उसके हाथ तथा पैर बाँध दिए गए ताकि वह भाग न जाए। इस प्रकार इस निर्दोष बालिका को जीवित जला दिया गया।”

प्रश्न 10.
“बर्नियर के विवरणों ने अठारहवीं शताब्दी से पश्चिमी विचारकों को प्रभावित किया।” विवेचना कीजिए।
उत्तर:
बर्नियर के विवरणों द्वारा पश्चिमी विचारकों को प्रभावित करना बर्नियर के विवरणों ने अठारहवीं शताब्दी से निम्नलिखित पश्चिमी विचारकों को प्रभावित किया –

(1) मान्टेस्क्यू-फ्रांस का प्रसिद्ध दार्शनिक मान्टेस्क्यू बर्नियर के विवरणों से बड़ा प्रभावित हुआ। उसने बर्नियर के वृत्तांत का प्रयोग प्राच्य निरंकुशवाद के सिद्धान्त को विकसित करने में किया। इस सिद्धान्त के अनुसार एशिया (प्राच्य अथवा पूर्व) में शासक अपनी प्रजा के ऊपर असीम प्रभुत्व का उपभोग करते थे तथा प्रजा को दासता तथा गरीबी की स्थितियों में रखा जाता था। इस तर्क का आधार यह था कि सम्पूर्ण भूमि पर राजा का स्वामित्व होता था तथा निजी सम्पत्ति अस्तित्व में नहीं थी। इस दृष्टिकोण के अनुसार राजा और उनके अमीर वर्ग को छोड़कर प्रत्येक व्यक्ति कठिनाई से गुजारा कर पाता था।

(2) कार्ल मार्क्स प्रसिद्ध साम्यवादी विचारक कार्ल मार्क्स भी बर्नियर के विवरणों से प्रभावित हुआ। उसने उन्नीसवीं सदी के इस विचार को एशियाई उत्पादन शैली के सिद्धान्त के रूप में और आगे बढ़ाया। उसने यह तर्क प्रस्तुत किया कि भारत तथा अन्य एशियाई देशों में उपनिवेशवाद से पहले राज्य अधिशेष का अधिग्रहण कर लेता था। इसके फलस्वरूप एक ऐसे समाज का प्रादुर्भाव हुआ जो काफी स्वायत्त तथा आन्तरिक से समताण से बना था। इन ग्रामीण समुदायों पर से ( का नियन्त्रण होता था। जब तक अधिशेष की आपूर्ति निरन्तर जारी रहती थी इनकी स्वायत्तता का सम्मान किया जाता था। यह एक निष्क्रिय प्रणाली मानी जाती थी।

ग्रामीण समाज का चित्रण सच्चाई से दूर होना- परन्तु ग्रामीण समाज का यह चित्रण सच्चाई से बहुत दूर था सोलहवीं तथा सत्रहवीं शताब्दी में ग्रामीण समाज में चारित्रिक रूप से बड़े पैमाने पर सामाजिक और आर्थिक विभेद था। एक ओर बड़े जमींदार थे जो भूमि पर उच्चाधिकारों का उपभोग करते थे और दूसरी ओर ‘अस्पृश्य’ भूमिविहीन श्रमिक इन दोनों के बीच में बड़ा किसान था जो किराए के श्रम का प्रयोग करता था और माल उत्पादन में जुटा रहता था। इसके साथ ही कुछ छोटे किसान भी थे जो कठिनाई से ही अपने गुजारे योग्य उत्पादन कर पाते थे।

JAC Class 12 History Important Questions Chapter 5 यात्रियों के नज़रिए : समाज के बारे में उनकी समझ

प्रश्न 11.
फ्रांस्वा बर्नियर द्वारा की गई ‘पूर्व और पश्चिम की तुलना को उल्लेखित कीजिए।
अथवा
बर्नियर द्वारा वर्णित भारतीय यात्रा वृत्तान्त का विवेचन कीजिए।
अथवा
“बर्नियर का ग्रन्थ ‘ट्रेवल्स इन मुगल एम्पायर अपने गहन प्रेक्षण, आलोचनात्मक अन्तर्दृष्टि तथा गहन चिन्तन के लिए उल्लेखनीय है।” व्याख्या कीजिए।
उत्तर:
(1) फ्रांस्वा बर्नियर द्वारा की गई ‘पूर्व और पश्चिम’ की तुलना बर्नियर प्राय: भारत में जो देखता था, उसकी तुलना यूरोपीय स्थिति से करता था। वह यूरोपीय रा संस्कृति की श्रेष्ठता का प्रबल समर्थक था यूरोपीय स्थितियों के मुकाबले में वह भारत की स्थितियों को दयनीय दर्शाना चाहता था। यही कारण है कि लगभग प्रत्येक दृष्टान्त में बर्नियर ने भारत की स्थिति को यूरोप में हुए विकास की अ तुलना में दयनीय बताया।

(2) भूमि स्वामित्व का प्रश्न बर्नियर के अनुसार भारत और यूरोप के बीच मूल भिन्नताओं में से एक भारत में निजी भूस्वामित्व का अभाव था। बर्नियर के अनुसार भूमि = पर राजकीय स्वामित्व राज्य तथा उसके निवासियों, दोनों के लिए हानिकारक था। उसका विचार था कि मुगल साम्राज्य में सम्राट समस्त भूमि का स्वामी था जो इसे अपने अमीरों में बांटता था। इसके अर्थव्यवस्था और समाज दोनों के लिए विनाशकारी परिणाम होते थे।

(3) किसानों की दयनीय दशा-बर्नियर के अनुसार ग्रामीण अंचलों में रहने वाले कृषकों की दशा बड़ी दयनीय थी। यहाँ की खेती अच्छी नहीं थी और श्रमिकों के अभाव में कृषि योग्य भूमि का एक बड़ा भाग भी कृषि -विहीन रह जाता था। कई श्रमिक गवर्नरों के द्वारा किए गए अत्याचारों के कारण मौत के मुंह में चले जाते थे। अपने स्वामियों की माँगों को पूरा न करने के कारण अनेक किसानों को उनके गुजारा करने के साधनों से वंचित कर दिया जाता था तथा उनके बच्चों को दास बना लिया जाता था।

(4) मुगल साम्राज्य का स्वरूप- बर्नियर के अनुसार मुगल साम्राज्य का राजा ‘भिखारियों और क्रूर लोगों का राजा था। मुगल राज्य के शहर और नगर विनष्ट तथा ‘खराब वायु’ से दूषित थे और इसके खेत ‘झाड़ीदार’ तथा ‘घातक दलदल से भरे हुए थे। इसका मात्र एक ही कारण था राजकीय भू-स्वामित्व।

(5) एक अधिक जटिल सामाजिक सच्चाई एक ओर वर्नियर कहता है कि भारतीय शिल्पकारों के पास अपने उत्पादों के विस्तार के लिए कोई प्रोत्साहन नहीं था क्योंकि समस्त लाभ राज्य के खजाने में चला जाता था। इसलिए उत्पादन सर्वत्र पतनोन्मुख था परन्तु इसके साथ ही बर्नियर यह भी स्वीकार करता है कि सम्पूर्ण विश्व से बड़ी माश में बहुमूल्य धातुएँ भारत में आती थीं क्योंकि उत्पादों का सोने और चाँदी के बदले निर्यात होता था. भारत में एक समृद्ध व्यापारिक समुदाय भी था।

(6) भारतीय शहर बर्नियर के अनुसार मुगलकालीन शहर ‘शिविर नगर’ थे। शिविर नगरों से उसका अभिप्राय उन नगरों से था, जो अपने अस्तित्व और बने रहने के लिए राजकीय शिविर पर निर्भर थे उसका विचार था कि ये नगर राजकीय दरबार के आगमन के साथ अस्तित्व में आते थे और इसके अन्यत्र चले जाने के बाद तेजी से विलुप्त हो जाते थे।

(7) व्यावसायिक वर्ग अन्य शहरी समूहों में व्यावसायिक वर्ग जैसे चिकित्सक (हकीम एवं वैद्य), अध्यापक (पंडित या मुल्ला), अधिवक्ता (वकील), चित्रकार, वास्तुविद् संगीतकार, सुलेखक आदि सम्मिलित थे।

JAC Class 12 History Important Questions Chapter 5 यात्रियों के नज़रिए : समाज के बारे में उनकी समझ

प्रश्न 12.
इब्नबतूता और अल बिरूनी के भारत यात्रा-वृत्तान्तों की तुलना कीजिए।
उत्तर:
इब्नबतूता और अल बिरूनी के भारत यात्रा वृत्तान्तों की तुलना इब्नबतूता तथा अल बिरुनी के भारत यात्रा वृत्तान्तों की तुलना निम्न प्रकार से की जा सकती है-

(1) भिन्न-भिन्न कालों से सम्बन्धित यात्रा वृत्तान्त- अल बिरुनी का या वृत्तान्त ग्यारहवीं शताब्दी के भारतीय सामाजिक एवं सांस्कृतिक जीवन से सम्बन्धित है, जबकि इब्नबतूता का यात्रा-वृत्तान्त चौदहवीं शताब्दी के भारतीय सामाजिक एवं सांस्कृतिक जीवन से सम्बन्धित है।

(2) विषय अल बिरूनी ने अपनी प्रसिद्ध रचना ‘किताब-उल-हिन्द’ में भारतीय धर्म और दर्शन, त्योहारों, खगोल विज्ञान, रीति-रिवाज तथा प्रथाओं, सामाजिक जीवन, 7 भार तौल तथा मापन विधियों, मूर्तिकला, कानून, मापतन्त्र विज्ञान आदि विषयों का विवेचन किया है। इब्नबतूता ने न अपनी प्रसिद्ध रचना ‘रिहला’ में भारतीय सामाजिक तथा सांस्कृतिक जीवन, डाक व्यवस्था, भारतीय शहरों, बाजारों नारियल तथा पान, दास-दासियों, सती प्रथा, भारत की बा जलवायु लोगों के रहन-सहन, वेशभूषा, कृषि व्यापार आदि न विषयों का विवेचन किया है।

(3) भारतीय लोगों के धर्म, दर्शन और विज्ञान के के बारे में वर्णन करना अल बिरुनी संस्कृत भाषा का था। उसने यहाँ के लोगों के दर्शन, धर्म, विज्ञान और ना विचारों के ग्रन्थों का अच्छा ज्ञान प्राप्त कर लिया था। उसके द्वारा वर्णित भारत की जाति व्यवस्था का वर्णन उसके संस्कृत ग्रन्थों के अध्ययन से पूर्णतया प्रभावित था। परन्तु से इब्नबतूता संस्कृत और अन्य भारतीय भाषाओं, यहाँ के क रीति-रिवाजों, दर्शन आदि से अपरिचित थे।

(4) साहित्यिक ग्रन्थों एवं यात्राओं से अर्जित अनुभव को महत्त्वपूर्ण स्रोत मानना अल बिरूनी न साहित्यिक ग्रन्थों से प्राप्त अनुभव को ज्ञान का अधिक वाय महत्त्वपूर्ण खोत मानता था, परन्तु इब्नबतूता साहित्यिक नए ग्रन्थों के स्थान पर चाशओं से अर्जित अनुभव को महत्त्वपूर्ण नगर स्त्रोत मानता था।

(5) उद्देश्य अल बिरुनी के बाश-वृत्तान्त के उद्देश्य ये –

  • उन लोगों के लिए सहायक जो हिन्दुओं से धार्मिक विषयों पर चर्चा करना चाहते थे तथा
  • ऐसे लोगों के लिए एक सूचना का संग्रह जो उनके साथ सम्बद्ध होना चाहते थे इब्नबतूता के भारत यात्रा के वृत्तान्त का उद्देश्य अपरिचित वस्तुओं, राज्यों, घटनाओं आदि से अपने देशवासियों को परिचित कराना था। वह चाहता था कि श्रोता अथवा पाठक सुदूर देशों के वृत्तान्तों से पूरी तरह से प्रभावित हो सकें। इसी कारण इब्नबतूता ने पान और नारियल डाक- व्यवस्था, भारतीय शहरों के वैभव आदि के बारे में विस्तार से लिखा।

प्रश्न 13.
एक ओर बर्नियर ने भारतीय समाज को दरिद्र लोगों के समरूप जनसमूह से निर्मित वर्णित किया है, तो दूसरी ओर उसने एक अधिक सामाजिक आर्थिक सच्चाई को भी उजागर किया है। विवेचना कीजिये।
उत्तर:
भारतीय समाज दरिद्र लोगों के जनसमूह के रूप में बर्नियर ने भारतीय समाज को दरिद्र लोगों के जनसमूह के रूप में दर्शाते हुए लिखा है कि भारत के बड़े ग्रामीण अंचलों में से कई अंचल रेतीली भूमियाँ या बंजर पर्वत है। यहाँ की खेती भी अच्छी नहीं है प्रान्तीय गवर्नरों के अत्याचारों के कारण अनेक गरीब मजदूर मर जाते हैं। जब गरीब लोग अपने भू-स्वामियों की मांगों को पूरा नहीं कर पाते, तो उन्हें न केवल जीवन निर्वहन के साधनों से वंचित कर दिया जाता है, बल्कि उनके बच्चों को दास भी बना लिया जाता है। विवश होकर बहुत से गरीब किसान गाँव छोड़कर चले जाते हैं।

(1) बहुमूल्य धातुओं का भारत में आना एक अधिक जटिल सामाजिक और आर्थिक सच्चाई को दर्शाना- दूसरी ओर बर्नियर भारत में व्याप्त एक अधिक प्रसिद्ध सामाजिक और आर्थिक सच्चाई को दर्शाते हुए लिखता है कि शिल्पकारों के पास अपने उत्पादों को उत्तम बनाने के लिए कोई प्रोत्साहन नहीं था, क्योंकि सारा मुनाफा राज्य को होता था फिर भी सम्पूर्ण विश्व से बड़ी मात्रा में बहुमूल्य धातुएँ भारत में आती थीं क्योंकि उत्पादों का सोने और चाँदी के बदले निर्यात होता था। इससे देश में सोना और चाँदी इकट्ठा होता था। बर्नियर के अनुसार भारत में एक समृद्ध व्यापारिक समुदाय का भी अस्तित्व था जो लम्बी दूरी के विनिमय में संलग्न था।

(2) उपजाऊ भूमि और उन्नत शिल्प-बर्नियर ने लिखा है कि भारत का एक बड़ा भू-भाग अत्यन्त उपजाऊ है। बंगाल मित्र से न केवल चावल, मकई तथा जीवन की अन्य आवश्यक वस्तुओं के उत्पादन में बल्कि रेशम कपास, नील आदि के उत्पादन में भी आगे है भारत के अनेक हिस्सों में खेती अच्छी होती है। यहाँ के शिल्पकार अनेक | वस्तुओं के उत्पादन में संलग्न रहते हैं। ये शिल्पकार गलीचों, जरी कसीदाकारी, कढ़ाई, सोने और चांदी के वस्त्रों तथा विभिन्न प्रकार के रेशम और सूती वस्त्रों के निर्माण का कार्य करते हैं। रेशमी तथा सूती वस्त्रों का प्रयोग केवल भारत में ही नहीं होता, अपितु ये विदेशों में भी निर्यात किये जाते हैं।

(3) भारत में सोना और चाँदी का संग्रह होना- बर्नियर ने यह भी लिखा है कि सम्पूर्ण विश्व के सभी भागों में संचलन के बाद सोना और चाँदी का भारत में आकर कुछ सीमा तक संग्रह हो जाता है।

JAC Class 12 History Important Questions Chapter 5 यात्रियों के नज़रिए : समाज के बारे में उनकी समझ

प्रश्न 14.
अल बिरूनी ने ब्राह्मणवादी व्यवस्था की अपवित्रता की मान्यता को क्यों अस्वीकार कर दिया? क्या जाति व्यवस्था के नियमों का पालन पूर्ण कठोरता से किया जाता था? अल बिरूनी ने भारत की वर्ण व्यवस्था का वर्णन किस प्रकार किया है?
उत्तर:
यद्यपि अल – विरुनी ने भारतीय सामाजिक व्यवस्था में ब्राह्मणों द्वारा निर्मित जाति व्यवस्था को स्वीकार किया और उसकी मान्यता के लिए अन्य देशों के समुदायों में इस व्यवस्था के प्रतिरूपों के उदाहरणों को भी प्रस्तुत किया, फिर भी वह अपवित्रता की मान्यता को स्वीकार न कर अल बिरुनी ने लिखा है कि, “हर वह वस्तु जो अपवित्र हो जाती है, अपनी खोई हुई पवित्रता को पुनः पाने का प्रयास करती है और सफल होती है सूर्य हवा को स्वच्छ करता है और समुद्र में नमक पानी को गन्दा होने से बचाता है।” अल-विरुनी जोर देकर कहता है कि यदि ऐसा नहीं होता तो पृथ्वी पर जीवन असम्भव हो जाता। उसके अनुसार अपवित्रता की अवधारणा प्राकृतिक नियमों के विरुद्ध है।

अल बिरूनी ने जाति व्यवस्था के सम्बन्ध में जो भी विवरण दिया है; वह पूर्णतया संस्कृत ग्रन्थों के अध्ययन से प्रभावित है। जिन नियमों का वर्णन इन ग्रन्थों में ब्राह्मणवादी जाति-व्यवस्था को संचालित करने हेतु किया गया है; वह वास्तविक रूप में समाज में उतनी कठोरता से लागू नहीं थी। इनमें लचीलापन था उदाहरण हेतु परित्यक्त लोग जिन्हें अंत्यज कहते थे जो इस जाति व्यवस्था में शामिल नहीं थे, आर्थिक तन्त्र में उन्हें भी शामिल किया गया था। भले ही उनसे सस्ता श्रम प्राप्त करने के लिए ऐसा किया जाता हो।
अल- विरुनी की भारत की सामाजिक व्यवस्था की जानकारी प्राचीन भारतीय संस्कृत ग्रन्थों पर आधारित थी।

इसी आधार पर अल बिरूनी ने भारत की वर्ण व्यवस्था का वर्णन निम्न प्रकार से किया है –

  • ब्राह्मण अल बिरूनी लिखता है कि ब्राह्मण सबसे सर्वोच्च वर्ण था क्योंकि हिन्दू ग्रन्थों की मान्यताओं के अनुसार इनकी उत्पत्ति आदि देव ब्रह्मा के मुख से हुई और मुख का स्थान सबसे उच्च है; इसी कारण हिन्दू जाति में से सबसे उच्च माने जाते हैं।
  • क्षत्रिय क्षत्रियों की उत्पत्ति आदि देव ब्रह्मा के कन्धों व हाथों से मानी गई है। मुख के बाद द्वितीय स्थान कन्धों व याँहों का है अतः उन्हें वर्ण व्यवस्था में ब्राह्मणों से कुछ नीचे द्वितीय स्थान पर रखा गया।
  • वैश्य वैश्य वर्ण की उत्पत्ति ब्रह्मा के उदर व जंघा भाग से मानी गई इसलिए इन्हें तीसरे स्थान पर रखा गया।
  • शूद्र-शूद्र वर्ण की उत्पत्ति ब्रह्मा के चरणों से मानी गयी है, अतः वैश्य और शूद्रों के बीच अल बिरुनी अधिक अन्तर नहीं मानता था। अल बिरुनी के अनुसार, यद्यपि वर्ग-भेद तो था फिर भी सभी लोग एक साथ एक ही शहर या गांव में समरसता के साथ रहते थे।

JAC Class 12 History Important Questions Chapter 4 विचारक, विश्वास और इमारतें : सांस्कृतिक विकास

Jharkhand Board JAC Class 12 History Important Questions Chapter 4 विचारक, विश्वास और इमारतें : सांस्कृतिक विकास Important Questions and Answers.

JAC Board Class 12 History Important Questions Chapter 4 विचारक, विश्वास और इमारतें : सांस्कृतिक विकास

बहुविकल्पीय प्रश्न (Multiple Choice Questions)

1. साँची का स्तूप किस राज्य में स्थित है –
(अ) उत्तर प्रदेश
(स) मध्य प्रदेश
(ब) बिहार
(द) गुजरात
उत्तर:
(स) मध्य प्रदेश

2. ऋग्वेद का संकलन कब किया गया?
(अ) 2500-2000 ई. पूर्व
(ब) 2000-1500 ई. पूर्व
(स) 1000-500 ई. पूर्व
(द) 1500 से 1000 ई. पूर्व
उत्तर:
(द) 1500 से 1000 ई. पूर्व

3. महावीर स्वामी से पहले कितने शिक्षक (तीर्थंकर) हो चुके थे?
(अ) 24
(स) 14
(ब) 20
(द) 23
उत्तर:
(द) 23

JAC Class 12 History Important Questions Chapter 4 विचारक, विश्वास और इमारतें : सांस्कृतिक विकास

4. बौद्धसंघ में सम्मिलित होने वाली प्रथम भिक्षुणी थी –
(अ) महामाया
(ब) पुन्ना
(स) महाप्रजापति गोतमी
(द) सुलक्षणी
उत्तर:
(स) महाप्रजापति गोतमी

5. जिन ग्रन्थों में बुद्ध की शिक्षाओं का संकलन किया गया है, उन्हें कहा जाता है –
(अ) बौद्धग्रन्थ
(स) महावंश
(ब) दीपवंश
(द) त्रिपिटक
उत्तर:
(द) त्रिपिटक

6. किस ग्रन्थ में बौद्ध भिक्षुओं और भिक्षुणियों के लिए नियम संकलित किए गए हैं?
(अ) अभिधम्मपिटक
(ब) विनयपिटक
(स) सुत्तपिटक
(द) बुद्धचरित
उत्तर:
(ब) विनयपिटक

7. अमरावती के स्तूप की खोज सर्वप्रथम कब हुई ?
(अ) 1896
(ब) 1796
(स) 1717
(द) 1817
उत्तर:
(ब) 1796

8. साँची की खोज कब हुई?
(अ) 1818 ई.
(ब) 1718 ई.
(स) 1918 ई.
(द) 1878 ई.
उत्तर:
(अ) 1818 ई.

9. किस अंग्रेज लेखक ने साँची पर लिखे अपने महत्त्वपूर्ण ग्रन्थों को सुल्तान जहाँ बेगम को समर्पित किया?
(अ) जेम्स टॉड
(ब) जॉर्ज थॉमस
(स) जॉन मार्शल
(द) जरथुस्व
उत्तर:
(स) जॉन मार्शल

10. किस दार्शनिक का सम्बन्ध चूनान से है?
(अ) सुकरात
(ख) खुंगत्सी
(स) बुद्ध
(द) कनिंघम
उत्तर:
(अ) सुकरात

11. समकालीन बौद्ध ग्रन्थों में हमें कितने सम्प्रदायों या चिन्तन परम्पराओं की जानकारी मिलती है?
(अ) 10
(स) 94
(ब) 24
(द) 64
उत्तर:
(द) 64

JAC Class 12 History Important Questions Chapter 4 विचारक, विश्वास और इमारतें : सांस्कृतिक विकास

12. किस देश से दीपवंश एवं महावंश जैसे क्षेत्र विशेष के बौद्ध इतिहास का सम्बन्ध है-
(अ) भारत
(स) चीन
(ब) नेपाल
(द) श्रीलंका
उत्तर:
(द) श्रीलंका

13. त्रिपिटक की रचना कब हुई थी?
(अ) बुद्ध के जन्म से पूर्व
(ब) बुद्ध के महापरिनिर्वाण प्राप्त करने के पश्चात्
(स) बुद्ध के जीवन काल में
(द) इनमें से कोई नहीं
उत्तर:
(ब) बुद्ध के महापरिनिर्वाण प्राप्त करने के पश्चात्

14. महात्मा बुद्ध की शिक्षाओं का संकलन निम्नलिखित में से किसमें है?
(अ) सुत्तपिटक
(ब) विनय पिटक
(स) जातक
(द) इनमें से कोई नहीं
उत्तर:
(अ) सुत्तपिटक

15. महात्मा बुद्ध का जन्म हुआ था –
(अ) कपिलवस्तु
(ब) लुम्बिनी
(स) सारनाथ
(द) बोधगया
उत्तर:
(ब) लुम्बिनी

रिक्त स्थानों की पूर्ति कीजिए।

1. भोपाल की नवाब शाहजहाँ बेगम की आत्मकथा ………….. है।
2. शाहजहाँ बेगम की उत्तराधिकारी ……………. थीं।
3. ………… और …………… जैसे जटिल यज्ञ सरदार और राजा किया करते थे।
4. महात्मा बुद्ध के दर्शन से जुड़े विषय …………. पिटक में आए।
5. ज्यादातर पुराने बौद्ध ग्रन्थ …………… भाषा में हैं।
6. बौद्ध धर्म के पूर्व एशिया में फैलने के पश्चात् …………. और ………… जैसे तीर्थयात्री बौद्ध ग्रन्थों की खोज में चीन से भारत आए।
7. वे महापुरुष जो पुरुषों और महिलाओं को जीवन की नदी के पार पहुँचाते हैं उन्हें …………….. कहते हैं।
उत्तर:
1. ताज-उल- इकबाल
2. सुल्तान जहाँ बेगम
3. राजसूय, अश्वमेध
4. अभिधम्म
5. पालि
6. फा- शिएन, श्वैन त्सांग
7. तीर्थंकर

अतिलघूत्तरात्मक प्रश्न

प्रश्न 1.
महात्मा बुद्ध ने अपना प्रथम उपदेश कहाँ दिया?
उत्तर:
महात्मा बुद्ध ने अपना प्रथम उपदेश सारनाथ में दिया।

प्रश्न 2.
जैन दर्शन की सबसे महत्त्वपूर्ण अवधारणा क्या है?
उत्तर:
जैन दर्शन के अनुसार सम्पूर्ण संसार प्राणवान है

JAC Class 12 History Important Questions Chapter 4 विचारक, विश्वास और इमारतें : सांस्कृतिक विकास

प्रश्न 3.
मन्दिर स्थापत्य कला में गर्भगृह एवं शिखर से आप क्या समझते हैं?
उत्तर:
(1) गर्भगृह- मन्दिर का चौकोर कमरा
(2) शिखर- गर्भगृह के ऊपर ढाँचा।

प्रश्न 4.
बौद्ध संघ की संचालन पद्धति को स्पष्ट कीजिये।
उत्तर:
बौद्ध संघ की संचालन पद्धति गणों और संघों की परम्परा पर आधारित थी। लोग बातचीत के द्वारा एकमत होते थे।

प्रश्न 5.
बुद्ध के जीवन से जुड़े बोधगया एवं सारनाथ नामक स्थानों का महत्त्व बताइये।
उत्तर:
(1) बोधगया में. बुद्ध ने ज्ञान प्राप्त किया था।
(2) सारनाथ में बुद्ध ने प्रथम उपदेश दिया था।

प्रश्न 6.
यदि आप भोपाल की यात्रा करेंगे, तो वहाँ किस बौद्धकालीन स्तूप को देखना चाहेंगे?
उत्तर:
साँची के स्तूप को।

प्रश्न 7.
महात्मा बुद्ध का बचपन का नाम क्या था?
उत्तर:
सिद्धार्थ

प्रश्न 8.
साँची का स्तूप कहाँ स्थित है?
उत्तर:
साँची का स्तूप मध्य प्रदेश की राजधानी भोपाल से 20 मील उत्तर-पूर्व में स्थित साँची कनखेड़ा नामक एक गाँव में स्थित है।

प्रश्न 9.
थेरीगाथा बौद्ध ग्रन्थ किस पिटक का हिस्सा है?
उत्तर:
सुत्तपिटक।

प्रश्न 10.
कैलाशनाथ मन्दिर कहाँ स्थित है?
उत्तर:
महाराष्ट्र में।

JAC Class 12 History Important Questions Chapter 4 विचारक, विश्वास और इमारतें : सांस्कृतिक विकास

प्रश्न 11.
महावीर स्वामी का बचपन का नाम क्या
उत्तर:
वर्धमान।

प्रश्न 12.
ऐसे तीन स्थानों का उल्लेख कीजिये, जहाँ स्तूप बनाए गए थे।
उत्तर:
(1) भरहुत
(2) साँची
(3) अमरावती।

प्रश्न 13.
साँची के स्तूप को आर्थिक अनुदान देने वाले दो शासकों के नाम लिखिए।
उत्तर:
(1) शाहजहाँ बेगम
(2) सुल्तानजहाँ बेगम।

प्रश्न 14.
बुद्ध की शिक्षाएँ किन ग्रन्थों में संकलित हैं?
उत्तर:
त्रिपिटक में

प्रश्न 15.
साँची के स्तूप को किसने संरक्षण प्रदान किया?
उत्तर:
साँची के स्तूप को भोपाल के शासकों ने संरक्षण प्रदान किया जिनमें शाहजहाँ बेगम एवं सुल्तानजहाँ बेगम प्रमुख हैं।

प्रश्न 16.
“इस देश की प्राचीन कलाकृतियों की लूट होने देना मुझे आत्मघाती और असमर्थनीय नीति लगती है।” यह कथन किस पुरातत्त्ववेत्ता का है?
उत्तर:
एच. एच. कोल।

प्रश्न 17.
ईसा पूर्व प्रथम सहस्राब्दी का काल विश्व इतिहास में एक महत्त्वपूर्ण मोड़ क्यों माना जाता है ?
उत्तर:
क्योंकि इस काल में ईरान में जरस्थुस्व, चीन में खुंगस्ती, यूनान में सुकरात, प्लेटो, अरस्तू एवं भारत में महावीर व बुद्ध जैसे चिन्तकों का उदय हुआ।

प्रश्न 18.
अजन्ता की गुफाएँ कहाँ स्थित हैं?
उत्तर:
महाराष्ट्र में

प्रश्न 19.
पौराणिक हिन्दू धर्म में किन दो प्रमुख देवताओं की पूजा प्रचलित थी?
उत्तर:
(1) विष्णु
(2) शिव।

प्रश्न 20.
सबसे प्राचीन कृत्रिम गुफाओं का निर्माण किस शासक ने करवाया था ?
उत्तर:
अशोक ने आजीविक सम्प्रदाय के सन्तों हेतु निर्माण करवाया था।

JAC Class 12 History Important Questions Chapter 4 विचारक, विश्वास और इमारतें : सांस्कृतिक विकास

प्रश्न 21.
धेरी का क्या अभिप्राय है?
उत्तर:
थेरी का अर्थ है ऐसी महिलाएँ जिन्होंने निर्वाण प्राप्त कर लिया हो।

प्रश्न 22.
बौद्ध धर्म की दो शिक्षाएँ बताइये।
उत्तर:
(1) विश्व अनित्य है।
(2) यह संसार दुःखों का घर है।

प्रश्न 23.
जैन धर्म की दो शिक्षाएँ बताइये।
उत्तर:
(1) जीवों के प्रति अहिंसा
(2) कर्मवाद और पुनर्जन्म में विश्वास।

प्रश्न 24.
बौद्ध संघ में सम्मिलित होने वाली प्रथम महिला कौन थी?
उत्तर:
बुद्ध की उपमाता महाप्रजापति गोतमी।

प्रश्न 25.
जैन धर्म के महापुरुष क्या कहलाते थे?
उत्तर:
तीर्थंकर।

प्रश्न 26.
जैन धर्म के 23वें तीर्थकर कौन थे?
उत्तर:
पार्श्वनाथ

प्रश्न 27.
जैन दर्शन की सबसे महत्त्वपूर्ण अवधारणा क्या है?
उत्तर:
जैन दर्शन की सबसे महत्त्वपूर्ण अवधारणा यह है कि सम्पूर्ण विश्व प्राणवान है।

प्रश्न 28.
राजसूय एवं अश्वमेध यज्ञों का अनुष्ठान किनके द्वारा कराया जाता था ?
उत्तर:
ब्राह्मण पुरोहितों द्वारा

प्रश्न 29.
समकालीन बौद्ध ग्रन्थों में कितने सम्प्रदायों एवं चिन्तन परम्पराओं का उल्लेख मिलता है?
उत्तर:
64 सम्प्रदाय या चिन्तन परम्पराओं का।

प्रश्न 30.
कुटागारशालाओं का शाब्दिक अर्थ बताइये।
उत्तर:
नुकीली छत वाली झोंपड़ी।

JAC Class 12 History Important Questions Chapter 4 विचारक, विश्वास और इमारतें : सांस्कृतिक विकास

प्रश्न 31.
शिक्षक अपने दार्शनिक विचारों की चर्चा कहाँ करते थे?
उत्तर:
कुटागारशालाओं या उपवनों में।

प्रश्न 32.
महावीर स्वामी तथा बुद्ध ने किसका विरोध किया?
उत्तर:
वेदों के प्रभुत्व का।

प्रश्न 33.
चीनी यात्री फा-शिएन तथा श्वैन-त्सांग ने भारत की यात्रा क्यों की?
उत्तर:
बौद्ध ग्रन्थों की खोज के लिए।

प्रश्न 34.
प्रमुख नियतिवादी बौद्ध भिक्षु कौन थे?
उत्तर:
मक्खलि गोसाल।

प्रश्न 35.
प्रमुख भौतिकवादी बौद्ध दार्शनिक कौन थे?
उत्तर:
अजीत केसकम्बलिन।

प्रश्न 36.
महावीर स्वामी से पहले कितने तीर्थकर हो चुके थे?
उत्तर:
231

प्रश्न 37.
अमरावती के स्तूप की खोज कब हुई ?
उत्तर:
1796 ई. में

JAC Class 12 History Important Questions Chapter 4 विचारक, विश्वास और इमारतें : सांस्कृतिक विकास

प्रश्न 38.
विष्णु के कितने अवतारों की कल्पना की गई ?
उत्तर:
दस अवतारों की।

प्रश्न 39.
बराबर (बिहार) की गुफाओं का निर्माण किसने करवाया था ?
उत्तर:
अशोक ने।

प्रश्न 40.
अशोक ने किस सम्प्रदाय के सन्तों के लिए बराबर की गुफाओं का निर्माण करवाया था?
उत्तर:
आजीविक सम्प्रदाय के सन्तों के लिए।

प्रश्न 41.
कैलाशनाथ के मन्दिर कहाँ स्थित हैं?
उत्तर:
एलोरा (महाराष्ट्र) में।

प्रश्न 42.
कैलाशनाथ के मन्दिर का निर्माण कब करवाया गया था?
उत्तर:
आठवीं शताब्दी में।

प्रश्न 43.
यूनानी शैली से प्रभावित मूर्तियाँ किन क्षेत्रों से प्राप्त हुई हैं?
उत्तर:
तक्षशिला और पेशावर से।

प्रश्न 44.
प्रतीकों द्वारा बुद्ध की स्थिति किस प्रकार दिखाई जाती है? दो उदाहरण दीजिये।
उत्तर:
बुद्ध के ‘अध्यान की दशा’ को ‘रिक्त स्थान’ तथा ‘महापरिनिर्वाण’ को ‘स्तूप’ के द्वारा दिखाया जाता है।

प्रश्न 45.
ऐसे चार सामाजिक समूहों के नाम बताइये जिनमें से बुद्ध के अनुयायी आए।
उत्तर:
(1) राजा
(2) धनवान
(3) गृहपति तथा
(4) सामान्य जन

प्रश्न 46.
बुद्ध द्वारा गृह त्याग के क्या कारण थे?
उत्तर:
नगर का भ्रमण करते समय एक वृद्ध व्यक्ति, एक रोगी, एक लाश और संन्यासी को देख कर बुद्ध ने घर त्याग दिया।

प्रश्न 47.
लुम्बिनी एवं कुशीनगर नामक स्थानों का महत्त्व बताइये।
उत्तर:
लुम्बिनी में बुद्ध का जन्म हुआ। कुशीनगर में बुद्ध ने निर्वाण प्राप्त किया। ये बौद्ध धर्म के पवित्र स्थान हैं।

प्रश्न 48.
जैन धर्म के अनुसार कर्मफल से मुक्ति कैसे पाई जा सकती है ?
उत्तर:
त्याग और तपस्या के द्वारा।

प्रश्न 49.
आपकी दृष्टि में बौद्ध धर्म के तेजी से प्रसार के क्या कारण थे? किन्हीं दो कारणों का उल्लेख कीजिये।
उत्तर:
(1) लोग समकालीन धार्मिक प्रथाओं से असन्तुष्ट थे।
(2) बौद्ध धर्म ने जाति प्रथा का विरोध किया, सामाजिक समानता पर बल दिया।

प्रश्न 50.
हीनयान और महायान बौद्ध सम्प्रदायों में अन्तर स्पष्ट कीजिये।
उत्तर:
महायानी बुद्ध को देवता मान कर उनकी पूजा करते थे, परन्तु हीनयानी बुद्ध को अवतार नहीं मानते थे।

JAC Class 12 History Important Questions Chapter 4 विचारक, विश्वास और इमारतें : सांस्कृतिक विकास

प्रश्न 51.
साँची का स्तूप कहाँ स्थित है?
उत्तर:
साँची का स्तूप भोपाल से बीस मील दूर उत्तर- पूर्व में स्थित साँची कनखेड़ा नामक एक गाँव में स्थित है।

प्रश्न 52.
ईसा पूर्व प्रथम सहस्राब्दी में विश्व में किन प्रसिद्ध चिन्तकों का उद्भव हुआ?
उत्तर:
ईरान में जरथुस्व, चीन में खुंगत्सी, यूनान में सुकरात, प्लेटो और अरस्तू तथा भारत में महावीर और बुद्ध का उद्भव हुआ।

प्रश्न 53.
ऋग्वेद का संकलन कब किया गया?
उत्तर:
ग्वेद का संकलन 1500 से 1000 ई. पूर्व में किया गया।

प्रश्न 54.
नियतिवादियों और भौतिकवादियों में क्या अन्तर था?
उत्तर:
नियतिवादियों के अनुसार सब कुछ पूर्व निर्धारित है परन्तु भौतिकवादियों के अनुसार दान, यज्ञ या चढ़ावा निरर्थक हैं।

प्रश्न 55.
कर्म के चक्र से मुक्ति के लिए क्या किया जाना आवश्यक है?
उत्तर:
कर्म के चक्र से मुक्ति के लिए त्याग और तपस्या किया जाना आवश्यक है।

प्रश्न 56.
जैन धर्म के पाँच व्रतों का उल्लेख कीजिए।
उत्तर:
(1) हत्या न करना
(2) चोरी न करना
(3) झूठ न बोलना
(4) ब्रह्मचर्य
(5) धन संग्रह न करना।

प्रश्न 57.
बुद्ध के संदेश भारत के बाहर किन-किन देशों में फैले ? नाम लिखिए।
अथवा
बौद्ध धर्म का प्रसार किन देशों में हुआ?
उत्तर:
बौद्ध धर्म का प्रसार सम्पूर्ण उपमहाद्वीप मध्य एशिया, चीन, कोरिया, जापान, श्रीलंका, म्यांमार, थाइलैंड तथा इंडोनेशिया में हुआ।

प्रश्न 58.
नगर का भ्रमण करते समय किन को देखकर सिद्धार्थ ने घर त्यागने का निश्चय कर लिया?
उत्तर:
एक वृद्ध व्यक्ति, एक रोगी, एक लाश और एक संन्यासी को देखकर सिद्धार्थ ने घर त्यागने का निश्चय कर लिया।

प्रश्न 59.
बौद्ध धर्म के अनुसार ‘संघ’ से क्या अभिप्राय है?
उत्तर:
युद्ध द्वारा स्थापित संघ भिक्षुओं की एक संस्था थी जो धम्म के शिक्षक बन गए।

प्रश्न 60.
संघ के भिक्षुओं की दैनिक चर्या का उल्लेख कीजिए।
उत्तर:
(1) भिक्षु सादा जीवन बिताते थे।
(2) वे उपासकों से भोजन दान पाने के लिए एक कटोरा रखते थे।

JAC Class 12 History Important Questions Chapter 4 विचारक, विश्वास और इमारतें : सांस्कृतिक विकास

प्रश्न 61.
बौद्ध धर्म के शीघ्र प्रसार के दो कारण बताइये।
उत्तर:
(1) लोग समकालीन धार्मिक प्रथाओं से असन्तुष्ट थे
(2) बौद्ध धर्म सामाजिक समानता पर बल देता था।

प्रश्न 62.
‘चैत्य’ किसे कहते हैं?
उत्तर:
शवदाह के बाद शरीर के कुछ अवशेष टीलों पर सुरक्षित रख दिए जाते थे। ये टीले चैत्य कहे जाने लगे।

प्रश्न 63.
बुद्ध के जीवन से जुड़े चार स्थानों का उल्लेख कीजिए।
उत्तर:
(1) लुम्बिनी ( बुद्ध का जन्म स्थान)
(2) बोधगया (ज्ञान प्राप्त होना)
(3) सारनाथ (प्रथम उपदेश देना)
(4) कुशीनगर (निर्वाण प्राप्त करना)।

प्रश्न 64.
स्तूप’ किसे कहते हैं?
उत्तर:
कुछ पवित्र स्थानों पर बुद्ध से जुड़े कुछ अवशेष जैसे उनकी अस्थियाँ गाड़ दी गई थीं। ये टीले स्तूप कहलाये।

प्रश्न 65.
किन लोगों के द्वारा स्तूपों को दान दिया जाता था?
उत्तर:
(1) राजाओं के द्वारा (जैसे सातवाहन वंश के राजा)
(2) शिल्पकारों तथा व्यापारियों की श्रेणियों द्वारा
(3) महिलाओं और पुरुषों के द्वारा।

प्रश्न 66.
स्तूप की संरचना के प्रमुख तत्त्वों का उल्लेख कीजिए।
उत्तर:
(1) मिट्टी का टीला (अंड)
(2) हर्मिका (उन्जे जैसा ढाँचा)
(3) यष्टि (हर्मिका से निकला मस्तूल)
(4) वेदिका (टीले के चारों ओर बनी वेदिका)।

प्रश्न 67.
महायान मत में बोधिसत्तों की अवधारणा को स्पष्ट कीजिए।
उत्तर:
बोधिसता परम करुणामय जीव थे जो अपने सत्कार्यों से पुण्य कमाते थे और इससे दूसरों की सहायता करते थे।

प्रश्न 68.
महायान मत से आप क्या समझते हैं?
उत्तर:
महायान मत में बुद्ध और बोधिसत्तों की मूर्तियों की पूजा की जाती थी।

प्रश्न 69.
हीनयान या थेरवाद’ से क्या अभिप्राय है?
उत्तर:
पुरानी बौद्ध परम्परा के अनुवायी स्वयं को धेरवादी कहते थे। वे पुराने, प्रतिष्ठित शिक्षकों के बताए रास्ते पर चलते थे।

प्रश्न 70.
कैलाशनाथ मन्दिर के बारे में आप क्या जानते हैं?
उत्तर:
कैलाशनाथ मन्दिर एलोरा (महाराष्ट्र) में स्थित है यह सारा ढाँचा एक चट्टान को काट कर तैयार किया गया है।

प्रश्न 71.
जैन धर्म के दो सिद्धान्तों का उल्लेख कीजिए
उत्तर:
(1) जीवों के प्रति अहिंसा
(2) जन्म और पुनर्जन्म का चक्र कर्म के द्वारा निर्धारित होना।

JAC Class 12 History Important Questions Chapter 4 विचारक, विश्वास और इमारतें : सांस्कृतिक विकास

प्रश्न 72.
जैन विद्वानों ने किन भाषाओं में अपने ग्रन्थों की रचना की?
उत्तर:
जैन विद्वानों ने प्राकृत, संस्कृत, तमिल आदि भाषाओं में अपने ग्रन्थों की रचना की।

प्रश्न 73.
बौद्ध धर्म के दो सिद्धान्तों का उल्लेख कीजिये।
उत्तर:
(1) विश्व अनित्य है और निरन्तर बदल रहा है।
(2) विश्व आत्माविहीन है क्योंकि यहाँ कुछ भी स्थायी या शाश्वत नहीं है।

प्रश्न 74.
स्तूप क्या हैं?
उत्तर:
स्तूप बुद्ध धर्म से जुड़े पवित्र टीले हैं। इनमें बुद्ध के शरीर के अवशेष अथवा उनके द्वारा प्रयोग की गई किसी वस्तु को गाड़ा गया था।

प्रश्न 75.
चैत्य क्या थे?
उत्तर:
शवदाह के पश्चात् बौद्धों के शरीर के कुछ अवशेष टीलों पर सुरक्षित रख दिए जाते थे। अन्तिम संस्कार से जुड़े इन टीलों को चैत्य कहा जाता था।

प्रश्न 76.
ऋग्वेद में किन सूक्तियों का संग्रह है?
उत्तर:
ऋग्वेद में इन्द्र, अग्नि, सोम आदि देवताओं की स्तुति से सम्बन्धित सूक्तियों का संग्रह है।

प्रश्न 77.
नियतिवादी कौन थे?
उत्तर:
नियतिवादी वे लोग थे जो विश्वास करते थे कि सुख और दुःख पूर्व निर्धारित मात्रा में माप कर दिए गए हैं।

प्रश्न 78.
भौतिकवादी कौन थे?
उत्तर:
भौतिकवादी लोग यह मानते थे कि दान, यज्ञ या चढ़ावा निरर्थक हैं। इस दुनिया या दूसरी दुनिया का अस्तित्व नहीं होता।

JAC Class 12 History Important Questions Chapter 4 विचारक, विश्वास और इमारतें : सांस्कृतिक विकास

प्रश्न 79.
‘सन्तचरित्र’ से क्या अभिप्राय है?
उत्तर:
संतचरित्र किसी संत या धार्मिक महापुरुष की जीवनी है। संतचरित्र संत की उपलब्धियों का गुणगान करते हैं।

प्रश्न 80.
विनयपिटक तथा सुत्तपिटक में किन शिक्षाओं का संग्रह था ?
उत्तर:
(1) विनयपिटक में बौद्ध मठों में रहने वाले लोगों के लिए नियमों का संग्रह था।
(2) सुत्तपिटक में महात्मा बुद्ध की शिक्षाओं का संग्रह था।

प्रश्न 81.
अभिधम्मपिटक’ से आप क्या समझते हैं ?
उत्तर:
अभिधम्मपिटक’ नामक ग्रन्थ में बौद्ध दर्शन से सम्बन्धित सिद्धान्तों का संग्रह था।

प्रश्न 82.
श्रीलंका के इतिहास पर प्रकाश डालने वाले बौद्ध ग्रन्थों का उल्लेख कीजिये।
उत्तर:
‘दीपवंश’ (द्वीप का इतिहास) तथा ‘महावंश’ (महान इतिहास) से श्रीलंका के इतिहास पर प्रकाश पड़ता है।

प्रश्न 83.
स्तूप को संस्कृत में क्या कहा जाता है?
उत्तर:
स्तूप को संस्कृत में टीला कहा जाता है।

प्रश्न 84.
कौनसा अनूठा ग्रन्थ सुत्तपिटक का हिस्सा
उत्तर:
थेरीगाथा ग्रन्थ सुत्तपिटक का हिस्सा है।

प्रश्न 85.
बिना अलंकरण वाले प्रारम्भिक स्तूप कौन- कौनसे हैं?
उत्तर:
साँची और भरहुत स्तूप बिना अलंकरण वाले प्रारम्भिक स्तूप हैं।

प्रश्न 86.
जैन धर्म की सबसे महत्वपूर्ण अवधारणा क्या है?
उत्तर:
जैन धर्म के अनुसार सम्पूर्ण संसार प्राणवान है; पत्थर, चट्टान और जल में भी जीवन होता है। प्रश्न 87. अमरावती का स्तूप किस राज्य में है? उत्तर- अमरावती का स्तूप गुंटूर (आन्ध्र प्रदेश) में स्थित है।

प्रश्न 88.
जेम्स फर्ग्युसन ने वृक्ष और सर्प पूजा का केन्द्र किसे माना ?
उत्तर:
जेम्स फर्ग्युसन ने वृक्ष और सर्प पूजा का केन्द्र साँची को माना।

JAC Class 12 History Important Questions Chapter 4 विचारक, विश्वास और इमारतें : सांस्कृतिक विकास

प्रश्न 89.
वराह ने किसकी रक्षा की थी?
उत्तर:
वराह ने पृथ्वी की रक्षा की थी।

प्रश्न 90.
बौद्धों का सबसे विशाल और शानदार स्तूप कौनसा था ?
उत्तर:
बौद्धों का सबसे विशाल और शानदार स्तूप अमरावती का स्तूप है।

प्रश्न 91.
भक्ति से क्या आशय है?
उत्तर:
भक्ति एक प्रकार की आराधना है। इसमें उपासक एवं ईश्वर के मध्य के रिश्ते को प्रेम तथा समर्पण का रिश्ता माना जाता है।

लघुत्तरात्मक प्रश्न

प्रश्न 1.
बौद्ध धर्म के व्यावहारिक पक्ष के बारे में सुत्तपिटक के उद्धरण पर प्रकाश डालिए।
उत्तर:
(1) मालिक को अपने नौकरों और कर्मचारियों की पाँच प्रकार से देखभाल करनी चाहिए उनकी क्षमता के अनुसार उन्हें काम देकर उन्हें भोजन और मजदूरी देकर, बीमार पड़ने पर उनकी परिचर्या करने, उनके साथ स्वादिष्ट भोजन बाँट कर और समय-समय पर उन्हें छुट्टी देकर।
(2) कुल के लोगों को पाँच तरह से श्रमणों और ब्राह्मणों की देखभाल करनी चाहिए अनुराग द्वारा, सदैव पुस् घर खुले रखकर तथा उनकी दिन-प्रतिदिन की आवश्यकताओं दिन की पूर्ति करके।

प्रश्न 2.
बौद्ध एवं जैन धर्म में कितनी समानता थी?
उत्तर:

  1. दोनों धर्म कर्मवाद और पुनर्जन्मवाद में विश्वास करते हैं।
  2. दोनों धर्म अहिंसा के सिद्धान्त में विश्वास करते की में हैं।
  3. दोनों धर्म अनीश्वरवादी हैं।
  4. दोनों धर्मों ने यज्ञों, बहुदेववाद और कर्मकाण्डों गी का विरोध किया।
  5. दोनों धर्म निर्वाण प्राप्त करने पर बल देते हैं।
  6. दोनों धर्म निवृत्तिमार्गी हैं और संसार त्याग पर बल देते हैं।

प्रश्न 3.
साँची का स्तूप यूरोप के लोगों को विशेषकर में रुचिकर क्यों लगता है?
उत्तर:
भोपाल से बीस मील उत्तर पूर्वी की ओर एक पहाड़ी की तलहटी में साँची का स्तूप स्थित है। इस स्तूप की पत्थर की वस्तुएँ, बुद्ध की मूर्तियाँ तथा प्राचीन तोरणद्वार आदि यूरोप के लोगों को विशेषकर रुचिकर लगते हैं जिनमें मेजर अलेक्जैंडर कनिंघम एक हैं। मेजर अलेक्जेंडर कनिंघम ने इस स्थान के चित्र बनाए। उन्होंने यहाँ के अभिलेखों को पढ़ा और गुम्बदनुमा ढाँचे के बीचों-बीच खुदाई की। उन्होंने इस खोज के निष्कर्षो को एक अंग्रेजी पुस्तक में लिखा।

प्रश्न 4.
साँची का पूर्वी तोरणद्वार भोपाल राज्य से बाहर जाने से कैसे बचा रहा?
उत्तर:
साँची के स्तूप का पूर्वी तोरणद्वार सबसे अच्छी दशा में था अतः फ्रांसीसियों ने सांची के पूर्वी तोरणद्वार को फ्रांस के संग्रहालय में प्रदर्शित करने के लिए शाहजहाँ बेगम से इसे फ्रांस ले जाने की अनुमति माँगी। अंग्रेजों ने भी इसे इंग्लैण्ड ले जाने का प्रयास किया। सौभाग्वश फ्रांसीसी और अंग्रेज दोनों ही इसकी प्रतिकृतियों से सन्तुष्ट हो गए, जो बड़ी सावधानीपूर्वक प्लास्टर से बनाई गई थीं। इस प्रकार मूल कृति भोपाल राज्य में अपने स्थान पर ही बनी रही।

प्रश्न 5.
भोपाल के शासकों ने साँची स्तूप के संरक्षण के लिए क्या उपाय किये?
उत्तर:

  1. भोपाल के शासकों शाहजहाँ बेगम तथा उनकी उत्तराधिकारी सुल्तानजहाँ बेगम ने साँची के स्तूप के रख-रखाव के लिए प्रचुर धन का अनुदान किया।
  2. सुल्तानजहाँ बेगम ने वहाँ पर एक संग्रहालय और अतिथिशाला बनाने के लिए अनुदान दिया।
  3. जान मार्शल द्वारा साँची के स्तूप पर लिखी गई पुस्तक के प्रकाशन में भी सुल्तानजहाँ बेगम ने अनुदान दिया।
  4. साँची के स्तूप को भोपाल राज्य में बनाए रखने में शाहजहाँ बेगम ने योगदान दिया।

प्रश्न 6.
” ईसा पूर्व प्रथम सहस्राब्दी का काल विश्व इतिहास में एक महत्त्वपूर्ण मोड़ माना जाता है।” व्याख्या कीजिए।
उत्तर:
इस काल में ईरान में जरथुस्त्र, चीन में खंगत्सी, यूनान में सुकरात, प्लेटो और अरस्तू तथा भारत में महावीर, गौतम बुद्ध एवं कई अन्य चिन्तकों का उदय हुआ। उन्होंने जीवन के रहस्यों को समझने का प्रयास किया। उन्होंने मानव तथा विश्व व्यवस्था के बीच सम्बन्ध को भी समझने का प्रयास किया। इसी काल में गंगा घाटी में नये राज्य और शहर उभर रहे थे और सामाजिक एवं आर्थिक जीवन में कई प्रकार के परिवर्तन हो रहे थे जिन्हें ये चिन्तक समझने का प्रयास कर रहे थे।

JAC Class 12 History Important Questions Chapter 4 विचारक, विश्वास और इमारतें : सांस्कृतिक विकास

प्रश्न 7.
प्राचीन युग में प्रचलित यज्ञों की परम्परा का वर्णन कीजिए।
उत्तर:
पूर्व वैदिक काल में यज्ञों की परम्परा प्रचलित थी यज्ञों के समय ऋग्वैदिक देवताओं की स्तुति सूतों का न उच्चारण किया जाता था और लोग पशु, पुत्र, स्वास्थ्य, दीर्घ आयु आदि के लिए प्रार्थना करते थे आरम्भिक यह सामूहिक नेरूप से किए जाते थे। बाद में (लगभग 1000 ई. पूर्व से 500 ई. पूर्व) कुछ यज्ञ घरों के स्वामियों द्वारा किए जाते थे राजसूष और अश्वमेध जैसे जटिल यज्ञ सरदार और राजा ही किया करते थे।

प्रश्न 8.
वैदिक परम्परा के अन्तर्गत तथा वैदिक र परम्परा के बाहर छठी शताब्दी ई. पूर्व में उठे प्रश्नों और विवादों का उल्लेख कीजिए।
उत्तर:
छठी शताब्दी ई. पूर्व में लोग जीवन का अर्थ, मृत्यु के पश्चात् जीवन की सम्भावना और पुनर्जन्म के बारे ट में जानने के लिए उत्सुक थे क्या पुनर्जन्म अतीत के कर्मों । के कारण होता था? इस प्रकार के प्रश्नों पर खूब बाद- ही विवाद होता था। चिन्तक परम यथार्थ की प्रकृति को समझने और प्रकट करने में संलग्न थे। वैदिक परम्परा से बाहर के पण कुछ दार्शनिक यह प्रश्न उठा रहे थे कि सत्य एक होता है या अनेक। कुछ लोग यज्ञों के महत्त्व के बारे में विचार कर था रहे थे।

प्रश्न 9.
छठी शताब्दी ई. पूर्व में चिन्तकों में होने वाले वाद-विवादों तथा चर्चाओं की विवेचना कीजिए।
उत्तर:
छठी शताब्दी ई.पूर्व में विभिन्न सम्प्रदाय के शिक्षक एक स्थान से दूसरे स्थान पर घूम-घूमकर अपने दर्शन या विश्व के विषय में अपने दृष्टिकोण को लेकर एक-दूसरे से तथा सामान्य लोगों से तर्क-वितर्क करते थे। इस प्रकार की चर्चाएँ कुटागारशालाओं या ऐसे उपवनों में होती थीं जहाँ घुमक्कड़ चिन्तक ठहरा करते थे। यदि एक शिक्षक अपने प्रतिद्वन्द्वी को अपने तर्कों से सहमत कर लेता था, तो वह अपने अनुयायियों के साथ उसका शिष्य बन जाता था।

प्रश्न 10.
छठी शताब्दी ई. पूर्व में शिक्षक वैदिक धर्म के किन सिद्धान्तों पर प्रश्न उठाते थे?
उत्तर:
छठी शताब्दी ई. पूर्व में अनेक शिक्षक वेदों के प्रभुत्व पर प्रश्न उठाते थे इन शिक्षकों में महावीर स्वामी तथा बुद्ध भी सम्मिलित थे। उन्होंने यह विचार भी प्रकट किया कि जीवन के दुःखों से मुक्ति का प्रयास प्रत्येक व्यक्ति स्वयं कर सकता था। यह बात ब्राह्मणवाद से बिल्कुल भिन्न थी क्योंकि ब्राह्मणवाद की यह मान्यता थी कि किसी व्यक्ति का अस्तित्व उसकी जाति और लिंग से निर्धारित होता था।

प्रश्न 11.
आत्मा की प्रकृति और सच्चे यज्ञ के बारे में उपनिषदों में क्या कहा गया है?
उत्तर:
(1) आत्मा की प्रकृति छान्दोग्य उपनिषद में आत्मा की प्रकृति के आरे में कहा गया है कि “यह आत्मा धान या यव या सरसों या बाजरे के बीज की गिरी से भी छोटी है मन के अन्दर छुपी यह आत्मा पृथ्वी से भी विशाल, क्षितिज से भी विस्तृत, स्वर्ग से भी बड़ी है। और इन सभी लोकों से भी बड़ी है।”

(2) सच्चा यज्ञ ही एक यज्ञ है। बहते यह (पवन) जो बह रहा है, निश्चय बहते यह सबको पवित्र करता है। इसलिए यह वास्तव में यज्ञ है।

प्रश्न 12.
बौद्ध धर्म की शिक्षाओं के प्रसार में चीनी और भारतीय विद्वानों के योगदान का वर्णन कीजिये।
उत्तर:
जब बौद्ध धर्म पूर्वी एशिया में फैल गया तब फा-शिएन और श्वेन त्सांग जैसे चीनी यात्री बौद्ध ग्रन्थों की खोज में भारत आए ये पुस्तकें वे अपने देश ले गए, जहाँ विद्वानों ने इनका अनुवाद किया। भारत के बौद्ध शिक्षक भी अनेक देशों में गए बुद्ध की शिक्षाओं का प्रसार करने हेतु वे कई ग्रन्थ अपने साथ ले गए। कई सदियों तक ये पाण्डुलिपियाँ एशिया के विभिन्न देशों में स्थित बौद्ध- विहारों में संरक्षित थीं।

प्रश्न 13.
त्रिपिटकों पर एक संक्षिप्त टिप्पणी लिखिए।
उत्तर:
बौद्धों के धार्मिक सिद्धान्त त्रिपिटकों में संकलित –

  1. विनयपिटक – इसमें संघ या बौद्ध मठों में रहने वाले बौद्ध भिक्षुओं और भिक्षुणियों के आचरण सम्बन्धी नियमों का वर्णन है।
  2. सुत्तपिटक – इसमें महात्मा बुद्ध की शिक्षाओं का संग्रह है।
  3. अभिधम्मपिटक – इसमें बौद्ध दर्शन से जुड़े विषय संकलित हैं।

प्रश्न 14.
नियतिवादियों के सिद्धान्तों का वर्णन कीजिए।
उत्तर:
(1) नियतिवादियों के अनुसार सब कुछ पूर्व निर्धारित है। सुख और दुःख पूर्व निर्धारित मात्रा में माप कर दिए गए हैं। इन्हें संसार में बदला नहीं जा सकता। इन्हें बढ़ाया या घटाया नहीं जा सकता।
(2) बुद्धिमान लोग यह विश्वास करते हैं कि वे सद्गुणों तथा तपस्या द्वारा अपने कर्मों से मुक्ति प्राप्त कर लेंगे। मूर्ख लोग उन्हीं कार्यों को करके मुक्ति प्राप्त करने की आशा करते हैं नहीं है।

JAC Class 12 History Important Questions Chapter 4 विचारक, विश्वास और इमारतें : सांस्कृतिक विकास

प्रश्न 15.
भौतिकवादियों के सिद्धान्तों का विवेचन कीजिए।
उत्तर:
(1) संसार में दान, यज्ञ या चढ़ावा जैसी कोई चीज नहीं है।
(2) मनुष्य चार तत्वों से बना है जब वह मरता है, तब मिट्टी वाला अंश पृथ्वी में जल वाला अंश जल में, गर्मी वाला अंश आग में तथा साँस का अंश वायु में वापिस मिल जाता है और उसकी इन्द्रियाँ अंतरिक्ष का हिस्सा बन जाती हैं।
(3) दान देने का सिद्धान्त मूर्खों का सिद्धान्त है, यह खोखला झूठ है। मूर्ख हो या विद्वान् दोनों ही कट कर नष्ट हो जाते हैं। मृत्यु के बाद कुछ नहीं बचता।

प्रश्न 16.
जैन धर्म की प्रमुख शिक्षाओं का वर्णन कीजिये।
उत्तर:
(1) सम्पूर्ण विश्व प्राणवान है। पत्थर चट्टान, जल आदि में भी जीवन है।
(2) जीवों के प्रति अहिंसा का पालन करना चाहिए। मनुष्यों, जानवरों, पेड़-पौधों, कीड़े-मकोड़ों को नहीं मारना चाहिए।
(3) जन्म और पुनर्जन्म का चक्र कर्म के द्वारा निर्धारित होता है।
(4) पाँच व्रतों का पालन करना चाहिए –

  • हत्या न करना
  • चोरी नहीं करना
  • झूठ न बोलना
  • ब्रह्मचार्य
  • धन संग्रह न करना।

प्रश्न 17.
गौतम बुद्ध की जीवनी का संक्षेप में वर्णन कीजिए।
अथवा
गीतम बुद्ध पर एक संक्षिप्त टिप्पणी लिखिए।
अथवा
गौतम बुद्ध के जीवन का वर्णन कीजिये।
उत्तर:
बुद्ध के बचपन का नाम सिद्धार्थ था। वह शाक्य कबीले के सरदार के पुत्र थे। एक दिन नगर का भ्रमण करते समय सिद्धार्थ को एक रोगी, वृद्ध, मृतक तथा संन्यासी के दर्शन हुए जिससे उनका संसार के प्रति वैराग्य और बढ़ गया। अतः सिद्धार्थ महल त्याग कर सत्य की खोज में निकल गए। प्रारम्भ में उन्होंने 6 वर्ष तक कठोर तपस्या की। अन्त में उन्होंने एक वृक्ष के नीचे बैठकर चिन्तन करना शुरू किया और सच्चा ज्ञान प्राप्त किया। इसके बाद वह अपनी शिक्षाओं का प्रचार करने लगे।

प्रश्न 18.
बौद्ध धर्म की शिक्षाओं का वर्णन कीजिए।
अथवा
गौतम बुद्ध के उपदेशों का वर्णन कीजिये।
उत्तर:
(1) बौद्ध दर्शन के अनुसार विश्व अनित्य है और निरन्तर बदल रहा है। यह आत्माविहीन है क्योंकि यहाँ कुछ भी स्थायी अथवा शाश्वत नहीं है।
(2) इस क्षणभंगुर संसार में दुःख मनुष्य के जीवन का अन्तर्निहित तत्त्व है।
(3) घोर तपस्या और विषयासक्ति के बीच मध्यम मार्ग का अनुसरण करते हुए मनुष्य दुनिया के दुःखों से मुक्ति पा सकता है।
(4) बुद्ध की मान्यता थी कि समाज का निर्माण मनुष्यों ने किया था, न कि ईश्वर ने

प्रश्न 19.
‘बौद्ध संघ’ पर एक संक्षिप्त टिप्पणी लिखिए।
उत्तर:
बुद्ध ने अपने शिष्यों के लिए ‘संघ’ की स्थापना की संघ बौद्ध भिक्षुओं की एक ऐसी संस्था थी जो धम्म के शिक्षक बन गए। ये भिक्षु एक सादा जीवन बिताते थे। प्रारम्भ में केवल पुरुष ही संघ में सम्मिलित हो सकते थे, बाद में महिलाओं को भी संघ में सम्मिलित होने की अनुमति दे दी गई। कई स्वियों जो संघ में आईं, वे धम्म की उपदेशिकाएँ बन गई। संघ में सभी को समान दर्जा प्राप्त था। संघ की संचालन पद्धति गणों और संघ की परम्परा पर आधारित थी।

JAC Class 12 History Important Questions Chapter 4 विचारक, विश्वास और इमारतें : सांस्कृतिक विकास

प्रश्न 20.
संघ में रहने वाले भिक्षुओं का जीवन कैसा था?
उत्तर:
संघ में रहने वाले बौद्ध भिक्षु सादा जीवन बिताते थे। उनके पास जीवनयापन के लिए अत्यावश्यक वस्तुओं के अतिरिक्त कुछ नहीं होता था। वे दिन में केवल एक बार उपासकों से भोजनदान पाने के लिए एक कटोरा रखते थे। वे दान पर निर्भर थे, इसलिए उन्हें भिक्खु कहा जाता था। संघ में रहते हुए वे बौद्ध ग्रन्थों का अध्ययन करते थे।

प्रश्न 21.
महात्मा बुद्ध ने महिलाओं को भी संघ में सम्मिलित होने की अनुमति क्यों दी?
उत्तर:
प्रारम्भ में केवल पुरुष ही बौद्ध संघ में सम्मिलित हो सकते थे परन्तु कालान्तर में अपने प्रिय शिष्य आनन्द के अनुरोध पर महात्मा बुद्ध ने स्त्रियों को भी संघ में सम्मिलित होने की अनुमति प्रदान कर दी। बुद्ध की उपमाता महाप्रजापति गोतमी संघ में सम्मिलित होने वाली पहली भिक्षुणी थी। संघ में आने वाली कई स्त्रियाँ धम्म की उपदेशिकाएँ बन गई। आगे चलकर वे घेरी बनीं, जिसका अर्थ है- निर्वाण प्राप्त करने वाली महिलाएँ।

प्रश्न 22.
बुद्ध के अनुयायी किन सामाजिक वर्गों से सम्बन्धित थे?
उत्तर:
बुद्ध के अनुयायियों में कई सामाजिक वर्गों के लोग सम्मिलित थे। इनमें राजा, धनवान, गृहपति, व्यापारी, सामान्यजन कर्मकार, दास, शिल्पी आदि शामिल थे। इनमें स्त्री और पुरुष दोनों सम्मिलित थे। एक बार बौद्ध संघ में आ जाने पर सभी को बराबर माना जाता था क्योंकि भिक्षु और भिक्षुणी बनने पर उन्हें अपनी पुरानी पहचान को त्यागना पड़ता था।

प्रश्न 23.
भिक्षुओं और भिक्षुणियों के लिए निर्धारित नियमों का वर्णन कीजिए।
उत्तर:
(1) जब कोई भिक्षु एक नया कम्बल या गलीचा बनाएगा, तो उसे इसका प्रयोग कम से कम छः वर्षों तक करना पड़ेगा।
(2) यदि कोई भिक्षु किसी गृहस्थ के घर जाता है और उसे भोजन दिया जाता है, तो वह दो से तीन कटोरा भर ही स्वीकार कर सकता है।
(3) यदि बिहार में ठहरा हुआ भिक्षु प्रस्थान के पहले अपने बिस्तर को नहीं समेटता है, तो उसे अपराध स्वीकार करना होगा।

प्रश्न 24.
बौद्ध धर्म के तेजी से प्रसार होने के क्या कारण थे?
उत्तर:
(1) लोग समकालीन धार्मिक प्रथाओं से असन्तुष्ट थे।
(2) बौद्ध धर्म ने जन्म पर आधारित वर्ण व्यवस्था का विरोध किया और सामाजिक समानता पर बल दिया।
(3) बौद्ध धर्म में अच्छे आचरण और मूल्यों को महत्त्व दिया गया। इससे स्त्री और पुरुष इस धर्म की ओर आकर्षित हुए।
(4) बौद्ध धर्म ने निर्बल लोगों के प्रति दयापूर्ण और मित्रतापूर्ण व्यवहार को महत्त्व दिया।

प्रश्न 25.
चैत्य से क्या अभिप्राय है?
उत्तर:
अत्यन्त प्राचीनकाल से ही लोग कुछ स्थानों को पवित्र मानते थे ऐसे स्थानों पर जहाँ प्रायः विशेष वनस्पति होती थी, अनूठी चट्टानें थीं या आश्चर्यजनक प्राकृतिक सौन्दर्य था वहाँ पवित्र स्थल बन जाते थे ऐसे कुछ स्थलों पर एक छोटी-सी वेदी भी बनी रहती थी, जिन्हें कभी-कभी चैत्य कहा जाता था शवदाह के बाद शरीर के कुछ अवशेष टीलों पर सुरक्षित रख दिए जाते थे। अन्तिम संस्कार से जुड़े टीले चैत्य के रूप में जाने गए।

प्रश्न 26.
बौद्ध साहित्य में वर्णित कुछ चैत्यों का उल्लेख कीजिये।
उत्तर:
बौद्ध साहित्य में कई चैत्यों का उल्लेख मिलता है। इसमें बुद्ध के जीवन से सम्बन्धित स्थानों का भी उल्लेख है, जैसे लुम्बिनी (जहाँ बुद्ध का जन्म हुआ), बोधगया (जहाँ बुद्ध ने ज्ञान प्राप्त किया), सारनाथ (जहाँ उन्होंने उपदेश दिया) और कुशीनगर (जहाँ बुद्ध ने निब्बान प्राप्त किया। धीरे-धीरे ये समस्त स्थान पवित्र स्थल बन गए और यहाँ अनेक चैत्य बनाए गए।

प्रश्न 27.
स्तूपों का निर्माण क्यों किया जाता था?
उत्तर:
स्तूप बनाने की परम्परा बुद्ध से पहले की रही होगी। परन्तु वह बौद्ध धर्म से जुड़ गई। चूंकि स्तूपों में ऐसे अवशेष रहते थे, जिन्हें पवित्र समझा जाता था। इसलिए समूचा स्तूप ही बुद्ध और बौद्ध धर्म के प्रतीक के रूप में प्रतिष्ठित हुआ। अशोक ने बुद्ध को अवशेषों के हिस्से प्रत्येक महत्त्वपूर्ण नगर में बाँट कर उनके ऊपर स्तूप बनाने का आदेश दिया था।

JAC Class 12 History Important Questions Chapter 4 विचारक, विश्वास और इमारतें : सांस्कृतिक विकास

प्रश्न 28.
स्तूपों की संरचना का वर्णन कीजिये।
उत्तर:
(1) स्तूप का जन्म एक गोलाई लिए हुए मिट्टी के टीले से हुआ जिसे बाद में अंड कहा गया।
(2) अंड के ऊपर एक ‘हर्मिका’ होती थी। वह छज्जे जैसा ढांचा होता था।
(3) हर्मिका से एक मस्तूल निकलता था, जिसे ‘वष्टि’ कहते थे, जिस पर प्रायः एक छत्री लगी होती थी।
(4) टीले के चारों ओर एक वेदिका होती थी जो पवित्र स्थल को सामान्य दुनिया से अलग करती थी।

प्रश्न 29.
साँची और भरहुत के स्तूपों का वर्णन कीजिए।
उत्तर:
साँची और भरहुत के स्तूप बिना अलंकरण के हैं, उनमें केवल पत्थर की वेदिकाएँ तथा तोरणद्वार हैं। ये पत्थर की वेदिकाएँ किसी बाँस के या काठ के घेरे के समान थीं और चारों दिशाओं में खड़े तोरणद्वार पर खूब नक्काशी की गई थी। उपासक पूर्वी तोरणद्वार से प्रवेश करके टीले को दाई ओर देखते हुए दक्षिणावर्त परिक्रमा करते थे, मानो वे आकाश में सूर्य के पथ का अनुकरण कर रहे हों। बाद में स्तूप के टीले पर भी अलंकरण और नक्काशी की जाने लगी।

प्रश्न 30.
स्तूप किन लोगों के दान से बनाए गए ?
उत्तर:
(1) स्तूपों के निर्माण के लिए कुछ दान राजाओं के द्वारा दिए गए।
(2) कुछ दान शिल्पकारों और व्यापारियों की श्रेणियों द्वारा दिए गए।
(3) साँची के एक तोरण द्वार का भाग हाथीदांत का काम करने वाले शिल्पकारों के दान से बनाया गया था
(4) स्तूपों के निर्माण के लिए बौद्ध भिक्षुओं और भिक्षुणियों ने भी दान दिया।
(5) स्तूपों के निर्माण के लिए सैकड़ों महिलाओं और पुरुषों के द्वारा दान दिए गए।

प्रश्न 31.
अमरावती स्तूप की खोज किस प्रकार की गई ?
उत्तर:
1796 में एक स्थानीय राजा को अचानक अमरावती के स्तूप के अवशेष मिल गये। उन्होंने उसके पत्थरों से एक मन्दिर बनवाने का निश्चय किया। कुछ वर्षों बाद कालिन मेकेंजी नामक एक अंग्रेज अधिकारी इस क्षेत्र से गुजरे। उन्हें वहाँ नई मूर्तियाँ मिलीं और उन्होंने उनका चित्रांकन किया। 1854 ई. में गुन्टूर के कमिश्नर ने अमरावती की यात्रा की। वे वहाँ से कई मूर्तियाँ और उत्कीर्ण पत्थर मद्रास ले गए। उन्होंने पश्चिमी तोरणद्वार को भी खोज निकाला।

प्रश्न 32.
साँची क्यों बच गया, जबकि अमरावती नष्ट हो गया?
उत्तर:
सम्भवतः अमरावती की खोज थोड़ी पहले हो गई थी। 1818 में जब साँची की खोज हुई, इसके तीन तोरणद्वार तब भी खड़े थे। चौथा तोरणद्वार वहीं पर गिरा हुआ था। उस समय भी यह सुझाव आया कि तोरणद्वारों को पेरिस या लन्दन भेज दिया जाए। परन्तु कई कारणों से साँची का स्तूप वहीं बना रहा और आज भी बना हुआ है। दूसरी ओर अमरावती का महाचैत्व अब केवल एक छोटा- सा टीला है जिसका सम्पूर्ण गौरव नष्ट हो चुका है।

प्रश्न 33.
इतिहासकार किसी मूर्तिकला की व्याख्या लिखित साक्ष्यों के साथ तुलना के द्वारा करते हैं। स्पष्ट कीजिए।
उत्तर:
प्रस्तुत चित्र पहली बार देखने पर तो इस मूर्तिकला के अंश में फूस की झोंपड़ी तथा पेड़ों वाले ग्रामीण दृश्य का चित्रण दिखाई देता है परन्तु कला इतिहासकार इसे ‘वेसान्तर जातक’ से लिया गया एक दृश्य बताते हैं यह कहानी एक ऐसे दानी राजकुमार के बारे में है जो अपना सर्वस्व एक ब्राह्मण को सौंप कर स्वयं अपनी पत्नी तथा बच्चों के साथ वन में रहने के लिए चला गया। इस उदाहरण से स्पष्ट है कि प्राय: इतिहासकार किसी मूर्तिकला की व्याख्या लिखित साक्ष्यों के साथ तुलना के द्वारा करते हैं। (पाठ्यपुस्तक का चित्र 4.13, पेज 100)

प्रश्न 34.
“कई प्रारम्भिक मूर्तिकारों ने बुद्ध को मानव के रूप में न दिखाकर उनकी उपस्थिति प्रतीकों के माध्यम से दर्शाने का प्रयास किया।” स्पष्ट कीजिए।
उत्तर:
कई प्रारम्भिक मूर्तिकारों ने बुद्ध को मानव के रूप में न दिखाकर उनकी उपस्थिति प्रतीकों के माध्यम से दर्शाने का प्रयास किया। उदाहरण के लिए ‘रिक्त स्थान’ बुद्ध के ध्यान की दशा तथा ‘स्तूप’ ‘महापरिनिब्बान’ के प्रतीक बन गए। चक्र बुद्ध द्वारा सारनाथ में दिए गए पहले उपदेश का प्रतीक था। पेड़ बुद्ध के जीवन की एक पटना का प्रतीक था ऐसे प्रतीकों को समझने के लिए यह आवश्यक है कि इतिहासकार इन कलाकृतियों के निर्माताओं की परम्पराओं से परिचित हो ।

JAC Class 12 History Important Questions Chapter 4 विचारक, विश्वास और इमारतें : सांस्कृतिक विकास

प्रश्न 35.
साँची स्तूप के तोरणद्वार के किनारे पर एक वृक्ष पकड़कर झूलती हुई महिला की मूर्ति से किस लोक परम्परा का बोध होता है?
उत्तर:
प्रारम्भ में विद्वानों को इस मूर्ति के महत्त्व के बारे में कुछ असमंजस था। इस मूर्ति से त्याग और तपस्या के भाव प्रकट नहीं होते थे। परन्तु लोक साहित्यिक परम्पराओं के अध्ययन के द्वारा वे इस निष्कर्ष पर पहुँचे कि यह संस्कृत भाषा में वर्णित ‘शालभंजिका’ की मूर्ति है। लोक परम्परा के अनुसार लोग यह मानते थे कि इस स्वी द्वारा हुए जाने से वृक्षों में फूल खिल उठते थे और फल होने लगते थे। यह एक शुभ प्रतीक माना जाता था और इसी कारण स्तूप के अलंकरण में यह प्रयुक्त हुआ ।

प्रश्न 36.
“साँची की मूर्तियों में पाए गए कई प्रतीक या चिह्न लोक परम्पराओं से उभरे थे।” व्याख्या कीजिए।
उत्तर:
साँची की मूर्तियों में पाए गए कई प्रतीक या चिह्न लोक परम्पराओं से उभरे थे। उदाहरण के लिए, साँची में जानवरों के कुछ सुन्दर उत्कीर्णन पाए गए हैं। इन जानवरों में हाथी, घोड़े, बन्दर और गाय-बैल सम्मिलित हैं। ऐसा प्रतीत होता है कि यहाँ पर लोगों को आकर्षित करने के लिए जानवरों का उत्कीर्णन किया गया था। इसके अतिरिक्त जानवरों का मनुष्यों के गुणों के प्रतीक के रूप में प्रयोग किया जाता था। उदाहरणार्थ, हाथी शक्ति और ज्ञान के प्रतीक माने जाते थे।

प्रश्न 37.
चित्र 4.19 पृष्ठ 102 पर चित्रित कमलदल और हाथियों के बीच एक महिला की मूर्ति से किस लोक परम्परा के बारे में जानकारी मिलती है?
उत्तर:
इस चित्र में हाथी महिला के ऊपर जल छिड़क रहे हैं, जैसे वे उनका अभिषेक कर रहे हो कुछ इतिहासकार इस महिला को बुद्ध की माँ माया मानते हैं तो दूसरे इतिहासकार इसे एक लोकप्रिय देवी गजलक्ष्मी मानते हैं। गजलक्ष्मी सौभाग्य लाने वाली देवी थी, जिन्हें प्राय: हाथियों से सम्बन्धित किया जाता है। यह भी सम्भव है कि इन उत्कीर्णित मूर्तियों को देखने वाले उपासक इसे माया और गजलक्ष्मी दोनों से ही जोड़ते थे।

प्रश्न 38.
अजन्ता की चित्रकारी की प्रमुख विशेषताओं का वर्णन कीजिए।
उत्तर:
प्राचीन काल में चित्रकारी जैसे सम्प्रेषण के अन्य माध्यमों का भी प्रयोग किया जाता था। इन चित्रों में जो सबसे अच्छी दशा में बचे हुए हैं, वे गुफाओं की दीवारों पर बने चित्र हैं। इनमें अजन्ता (महाराष्ट्र) की चित्रकारी अत्यन्त प्रसिद्ध है। अजन्ता के चित्रों में जातकों की कथाएँ चित्रित हैं। इनमें राजदरबार का जीवन, शोभायात्राएँ, काम करते हुए स्वी- पुरुष और त्यौहार मनाने के चित्र दिखाए गए हैं अजन्ता के कुछ चित्र अत्यन्त स्वाभाविक एवं सजीव लगते हैं।

प्रश्न 39.
बौद्धमत में महायान के विकास का वर्णन कीजिए।
अथवा
महायान बौद्ध धर्म के विकास का संक्षेप में वर्णन कीजिये।
उत्तर:
ईसा की प्रथम शताब्दी के पश्चात् यह विश्वास किया जाने लगा कि बुद्ध लोगों को मुक्ति दिलवा सकते थे। साथ-साथ बोधिसत्त की अवधारणा भी विकसित होने लगी। बोधिसत्तों को परम दयालु जीव माना गया जो अपने सत्कार्यों से पुण्य कमाते थे परन्तु वे इस पुण्य का प्रयोग दूसरों का कल्याण करने में करते थे। इस प्रकार बौद्धमत में बुद्ध और बोधिसत्वों की मूर्तियों की पूजा इस परम्परा का एक महत्त्वपूर्ण अंग बन गई। इस नई परम्परा को ‘महायान’ कहा गया।

प्रश्न 40.
पौराणिक हिन्दू धर्म के उदय का वर्णन कीजिए।
उत्तर:
हिन्दू धर्म में वैष्णव एवं शैव परम्पराएँ सम्मिलित हैं। वैष्णव परम्परा में विष्णु को सबसे महत्वपूर्ण देवता माना जाता है और शैव परम्परा में शैव को परमेश्वर माना जाता है। इनके अन्तर्गत एक विशेष देवता की पूजा को विशेष महत्व दिया जाता था। इस प्रकार की आराधना में उपासना और ईश्वर के बीच का सम्बन्ध प्रेम और समर्पण का सम्बन्ध माना जाता था। इसे भक्ति कहते हैं।

JAC Class 12 History Important Questions Chapter 4 विचारक, विश्वास और इमारतें : सांस्कृतिक विकास

प्रश्न 4.
‘अवतारवाद की अवधारणा’ से आप क्या समझते हैं ?
उत्तर:
वैष्णववाद में दस अवतारों की कल्पना की गई है यह माना जाता था कि पापियों के बढ़ते प्रभाव के कारण जब संसार में अराजकता, अव्यवस्था और विनाश की स्थिति आ जाती थी, तब विश्व की रक्षा के लिए भगवान अलग-अलग रूपों में अवतार लेते थे। सम्भवतः पृथक् पृथक् अवतार देश के भिन्न-भिन्न भागों में लोकप्रिय थे। इन समस्त स्थानीय देवताओं को विष्णु का रूप मान लेना एकीकृत धार्मिक परम्परा के निर्माण का एक महत्त्वपूर्ण तरीका था।

प्रश्न 42.
“वैष्णववाद में अनेक अवतारों के इर्द- गिर्द पूजा पद्धतियाँ विकसित हुई।” व्याख्या कीजिये।
अथवा
वैष्णववाद में अनेक अवतारों के इर्द-गिर्द पूजा – पद्धतियाँ किस प्रकार विकसित हुई? विवेचना कीजिये।
उत्तर:
वैष्णववाद में कई अवतारों को मूर्तियों के रूप में दिखाया गया है। अन्य देवताओं की मूर्तियों का भी निर्माण हुआ। शिव को उनके प्रतीक लिंग के रूप में बनाया जाता था। परन्तु उन्हें कई बार मनुष्य के रूप में भी दिखाया गया है। ये समस्त चित्रण देवताओं से जुड़ी हुई मिश्रित अवधारणाओं पर आधारित थे। उनकी विशेषताओं और प्रतीकों को उनके शिरोवस्त्र, आभूषणों, आयुषों (हथियार और हाथ में धारण किए गए अन्य शुभ अस्त्र) और बैठने की शैली से दर्शाया जाता था।

प्रश्न 43.
प्रारम्भिक मन्दिरों की विशेषताओं का वर्णन कीजिए।
उत्तर:
(1) शुरू के मन्दिर एक चौकोर कमरे के रूप में थे जिन्हें गर्भगृह कहा जाता था। इनमें एक दरवाजे से उपासक मूर्ति की पूजा करने के लिए भीतर प्रविष्ट हो
सकता था।
(2) धीरे-धीरे गर्भगृह के ऊपर एक ऊँचा ढाँचा बनाया जाने लगा जिसे शिखर कहा जाता था।
(3) मन्दिर की दीवारों पर प्रायः भित्तिचित्र उत्कीर्ण किए जाते थे।
(4) बाद में मन्दिरों में विशाल सभा स्थल, ऊंची दीवारों और तोरणों का भी निर्माण किया जाने लगा।
(5) कुछ पहाड़ियों को काट कर कुछ गुफा मन्दिर भी बनाये गये।

प्रश्न 44.
उन्नीसवीं शताब्दी के यूरोपीय विद्वान यूनानी शैली से प्रभावित भारतीय मूर्तियों से क्यों प्रभावित हुए?
उत्तर:
उन्नीसवीं शताब्दी के यूरोपीय विद्वानों ने मूर्तियों की तुलना प्राचीन यूनान की कला परम्परा से की। वे बुद्ध और बोधिसतों की मूर्तियों की खोज से काफी उत्साहित हुए। इसका कारण यह था कि ये मूर्तियाँ यूनानी मूर्तिकला से प्रभावित थीं। ये मूर्तियाँ यूनानी मूर्तियों से काफी मिलती- जुलती थीं। चूँकि वे विद्वान यूनानी परम्परा से परिचित थे, इसलिए उन्होंने इन्हें भारतीय मूर्तिकला का सर्वश्रेष्ठ नमूना माना।

प्रश्न 45.
कृत्रिम गुफाएँ बनाने की परम्परा का वर्णन कीजिए।
उत्तर:
प्राचीनकाल में कुछ मन्दिर पहाड़ियों को काट कर खोखला कर कृत्रिम गुफाओं के रूप में बनाए गए थे। कृत्रिम गुफाएँ बनाने की परम्परा काफी प्राचीन थी। सबसे प्राचीन कृत्रिम गुफाएँ ई. पूर्व तीसरी सदी में अशोक के आदेश से बराबर ( बिहार ) की पहाड़ियों में आजीविक सम्प्रदाय के संतों के लिए बनाई गई थीं कृत्रिम गुफा बनाने का सबसे विकसित रूप हमें आठवीं शताब्दी के कैलाशनाथ के मन्दिर में दिखाई देता है। इसे पूरी पहाड़ी काटकर बनाया गया था।

प्रश्न 46.
पुरातत्त्ववेत्ता एच. एच. कोल प्राचीन कलाकृतियों के बारे में क्या सोच रखते थे?
उत्तर:
पुरातत्त्ववेत्ता एच. एच. कोल प्राचीन कलाकृतियों को उठा ले जाने के विरुद्ध थे। वे इस लूट को आत्मघाती मानते थे। उनका मानना था कि संग्रहालयों में मूर्तियों की प्लास्टर कलाकृतियाँ रखी जानी चाहिए जबकि असली कृतियाँ खोज के स्थान पर ही रखी जानी चाहिए।

प्रश्न 47.
जैन धर्म के अहिंसावादी स्वरूप के विषय में आप क्या जानते हैं? स्पष्ट कीजिए।
उत्तर:
हॉपकिन्स नामक विद्वान ने कहा था कि जैन धर्म कीट-पतंगों का भी पोषण करता है, पूर्णतया सत्य है। क्योंकि जैन धर्म में अहिंसा पर अत्यधिक बल दिया गया है। विश्व के अस्तित्व के लिए प्रत्येक अणु महत्त्वपूर्ण है। इसके लिए जैन दार्शनिकों ने मत दिए कि व्यक्ति को मन, कर्म तथा वचन से पूर्ण अहिंसावादी होना चाहिए। मनुष्य को जाने-अनजाने में भी प्राणियों के प्रति किसी भी हिंसावादी गतिविधि का सम्पादन नहीं करना चाहिए। इसी के फलस्वरूप अनेक जैन सन्त मुँह पर पट्टी बाँधते हैं; जिससे उनके श्वास लेते समय कोई कीटाणु उनके मुँह में न चला जाए।

JAC Class 12 History Important Questions Chapter 4 विचारक, विश्वास और इमारतें : सांस्कृतिक विकास

प्रश्न 48.
गौतम बुद्ध एक महान् समाज सुधारक थे। स्पष्ट कीजिए।
उत्तर:
बौद्ध धर्म का उद्भव एक क्रान्तिकारी धर्म के रूप में हुआ था। इस धर्म के प्रवर्तक मध्यमार्गी बुद्ध थे। गौतम बुद्ध की विचारधारा, उपदेश तथा कार्यप्रणाली अत्यन्त सरल थी। गौतम बुद्ध ने जातिवाद विशेषकर जन्म पर आधारित जातिवाद का घोर विरोध किया तथा जन्म के स्थान पर कर्म को व्यक्ति की पहचान बताया।

एक स्थान पर गौतम बुद्ध अपने शिष्य आनन्द से कहते हैं कि साधक की जाति क्या पूछता है कर्म पूछ गौतम बुद्ध की यह विचारधारा तत्कालीन अतिवादी तथा ब्राह्मणवादी व्यवस्था के विरुद्ध जनसामान्य की सबसे बड़ी आवश्यकता थी। बुद्ध ने जनता की इस आवश्यकता को समझा और ऐसी कर्मकाण्डीय ब्राह्मणवादी व्यवस्था का विरोध किया; जिसमें जनसामान्य का कोई स्थान नहीं था।

प्रश्न 49.
बौद्ध संघों की स्थापना क्यों की गयी?
उत्तर:
बौद्ध धर्म की व्यापक लोकप्रियता के कारण शिष्यों की संख्या तीव्रता से बढ़ने लगी। शिष्यों में ही कुछ ऐसे शिष्य जिनका आत्मिक विकास उच्च था; वे धम्म के शिक्षक बन गये। संघ की स्थापना धम्म के शिक्षकों के की गयी संघ में सदाचरण और नैतिकता पर बल दिया जाता था। हेतु प्रारम्भ में महिलाओं का संघ में प्रवेश वर्जित था परन्तु बाद में महात्मा बुद्ध ने महिलाओं को भी संघ में प्रवेश की अनुमति दे दी। महाप्रजापति गौतमी जो कि बुद्ध की उपमाता थीं, संघ में प्रवेश करने वाली पहली महिला थीं।

प्रश्न 50.
जैन तथा बौद्ध धर्म में क्या भिन्नताएँ हैं? संक्षेप में लिखिए।
उत्तर:
जैन तथा बौद्ध धर्म में अनेक समानताएँ थीं परन्तु कुछ असमानताएँ भी थीं। प्रमुख भिन्नताओं का विवरण निम्नलिखित है –

  1.  जैन धर्म आत्मवादी है परन्तु बौद्ध धर्म अनात्मवादी
  2. जैनों के साहित्य को आगम कहा जाता है; बौद्धों के साहित्य को त्रिपिटक कहा जाता है।
  3. जैन धर्म अत्यधिक अहिंसावादी है बौद्ध धर्म मध्यमार्गी है।
  4. जैन धर्म के अनुसार मोक्ष मृत्यु के पश्चात् ही सम्भव है वहीं बौद्ध धर्म के अनुसार इसी जन्म में निर्वाण सम्भव है।
  5. जैन धर्म में तीर्थंकरों की उपासना होती है परन्तु बौद्ध धर्म में बुद्ध और बोधिसत्वों की आराधना की जाती है।

प्रश्न 51.
बौद्ध मूर्तिकला की प्रतीकात्मक पद्धति क्या थी? इन प्रतीकों को समझ पाना एक दुष्कर कार्य क्यों था?
उत्तर:
बौद्ध मूर्तिकला की मूर्तियों को स्पष्ट रूप से समझ पाना एक दुष्कर कार्य इसलिए है कि इतिहासकार केवल इस बात का अनुमान ही लगा सकते हैं कि मूर्ति बनाते समय मूर्तिकार का दृष्टिकोण क्या था इतिहासकारों के ‘अनुमान के अनुसार आरम्भिक मूर्तिकारों ने महात्मा बुद्ध को मनुष्य के रूप में न दिखाकर उन्हें प्रतीकों के माध्यम से प्रकट करने का प्रयास किया है। चित्र में दिखाए गए रिक्त स्थान को इतिहासकार बुद्ध की ध्यान अवस्था के रूप में बताते हैं; क्योंकि ध्यान की महादशा में अन्तर में रिक्तता की अनुभूति होती है स्तूप को महानिर्वाण की दशा के रूप में व्याख्यायित किया है।

महानिर्वाण का अर्थ है- विराट में समा जाना चक्र को बुद्ध द्वारा सारनाथ में दिए गए पहले प्रवचन का प्रतीक माना गया है; इसके अनुसार यहीं से बौद्ध धर्म के प्रचार-प्रसार का चक्र घूमा पेड़ का अर्थ मात्र पेड़ के रूप में नहीं बल्कि वह बीज की पूर्ण परिपक्वता का प्रतीक है एक बीज जिसकी सम्भावना वृक्ष बनने की है वह अपनी पूर्णता को प्राप्त हुआ इसी प्रकार बुद्ध अपने जीवन में सम्पूर्णता को प्राप्त हुए।

प्रश्न 52.
साँची के प्रतीक लोक-परम्पराओं से जुड़े थे; संक्षिप्त विवेचना कीजिए।
उत्तर:
साँची की मूर्तियों में प्राप्त उत्कीर्णन में लोक परम्परा से जुड़े हुए बहुत से प्रतीकों का चिशंकन है। मूर्तियों को आकर्षक और सुन्दर दर्शाने हेतु विविध प्रतीकों जैसे हाथी, घोड़ा, बन्दर, गाय, बैल आदि जानवरों का उत्कीर्णन जीवन्त रूप से किया गया है। हाथी को शक्ति तथा ज्ञान का प्रतीक कहा गया है। इसी प्रकार एक स्वी तोरणद्वार के किनारे एक पेड़ पकड़कर झूल रही है। यह शालभंजिका की मूर्ति है, लोक परम्परा में शाल भंजिका को शुभ प्रतीक माना जाता है। वामदल और हाथियों के बीच एक महिला को एक अन्य मूर्ति में दिखाया गया है।

हाथी उस महिला के ऊपर जल छिड़क रहे हैं; जैसे उसका अभिषेक कर रहे हैं। इस महिला को बुद्ध की माँ माना जाता है। कुछ इतिहासकारों के अनुसार यह महिला सौभाग्य की देवी गजलक्ष्मी है। इतिहासकारों का इस सम्बन्ध में अपना दृष्टिकोण है। सर्पों का उत्कीर्णन भी कई स्तम्भों पर पाया जाता है। इस प्रतीक को भी लोक परम्परा से जुड़ा हुआ माना जाता है। प्रारम्भिक इतिहासकार जेम्स फर्गुसन के अनुसार साँची में वृक्षों और सर्पों की पूजा की जाती थी। वे बौद्ध साहित्य से अनभिज्ञ थे, उन्होंने उत्कीर्णित मूर्तियों का अध्ययन करके अपना यह निष्कर्ष निकाला था।

JAC Class 12 History Important Questions Chapter 4 विचारक, विश्वास और इमारतें : सांस्कृतिक विकास

प्रश्न 53.
अतीत से प्राप्त चित्रों की क्या विशेषताएँ थीं?
उत्तर:
अजन्ता और एलोरा की गुफाओं में बने भित्ति चित्र सम्पूर्ण विश्व के आकर्षण का केन्द्र है। चित्रकारी भी मूर्तिकला की भाँति सम्प्रेषण का एक माध्यम है। जातक की कथाओं का चित्रण बहुत ही सुन्दर ढंग से अजन्ता के चित्रों में दिखाया गया है। इन कथाओं में राजदरबार का चित्रण, शोभा यात्राएं, त्यौहारों का उत्सव और कार्य करते हुए पुरुषों और महिलाओं का चित्रांकन है। यह चित्र अत्यन्त ही सुन्दर और सजीव है हर्षोल्लास, उमंग, प्रसन्नता, प्रेम की भावनाओं की अभिव्यक्ति इतनी कुशलता से और जीवन्तता से की गई है कि यह लगता है कि चित्र बोल पड़ेंगे। कलाकारों ने इन्हें त्रिविम रूप से चित्रित किया गया है; इसके लिए आभा भेद तकनीक का प्रयोग करके इन्हें सजीवता प्रदान की है।

प्रश्न 54.
हीनयान तथा महायान में मुख्य अन्तरों को समझाइए।
उत्तर:
हीनयान तथा महायान में अनेक मूलभूत अन्तर हैं। इनमें से प्रमुख अन्तरों का विवरण निम्नलिखित है –

  1. हीनयान प्राचीन, रूढ़िवादी तथा मूल मत है जबकि महायान बौद्ध मत का संशोधित तथा सरल रूप है जो चतुर्थ बौद्ध संगीति में प्रकाश में आया।
  2. हीनयानी बुद्ध की स्तुति प्रतीकों के माध्यम से करते हैं वहीं महायानी मूर्ति पूजा में विश्वास करते हैं।
  3. हीनयान मत रूढ़िवादी है जबकि महायान अत्यधिक सरल मत है।
  4. हीनयान में बुद्ध की स्तुति की जाती है; वहीं महायान में बुद्ध के साथ-साथ बोधिसत्वों की आराधना भी की जाती है।
  5. हीनयान ज्ञान को महत्त्व देता है जबकि महायान करुणा को।

प्रश्न 55.
प्राचीन भारतीय कला की पृष्ठभूमि और महत्त्व को 19वीं सदी के यूरोपीय विद्वान प्रारम्भ में क्यों नहीं समझ सके ? उनकी समस्या का निराकरण किस प्रकार हुआ?
उत्तर:
प्रत्येक देश की धार्मिक आस्थाओं, धारणाओं, परम्पराओं आदि में अन्तर होते हैं उनके सोचने और समझने के ढंग और प्रतिमान अलग-अलग होते हैं। इसलिए यूरोपीय विद्वानों ने जब प्राचीन भारतीय मूर्तिकला की यह मूर्तियाँ जो कई हाथों, कई सिरों या अर्द्ध मानव के रूप में निर्मित थीं देखीं तो उन्हें यह विचित्र प्रतीत हुई। आराध्य देवों के यह रूप उनकी कल्पना से परे थे। फिर भी इन आरम्भिक यूरोपीय विद्वानों ने इन विभिन्न रूपों वाली आराध्य देवों की मूर्तियों को समझने हेतु प्रयास किए।

यूरोपीय विद्वानों ने प्राचीन यूनानी कला परम्परा की पृष्ठभूमि को आधार बनाकर इन मूर्तियों की यूनानी मूर्तियों से तुलना की तथा उन्हें समझने का प्रयास किया। लेकिन जब उन्होंने बौद्ध धर्म की कला परम्परा, बौद्ध धर्म की बुद्ध और बोधसत्व की मूर्तियाँ देखीं तो वे बहुत प्रोत्साहित हुए। इन मूर्तियों की उत्कृष्टता को देखकर हैरान रह गए, उन्हें लगा ये मूर्तियाँ यूनानी प्रतिमानों के अनुरूप हैं। इस प्रकार इस मूर्तिकला की अनजानी व अपरिचित पृष्ठभूमि और इसके अपरिचित महत्त्व को उन्होंने परिचित यूनानी मूर्तिकला के आधार पर समझने का प्रयास किया।

निबन्धात्मक प्रश्न

प्रश्न 1.
साँची के स्तूप के संरक्षण में भोपाल की शासिकाओं के योगदान का वर्णन कीजिये।
उत्तर:
साँची के स्तूप के संरक्षण में भोपाल की शासिकाओं का योगदान साँची के स्तूप के संरक्षण में भोपाल की शासिकाओं के योगदान का वर्णन निम्नलिखित बिन्दुओं के अन्तर्गत किया जा सकता है –
(1) साँधी के स्तूप के रख-रखाव के लिए धन का अनुदान देना- भोपाल की शासिकाओं- शाहजहाँ बेगम तथा उनकी उत्तराधिकारी सुल्तान जहाँ बेगम की पुरातात्विक स्थलों के संरक्षण में बड़ी रुचि थी। उन्होंने साँची के विश्व प्रसिद्ध स्तूप के रख-रखाव के लिए प्रचुर धन का अनुदान दिया।

(2) जान मार्शल द्वारा रचित पुस्तक के प्रकाशन के लिए अनुदान देना जान मार्शल ने साँची स्तूप पर एक महत्वपूर्ण ग्रन्थ की रचना की। उन्होंने सुल्तानजहाँ की पुरातात्विक स्थलों के प्रति रुचि को देखते हुए अपना प्रसिद्ध ग्रन्थ सुल्तानजहाँ बेगम को समर्पित किया। सुल्तानजहाँ बेगम ने इस ग्रन्थ के विभिन्न खण्डों के प्रकाशन हेतु धन का अनुदान दिया।

(3) संग्रहालय और अतिथिशाला का निर्माण करवाया सुल्तानजहाँ बेगम ने स्तूप स्थल पर एक संग्रहालय तथा अतिथिशाला के निर्माण के लिए प्रचुर धन का अनुदान दिया।

(4) साँची के स्तूप को भोपाल राज्य में सुरक्षित रखना – उन्नीसवीं शताब्दी के यूरोपीय लोगों में साँची के स्तूप को लेकर बड़ी रुचि थी। प्रारम्भ में फ्रांसीसियों ने सबसे अच्छी दशा में बचे साँची के पूर्वी तोरणद्वार को फ्रांस के संग्रहालय में प्रदर्शित करने हेतु शाहजहाँ बेगम से फ्रांस ले जाने की अनुमति माँगी।

इसके बाद अंग्रेजों ने भी साँची के पूर्वी तोरणद्वार को इंग्लैण्ड ले जाने की अनुमति माँगी। परन्तु शाहजहाँ बेगम ने उन्हें साँची के स्तूप की प्लास्टर प्रतिकृतियाँ देकर सन्तुष्ट कर दिया। इस प्रकार शाहजहाँ बेगम के प्रयासों के परिणामस्वरूप साँची के स्तूप की मूलकृति भोपाल राज्य में अपने स्थान पर ही सुरक्षित रही। इस प्रकार यह स्तूप समूह बना रहा है तो इसके पीछे भोपाल की बेगमों के विवेकपूर्ण निर्णयों की बड़ी भूमिका रही है।

प्रश्न 2.
बौद्ध ग्रन्थ किस प्रकार तैयार और संरक्षित किए जाते थे?
उत्तर:
बौद्ध ग्रंथों को तैयार एवं संरक्षित करना बौद्ध धर्म के संस्थापक महात्मा बुद्ध अन्य शिक्षकों की भाँति चर्चा और वार्तालाप करते हुए मौखिक शिक्षा देते थे। स्त्री, पुरुष और बच्चे इन प्रवचनों को सुनते थे और इन पर चर्चा करते थे। बुद्ध की शिक्षाओं को उनके जीवनकाल में लिखा नहीं गया। उनकी मृत्यु के बाद पांचवीं चौथी सदी ई. पूर्व में उनके शिष्यों ने वरिष्ठ श्रमणों की एक सभा वैशाली में आयोजित की। वहाँ पर ही उनकी शिक्षाओं का संकलन किया गया। इन संग्रहों को ‘त्रिपिटक’ कहा जाता था।

(1) त्रिपिटक त्रिपिटक का शाब्दिक अर्थ है-भिन्न प्रकार के ग्रन्थों को रखने के लिए ‘तीन टोकरियाँ’ त्रिपिटक तीन हैं-

  • विनयपिटक – इसमें संघ या बौद्ध मठों में रहने वाले बौद्ध भिक्षुओं और भिक्षुणियों के लिए आचरण सम्बन्धी नियमों का संग्रह है।
  • सुत्तपिटक – इसमें बुद्ध की शिक्षाओं का संग्रह है।
  • अभिधम्मपिटक इसमें बौद्धधर्म के दार्शनिक सिद्धान्तों का विवेचन है।

प्रत्येक पिटक के अन्दर कई ग्रन्थ होते थे-बाद के युगों में बौद्ध विद्वानों ने इन ग्रन्थों पर टीकाएँ लिखीं।

(2) दीपवंश तथा महावंश-जब बौद्ध धर्म का श्रीलंका जैसे नए क्षेत्रों में प्रसार हुआ, तब दीपवंश ( द्वीप का इतिहास) तथा महावंश (महान इतिहास) जैसे क्षेत्र विशेष के बौद्ध इतिहास को लिखा गया। इनमें से कई रचनाओं में बुद्ध की जीवनी लिखी गई है। अधिकांश पुराने ग्रन्थ पालि में हैं। कालान्तर में संस्कृत में भी ग्रन्थों की रचना की गई।

(3) बौद्ध ग्रन्थों का संरक्षण जब बौद्ध धर्म का पूर्वी एशिया में प्रसार हुआ, तब फा-शिएन और श्वैन-त्सांग जैसे तीर्थयात्री बौद्धग्रन्थों की खोज में चीन से भारत आए थे। वे अनेक बौद्ध ग्रन्थ अपने देश ले गए जहाँ विद्वानों ने इनका अनुवाद किया। भारत के बौद्ध शिक्षक भी दूर-दराज के देशों में गए। बुद्ध की शिक्षाओं का प्रचार-प्रसार करने के लिए वे अनेक ग्रन्थ भी अपने साथ ले गए। कई शताब्दियों तक ये पांडुलिपियाँ एशिया के भिन्न-भिन्न देशों में स्थित बौद्ध- विहारों में संरक्षित थीं। पालि, संस्कृत, चीनी और तिब्बती भाषाओं में लिखे इन ग्रन्थों से आधुनिक अनुवाद तैयार किए गए हैं।

प्रश्न 3.
नियतिवादियों एवं भौतिकवादियों के विचारों की विवेचना कीजिये।
उत्तर:
नियतिवादियों के विचार मक्खलिगोसाल नामक व्यक्ति नियतिवादियों के प्रमुख दार्शनिक थे। नियतिवादियों के प्रमुख विचार निम्नलिखित थे
(1) सब कुछ पूर्व निर्धारित है नियतिवादियों के अनुसार सब कुछ पूर्व निर्धारित है। सुख और दुःख पूर्व निर्धारित मात्रा में माप कर दिए गए हैं। स्न्नों संका में बदला नहीं जा सकता। इन्हें बढ़ाया या घटाया नहीं जा सकता। जैसे धागे का गोला फेंक देने पर लुढ़कते लुढ़कते अपनी पूरी लम्बाई तक खुलता जाता है, उसी प्रकार मूर्ख और विद्वान दोनों ही पूर्व निर्धारित मार्ग से होते हुए दुःखों का निदान करेंगे।

(2) कर्म – मुक्ति की निरर्थक आशा करना- नियतिवादियों का कहना है कि बुद्धिमान लोग यह विश्वास करते हैं कि वे अपने सद्गुणों एवं तपस्या के बल पर कर्म मुक्ति प्राप्त करेंगे। इसी प्रकार मूर्ख लोग उन्हीं कार्यों को सम्पन्न करके शनैः-शनैः कर्म मुक्ति प्राप्त करने की आशा करते हैं। परन्तु उनका यह सोचना गलत है तथा दोनों में से कोई भी कुछ नहीं कर सकता। इसलिए लोगों के भाग्य में जो कुछ लिखा है, उसे उन्हें भोगना ही पड़ेगा।

भौतिकवादियों के विचार अजीत केसकंबलिन नामक व्यक्ति भौतिकवादियों के प्रमुख दार्शनिक थे भौतिकवादियों के प्रमुख विचार निम्नलिखित है-
(1) संसार में दान, यज्ञ या चढ़ावा जैसी कोई वस्तु नहीं होती। इस दुनिया या दूसरी दुनिया जैसी किसी वस्तु का कोई अस्तित्व नहीं होता।
(2) मनुष्य चार तत्त्वों से बना होता है जब उसकी मृत्यु होती है, तब मिट्टी वाला अंश पृथ्वी में, जल वाला अंश जल में, गर्मी वाला अंश आग में तथा साँस का अंश वायु में वापस मिल जाता है और उसकी इन्द्रियाँ अन्तरिक्ष का भाग बन जाती हैं।
(3) दान देने की बात मूर्खों का सिद्धान्त है, यह सिद्धान्त असत्य है। कुछ मूर्ख एवं विद्वान दोनों ही कट कर नष्ट हो जाते हैं। मृत्यु के बाद कोई नहीं बचता।

JAC Class 12 History Important Questions Chapter 4 विचारक, विश्वास और इमारतें : सांस्कृतिक विकास

प्रश्न 4.
महात्मा बुद्ध की जीवनी का वर्णन कीजिए।
उत्तर:
महात्मा बुद्ध की जीवनी महात्मा बुद्ध बौद्ध धर्म के संस्थापक और प्रवर्तक थे। बुद्ध का जन्म 563 ई. पू. में कपिलवस्तु के निकट लुम्बिनी नामक वन में हुआ था। इनके पिता का नाम शुद्धोधन था, जो शाक्य गणराज्य के प्रधान थे। इनकी माता का नाम महामाया (मायादेवी) था। बुद्ध के बचपन का नाम सिद्धार्थ था।

16 वर्ष की आयु में सिद्धार्थ का विवाह यशोधरा नामक एक सुन्दर कन्या से किया गया। कुछ समय बाद उनके यहाँ पुत्र ने जन्म लिया, जिसका नाम राहुल रखा गया। परन्तु सिद्धार्थ की संसार से विरक्ति बढ़ती गई। महाभिनिष्क्रमण-नगर-दर्शन हेतु विभिन्न अवसरों पर बाहर जाते हुए सिद्धार्थ ने एक वृद्ध व्यक्ति, रोगी, मृतक एवं संन्यासी को देखा जिन्हें देखकर उन्हें संसार से विरक्ति हो गई। अन्त में 29 वर्ष की आयु में सिद्धार्थ अपनी पत्नी, अपने पुत्र तथा राजकीय वैभव को छोड़कर ज्ञान की खोज में निकल पड़े।

यह घटना ‘महाभिनिष्क्रमण’ के नाम से प्रसिद्ध है। ज्ञान की प्राप्ति प्रारम्भ में सिद्धार्थ ने वैशाली के ब्राह्मण विद्वान आलारकालाम तथा राजगृह के विद्वान् उद्रक रामपुत्त से ज्ञान प्राप्त करने का प्रयास किया, परन्तु उनकी जिज्ञासा शान्त नहीं हुई। इसके बाद वे उरुवेल के जंगल में अपने पाँच ब्राह्मण साथियों के साथ कठोर तपस्या करने लगे, परन्तु यहाँ भी उनके हृदय को शान्ति नहीं मिली। उनका शरीर सूख सूख कर काँटा हो गया, परन्तु उन्हें ज्ञान की प्राप्ति नहीं हुई। अतः उन्होंने भोजन आदि ग्रहण करना शुरू कर दिया जिससे नाराज होकर उनके पाँचों साथी सिद्धार्थ का साथ छोड़कर वहाँ से चले गए।

अन्त में सिद्धार्थ एक पीपल के वृक्ष के नीचे ध्यान की अवस्था में बैठ गए। सात दिन अखण्ड समाधि में लीन रहने के बाद वैशाखी पूर्णिमा की रात को उन्हें ‘ज्ञान’ प्राप्त हुआ, उन्हें सत्य के दर्शन हुए और वे ‘बुद्ध’ कहलाने लगे। इस घटना को ‘सम्बोधि’ कहा जाता है। जिस वृक्ष के नीचे सिद्धार्थ को ज्ञान प्राप्त हुआ, उसे ‘बोधिवृक्ष’ कहा जाने लगा और गया ‘बोधगया’ के नाम से प्रसिद्ध हुआ। धर्म का प्रचार ज्ञान प्राप्त करने के पश्चात् बुद्ध ने सारनाथ पहुँचकर उन पाँच ब्राह्मणों को उपदेश दिया, जो उन्हें छोड़कर चले आए थे। ये पाँचों ब्राह्मण बुद्ध के अनुयायी बन गए। इस घटना को ‘धर्मचक्र प्रवर्तन’ कहते हैं। इसके बाद बुद्ध ने काशी, कोशल, मगध, वज्जि प्रदेश, मल्ल, वत्स आदि में अपने धर्म की शिक्षाओं का प्रचार किया।

उनके अनुयायियों की संख्या दिन-प्रतिदिन बढ़ती गई। उनके अनुयायियों में अनेक राजा, व्यापारी, ब्राह्मण, विद्वान, सामान्यजन कर्मकार, दास, शिल्पी आदि सम्मिलित थे। महापरिनिर्वाण लगभग 45 वर्ष तक बुद्ध अपने धर्म का प्रचार करते रहे। अन्त में 483 ई. पू. में 80 वर्ष की आयु में कुशीनगर में बुद्ध का देहान्त हो गया। बौद्ध परम्परा के अनुसार यह घटना ‘महापरिनिर्वाण’ कहलाती है।

JAC Class 12 History Important Questions Chapter 4 विचारक, विश्वास और इमारतें : सांस्कृतिक विकास

प्रश्न 5.
बुद्ध की शिक्षाओं का विवेचन कीजिए। अथवा
उत्तर:
बौद्ध धर्म के प्रमुख सिद्धान्तों का वर्णन कीजिए। बुद्ध की शिक्षाएँ (बौद्ध धर्म के सिद्धान्त ) बुद्ध की प्रमुख शिक्षाएँ निम्नलिखित हैं –
(1) विश्व अनित्य है – बौद्ध दर्शन के अनुसार विश्व अनित्य है और निरन्तर परिवर्तित हो रहा है। यह आत्माविहीन है क्योंकि यहाँ कुछ भी स्थायी या शाश्वत नहीं है।
(2) मध्यम मार्ग – बुद्ध का कहना था कि न तो मनुष्य को घोर तपस्या करनी चाहिए, न ही अधिक भोग- विलास में लिप्त रहना चाहिए।
(3) भगवान का होना या न होना अप्रासंगिक- बौद्ध धर्म की प्रारम्भिक परम्पराओं में भगवान का होना या न होना अप्रासंगिक था।
(4) समाज का निर्माण मनुष्यों द्वारा किया जाना- बुद्ध मानते थे कि समाज का निर्माण मनुष्यों ने किया था, न कि ईश्वर ने
(5) व्यक्तिगत प्रवास पर बल बुद्ध के अनुसार व्यक्तिगत प्रयास से सामाजिक परिवेश को बदला जा सकता था।
(6) व्यक्ति केन्द्रित हस्तक्षेप तथा सम्यक् कर्म पर बल बुद्ध ने जन्म-मृत्यु के चक्र से मुक्ति, आत्म-ज्ञान और निर्वाण के लिए आत्मकेन्द्रित हस्तक्षेप और सम्यक् कर्म पर बल दिया।
(7) निर्वाण बौद्ध धर्म के अनुसार निर्वाण का अर्थ था अहं और इच्छा का समाप्त हो जाना जिससे गृह त्याग करने वालों के दुःख के चक्र का अन्त हो सकता था।
(8) चार आर्य सत्य बौद्ध धर्म के अनुसार चार था। आर्य सत्य निम्नलिखित हैं –

  • दुःख – यह संसार दुःखमय है। संसार में सर्वत्र दुःख ही दुःख है।
  • दुःख समुदय-दुःख और कष्टों का कारण तृष्णा है।
  • दु:ख निरोध- तृष्णा नष्ट कर देने से दुःखों से. मुक्ति प्राप्त हो सकती है।
  • दु:ख निरोध का मार्ग-तृष्णा के विनाश के लिए अष्टांगिक मार्ग का अनुसरण करना चाहिए।

(9) अष्टांगिक मार्ग-अष्टांगिक मार्ग की मुख्य आठ बातें निम्नलिखित हैं –

  • सम्यक् दृष्टि (सत्य, विश्वास या दृष्टिकोण)
  • सम्यक् संकल्प (सत्य संकल्प या विचार)
  • सम्यक् वाणी (मधुर एवं सत्य वचन बोलना )
  • सम्यक् कर्मान्त (सदाचारपूर्ण आचरण करना)
  • सम्यक् आजीव (सदाचारपूर्ण साधनों से जीविका का निर्वाह करना)
  • सम्यक् व्यायाम ( निरन्तर सद्प्रयास )
  • सम्यक् स्मृति (दुर्बलताओं की निरन्तर स्मृति)
  • सम्यक् समाधि (चित्त की एकाग्रता)।

(10) स्वावलम्बन पर बल बुद्ध का कहना था कि मनुष्य स्वयं अपने भाग्य का निर्माता है।

प्रश्न 6.
बौद्ध संघ पर एक संक्षिप्त निबन्ध लिखिए।
अथवा
बुद्ध के अनुयायियों के बारे में आप क्या जानते हैं ?
उत्तर:
बौद्ध संघ (बुद्ध के अनुयायी ) महात्मा बुद्ध के उपदेशों से प्रभावित होकर उनके अनुयायियों की संख्या बढ़ती चली गई। धीरे-धीरे उनके शिष्यों का दल तैयार हो गया। उन्होंने अपने शिष्यों के लिए बौद्ध संघ की स्थापना की संघ ऐसे बौद्ध भिक्षुओं की एक संस्था थी, जो धम्म के शिक्षक बन गए। ये बौद्ध भिक्षु एक सादा जीवन व्यतीत करते थे। उनके पास जीवनयापन के लिए अत्यावश्यक वस्तुओं के अलावा कुछ नहीं होता था। वे दिन में एक बार भोजन करते थे। इसके लिए वे उपासकों से भोजन दान प्राप्त करने के लिए एक कटोरा रखते थे। चूंकि वे दान पर निर्भर थे, इसलिए उन्हें भिक्षु कहा जाता था।

(1) महिलाओं को संघ में सम्मिलित करना प्रारम्भ में केवल पुरुष ही संघ में सम्मिलित हो सकते थे परन्तु बाद में महिलाओं को भी संघ में सम्मिलित होने की अनुमति दे दी गई। बौद्ध ग्रन्थों से ज्ञात होता है कि अपने प्रिय शिष्य आनन्द के अनुरोध पर बुद्ध ने महिलाओं को संघ में सम्मिलित होने की अनुमति प्रदान कर दी। बुद्ध की उपमाता महाप्रजापति गोतमी संघ में सम्मिलित होने वाली प्रथम भिक्षुणी थी। संघ में सम्मिलित होने वाली कई स्त्रिय धम्म की उपदेशिकाएँ बन गई। कालान्तर में ये धेरी बनीं, जिसका अर्थ है-ऐसी महिलाएँ जिन्होंने निर्वाण प्राप्त कर लिया हो।

(2) बुद्ध के अनुयायियों का विभिन्न सामाजिक वर्गों से सम्बन्धित होना बुद्ध के अनुयायी विभिन्न सामाजिक वर्गों से सम्बन्धित थे। इनमें राजा, धनवान, गृहपति और सामान्य जन कर्मकार, दास, शिल्पी सभी सम्मिलित थे। संघ में सम्मिलित होने वाले सभी भिक्षुओं तथा भिक्षुणियों को बराबर माना जाता था क्योंकि भिक्षु बनने पर उन्हें अपनी पुरानी पहचान को त्याग देना पड़ता था।

JAC Class 12 History Important Questions Chapter 4 विचारक, विश्वास और इमारतें : सांस्कृतिक विकास

(3) संघ की संचालन पद्धति संघ की संचालन पद्धति गणों और संघों की परम्परा पर आधारित थी। इसके अन्तर्गत लोग वार्तालाप के द्वारा एकमत होने का प्रयास करते थे एकमत न होने पर मतदान द्वारा निर्णय लिया जाता था।

प्रश्न 7.
स्तूपों की खोज किस प्रकार हुई ? अमरावती तथा साँची की नियति बताइये।
अथवा
स्तूपों की खोज किस प्रकार हुई ? साँची क्यों बच गया, जबकि अमरावती नष्ट हो गया?
उत्तर:
स्तूपों की खोज बौद्ध कला में स्तूपों का महत्त्वपूर्ण स्थान है। साँची और अमरावती के स्तूप तत्कालीन वास्तुकला के उत्कृष्ट नमूने हैं।
(1) साँची के स्तूप की खोज-साँची का स्तूप एक पहाड़ी के ऊपर बना हुआ है जो मुकुट जैसा दिखाई देता है 1818 ई. में साँची की खोज हुई।

(2) अमरावती के स्तूप की खोज- 1796 ई. में स्थानीय राजा एक मन्दिर का निर्माण करना चाहते थे। उन्हें अचानक अमरावती के स्तूप के अवशेष मिल गए। उन्होंने इसके पत्थरों का प्रयोग करने का निश्चय किया। कुछ वर्षों के बाद कॉलिन मेकेंजी नामक एक अंग्रेज अधिकारी को इस क्षेत्र से गुजरने का अवसर मिला। यद्यपि उन्होंने यहाँ कई मूर्तियाँ प्राप्त की और उनका विस्तृत चिशंकन भी किया, परन्तु उनकी रिपोर्ट कभी प्रकाशित नहीं हुई।

(3) गुन्टूर के कमिश्नर द्वारा अमरावती की यात्रा करना- 1854 ई. में गुन्टूर (आन्ध्र प्रदेश) के कमिश्नर ने अमरावती की यात्रा की। उन्होंने कई मूर्तियों तथा उत्कीर्ण पत्थरों को एकत्रित किया और वे उन्हें मद्रास ले गए।

(4) अमरावती के स्तूप के उत्कीर्ण पत्थरों को विभिन्न स्थानों पर ले जाना-1850 के दशक में अमरावती के स्तूप के उत्कीर्ण पत्थरों को भिन्न-भिन्न स्थानों पर ले जाया गया। कुछ उत्कीर्ण पत्थर कोलकाता में एशियाटिक सोसायटी ऑफ बंगाल पहुंचे, तो कुछ पत्थर मद्रास के इण्डिया ऑफिस पहुँचे। कुछ पत्थर तो लन्दन तक पहुँच गए। कई अंग्रेज अधिकारियों ने अपने बागों में अमरावती की मूर्तियाँ स्थापित कीं।

(5) साँची क्यों बच गया, जबकि अमरावती नष्ट हो गया – सम्भवतः अमरावती की खोज थोड़ी पहले हो गई थी तब तक विद्वान इस बात के महत्त्व को नहीं समझ पाए थे कि किसी पुरातात्विक अवशेष को उठाकर ले जाने की बजाय खोज की जगह पर ही संरक्षित करना बड़ा महत्त्वपूर्ण था। 1818 में सांची की खोज हुई उस समय तक भी इस स्तूप के तीन तोरणद्वार खड़े थे।

चौथा तोरण द्वार वहीं पर गिरा हुआ था। इसके अतिरिक्त टीला भी अच्छी दशा में था। उस समय कुछ विदेशियों ने यह सुझाव दिन था कि तोरणद्वारों को पेरिस या लन्दन भेज दिया जाए। परन्तु कई कारणों से साँची का स्तूप वहीं बना रहा और आज भी बना हुआ है। दूसरी ओर अमरावती का महाचैत्य अब केवल एक छोटा सा टीला है, जिसका सम्पूर्ण गौरव नष्ट हो चुका है।

प्रश्न 8.
पौराणिक हिन्दू धर्म की विशेषताओं का वर्णन कीजिए।
उत्तर:
पौराणिक हिन्दू धर्म की विशेषताएँ पौराणिक हिन्दू धर्म की विशेषताओं का वर्णन अग्रानुसार
(1) पौराणिक हिन्दू धर्म का उदय पौराणिक हिन्दू धर्म में भी मुक्तिदाता की कल्पना विकसित हो रही थी। इस पौराणिक हिन्दू धर्म में दो परम्पराएँ प्रमुख थीं –

  • वैष्णववाद तथा
  • शैववाद वैष्णववाद में विष्णु को सबसे प्रमुख देवता माना जाता है और शैववाद में शिव परमेश्वर माने गए हैं।

(2) अवतारवाद – वैष्णववाद में कई अवतारों के चारों ओर पूजा पद्धतियाँ विकसित हुई इस परम्परा के अन्दर दस अवतारों की कल्पना की गई है। लोगों में यह मान्यता प्रचलित थी कि पापियों के बढ़ते प्रभाव के कारण जब संसार में अराजकता, अव्यवस्था और विनाश की स्थिति उत्पन्न हो जाती थी, तब संसार की रक्षा के लिए भगवान अलग-अलग रूपों में अवतार लेते थे।

(3) अवतारों को मूर्तियों में दिखाना- कई अवतारों को मूर्तियों के रूप में दिखाया गया है। अन्य देवताओं की मूर्तियों का भी निर्माण किया गया। शिव को उनके प्रतीक लिंग के रूप में बनाया जाता था परन्तु उन्हें कई बार मनुष्य के रूप में भी दर्शाया गया है।

(4) पुराणों की कहानियाँ – इन मूर्तियों के उत्कीर्णन का अर्थ समझने के लिए इतिहासकारों को इनसे जुड़ी हुई कहानियों से परिचित होना पड़ता है। कई कहानियाँ प्रथम सहस्राब्दी के मध्य से ब्राह्मणों द्वारा रचित पुराणों में पाई जाती हैं। इनमें देवी-देवताओं की भी कहानियाँ हैं। प्रायः इन्हें संस्कृत श्लोकों में लिखा गया था। इन्हें ऊंची आवाज में पढ़ा जाता था जिसे कोई भी सुन सकता था।

यद्यपि महिलाओं और शूद्रों (हरिजनों) को वैदिक साहित्य पढ़ने – सुनने की अनुमति नहीं थी. परन्तु वे पुराणों को सुन सकते थे। पुराणों की अधिकांश कहानियाँ लोगों के आपसी मेल- मिलाप से विकसित हुई। पुजारी, व्यापारी और सामान्य स्त्री-पुरुष एक स्थान से दूसरे स्थान पर आते-जाते हुए, अपने विश्वासों और विचारों का आदान-प्रदान करते थे। उदाहरण के लिए, वासुदेव कृष्ण मथुरा क्षेत्र के महत्त्वपूर्ण देवता थे। कई शताब्दियों के दौरान उनकी पूजा देश के दूसरे प्रदेशों में भी प्रचलित हो गई।

JAC Class 12 History Important Questions Chapter 4 विचारक, विश्वास और इमारतें : सांस्कृतिक विकास

प्रश्न 9.
अतीत की समृद्ध दृश्य परम्पराओं को समझने के लिए यूरोपीय विद्वानों द्वारा किये गये प्रयासों की विवेचना कीजिए।
उत्तर:
अतीत की समृद्ध दृश्य-परम्पराओं को समझने के प्रयास अतीत की समृद्ध दृश्य पराम्पराएँ ईंट और पत्थर से निर्मित स्थापत्य कला, मूर्तिकला और चित्रकला के रूप में हमारे सामने आई हैं। इनमें से कुछ कलाकृतियाँ नष्ट हो गई हैं। परन्तु जो बची हैं, और संरक्षित हैं वे हमें इन सुन्दर कलाकृतियों के निर्माताओं – कलाकारों, मूर्तिकारों, राजगीरों और वास्तुकारों के दृष्टिकोण से परिचित कराती हैं। परन्तु उनके दृष्टिकोण को समझना सरल नहीं है। हम कभी भी यह बात पूर्ण रूप से नहीं समझ सकते कि इन प्रतिकृतियों को देखने और पूजने वाले लोगों के लिए इनका क्या महत्त्व था।

यूरोपीय विद्वानों के प्रयास – उन्नीसवीं शताब्दी में यूरोपीय विद्वानों ने जब देवी-देवताओं की मूर्तियाँ देखीं तो वे उनकी पृष्ठभूमि और महत्त्व को नहीं समझ पाए। कई सिरों, हाथों वाली या मनुष्य और जानवर के रूपों को मिलाकर बनाई गई मूर्तियाँ उन्हें खराब लगती थीं और कई बार उन्हें इन मूर्तियों से घृणा होने लगती थी। प्राचीन यूनान की कला परम्परा से तुलना करना- इन प्रारम्भिक यूरोपीय विद्वानों ने ऐसी विचित्र मूर्तियों का अभिप्राय समझने के लिए उनकी तुलना प्राचीन यूनान की कला परम्परा से की। ये विद्वान् यूनानी परम्परा से परिचित थे।

यद्यपि वे प्रारम्भिक भारतीय मूर्तिकला को यूनान की कला से निम्न स्तर का मानते थे, फिर भी वे बुद्ध और बोधिसत्त की मूर्तियों की खोज से काफी प्रोत्साहित हुए। इसका कारण यह था कि ये मूर्तियाँ यूनानी आदर्शों से प्रभावित थीं। ये मूर्तियाँ अधिकतर उत्तर-पश्चिम के नगरों तक्षशिला और पेशावर में मिली थीं। इन प्रदेशों में ईसा से दो सौ वर्ष पहले भारतीय यूनानी शासकों ने अपने राज्य स्थापित किए थे। ये मूर्तियाँ यूनानी मूर्तियों से काफी मिलती- जुलती थीं। चूंकि ये विद्वान यूनानी परम्परा से परिचित थे, इसलिए उन्होंने इन्हें भारतीय मूर्तिकला का सर्वश्रेष्ठ नमूना माना।

कलाकृतियों के महत्त्व को समझने के लिए लिखित ग्रन्थों से जानकारी प्राप्त करना मूर्तियों के महत्त्व और संदर्भ को समझने के लिए कला के इतिहासकार प्राय: लिखित ग्रन्थों से जानकारी प्राप्त करते हैं भारतीय मूर्तियों की यूनानी मूर्तियों से तुलना कर निष्कर्ष निकालने की अपेक्षा यह तरीका अधिक अच्छा है। परन्तु यह बहुत सरल तरीका नहीं है।

कला – इतिहासकारों व पुराणों में इस कथा को पहचानने के लिए काफी खोजबीन की है। परन्तु उनमें काफी मतभेद हैं। कुछ इतिहासकारों के अनुसार इस कथा में गंगा नदी के स्वर्ग से अवतरण का चित्रण है। उनका कहना है कि चट्टान की सतह के बीच प्राकृतिक दरार शायद नदी को दर्शा रही है। यह कथा महाकाव्यों और पुराणों में वर्णित है। परन्तु अन्य विद्वानों की मान्यता है कि यहाँ पर दिव्यास्य प्राप्त करने के लिए महाभारत में वर्णित अर्जुन की तपस्या का चित्रण है। उनका कहना है कि मूर्तियों के बीच एक साधु की मूर्ति केन्द्र में रखी गई है।

प्रश्न 10.
लौकिक सुखों से आगे, वर्धमान महावीर की शिक्षाओं का विस्तार से वर्णन कीजिए।
उत्तर:
वर्धमान महावीर की शिक्षाएँ –
(1) सारा संसार संजीव है-सम्पूर्ण विश्व प्राणवान है। यह जैन धर्म की सबसे प्रमुख अवधारणा है। वर्धमान महावीर के अनुसार सम्पूर्ण विश्व में कुछ भी निर्जीव नहीं है। यहाँ तक कि पत्थर, चट्टान, जल में भी जीवन होता है। यह भगवान महावीर की गहन अन्तर्दृष्टि थी।

(2) अहिंसा-अहिंसा का सिद्धान्त जैन दर्शन का केन्द्रबिन्दु है। जीवों के प्रति दयाभाव रखना चाहिए, हमें किसी भी जानवर, मनुष्य, पेड़-पौधे, यहाँ तक कि कीट- पतंगों को भी नहीं मारना चाहिए। महावीर के अनुसार आत्मा केवल मनुष्यों में ही नहीं बल्कि पशुओं, कीड़ों, पेड़-पौधों आदि में भी होती हैं।

JAC Class 12 History Important Questions Chapter 4 विचारक, विश्वास और इमारतें : सांस्कृतिक विकास

(3) जीवन चक्र और कर्मवाद जैन दर्शन के अनुसार जन्म और पुनर्जन्म का चक्र कर्मानुसार निर्धारित होता है। ‘कर्मों के अनुरूप ही पुनर्जन्म होता है।

(4) तप आवागमन के बन्धन से छुटकारा पाने का एक ही मार्ग त्याग और तपस्या है। मुक्ति का प्रयास संसार को त्याग कर विहारों में निवास कर तपस्या द्वारा ही फलीभूत होगा।

(5) पंच महाव्रत- जैन धर्म में पंचमहाव्रतों का सिद्धान्त दिया गया है; जैसे कि अहिंसा, अमृषा, अस्तेय, अपरिग्रह, इन्द्रिय निग्रह।

(6) त्रि-रत्न-पंचमहाव्रत के अतिरिक्त वर्धमान महावीर ने निर्वाण प्राप्ति हेतु त्रिरत्न नामक तीन सिद्धान्तों- सत्य, विश्वास, सत्यज्ञान और सत्यकार्य को अपनाने की शिक्षा भी दी है।

(7) ईश्वर की अवधारणा से मुक्ति वर्धमान महावीर ईश्वर के अस्तित्व में बिल्कुल भी विश्वास नहीं करते थे। वह यह नहीं मानते थे कि ईश्वर ने संसार की रचना की है। जैन धर्म आत्मा के अस्तित्व में विश्वास रखता है।

(8) वेदों में विश्वास जैन धर्म को मानने वाले वेदों को ईश्वरी ज्ञान नहीं मानते वे वेदों में मुक्ति हेतु दिए साधनों यह जप, तप, हवन आदि को व्यर्थ समझते हैं।

(9) जाति पाँति में अविश्वास जैन धर्म के अनुयायी जाति पाँति में विश्वास नहीं करते हैं।

(10) स्याद्वाद – जैन धर्म में स्याद्वाद प्रमुख है; कोई भी मनुष्य सम्पूर्ण सत्य के ज्ञान का दावा नहीं कर सकता। सत्यं बहुत व्यापक है, इसके अनेक पक्ष हैं। देश, काल और परिस्थिति के अनुसार मनुष्य को सत्य का आंशिक ज्ञान प्राप्त होता है।

प्रश्न 11.
बौद्ध कैसे की जाती थी? महात्मा धर्म का लेखन और इनकी सुरक्षा
उत्तर:
बुद्ध तथा उनके अनुयायी लोगों में वार्तालाप व वाद-विवाद द्वारा मौखिक रूप से अपनी शिक्षाओं का प्रसार करते थे। महात्मा बुद्ध के जीवन काल में वक्तव्यों का लेखन नहीं किया गया। महात्मा बुद्ध के देह त्याग के उपरान्त पाँचवीं चौथी सदी ईसा पूर्व में उनके शिष्यों ने वैशाली में एक सभा का आयोजन किया। शिक्षाओं का संकलन पुस्तकों के रूप में किया गया; जिन्हें त्रिपिटक, जिसका अर्थ है विभिन्न प्रकार के ग्रन्थों को रखने की तीन टोकरियाँ कहा गया। तदुपरान्त बौद्ध धर्म के विद्वानों द्वारा इन पर टिप्पणियाँ लिखी गयीं।
त्रिपिटक त्रिपिटक में तीन पिटक सम्मिलित हैं –

(1) विनयपिटक विनयपिटक में भिक्षु और भिक्षुणी जो कि बौद्ध मठों या संघों में रहते थे उनके लिए आचार संहिता थी। उन्हें किस प्रकार का आचरण करना चाहिए। इस सम्बन्ध में विनयपिटक में व्यापक नियम दिये गये हैं।

JAC Class 12 History Important Questions Chapter 4 विचारक, विश्वास और इमारतें : सांस्कृतिक विकास

(2) सुत्तपिटक – सुतपिटक में महात्मा बुद्ध की शिक्षाएँ दी गई हैं, जो उन्होंने समाज के प्रत्येक पक्ष को ध्यान में रखते हुए दी हैं।

(3) अभिधम्मपिटक अभिधम्मपिटक में दर्शनशास्त्र से सम्बन्धित विषयों की गहन व्याख्याएँ सम्मिलित हैं।
(i) नए ग्रन्थ धीरे-धीरे बौद्ध धर्म का विस्तार श्रीलंका तक फैल गया तो नए ग्रन्थों जैसे दीपवंश और महावंश नामक ग्रन्थों की रचना की गई। इन ग्रन्थों में क्षेत्र विशेष से सम्बन्धित बौद्ध साहित्य प्राप्त होता है। कई रचनाओं में महात्मा बुद्ध की जीवनी का भी समावेश है।
(ii) ग्रन्थों की सुरक्षा बौद्ध धर्म का प्रचार-प्रसार जब पूर्व एशिया तक फैल गया तो इससे आकर्षित होकर फा-शिएन और श्वेन त्सांग नामक चीनी तीर्थयात्री बौद्ध ग्रन्थों की खोज में चीन से भारत आए।

वे इनमें से कई ग्रन्थों को अपने साथ चीन ले गए और वहाँ इसका अनुवाद चीन की भाषा में किया। भारतीय बौद्ध धर्म के प्रचारक भी देश-विदेश में बौद्ध धर्म की शिक्षाओं का प्रचार-प्रसार करने के लिए कई ग्रन्थों को साथ ले गए। एशिया में फैले विभिन्न बौद्ध विहारों में यह पाण्डुलिपियाँ वर्षों तक संरक्षित रहीं। बौद्ध धर्म के प्रमुख केन्द्र तिब्बत के ल्हासा मठ में बौद्ध धर्म की पालि संस्कृत, चीनी, तिब्बती भाषा की तमाम पाण्डुलिपियाँ आज भी संरक्षित हैं।